You are on page 1of 629

Atoms, Molecules & Stoichiometry

ATOMS, MOLECULES & STOICHIOMETRY 1a


Structured / Free Response Questions )
1 A sample containing ammonium sulfate, (NH4)2SO4, was warmed with 250 cm3 of
0.800 mol dm−3 aqueous sodium hydroxide. After the evolution of ammonia had
ceased, the excess of sodium hydroxide solution was neutralised by 85.0 cm3 of 0.500
mol dm−3 hydrochloric acid. Calculate the mass of ammonium sulfate in the sample?

2 A student wants to collect 80 cm3 of carbon dioxide gas from the reaction between
dilute hydrochloric acid and powdered calcium carbonate at room temperature. Given
that 50 cm3 of 2 mol dm−3 of dilute hydrochloric acid was used, calculate the mass of
powdered calcium carbonate to be used, assuming that the yield of carbon dioxide
was 90 %.

3 In a titration experiment, a standard solution of aqueous sulfuric acid was to be used


to accurately determine the concentration of an aqueous solution of sodium hydroxide.

In the preparation of the standard solution of aqueous sulfuric acid, a suitable volume
23
of concentrated sulfuric acid would be diluted with deionised water in a 250 cm3
volumetric flask.

By taking the following points into consideration,


LL

• The approximate concentration of sodium hydroxide was 0.1 mol dm−3.


• 25.0 cm3 of sodium hydroxide was pipetted out.
• A titration value of 25.00 cm3 of aqueous sulfuric acid is to be obtained.

(a) Calculate the concentration of the diluted aqueous sulfuric acid in the standard
solution.
(b) Determine the volume of 10 mol dm−3 of concentrated sulfuric acid to be used
in the preparation of 250 cm3 of standard solution.

4 [IJC 2011/III/2(c)]
(i) A large portion of the copper produced in the world is obtained by the smelting of
chalcopyrite. It is an iron-containing mineral with the molecular formula of CuFeS2.
Smelting of chalcopyrite can be represented by the following equation:

3CuFeS2 + 8O2 3Cu + FeO + Fe2O3 + 6SO2

Calculate the volume of sulfur dioxide gas released into the atmosphere at r.t.p. if 1
tonne of the rock sample is smelted. It is estimated that the rock contains 1.5% by
mass of chalcopyrite. [1 tonne = 1000 kg]

(ii) Why is it important to remove SO2 from the products of smelting before release into
the atmosphere?
[3]
1
Atoms, Molecules & Stoichiometry

5 [TJC 2009/III/4(a)]

By harnessing the benefits of nitrates, Mankind has made progress over the centuries.
A common Group II nitrate, magnesium nitrate is being used in ceramics, printing, chemical
and agriculture industries.

This nitrate can be produced commercially via two reactions:


The first reaction involves magnesium oxide and nitric acid. In the second reaction,
magnesium nitrate can be formed using magnesium hydroxide and ammonium nitrate.
Also, it was determined that magnesium nitrate decomposes at 330 °C.

(i) Calculate the amount of magnesium nitrate produced if 0.500 g of magnesium oxide is
reacted with 50 cm3 of 0.100 mol dm-3 nitric acid.

(ii) In the second reaction, a gaseous side product is formed.


This gaseous side product forms a white solid when reacted with hydrogen chloride
gas.
Calculate the volume of gas that will be evolved at room temperature and pressure if
1.5 g of magnesium nitrate is formed in the second reaction.
[4]

6 [SRJC 2011/III/1(b)]
Acrylamide is a compound that is found in many deep fried foods such as French fries and
23
potato chips. Lately, it has been added to the list of substances of very high concern in 2010
as it is believed to be carcinogenic.

(i) Acrylamide contains 50.7% by mass of carbon, 7.0% by mass of hydrogen, 19.8% by
LL

mass of nitrogen and 22.5% by mass of oxygen. Given that the relative molecular
mass of acrylamide is 71.0, determine the molecular formula of acrylamide.

(ii) Apart from deep fried food, acrylamide can also be found in beverages such as prune
juice and brewed coffee. Studies have shown that a person can safely ingest up to 0.5
mg / kg body weight in a day without any observed adverse effects to his or her
nervous system. Given that 1 kg of brewed coffee contains 1.2 × 10−7 mol of
acrylamide, calculate the maximum volume of coffee a 50 kg woman can drink per
day. You may assume that coffee has the same density as water.
[4]

7 [NYJC 2011/III/3(d)]
A is a compound of a Group II element M. When A was treated with dilute hydrochloric acid,
a colourless gas, B was evolved, which turned moist litmus paper red.
On bubbling B through lime water, a precipitate, C was formed, but passage of further gas
resulted in a clear solution, D.
On strong heating, 1.9735 g of A decomposed, giving 1.5334 g of E its oxide, a white solid.
The sample of E was dissolved in water and made up to 250 cm3 in a standard flask. 25.0
cm3 aliquots were titrated with acid and required 20.30 cm3 of 0.0985 mol dm─3 hydrochloric
acid.

(i) Identify the compounds B, C and D.


(ii) Calculate the relative molecular mass of A.

2
Atoms, Molecules & Stoichiometry

(iii) Consider the identity of A and calculate the relative atomic mass of the Group II
element M in A. Hence identify M.
[7]

8 [VJC 2011/II/2(c)]
0.500 g of a Group II iodate(V) salt, M(IO3)2, was heated and decomposed to give a white
solid, a purple gas, and a colourless gas that rekindles a glowing splint.

(i) Identify the 3 decomposition products.

(ii) Write a balanced equation for the decomposition reaction, including state symbols.

(iii) Given that 0.0532 g of the white solid was collected, calculate the relative atomic
mass of the metal, and hence deduce its identity.
[4]

9 [MJC 2011/III/1(d)]
The hexadentate ligand EDTA4− forms a complex both with magnesium ions and calcium
ions. With a suitable indicator, this can be used to determine the total concentration of
magnesium ions and calcium ions in a mixture.

A sample of a mixture of anhydrous magnesium chloride and anhydrous calcium chloride


had a mass of 0.344 g and was dissolved in water to make 100 cm3 of solution. A 10 cm3
23
portion of this solution required 33.3 cm3 of 0.010 mol dm−3 EDTA4− for complete reaction.

For this calculations, you may assume that 1 mole of EDTA4− reacted with 1 mole of M2+ for
LL

this calculation where M is Mg or Ca.

Calculate the mass of the anhydrous magnesium chloride in the mixture.


[3]
10 [MJC 07 Common Test]
The table below lists the percentage by mass of phosphorus in some common fertilizers:

Fertilizer % by mass of phosphorus


Ca(H2PO4)2 26.5
X 23.5

Phosphorus fertilizers are derived from the phosphate rock called fluoropatite, Ca5(PO4)3F.
Fluoropatite is insoluble in water, so it must be converted to water-soluble calcium
dihydrogen phosphate, Ca(H2PO4)2.

2Ca5(PO4)3F(s) + 7H2SO4(aq) → 3Ca(H2PO4)2(aq) + 7CaSO4(aq) + 2HF(aq)

(a) If the yield of Ca(H2PO4)2 cannot exceed 75%, what is the minimum mass of
fluoropatite required to produce 4 kg of calcium sulfate?
[2]

3
Atoms, Molecules & Stoichiometry

(b) Explain, with reasoning, why the concentration of sulfuric acid needed is
approximately 2 mol dm−3 if 50 dm3 of this sulfuric acid is allowed to react with the
same amount of fluoropatite in (a).
[2]

(c) Another common fertilizer X which contains N, H, P and O was subjected to


quantitative analysis and the following data was obtained. A 0.250 g sample was
processed to liberate all its N as NH3 and 0.0644 g of NH3 was obtained. A 0.712 g
sample was carried through a series of reaction until all its hydrogen is converted to
1.77g of HCl. The percentage of phosphorus in fertilizer X was found to be 23.5%.
Calculate the empirical formula of fertilizer X.
[3]

11 Ascorbic acid (Vitamin C) is an organic compound containing carbon, hydrogen and oxygen.
When 1.000g of ascorbic acid was completely burnt in oxygen, 1.500g of carbon dioxide
and 0.405g of water were formed. Calculate the empirical formula of ascorbic acid and
determine its molecular formula if its molar mass is 176g mol-1.

23
LL

4
Atoms, Molecules & Stoichiometry

Multiple Choice Questions (Section A)

1 [ACJC 2011/I/1]

23
LL

2 [ACJC 2011/I/2]

5
Atoms, Molecules & Stoichiometry

3 [ACJC 2011/I/29]

4 [CJC 2011/I/11]

23
LL

5 [CJC 2011/I/2]

6
Atoms, Molecules & Stoichiometry

6 [AJC 2009/I/1]

7 [HCI 2009/I/2]

8 [NJC 2011/I/6]
23
LL

9 [ACJC 2009/I/1]

7
Atoms, Molecules & Stoichiometry

10 [PJC 2011/I/1]

23
LL

11 [ACJC 2009/I/2]

8
Atoms, Molecules & Stoichiometry

12 [RI 2011/I/1]

13 [VJC 2011/I/1]

14 [TJC 2011/I/1]
23
LL

15 [VJC 2009/I/1]

9
Atoms, Molecules & Stoichiometry

16 [MJC 2009/I/1]

17 [TJC 2009/I/2]

23
LL

18 [PJC H1 2009/I/1]

19 [NYJC H1 2009/I/2]

10
Atoms, Molecules & Stoichiometry

20 [IJC 2009/I/1]

21 [CJC 2009/I/1]

22 [SAJC H1 2009/I/1]
23
LL

23 [HCI 2010/I/2]

11
Atoms, Molecules & Stoichiometry

24 [NJC 2011/I/5]

25 [NJC 2011/I/4]
23
LL

12
Atoms, Molecules & Stoichiometry

Multiple Choice Questions (Section B)

1 [Nov 2011/I/31]

2 [Nov 2009/I/31]
Three organic molecules each have

• Three elements;
23
• The composition, by mass, C,54.5%; H, 9.1%

What could these molecules be?


LL

1. CH3CH2CH2CO2H
2. OHCCH2CH2CH2OH
3. CH3CH=CHCH2SH

3 [IJC 2011/I/31]

13
Atoms, Molecules & Stoichiometry

4 [HCI 2009/I/38]

5 [SRJC 2009/I/31] 23
LL

14
Atoms, Molecules & Stoichiometry

ATOMS, MOLECULES & STOICHIOMETRY SUGGESTED SOLUTIONS

Structured/Free Response Questions


1

1) (NH4)2SO4 reacts with NaOH to give NH3. By inspection, a salt and water are also formed.
2) Write the balanced equation for the reaction of HCl with NaOH.

HCl + NaOH NaCl + H2O …(1)

85.0
Amount of HCl reacted = x 0.500 = 4.25 x 10-2 mol
1000

From equation (1), amount of NaOH that reacted with HCl = 4.25 x 10-2 mol

(NH4)2SO4 + 2NaOH 2NH3 + Na2SO4 + 2H2O …(2)

250
Initial amount of NaOH = x 0.800 = 0.200 mol
1000
23
Amount of NaOH that reacted with (NH4)2SO4 = 0.200 - 4.25 x 10-2 = 0.1575
mol
LL

From equation (2), amount of (NH4)2SO4 = ½ x 0.1575 = 7.875 x 10-2 mol

Molar mass of (NH4)2SO4 = 2 x (14.0 + 4 x 1.0) + 32.1 + 4 x 16.0 = 132.1 g


mol-1

Mass of (NH4)2SO4 in the sample = 7.875 x 10-2 x 132.1 = 10.4 g

2 Given that 80 cm3 of CO2 was to be collected and the yield was only 90 %,
80 10−3 100
No. of moles of CO2 to be collected =  = 3.704 x 10−3 mol
24 90
CaCO3 + 2 HCl → CaCl2 + CO2 + H2O
50
No. of moles of HCl used =  2.0 = 0.100 mol
1000
Hence, HCl is in excess and CaCO3 is the limiting reagent.
No. of moles of CaCO3 required = 3.704 x 10−3 mol
Mass of CaCO3 required = 3.704 x 10−3 x 100.1 = 0.371 g

15
Atoms, Molecules & Stoichiometry

3 H2SO4 + 2 NaOH → Na2SO4 + 2 H2O

Since the approximate concentration of NaOH is 0.1 mol dm−3 and 25.0 cm3 was
used,
25
No. of moles of NaOH =  0.1 = 0.00250 mol
1000
No. of moles of H2SO4 reacted with NaOH = ½ x 0.00250 = 0.00125 mol
Volume of H2SO4 used in titration = 25.00 cm3
(a) Concentration of H2SO4 in the diluted standard solution
0.00125
= 1000 = 0.0500 mol dm−3
25.00
(b) No. of moles of H2SO4 in 250 cm3 of standard solution
0.0500
=  250 = 0.0125 mol
1000
Let the volume of original H2SO4 solution used be V cm3
Since only water was added to dilute the original H2SO4 solution to 250 cm3,
no. of moles of H2SO4 in V cm3 = no. of moles of H2SO4 in 250 cm3
23
No. of moles of 10 mol dm−3 H2SO4 in V cm3 = 0.0125 mol
0.0125
Hence, V = 1000 = 1.25 cm3
LL

10

4 (i) 1.50
Mass of CuFeS2 smelted =  1  10 6 = 1.5 x 104 g
100

1.50  10 4
Amount of CuFeS2 smelted = = 81.744 mol
[63.5 + 55.8 + 2(32.1)]
Mole ratio of CuFeS2 : SO2 = 3 : 6
Hence mole of SO2 = 2 x 81.744 = 163.488 mol
Vol. of SO2 = 163.488 x 24.0 dm3 = 3.92 x 104 dm3

(ii) SO2 in the atmosphere will result in the formation of acid rain, which is detrimental
to living things and will corrode buildings and cause air pollution.

16
Atoms, Molecules & Stoichiometry

5 (i) MgO (s) + 2HNO3 (aq) → Mg(NO3)2 (aq) + H2O (l)


0.500
Amount of MgO = = 0.0124 mol
24 .3 + 16 .0

50
Amount of HNO3 =  0.100 = 0.00500 mol (limiting reagent)
1000
0.005
Amount of Mg(NO3)2 produced = = 0.00250 mol
2

(ii) Mg(OH)2 + 2NH4NO3 → Mg(NO3)2 + 2NH3 + 2H2O


1 .5
Amount of Mg(NO3)2 = = 0.0101 mol
24 .3 + 28 .0 + 96 .0

Volume of NH3 produced = 0.0101 2  24 = 0.485 dm3

6 (i)
C H N O
Ar 12 1 14 16
% by mass 50.7 7.0 19.8 22.5
23
Ratio 4.225 7 1.414 1.406
3 5 1 1
LL

Let the molecular formula of acrylamide be C3nH5nNnOn.

Mr of acrylamide = 71.0 = n (3 × 12.0 + 5 × 1.0 + 14.0 + 16.0)


n =1

Hence, the molecular formula of acrylamide is C3H5NO.

(ii) Maximum mass of acrylamide the woman can take in per day = 0.5 × 10−3 × 50
= 0.025 g

Mass of acrylamide in 1 kg of coffee = 1.2 × 10−7 × 71.0


= 8.520 × 10−6 g per kg of coffee

Maximum mass of coffee the woman can drink per day = 0.025 ÷ 8.520 × 10−6
= 2934 kg

Maximum volume of coffee the woman can drink per day = 2934 ÷ 1 = 2930 dm3

7 (i) B CO2 C CaCO3 D Ca(HCO3)2

(ii) MO + H2O → M(OH)2


M(OH)2 + 2 HCl → MCl2 + 2H2O

17
Atoms, Molecules & Stoichiometry

nHCl for 25.0 cm3 aliquots = 0.0985 x 20.30/1000 = 0.0020 mol


nHCl reacting with E = 0.0020 x 250/25 = 0.020 mol
nMO = nM(OH)2 = ½ x 0.020 = 0.010 mol

MA = 1.9735 / 0.010 = 197.35 ≈ 197.4

(iii) A is likely to be a carbonate (since CO2 is evolved on heating)


Hence formula will be MCO3
Hence Ar of M will be 197.35 ─ 12.0 ─ 3(16.0) = 137.35
M is likely to be Barium.

8 (i) Metal oxide, iodine and oxygen gas

(ii) 2M(IO3)2(s) → 2MO(s) + 2I2(g) + 5O2(g)

(iii) nsalt = noxide


0.500 0.0532
=
Ar + 2(127 .0 + 3  16 .0) Ar + 16 .0
Ar = 23 .8
23
M is magnesium.
LL

9 Let the mass of the anhydrous magnesium chloride in the mixture = x g


x 0.344 − x
Hence total no. of moles of Mg2+ and Ca2+ = +
93.3 111 .1

No. of mole of EDTA4− = 0.000333

total no. of moles of Mg2+ and Ca2+ in 100 cm3 = 0.00333

x 0.344 − x
Hence 0.00333 = +
93.3 111 .1

x = 0.157 g

10 (a) 4000
Amt of CaSO4 = = 29.4 mol
136.2
29.4
Assuming 100% yield, theoretical amt of Ca5(PO4)3F required =  2 = 8.40 mol
7
8.40
Since % yield cannot exceed 75%, amt of Ca5(PO4)3F required = 100 = 11.2
75
mol

18
Atoms, Molecules & Stoichiometry

Mass of Ca5(PO4)3F required = 11.2 × 504.5 = 5.65 × 103 g

(b) Amt of H2SO4 = 2 × 50 = 100 mol


7
If 11.2 mol of Ca5(PO4)3F is used up, Amt of H2SO4 required = 11.2 = 39.2 mol
2
Therefore H2SO4 is in excess, hence the concentration of H2SO4 need not be
accurate, an approximate concentration of 2 mol dm−3 will ensure that H2SO4 is in
excess when 50 dm3 is used.

(c) 0.0644
Amt of NH3 = = 3.79 × 10−3 mol
17
Mass of nitrogen in the sample = 3.79 × 10−3 × 14.0 = 0.0531 g

0.0531
% mass of nitrogen in fertilizer X = 100% = 21.2%
0.25
1.77
Amt of HCl = 100% = 0.0485 mol
36.5
0.0485 1.0
% mass of hydrogen in fertilizer X = 100% =6.81%
0.712
% mass of oxygen in fertilizer X = 100−(6.81+21.2+23.5) = 48.5%
23
LL

Element H N O P
% mass 6.81 21.2 48.5 23.5
Mole ratio 6.81 21.2 48.5 23.5
1.0 14.0 16.0 31.0
Simplest mol 9 2 4 1
ratio

Empirical formula of fertilizer X = (NH4)2HPO4 or N2O4H9P

11 CxHyOz + mO2 → xCO2 + (y/2)H2O

Mass of C present in 1.000g of ascorbic acid = (1.500/44) x 12.0 = 0.4091g


Mass of H present in 1.000g of ascorbic acid = (0.405/18) x 1.0 = 0.0225g
Mass of O present in 1.000g of ascorbic acid = (1.000 – 0.4091 – 0.0225) = 0.5684

Element C H O
Mass (g) 0.4091 0.0225 0.5684
Molar mass (g mol- 12.0 1.0 16.0
1
)
Amount (mol) 0.0341 0.0225 0.0355
Ratio 3 4 3

19
Atoms, Molecules & Stoichiometry

Therefore the empirical formula for the ascorbic acid is C3H4O3.

3(12)a + 4(1)a + 3(16)a = 176


88a = 176
a =2

Therefore the molecular formula for the ascorbic acid is C6H8O6.

Multiple Choice Questions (Section A)


1. 2. 3. 4. 5. 6. 7. 8. 9. 10. 11. 12. 13. 14. 15. 16. 17. 18. 19. 20.
A A A D C B B A C A A C B D D B A C C A
21. 22. 23. 24. 25.
C B C A B

Multiple Choice Questions (Section B)


1. 2. 3. 4. 5.
D A D B B

Note:
23
Do not assume that all the given answers are correct.
Do discuss with your friends and check with your tutor should you think that any given
answer is incorrect.
LL

20
Redox Reactions

REDOX REACTIONS 1b
1
Structured / Free Response Questions
)
To standardise sodium thiosulfate solution, 25.0 cm3 of 0.0204 mol dm−3 of acidified
potassium manganate (VII) solution was added to an excess of acidified potassium iodide
solution. If the iodine liberated required 24.40 cm3 of sodium thiosulfate for reduction,
calculate the concentration of sodium thiosulfate in mol dm−3.

2 Ammonium iron (II) sulfate crystals have the formula, (NH4)2Fe(SO4)2•nH2O. In an experiment
to determine n, 8.565 g of the crystals was dissolved and made up to 250 cm3 of the solution
with distilled water and dilute sulfuric acid. A 25.0 cm3 aliquot of the solution was further
acidified and titrated against 22.50 cm3 of potassium dichromate solution of concentration
0.0125 mol dm−3.

(a) Calculate the concentration of (NH4)2Fe(SO4)2 solution in mol dm-3.

(b) Calculate the percentage of water of crystallisation in (NH4)2Fe(SO4)2•nH2O crystals.

(c) Calculate the value of n.


23
3 [MJC 2009/III/2(c)]
The purity of iodates (VII) can be estimated by adding an acidified solution of potassium
iodide and titrating the iodine produced with aqueous sodium thiosulphate.
LL

In one such analysis, a sample of 0.200 g of sodium iodate (VII), Na5IO6, was dissolved in
water and treated with an excess of acidified KI to yield iodine as the only product. The
iodine liberated required 22.0 cm3 of 0.100 mol dm-3 sodium thiosulphate to reach the end-
point.

Calculate the purity of the sodium iodate (VII) in the sample.

[ Given I2 + 2S2O32- → 2I- + S4O62- ; Mr of Na5IO6 = 338 ]


[3]

4 [YJC 2011/III/1(c)]
(a) Brass is an alloy of copper and zinc. It reacts with nitric acid to give a solution containing
Cu2+ and Zn2+ ions. The concentration of Cu2+ can be found by reaction with an excess of
potassium iodide
2Cu2+ (aq) + 4I - (aq) → 2CuI(s) + I2(aq)
2-
followed by titration of the iodine liberated against thiosulfate, S2O3 .

The iodine liberated by a piece of brass of mass 0.2055 g required 13.70 cm 3 of 0.150 mol
dm-3 thiosulfate solution.

1
Redox Reactions

Brass is an alloy of copper and zinc. It reacts with nitric acid to give a solution containing
Cu2+ and Zn2+ ions. The concentration of Cu2+ can be found by reaction with an excess of
potassium iodide

2Cu2+ (aq) + 4I- (aq) → 2CuI(s) + I2(aq)

followed by titration of the iodine liberated against thiosulfate, S2O32- .

(i) Write an equation to represent the titration reaction.

(ii) Calculate the percentage of copper in brass.


[4]

(b) When copper (I) oxide was reacted with hot dilute sulfuric acid a blue solution was produced
together with a brown precipitate.

(i) Suggest identities for the products and write, giving state symbols, a balanced
equation for the above reaction.

(ii) State what is unusual about this reaction.


[4]
5 [NJC 2011/II/4(a)(c)]
(a) Each year, about 1 × 106 tonnes of sulfuric acid are prepared starting from sulfur. Because
any unreacted materials are recycled, the overall percentage yield of H2SO4 is nearly 100%.
23
Calculate the mass of sulfur required to produce 1 × 106 tonnes of sulfuric acid annually.
Assume that the overall percentage yield of H2SO4 is 100%.
LL

(1 tonne = 1000 kg.)


[2]

1)(b) Nitrogen dioxide, NO2, (from car exhaust fumes) can react with sulfur dioxide, SO2, (from
the burning of fossil fuels) in the presence of water vapour in the atmosphere to produce
sulfuric acid (acid rain) and nitrogen monoxide, NO. The nitrogen monoxide is rapidly re-
oxidised to nitrogen dioxide by oxygen.

(i) Construct balanced equations for these two reactions.

(ii) Hence suggest the role played by nitrogen dioxide in the overall process.
[3]

6 [DHS 2011/II/2(b)]
In the determination of the mass percentage of ascorbic acid (C6H8O6) present in vitamin C
tablets, the following reaction involving iodine takes place.

C6H8O6 + I2 → C6H6O6 + 2 I– + 2H+

Due to the low solubility of iodine in water, it is generated in situ using potassium iodate(V),
potassium iodide and sulfuric acid. In this reaction, iodine is the only product.

(i) Write a balanced equation for the reaction between IO3– and I– to obtain I2.

2
Redox Reactions

(ii) A 3.0 g sample of the tablet is crushed and dissolved in 100.0 cm3 of dilute sulfuric
acid. To this solution, 1.08 g of potassium iodate(V) and 4.32 g of potassium iodide is
added. A 25.0 cm3 aliquot is withdrawn and 21.60 cm3 of 0.1 mol dm–3 thiosulfate is
required to titrate with the excess iodine, using starch as an indicator.

Calculate the total amount of iodine produced from reacting potassium iodate(V) with
potassium iodide.

(iii) Determine the total amount of excess iodine in the 100 cm3 solution.

(iv) Hence or otherwise, obtain the mass percentage of ascorbic acid in the sample.
[4]

7 In an experiment to determine the percentage purity of a sample of hydrated iron(II) sulfate


crystals, FeSO4•7H2O, m g of FeSO4•7H2O was dissolved in water and used to prepare a
100 cm3 solution in a volumetric flask.

The solution containing iron(II) ions was then titrated with a standard solution of potassium
manganate(VII) from a burette.

By considering the following points, determine a suitable mass of impure FeSO4•7H2O, m,


to be used to prepare the 100 cm3 solution.
• 25.0 cm3 of aqueous iron(II) ions is pipetted out.
23
• Concentration of aqueous potassium manganate(VII) is 0.020 mol dm−3.
• A titration value of 25.00 cm3 is to be obtained.
• Estimated percentage purity of FeSO4•7H2O crystals is 97 %
LL

8 [IJC 2011/III/5(b)]
A metallic salt containing the anion IxCly– decomposed when heated to form iodine vapour
and the metallic chloride. When 0.270g of this compound was heated, the iodine evolved
was bubbled into excess potassium iodide solution.

The dissolved iodine required 21.30 cm3 of 0.0600 moldm-3 of aqueous sodium thiosulfate
for complete reaction. When the residue was dissolved in water, the resultant solution
required 6.40 cm3 of 0.100 mol dm-3 aqueous silver nitrate for complete reaction.

(i) Determine the value of x and y in IxCly–.

(ii) Hence, determine the identity of the metallic element.


[6]

3
Redox Reactions

9 N2003/III/Q5(c)
Iodine and chlorine react together to form compound X (ICln).
When 0.0010 mol of X was reacted with KI (aq) in excess, all of its iodine was converted
into I2. The iodine liberated required 40.0 cm3 of 0.100 mol dm-3 sodium thiosulfate,
Na2S2O3, solution for complete reaction.

(i) How many moles of iodine is produced?

(ii) Hence, determine the value of n in ICln.

(iii) Write a balanced equation for the reaction between Cl2 and I2.
[3]

10 [YJC 2011/III/1(c)]
Under suitable conditions, SCl2 reacts with water to produce a yellow precipitate and a
solution J. Solution J is a mixture of SO2(aq) and compound K.

(i) Suggest the identity of compound K.

(ii) Write a balanced equation, including state symbols, for the reaction between SCl2 and
water.

(iii) Describe what would be observed when each of the following is added to separate
samples of solution J. Write balanced equation for any reaction that occurs.
23
• AgNO3(aq)
• acidified K2Cr2O7(aq)
LL

[6]

11 12.5 cm3 of a given solution of hydrogen peroxide were diluted to 500 cm3 with distilled
water. 25.0 cm3 of this diluted solution then required 22.50 cm3 of 0.0200 mol dm−3
potassium manganate(VII) solution for titration in acidic conditions.

(a) Calculate the concentration of the hydrogen peroxide solution

(b) The “volume strength” of H2O2 is the number of volumes of O2 obtained from 1 volume
of H2O2 at s.t.p. based on the disproportionation reaction: 2H2O2(aq) ⎯⎯→ 2H2O(l) +
O2(g)
For example, a “20 volume” H2O2 means 1 dm3 of H2O2, on decomposition, gives 20
dm3 of O2. Calculate the volume strength of the hydrogen peroxide solution.

4
Redox Reactions

12 [SAJC 2009/III/1(a)]
Acidified potassium manganate, KMnO4, is a strong oxidising agent which can oxidise both
X2+ and C2O42- in XC2O4. One of the products of this reaction is CO2.
15.00 cm3 of 0.1 mol dm-3 acidified KMnO4 was found to completely oxidise
25.0 cm3 of 0.1 mol dm-3 XC2O4 solution.
(i) Write a balanced equation for the reaction between MnO4- and C2O42- only. Hence,
determine the volume of KMnO4 needed to oxidise C2O42- only.

(ii) Using your answer in (a)(i), find the oxidation state of X in the product.
[5]

13 [VJC 2011/II/2(b)]
1.25 x 10-3 mol of an aqueous bromate salt containing the BrOm- anion was added to excess
potassium iodide. The resulting mixture was washed with chloroform to dissolve the iodine,
and the aqueous and organic layers were separated. The BrOm- anion was reduced to
bromide ions.

The iodine collected in the organic layer was titrated against 0.500 mol dm-3 sodium
thiosulfate. 14.90 cm3 of titrant was required to discharge the blue-black colour of the starch
indicator. Calculate the value of m.
23
[4]

14 [IJC 2011/III/1(c)]
LL

MnO2 undergoes the following reaction as shown in the schematic diagram below:

Step I: Oxidation of MnO2 in molten KOH with oxygen from the air produces a
green compound, A.

Step II: Reaction of A with dilute acid produced 0.174 g of MnO2 and a purple
solution of compound B was observed. A solution of compound B is
just decolourised by 40.0 cm3 of 0.500 mol dm-3 iron(II) sulphate.

(i) B contains potassium, manganese and oxygen only. Suggest the chemical formula of
compound B that causes the solution to appear purple.

(ii) Calculate the number of moles of MnO2 produced in step II.

(iii) Calculate the number of moles of compound B produced in step II, given that
compound B and iron(II) sulphate reacted in a 1:5 mole ratio.

5
Redox Reactions

(iv) With reference to step II of the above reaction scheme, let the oxidation number of
manganese in A be n, i.e. Mnn+. Using your answer to c(ii) and c(iii), write an
expression in n to show

(I) the no. of moles of electrons gained by Mnn+ when Mnn+ is reduced to MnO2
(II) the no. of moles of electrons lost by Mnn+ when Mnn+ is oxidised to B

(v) Hence, using your answer to c(iv), calculate the oxidation number of manganese in
compound A.
[5]

Multiple Choice Questions (Section A)


1 [AJC 2011/I/2]
20

23
2 [ACJC 2011/I/14]
LL

3 [IJC 2011/I/1]

6
Redox Reactions

4 [JJC 2011/I/2]

5 [AJC 2010/I/2]

23
LL

6 [CJC 2010/I/2]

7
Redox Reactions

7 [NJC 2010/I/1]

8 [NYJC 2011/I/2]

9 [SAJC 2010/I/2]
23
LL

10 [NJC 2010/I/12]

8
Redox Reactions

11 [PJC 2010/I/3]

12 [RVHS 2011/I/2]

13 [MJC 2010/I/1]
23
LL

14 [NJC 2010/I/7]

9
Redox Reactions

15 [TJC 2011/I/2]

16 [CJC 2009/I/3]

23
17 [RI 2011/I/2]
LL

18 [TPJC 2010/I/15]

10
Redox Reactions

19 [PJC H1 2009/I/2]

20 [CJC 2009/I/4]

23
Multiple Choice Questions (Section B)
LL

1 [JJC 2011/I/31]

11
Redox Reactions

2 [PJC 2011/I/31]

3 [JJC 2009/I/31]

23
LL

4 [NYJC 2009/I/31]

12
Redox Reactions

REDOX REACTIONS SUGGESTED SOLUTIONS

Structured/Free Response Questions

1 Oxidation: MnO4− + 8H+ + 5e → Mn2+ + 4H2O


Reduction: 2I− → I2 + 2e
Overall reaction: 2MnO4− + 16H+ + 10I− → 2Mn2+ + 8H2O + 5I2
25.0
Amount of KMnO4 used = x 0.0204 = 5.10 x 10−4 mol
1000 Hint!
1) Construct eqn between MnO4− and I−.
5 How much MnO4− reacted?
Amount of I2 liberated = x 5.10 x 10-4 = 1.275 x 10−3 mol 2)
2 3) How much I2 was liberated?
4) Construct eqn between I2 and S2O32-.
2S2O32− + I2 → S4O62− + 2I− 5) How much S2O32- reacted?

Amount of S2O32− reacted = 2 x 1.275 x 10-3 = 2.55 x 10-3 mol


2.55 x 10 −3
Concentration of Na2S2O3 = = 0.105 mol dm−3
24.40 x 10 −3
23
2 (a) Overall Eqn: Cr2O72− + 14H+ + 6Fe2+ ⎯⎯→ 2Cr3+ + 7H2O + 6Fe3+
22.50
Amount of Cr2O72- reacted = x 0.0125 = 2.813  10−4 mol Hint!
LL

1000 Write balanced equation. How


Amount of Fe2+ in 25.0 cm3 = 6  2.813  10−4 = 1.688  10−3 mol much Cr2O72− reacted? How
much Fe2+ reacted in 25.0 cm3?
1.688 x 10−3
Concentration of (NH4)2Fe(SO4)2 = x 1000 = 0.0675 mol dm−3
25.0
(b) Amount of (NH4)2Fe(SO4)2 in 250 cm3 = 1.688  10−2 mol
Mass of (NH4)2Fe(SO4)2 in 250 cm3 1.688  10−2  284.0 = 4.793 g

Mass of water of crystallisation in the 8.565 g of crystals = 8.565 – 4.793 = 3.772 g

% of water of crystallisation in the crystals = 3.772 ×100% = 44.0 %


8.565
What is the mass of
(NH4)2Fe(SO4)2 in 250 cm3?
(c) 8.565
Relative formula mass of (NH4)2Fe(SO4)2.nH2O = = 507.4
1.688 x10 −2
284.0 + 18n = 507.4 Hint!
What is the relative formula mass
n = 12
of (NH4)2Fe(SO4)2.nH2O?
Note: Water of crystallisation must be expressed to the nearest whole number

13
Redox Reactions

3 Overall equation : IO65- + 7I- + 12H+ → 4I2 + 6H2O

No of mol IO65- = 1/8 (2.20 x 10-3) = 2.75 x 10-4

Mass of sodium iodate (VII) = 2.75 x 10-4 x x 338 = 0.09295g

Percentage by mass = 0.09295/0.200 = 46.5%

4
(a) (i) I2 + S2O32- → 2 I- + S4O62-

(ii) Amount of used in S2O32- titration = 13.70/1000 x 0.150 = 2.055 x 10-3 mol
2 S2O32- ≡ I2 ≡ 2 Cu2+
Amount of Cu 2+ reacted = 2.055 x 10 -3 mol
Amount of Cu in brass = 2.055 x 10 -3 x 63.5 = 1.30 x10-1 g
Percentage of copper in brass is (1.30 x10-1/ 0.2055) x 100% = 63.5%

(b) (i) Cu2O(s) + H2SO4(aq) → CuSO4(aq) + Cu(s) + H2O (l)


Blue solution: CuSO4 ; Brown precipitate: Cu(s)

(ii) Copper is both oxidised and reduced in the same reaction.


This is a disproportionation reaction.
23
5
(a) Mr of H2SO4 = 98.1
LL

Amt of H2SO4 in 1 × 106 tonnes = = 1.019 ×1010 mol [1]


Since conversion is 100%,
Mass of S required = 1.019 ×1010 × 32.1
= 3.27 ×1011 g or 3.27 ×108 kg or 3.27 ×105 tonnes

(b) (i) NO2+ SO2 + H2O → H2SO4 + NO


2NO + O2 → 2NO2

(ii) NO2 is a catalyst as it is regenerated.

6 (i) 5I– + 6H+ + IO3– → 3I2 + 3H2O


OR

5KI + 3H2SO4+ KIO3 → 3I2 + 3H2O + 3K2SO4

(ii) No. of mol of KIO3 = 0.005044 mol


No. of mol of KI = 0.02600 mol
Since 5I–  IO3–, IO3– is the limiting reagent

Since IO3–  3I2,


Total amount of iodine produced = 0.005044 x 3
=0.015133 mol = 0.0151 mol

14
Redox Reactions

(iii) 21 .60
No. of mol of S2O32– = (0.1) =0.00216 mol in 25 cm3
1000
I2  2S2O32–
0.00216
No. of mol of excess I2 in 100 cm3 = ( 4 ) = 0.00432 mol
2

(iv)
No. of mol of iodine reacted with ascorbic acid = 0.015133 – 0.00432
= 0.010813 mol
Mass of ascorbic acid = 0.010813 x 176.0 = 1.9030 g
1.9030
Mass percentage of ascorbic acid = (100%) =63.4%
3

7 Molar mass of FeSO4•7H2O = 277.9

Equation of reaction:

MnO4− + 5 Fe2+ + 8 H+ → Mn2+ + 5 Fe3+ + 4 H2O

25.00
No. of moles of MnO4− used in the reaction = x 0.020 = 5.00 x 10−4 mol
1000
23
No. of moles of Fe2+ in 25.0 cm3 = 5 x 5.00 x 10−4 = 2.50 x 10−3 mol

No. of moles of Fe2+ in 100 cm3 = 2.50 x 10−3 x 4 = 0.0100 mol


LL

Mass of FeSO4•7H2O = 277.9 x 0.0100 = 2.779 g

Since percentage purity is approximately 97 %,

100
Mass of crystals to be weighed out = 2.779 x = 2.86 g
97

8 (i) I 2 + 2S2O32- → S4O62- + 2I-


0.0600
No. of moles of S2O32- =  21.30 = 1.278 10-3 mol
1000
1.278X10-3
No. of moles of I2 = = 6.3910-4 mol
2
No. of moles of I- = 1.278 10-3 mol
0.100
No. of moles of Silver nitrate =  6.40=6.39  10-4 mol
1000
Ag+(aq) + Cl-(aq) → AgCl (s)
No. of moles of Cl- = 6.39  10-4 mol
I-: Cl-, x: y = 2: 1
X=2
Y=1

15
Redox Reactions

(ii) No. of moles of IxCly- = 6.39 10-4 mol


0.27
Mr of the metallic salt = = 421.9
6.4 10-4
Ar of metal = 421.9 – (79.9  2) – 35.5 = 133
Identity: Caesium / Cs

9 (i) 40
Amt of S2O32– =  0.100 = 0.00400mol
100
Mol ratio of iodine : thiosulfate ions = 1:2
1
Amt of I2 =  0.00400 = 0.00200mol
2
(ii) Mole ratio of ICln : I2 = 0.0010 = 0.00200 = 1:2

Equation: ICln + 3KI ⎯→ 2I2 + 3KCl


n=3

(iii) 3 Cl2 + I2 ⎯→ 2 ICl3

10 (i) HCl

(ii) 2SCl2(l) + 2H2O(l) → SO2(aq) + S(s) + 4HCl(aq)


23
(iii) with AgNO3(aq)
white precipitate formed
Ag+(aq) + Cl−(aq) → AgCl(s) and/or Ag+(aq) + SO32−(aq) → Ag2SO3(s)
LL

with acidified K2Cr2O7


orange K2Cr2O7 turns green
Cr2O72− + 3SO2 + 2H+ → 2Cr3+ + 3SO42− + H2O

11 (a) 22.50
Amount of MnO4− reacted = x 0.0200 = 4.50 x 10-4 mol
1000
Reduction: MnO4− + 8H+ + 5e− ⎯→ Mn2+ + 4H2O Hint!
Begin by writing a balanced equation
Oxidation: H2O2 ⎯→ O2 + 2H+ + 2e– between H2O2 and MnO4-.

Equation: 2MnO4− + 5H2O2 + 6H+ ⎯⎯→ 2Mn2+ + 8H2O + 5O2


5
Amount of H2O2 reacted = x 4.50 x 10−4 = 1.125 x 10−3 mol
2
Amt. of H2O2 in 500 cm3
3 −3 diluted solution = amt. Of H2O2
Amount of H2O2 in 25.0 cm aliquot = 1.125 x 10 mol
in 12.5 cm3 given solution

500
Amount of H2O2 in 500 cm3 = 1.125 x 10−3 x = 0.0225 mol
25.0

Amount of H2O2 in 12.5 cm3 of given solution = 0.0225 mol

16
Redox Reactions

0.0225
Hence concentration of H2O2 solution = x 1000 = 1.80 mol dm−3
12.5

(b) Consider a 1 dm3 solution of H2O2,

Amount of H2O2 in 1 dm3 = 1.80 mol

From the above equation, the molar ratio of H2O2 to O = 2:1

Amount of O2 produced = ½ x 1.80 = 0.900 mol

Molar volume of O2 at s.t.p. = 22.4 dm3 mol-1

Volume of O2 produced at s.t.p. = 0.90 x 22.4 = 20.16 dm3

Hence, volume strength of H2O2 is 20.2 (3 s.f.)

12 (i) 5C2O42- + 2MnO4- + 16H+ → 10CO2 + 2Mn2+ + 8H2O


No. of moles of C2O42- ions 0.025 x 0.1 = 0.0025
Given C2O42- : MnO4- is 5:2,
No. of moles of MnO4- needed to oxidize C2O42- = 0.0025 x 2/5 = 0.001
23
Thus, Volume of MnO4- needed = (0.001 / 0.1) x 1000 = 10cm3

(ii) Volume of MnO4- ions needed to oxidize X2+ ions = 15 – 10 = 5cm3


LL

No. of moles of MnO4- ions = [(15 – 10)/1000] x 0.1 = 0.0005


No of moles of electrons absorbed by MnO4- = 0.0005 x 5 = 0.0025
Mole ratio of X2+ : e = 1 : 1
Since 1 mole of X2+ loses 1 mole of electrons,
Oxidation state of X in the product = +3
13 I2 + 2S2O32- → 2I- + S4O62-
14.90
nthiosulfate =  0.500 = 7.45 x 10-3 mol
1000
niodine = ½ nthiosulfate = 3.725 x 10-3 mol
niodine 3.725 10 −3
= = 3 (nearest whole number)
nbromate 1.25 10 −3
I2 + 2e- → 2I-
6 mol of electrons were transferred
Original oxidation state of Br in BrOm- = -1 + 6 = +5

5 + m(-2) = -1
m=3

17
Redox Reactions

14 (i) KMnO4

(ii) No. of moles of MnO2 = 0.174 / (54.9 + 16 x 2) = 0.002 mol

(iii) No. of moles of compound B = 40/1000 x 0.5 x 1/5 = 0.004 mol

(iv) Let the oxidation number of A be n.


[R]: Mn(n) + (n – 4) e– → Mn(IV)
0.002 0.002(n-4) 0.002
No of moles of electrons gained = 0.002 (n – 4)

[O]: Mn(n) → Mn(VII) + (7-n)e–


0.004 0.004 0.004 (7 – n)
No. of moles of electrons lost = 0.004 (7 – n)

(v) No. of moles of electrons gained = No. of moles of electrons lost

0.002 (n – 4) = 0.004 (7 – n)
n = +6

Multiple Choice Questions (Section A)


1. 2. 3. 4. 5. 6. 7. 8. 9. 10. 11. 12. 13. 14. 15. 16. 17. 18. 19. 20.
B A C A C C C D C C C C D A C C C A B D
23
Multiple Choice Questions (Section B)
1. 2. 3. 4.
LL

A A B D

Note: Do not assume that all the given answers are correct. Do discuss with your
friends and check with your tutor should you think that any given answer is incorrect.

18
Atomic Structure

ATOMIC STRUCTURE 2
Structured / Free Response Questions

1 [MI 2011/II/2]
Deuterium, 2D, is an isotope of hydrogen. D2O can be used in the elucidation of molecular
structures of substances.

(i) Deduce the total number of neutrons present in D2O.

(ii) Suggest how D2O is different from H2O.

(iii) Given the average relative atomic mass of hydrogen is 1.00015, assuming that 2D is
the only other isotope of 1H, find the percentage abundance of deuterium.
[3]

2 [NYJC 2011/III/3(c)]
The discovery of the Periodic Table was an essential requirement of the amazing growth in
theoretical and practical chemistry in the 19th century. Today’s version has the elements
arranged in order of atomic number rather than atomic mass.

Write the formula of the compound formed from the following ionic interactions:
23
(i) The ions form from the largest and smallest ionizable atoms in Period 2.

(ii) The 2+ ion and the 2− ion are both isoelectronic with the Period 3 noble gas.
LL

(iii) The 2+ ion is the smallest with a filled d subshell; the anion forms from the smallest
halogen.

(iv) The 2+ ion and the 1− ion are both isoelectronic with the atoms of a chemically
unreactive Period 4 element.
[2]

3 [SRJC 2011/II/5(c)]
Element G is in Period III.
The successive ionisation energies of element G is shown below.
1st 2nd 3rd 4th 5th 6th 7th 8th
I.E. / 999 2265 3331 5071 7008 8487 27107 31719
kJmol-1

(i) Deduce the group number in which G belongs.

(ii) Write down the electronic configuration of G in the +4 oxidation state.


[2]

1
Atomic Structure

4 [SAJC 2011/III/2(a)], [MJC 2011/III/2(a)]


Explain the following observations

(i) The fifth ionisation energy of chlorine is a smaller numerical value than the fifth
ionisation energy of sulfur.

(ii) The first ionisation of oxygen is less endothermic than that of the preceding element in
the Periodic Table.
[4]

5 [TJC 2011/III/1(c)]
Explain the following observation with the aid of relevant equations.
The second ionisation energies of the Group II elements decrease down the group. [2]

6 [YJC 2011/II/3(a)]
The graph of successive ionisation energy of magnesium and aluminium are given below:

23
LL

(i) Write an equation to represent the second ionisation energy of magnesium.

(ii) Explain why the ionisation energies increase for both magnesium and aluminium as
electrons are successively removed.

(iii) Explain why the third ionisation energy of magnesium is much higher than that of
aluminium.
[4]

2
Atomic Structure

7 [MJC 2009/II/2(a)]
When Mendeleev created the Periodic Table, there were uncertainties regarding the relative
atomic mass of tellurium, Te. It is now known that there are eight isotopes of tellurium.

isotope percentage abundance isotopic mass x


percentage abundance
tellurium-120 0.09 11
tellurium-122 2.46 300
tellurium-123 0.87 107
tellurium-124 4.61 572
tellurium-125 6.99 874
tellurium-126 18.71 2357
tellurium-128 31.79
tellurium-130

Complete the above table and give your answers to two decimal places.

Hence, calculate the relative atomic mass of tellurium to five significant figures.
[3]

8 [TJC 2009/II/3(a)]
The first six ionisation energies of three successive elements in a Periodic Table are
given below:
23
Ionisation energy in kJ mol−1
Element 1st 2nd 3rd 4th 5th 6th
LL

A 786 1580 3230 4360 16100 19800


B 1060 1900 2920 4960 6270 21300
C 1000 2260 3390 4540 7000 8500

(i) State and explain the group of the Periodic Table which element B belongs to.

(ii) Explain why the first ionisation energy of B is higher than the first ionisation
energies of A and C.
[5]

3
Atomic Structure

9 [IJC 2008/II/3]
The 68Ge isotope of the Group IV element Germanium is medically useful because it
undergoes a natural radioactive process to give a Gallium isotope, 68Ga, which can be used
to detect tumours. This transformation of Germanium occurs when an electron enters the
nucleus, changing a proton into a neutron.

(i) State the number of protons and neutrons in the 68Ga isotope formed.

(ii) While undergoing radioactive decay, the isotope will give out an alpha particle, α
which is the nucleus of helium atom.
The angle of deflection of the alpha particle, α, is 3o.

I. Draw the path for which the alpha particle will take when placed in an electric field
shown below.

23
II. Give reasons and suggest the angle of deflection of the deuterium ion, 12 D + .
[3]
LL

10 J90/3/2c
Use of the Data Booklet is relevant to this question

Orange street lamps contain sodium with a small amount of neon. The light is
produced when gaseous atoms are ionized in an electric field. When first turned
on, the lamps emit a red glow characteristic of neon, but after some time, the
orange glow of sodium predominates. Explain this phenomenon.

4
Atomic Structure

11 [RJC 2003 JC1 CT1]

Elements A, B, C, D, E, F, G and H are consecutive elements in the Periodic Table.

(a) The first ten successive ionisation energies of the element D are as follows:

496 4560 6910 9540 13400 16600 20100 25500 28900 141000

Deduce which group D belongs to in the Periodic Table.


[2]

(b) D is a Period 3 element. Explain why D cannot be a Period 2 element.


[1]

(c) The figure below shows the variation in second ionisation energy of elements A, B, C,
F, G and H. The corresponding values for D and E are missing.

23
LL

(i) With the aid of a relevant equation, explain what is meant by the second
ionisation energy of the element D.

(ii) Complete the graph in the above figure by predicting the second ionisation
energies of D and E.
[3]

5
Atomic Structure

12 [RJC 2006 JC1 CT1]

On Planet X, the number of subshells associated with each principal quantum number and
the respective energy levels of the subshells are similar to that on Earth. Each orbital
contains a maximum of two electrons. However, the number of orbitals that make up a
subshell may or may not be identical to that on Earth.

Figure 1 represents the sketch of the successive ionisation energies of all the electrons of
an element, Brownie, Bw, on Planet X.

23
LL

(a) (i) With the aid of a relevant equation, explain what is meant by the third ionisation
energy of Brownie, Bw.

(ii) By interpreting figure 1, suggest with reasoning, which period Brownie belongs
to.
[4]

(b) Given that Brownie is a first row d-block element with a half-filled set of d−orbitals,
deduce with reasoning, the number of p orbitals that make up a p subshell.
[2]

(c) State the full electronic configuration of Brownie, assuming that the Aufbau Principle,
the Pauli Exclusion Principle and Hund’s Rule are followed.
[1]

6
Atomic Structure

Multiple Choice Questions (Section A)


1 [AJC 2011/I/1]

23
LL

2 [AJC 2011/I/3]

3 [RVHS 2011/I/3]

7
Atomic Structure

4 [DHS 2011/I/9]

5 [AJC 2010/I/4]

23
LL

6 [HCI 2011/I/3]

8
Atomic Structure

7 [HCI 2011/I/16]

8 [MJC 2011/I/3]

9 [YJC 2011/I/2]

23
10 [YJC 2010/I/3]
LL

11 [NYJC 2010/I/2]

9
Atomic Structure

12 [NJC 2010/I/6]

23
LL

13 [RI 2010/I/1]

14 [NJC 2011/I/1]

10
Atomic Structure

15 [VJC 2011/I/2]

16 [SRJC 2011/I/3] 23
LL

17 [TPJC 2010/I/1]

11
Atomic Structure

18 [MJC 2007/I/7]

19 [IJC 2011/I/3]
23
LL

20 [IJC 2011/I/4]

12
Atomic Structure

Multiple Choice Questions (Section B)

1 [ACJC 2011/I/31]

23
2 [NYJC 2011/I/31]
LL

13
Atomic Structure

3 [ACJC 2010/I/31]

4 [PJC 2009/I/31]

23
5 [TPJC 2009/I/31]
LL

14
Atomic Structure

ATOMIC STRUCTURE SUGGESTED SOLUTIONS

Structured/Free Response Questions

1 (i) Total number of neutrons = 1 + 1 + 8 = 10

(ii) D2O has a slightly larger molecular mass than H2O.

(iii) Let percentage abundance of the 2D isotope be x.


x 100 - x
1.00015 = 100 X2+ 100 X1

x = 0.015%

2 (i) LiF

(ii) CaS

(iii) ZnF2

(iv) SrBr2
23
3 (i) Group VI

(ii) 1s2 2s2 2p6 3s2


LL

4 (i) Cl4+: [Ne]3s23p1


S4+: [Ne]3s2

The 5th electron is removed from the 3p subshell for chlorine and 3s subshell for
sulfur.
3p subshell is further from the nucleus as compared to the 3s subshell. Less
energy is required to remove the 5th electron from chlorine as compared to sulfur.

(ii) 1st IE of O is less endothermic than that of N due to inter-electron repulsion


between the paired electrons in the 2p orbital in O.

5 X+(g) → X2+(g) + e

Going down the X+ ions of Group II, both the nuclear charge and screening effect increase.
Less energy is required to remove the valence electron as it is increasingly further
from the nucleus.
Hence moving down the group, the valence electron in the X+ cation becomes further away
from the nucleus and is less strongly attracted, thus needing less energy for its removal.

15
Atomic Structure

6 (i) Mg+(g) → Mg2+(g) + e-


**must have state symbols

(ii) The successive removal of electrons is from an increasingly positive ion


where the distance between nucleus and outermost electron is decreasing
Hence the electrons are more tightly held by the nucleus as successive
electrons are removed.

(iii) For Mg, the third electron is removed from the n = 2 shell, while for Al it is the
n = 3 shell. Less energy is required for removal of 2s electron in Mg2+
compared to that in Al3+ as it is further from nucleus.
The net attractive force between the outermost electron and the nucleus for Mg2+ is
greater than that in Al3+

7 isotope percentage abundance isotopic mass x


percentage abundance
tellurium-128 31.79 4069
tellurium-130 34.48 4482

Relative atomic mass of Te

= 11 + 300 + 107 + 572 + 874 + 2357 + 4069.12 + 4482.40 = 127.7


100
23
8 (i) Element B belongs to Group V.
There is a big jump in I.E. between the 5th and 6th electron removed which
LL

implies that the 6th electron removed lies in an inner quantum shell and is more
strongly attracted by the nucleus. Hence more energy is required to remove the
6th electron.

(ii) Element A (Group IV) and B (Group V):


The nuclear charge of B is higher than that of A, while their screening effect is
approximately the same since both have the same number of occupied quantum
shells. The increase in nuclear charge outweighs the increase in screening effect
resulting in a higher effective nuclear charge for B. The valence electron to be
removed from B is more strongly attracted by the nucleus, therefore it has a higher
1st I.E. than A.

Element B (Group V)and C (Group VI):


The valence electron to be removed from C is one of the paired electrons in the
p orbital. It experiences inter-electron repulsion, therefore less energy is
required to remove it.

16
Atomic Structure

9 (i) Number of protons : ……31………


Number of neutrons: ……37……..

(ii)

The deflection remains the same at 3o

10 For neon: Ne (g) → Ne+ (g) + e 1st IE = 2080 kJ mol−1


For sodium: Na (g) → Na+ (g) + e 1st IE = 494 kJ mol−1

Ne: 1s2 2s2 2p6


Na: 1s2 2s2 2p6 3s1
Ionisation energy: Amt of energy required to remove the most loosely held electron
from one mole of gaseous atoms.

The outer electrons in Ne experiences a greater nuclear charge than Na as the


23
electrons in Na are shielded by an extra shell of electrons. Thus, the electrons in Ne
should require more energy to remove than Na. However, Na is a solid at room
temperature, hence energy must be taken in to vapourise Na prior to ionization.
Thus, the total amount of energy to vapourise Na and ionize Na exceeds that of the
LL

energy required to ionize Ne. Thus, the red glow due to ionization of Ne occurs
before the orange glow due to ionization of Na.

11 (a) There is a large difference between the first and second ionisation energies. This
indicates that the second electron to be removed comes from a inner principal
quantum shell, one which is lower in energy and closer to the nucleus. Hence there
exists one valence electron indicating that D belongs to Group I in the Periodic
Table.

(b) Since D belongs to Group I and each of its atoms has at least ten electrons, it
cannot be a Period 2 element (i.e. Li which contains only 3 electrons per atom).

(c) (i) D+(g) ⎯→ D2+(g) + e– H = 2nd ionisation energy of D


The second ionisation energy of the element D is the amount of energy required to
remove 1 mole of electrons from 1 mole of gaseous D+ ions to form 1 mole of
gaseous D2+ ions.

17
Atomic Structure

(ii)

12
(a) (i) Third ionisation energy (3rd IE) is the energy required to convert 1 mole of gaseous
Bw2+ ions to form 1 mole of gaseous Bw3+ ions.

Bw2+ (g) → Bw3+(g) + e− H = third ionisation energy

(ii) Since there are a total of 3 large increments between successive IEs, this implies
that the 3rd, 12th and 18th electron is removed from a different principal quantum
23
shell. Hence Brownie is a Period 4 element.

(b) In the Second principal quantum shell (12th to 17th IE), the small difference between the
LL

15th and 16th ionisation energy implies that the 16th electron is removed from a different
subshell (the s subshell).

In other words, the 12th to 15th electrons are removed from a p subshell.
Hence, the p subshell contains 4 electrons.
Since each orbital can accommodate a maximum number of 2 electrons, there are a total
of 2 p orbitals in each p subshell.

(c) Full Electronic configuration of Brownie is 1s22s22p43s23p43d34s2

Multiple Choice Questions (Section A)


1. 2. 3. 4. 5. 6. 7. 8. 9. 10. 11. 12. 13. 14. 15. 16. 17. 18. 19. 20.
B C D D B B A D B C C D D D D D B C D D

Multiple Choice Questions (Section B)


1. 2. 3. 4. 5.
C B B A D

Note:
Do not assume that all the given answers are correct.
Do discuss with your friends and check with your tutor should you think that any given
answer is incorrect.

18
Chemical Bonding

CHEMICAL BONDING
Structured / Free Response Questions
3
1 Explain the relative melting/boiling points in the three substances
a Melting point /oC
Ca 842
Cr 1907
K 64

b Boiling point /oC


HBr −67
HCl −85
HF 20

c Melting point /oC


CO2 −78
PbO2 290
SiO2 1600

2 a Explain the following as fully as you can:


b Graphite conducts electricity, but not diamond.
23
c Magnesium oxide is used in making bricks for lining high temperature furnaces;
sodium fluoride cannot be used for this purpose.
LL

d The bond energy of the nitrogen-nitrogen bond in N2 molecule is


944 kJ mol−1, whereas the bond energy of the carbon-oxygen bond in carbon
monoxide (which is isoelectronic with the N2 molecule), is 1074 kJ mol−1.

The relative molecular mass of ethanoic acid in benzene is 120.

3 [YJC 2011/III/2(a)]
The following table lists the boiling points of some unbranched alkanes and alcohols.

formula boiling points / C


C2H6 −88
C3H8 −42
C4H10 0
CH3OH 65
C2H5OH 78
C3H7OH 97
C4H9OH 118
C5H11OH 133

1
Chemical Bonding

(i) Explain the trend in the boiling points of the alkanes

(ii) If the intermolecular bonding in ethanol, C2H5OH, were to be similar to that of the
alkanes, predict a value for its boiling point.

(iii) Explain why the boiling points of the alcohols are much higher than those of the
alkanes.

(iv) A mixture containing two of the alcohols listed in the table above, is to be separated
using fractional distillation. The mixture is placed in the distillation apparatus at room
temperature and then gently heated. The first fraction is collected at 97.2 oC.

I Identify one alcohol from the table above that could not be present in the
mixture.

II By specifically referring to this experiment, explain why the alcohol identified in


(I) could not be present.

III Give a reason why the distillation flask should not be heated using a bunsen
burner.
[9]
23
4 [Modified YJC 2011/III/2(a), 5(a)(ii)]
Atmospheric carbon dioxide dissolves in water to form a weakly acidic solution containing
the hydrogencarbonate ions, HCO3−.
LL

(i) Draw a dot-and-cross diagram to illustrate the bonding in the hydrogencarbonate ion.
Your diagram should include the outer electrons only.

(ii) State the hybridisation of the carbon atom in the hydrogencarbonate ion.

(iii) Aqueous copper(II) sulfate contains [Cu(H2O)6]2+ ions. The diagram shows part of the
[Cu(H2O)6]2+ ion and the H-O-H bond angle in the water ligand.

Cu2+ O 107o

(iv) Explain why the H-O-H bond angle in the water ligand is 107o rather than 104.5o.
State the hybridisation state of the oxygen atom in the water ligand.
[5]

2
Chemical Bonding

5 [NYJC 2011/III/1(a)(ii), 4(b),(d)]

(a) Cyanogen, (CN)2 is called a pseudohalogen because several of its reactions are
similar to those shown by halogens Cl2, Br2 and I2.
Draw a dot-and-cross diagram of the cyanogen molecule, suggesting its shape. [2]

(b) Interhalogen compounds are formed between different Group VII elements. The
properties of interhalogen compounds tend to be intermediate between those of its
parent halogens. The following properties, however, are found in neither parent
halogens.

(i) Cl2 and I2 are soluble in non-polar solvent but ICl is insoluble. Explain why ICl is
insoluble in non-polar solvent.

(ii) Cl2 and I2 exists as discrete molecules but ICl3 dimerizes. Suggest a dot-cross
diagram for the dimer, given that it exists as a planar molecule.

(iii) Br2 and F2 are non-conductors of electricity but BrF3 is a good electrical
conductor in the molten state. With the aid of an equation, suggest a possible
reason why BrF3 can conduct electricity.
[3]

6 [MJC 2011/III/1(a)(ii), 4(b),(d)]


23
(a) The diagram below shows the structure of O3.
LL

(i) Explain why the boiling point of oxygen, O2, is lower than that of ozone.

(ii) The O-O bond length is 148 pm while the O=O bond length is 121 pm.
Suggest a reason why the bond length between the oxygen atoms in ozone is
actually 128 pm. (1 pm = 10−12 m)
[3]

(b) When ozone reacts with nitrogen dioxide, gaseous N2O5 and oxygen gas is produced.
N2O5 is a rare example of a compound that adopts two structures depending on the
conditions: as an ionic salt when in the solid state, but a molecule when in the
gaseous state.

(i) Gaseous N2O5 is a symmetrical molecule with the two nitrogen atoms bonded to
one central oxygen atom.

Draw a dot-and-cross diagram to show the bonding in N2O5 molecule. Use your
diagram to suggest the shape and the bond angle with respect to nitrogen.
Determine whether the molecule is polar or non-polar.

(ii) Draw a diagram to show the shape and bonding in N2O2.


[6]

3
Chemical Bonding

7 [MJC 2011/II/5(b)]
(a) An electride is a special type of ionic compound that can be formed from Group I
metals (such as Na and Cs) and has ‘trapped’ electrons taking on the role of anions. It
has the general formula [MLn]e , where M, L and e refer to the metal, ligand and
electron respectively. For example,

Na + 6NH3 → [Na(NH3)6]e

The structures of some electrides are studied and they are said to be remarkably
similar to that of sodium and caesium chlorides. The diagram below shows the crystal
lattice structures for both sodium chloride and caesium chloride.

23
crystal lattice of CsCl and NaCl
LL

In the crystal lattice of sodium and potassium chloride, the co-ordination number of
each ion is 6. However, in the crystal lattice of caesium chloride, CsCl, the
co-ordination number has a different value.

(i) From the diagrams, deduce the co-ordination number in CsCl lattice. Suggest
an explanation for the co-ordination number in the CsCl lattice being different
from those in NaCl and KCl.

(ii) It is proposed that [Na(NH3)6 ]e has the same crystal lattice structure as NaCl. In
the diagram below, label the particles present in the structure.

4
Chemical Bonding

(iii) Unlike Na which forms [Na(NH3)6]e, lithium forms the electride [Li(NH3)4]e
instead.
Suggest a reason why this is so.

(iv) Suggest whether an electride can conduct electricity in the solid state. Explain
your reasoning fully.
[5]

8 [SRJC 2011/III/1(e)(i), 2(a)(ii), SRJC 2011/II/4(c)(d)]


(a) When molecules of acrylamide are heated, they undergo polymerisation to form the
polymer, polyacrylamide. The structure of polyacrylamide is shown below:

Polyacrylamide is sometimes used in disposable diapers as it is able to absorb as


many times its mass, of water. However, when sodium chloride is added to the
polymer, the absorbed water is immediately released.
Explain.
23
[2]

(b) Haemoglobin has a quaternary structure that contains globular proteins and
LL

embedded haem groups. The diagram shows a simplified version of the haem group.
The various different side-groups are all shown as R.

R R

R R
N1 N2

Fe
N4 N3
R R

R R

Note that the four N atoms and the Fe ion are planar.

(i) State the difference between the type of bonds that occur between Fe ion and
the numbered N1 and N2 atoms.

(ii) State the type of hybridisation for the nitrogen atom labelled, N2 in the diagram
given.
[3]

5
Chemical Bonding

(c) A structural isomer of azomethane is NH2CH=CHNH2, identify the type of hybridisation


about the C atom.
[2]

(d) Explain, in terms of structure and bonding, why CH3CH3 is more soluble in
tetrachloromethane, CCl4, than NH2CH=CHNH2.
[2]

9 When BF3 is dissolved in HF, the solution became acidic. Suggest reasons for the
observations. Illustrate your answer with a diagram.
[3]

10 [VJC 2011/II/2(a), 3(g), RVHS 2011/II/3(b)]


(a) Oxoanions of Group VII elements have the general formula XOm-, where m = 1, 2, 3 or
4. These oxoanions are strong oxidizing agents.

Explain why fluorine does not form oxoanions.


[3]

(b) NO is readily oxidised to reddish-brown NO2. Both NO and NO2 chemically combine to
form N2O3.
Draw dot-and-cross diagrams to show the bonding in NO2 and N2O3.
[2]
23
(c) Explain in terms of structure and bonding, why N2O4 has a higher boiling point than
NO2.
[2]
LL

(d) Aluminium oxide has a melting point of 2072 °C while aluminium chloride sublimes at
180 °C. Explain this in terms of structure and bonding.
[2]

11 [TPJC 2011/II/2(c), 4(b), TJC 2011/III/3(d)]


(a) Salicylic acid and its isomer, 4-hydroxybenzoic acid, differ in their boiling points.
Suggest how they may differ in their boiling point, and account for this difference.

[2]

6
Chemical Bonding

12 [NJC 2011/II/2(c), NJC 2011/III/4(b), MJC 2009/III/3(d)]


(a) Selenium, Se, is in Group VI of the Periodic Table below sulfur.

How would you expect the bond angle in selenium dioxide, SeO2, to compare with that
of sulfur dioxide, SO2? Give your reasoning.
[2]

(b) Compound B is unstable.

(CH3)2CHCH2 H3C
O
C

CH

COOCH2CH3
B

Suggest the size of the bond angle in the three membered ring in B.
Hence explain why B is unstable.
[2]

(c) Ibuprofen is an over-the-counter painkiller for mild to moderate pain which also
23
reduces inflammation and fever.

CH3
LL

(CH3)2CHCH2 CH

COOH
Ibuprofen

In order for ibuprofen to act quickly, it must be quickly absorbed into the bloodstream
through the stomach lining. The more soluble it is in water, the faster the painkilling
action.

Explain why ibuprofen is not very soluble in water.


[2]

(d) Unlike chlorates (VII) and bromates (VII), which contain only the XO4- ion (X = Cl or
Br), iodates (VII) can contain the ions IO53- and IO65- as well as IO4- .

(i) Suggest a reason for this difference.

(ii) Draw the dot and cross diagram for IO65- and suggest its shape.
[3]

7
Chemical Bonding

Multiple Choice Questions (Section A)

1 [HCI 2008/I/16]

2 [MJC 2008/I/4]

23
LL

3 [MJC 2008/I/5]

8
Chemical Bonding

4 [HCI 2009/I/4]

5 [MJC 2011/I/4]

23
LL

6 [MJC 2011/I/5]

9
Chemical Bonding

7 [ACJC 2011/I/5]

8 [ACJC 2011/I/6]

23
9 [AJC 2011/I/5]
LL

10 [ACJC 2010/I/7]

10
Chemical Bonding

11 [AJC 2011/I/6]

23
12 [AJC 2010/I/7]
LL

13 [AJC 2010/I/8]

11
Chemical Bonding

14 [SAJC 2010/I/6]

15 [RI 2011/I/3]

23
LL

16 [RI 2011/I/4]

17 [SAJC 2011/I/6]

Sodium ethanoate has less covalent character in its ionic bonds than sodium chloride.

12
Chemical Bonding

18 [RI 2011/I/5]

19 [YJC 2011/I/4]
23
LL

13
Chemical Bonding

20 [SAJC 2011/I/4]

21 [SAJC 2011/I/5]

23
LL

22 [PJC 2009/I/9]

23 [NYJC 2011/I/5]

14
Chemical Bonding

24 [NYJC 2011/I/6]

25 [CJC 2010/I/6]

23
LL

15
Chemical Bonding

26 [HCI 2010/I/4]

27 [HCI 2010/I/5]

23
LL

16
Chemical Bonding

28 [MJC 2010/I/4]

29 [NYJC 2010/I/4]

23
30 [SAJC 2010/I/5]
LL

17
Chemical Bonding

Multiple Choice Questions (Section B)

1 [JJC 2009/I/33]

23
LL

2 [YJC H1 2009/I/27]

3 [MJC H1 2009/I/27]

18
Chemical Bonding

4 [SAJC 2009/I/31]

5 [YJC 2009/I/31]

6 [DHS H1 2009/I/27]
23
LL

7 [Nov 2008/I/31]
Carbon forms double bonds with each of the Group VI elements oxygen, sulfur and
selenium. In each case, the double bond is polar.

In the molecules carbon dioxide (CO2), carbonyl sulfide (COS) and carbonyl selenide
(COSe), the polarities of these bonds do not necessarily cancel.

Overall polarity of molecule


CO2 0
COS 0.71
COSe 0.73

Which factors could account for these observations?

1 The C=S bond is more polar than the C=Se bond.


2 The C=O bond is more polar than the C=S bond.
3 The C=Se bond is more polar than the C=O bond.

19
Chemical Bonding

8 [SAJC JC 1 BT 2007]
An ionic compound will show more covalent character if

1 the radius of the cation is large rather than small.


2 the radius of the anion is large rather than small.
3 the charge on the cation is large rather than small.

9 [HCI Prelims 2000]


Which of the following statements are not true?

1 Covalent bonds are not present in ionic compounds.


2 The bond energy of a single bond is one half that of a double bond between the
same atoms.
3 The number of covalent bonds formed by an atom equals the number of unpaired
23
electrons in the isolated, gaseous atom.

10 [NYJC H1 2009/I/30]
LL

20
Chemical Bonding

CHEMICAL BONDING SUGGESTED SOLUTIONS

Structured/Free Response Questions


1 a K has the lowest boiling point as K+ has lower charge density than Ca2+, also, there
are less number of delocalized electrons as K donates only one electron while Ca
donates two electrons. Thus, K+ attract the sea of delocalized electrons less strongly
than Ca2+. Metallic bonds in Ca is stronger than K and require more energy to break.

Cr has the highest boiling points as it has the greatest number of delocalized
electrons. This is because in addition to the 4s electrons, Cr can also utilize the 3d
electrons as the 3d and 4s subshell is close in energy. Also, Cr ions have higher
charge density as the 3d subshell is poor shielding leading to stronger attraction for
the delocalized electrons. Thus, the metallic bonds in Cr is the strongest.

b All three Group VI hydrogen compounds exists as discrete polar covalent molecules.

HF has a highest boiling point than HCl and HBr as the hydrogen bonds between HF
molecules are stronger than the permanent dipole-permanent dipole forces between
HCl and HBr molecules, hence require most energy to break.

HBr has a higher boiling point than HCl as HBr has a more electrons and a larger
electron cloud than HCl thus there is a greater ease of distortion of the electron cloud
of HBr and instantaneous dipoles more easily induced and are stronger, thus require
23
more energy to break.

NOTE: idid is NOT always the weakest intermolecular forces compared to Pdpd and
LL

H bonds.
Idid < Pd-pd< H bonds ONLY when the Mr and electron cloud sizes of the molecules
are similar.
Consider the example of NH3 and I2
NH3: H bonds + idid, NH3 is gas (Mr =17)
I2: Idid, I2 is solid (Mr = 2x127)
I2 has a very large e-cloud, idid strength can exceed the H bonds of NH3 in strength

c SiO2 has giant covalent structure


CO2 and PbO2 exist as simple molecules
Strong covalent bonds between Si and O atoms
Require more energy to break than instantaneous dipole induced dipole between
PbO2 molecules and CO2 molecules
SiO2 highest melting point

PbO2 has a more electrons and a larger electron cloud than CO2,
Greater ease of distortion of the electron cloud of PbO2
Instantaneous dipoles more easily induced and are stronger, thus require
more energy to break.
PbO2 has a higher melting point than CO2

21
Chemical Bonding

2 a Only 3 of the 4 valence electrons of C in graphite is used for bonding, compared to all
4 in diamond
Thus in graphite, there is delocalised electrons throughout the giant covalent lattice, to
act as mobile charge carriers.

b Mg2+ has a higher charge density than Na+, also, Mg2+ is smaller.
O2− has a higher charge and smaller radius than F−
q+q−
Since LE α , interionic distance between ions in MgO > NaF
r+ + r−
Ionic bond strength MgO > NaF, thus, require more energy to break, MgO has a
higher melting point to withstand the high temperature of furnaces.

c Both molecules have triple bond, thus same bond order of 3.


Bond lengths are similar
However, carbon oxygen bond in CO is polar while nitrogen-nitrogen bond in N2 is
non-polar, thus, the additional ionic character in the carbon oxygen bond in CO gives
it additional strength. (there is an additional attraction between the + C atom and the
− O atom on top of the attraction between the shared electrons and positive nuclei in
the covalent bond)

d Ethanoic acid dimerises in benzene as it is unable to form hydrogen bonds with


benzene. This is because benzene is non-polar. It forms the following structure, thus
the Mr is 120.
23
− + −
O H O
CH3C CCH3
LL

−O H +
O −

3 (i) Boiling points of alkanes increase with the increasing number of carbon atoms due to
greater number of electrons / more polarisable electron cloud.
Thus, the electron cloud is more polarisable giving rise to stronger instantaneous-
dipole induced-dipole / van der Waals forces of attractions.
Hence greater amount of energy is required to overcome the stronger instantaneous-
dipole induced-dipole / van der Waals forces of attractions.

(ii) −42 C (around the same as C3H8)


They have the same number of electrons / similar electron cloud size as C3H8.

(iii) Alcohols exhibit intermolecular hydrogen bonding, which are stronger than
instantaneous-dipole induced-dipole.
Hence greater amount of energy is required to overcome the hydrogen bonds in
alcohols.

22
Chemical Bonding

(iv) I methanol or ethanol

II If methanol (ethanol) had been present in the mixture it would have


condensed as the first fraction at 65 °C (78 °C); or
The boiling temperature of methanol (ethanol) is below 97 °C. Hence the
methanol (ethanol) could have boiled off before the first fraction is collected.

III Flammability of alcohol vapours or risk of fire or explosion

4 (i) H −
x
O
x
Cx
x
x
O O

(ii) sp2

(iii) Both oxygen atoms have tetrahedral arrangement of its four electron pairs.

The oxygen atom in isolated water molecule has 2 bond pairs and 2 lone pairs,
23
thus it has a bent shape with bond angle 104.5 .

When water is a ligand, the oxygen atom has 3 bond pairs and 1 lone pair, thus it
LL

has a trigonal pyramidal shape with bond angle 107.

This is because lone pair-lone pair repulsion is greater than bond pair-lone pair
repulsion which is greater than bond pair-bond pair repulsion.

sp3

5 (a)
x x x

x
Cx C
x x x

xN N
linear

(b) (i) ICl prefers to form stronger permanent dipole-permanent dipole interactions
between its polar molecules rather than weak Van der Waal’s forces with non-
polar solvent molecules.

(ii)
[NB: Since dimer is
planar, each iodine should
have 4bp and 2lp - square
planar]

23
Chemical Bonding

(iii) 2 BrF3 → [BrF2]+ + [BrF4]- or BrF3 → [BrF2]+ + F-


BrF3 ionizes/forms ions. The mobile ions (in molten state) are able to act as
charge carriers.

6 (a) (i) The electron cloud of O3 is greater/more polarisable than that of O2.

Boiling point of ozone is higher because more energy is required to


overcome the stronger van der Waals’ forces of attraction or id-id
interactions between ozone molecules than van der Waals’ forces of attraction
between oxygen molecules.

(ii) The pi electrons are delocalised over the 3 oxygen atoms, giving rise to
partial double bond character in the O-O bond.

(b) (i)

Shape: trigonal planar Bond angle: 120° abt each N central atom
23
It is a polar molecule.

(ii)
LL

7 (a) (i) The coordination number is 8.


Cs+ is a larger cation than Na+ or K+. More Cl- anions can surround the
larger Cs+ cation, thus the coordination number in CsCl (ie. 8) is different than
in NaCl and KCl.

(ii)

24
Chemical Bonding

(iii) Unlike Na, lithium is a period 2 element and has no energetically


accessible vacant 3d orbitals to accommodate more than eight electrons,
hence can only accommodate a maximum of 8 electrons.

(iv) There are no free mobile ions and the ‘trapped’ electrons are localised/ not
delocalised’ in the structure of electrides. Hence they are unable to conduct
electricity.

8 (a) The amide group in each subunit of polyacrylamide can form favourable
intermolecular hydrogen bonds with water molecules.

Upon addition of ionic sodium chloride, the amide groups can form ion-dipole
interactions with the ions which are stronger and consequently more favourable
than the intermolecular hydrogen bonds with water.
Or
Hence, the intermolecular hydrogen bonds are overcome or broken so that ion-
dipole interactions can be formed instead.

(b) (i) N1 – covalent bond


N2 – dative bond / coordinate bond

sp2
23
(ii)

(c) sp2
LL

(d) Both CH3CH3 and NH2CH=CHNH2 have simple molecular structure, with
intermolecular van der Waals’ forces of attraction and intermolecular hydrogen
bonds respectively.

CH3CH3 can form favourable van der Waals’ forces of attraction/id-id


interactions between solute and solvent molecules.
OR
Weaker van der Waal’s forces between CCl4 molecules are not strong enough to
displace the stronger hydrogen bonding between NH2CH=CHNH2 molecules.
9 Boron in BF3 has an empty energetically accessible orbital to form dative
bonding with the lone pair of electrons in F-.

HF ⇌ H+ + F-
BF3+ F- → BF4-

Since F- is constantly used to form BF4-with BF3, HF will dissociate to produce more
H+ and F-, hence increasing the acidity of the solution.

25
Chemical Bonding

10 (a) Fluorine is the most electronegative element, and is unable to form a compound
in which it has a positive oxidation number. In addition, fluorine is in Period 2 of
the Periodic Table and does not have energetically accessible d orbitals to
expand its octet structure to form more than 1 covalent bond.

(b)

(c) Both NO2 and N2O4 have simple molecular structure consisting of simple discrete
molecules held together by van der Waals’ / id-id interactions However, N2O4 has a
larger and more polarisable electron cloud, hence it has more electrons leading
to stronger van der Waals’ forces. So, a larger amount of heat energy is needed to
overcome the stronger van der Waals’ / id-id interactions, leading to a higher
boiling point.

(d) Aluminium oxide has a giant ionic structure while aluminium chloride has a simple
molecular structure. A larger amount of energy is needed to overcome the strong
electrostatic force of attraction between Al3+ ions and O2− ions than the weaker
23
van der Waals’ forces between aluminium chloride molecules.

11 Molecules of salicylic acid are able to form BOTH intramolecular and


LL

intermolecular hydrogen bonds, while molecules of 4-hydroxybenzoic acid are


able to form ONLY intermolecular hydrogen bonds.

The formation of intramolecular hydrogen bonds in salicylic acid reduces the


extent of formation of intermolecular hydrogen bonds.

During boiling, energy is required to break only the intermolecular hydrogen


bonds in order to separate the molecules.

Hence, the more extensive intermolecular hydrogen bonds in 4-hydroxybenzoic


acid will require more energy to overcome than the intermolecular hydrogen
bonds in salicylic acid, and 4-hydroxybenzoic acid has a higher melting point.

12 (a) Both SeO2 and SO2 have 2 bond pairs and 1 lone pair.
Se is less electronegative than S and hence the bonding pairs of electrons in
Se=O bonds are further away from Se central atom. The bonding pairs around
Se experiences less repulsion and can come closer together, hence the bond
angle is smaller.

26
Chemical Bonding

(b) Bond angle size is 60°


This angle size causes a ring strain/ angle strain the three membered ring making it
more easily broken, therefore more unstable.
OR
This small bond size results in more repulsion between the bond pairs around O.
This bond angle is less than the ideal bond angle of 104.5 ˚ for the –C-O-C- bond
angle or 109.5o for the sp3 hybridised carbon.

(c) Energy evolved from the unfavourable interactions formed between the bulky
non-polar hydrocarbon group (benzene or alkyl group) and water molecules is
insufficient to overcome hydrogen bonding between water molecules. The non-
polar hydrocarbon group also interferes with the extensiveness of hydrogen
bonding between the COOH group and water molecules.

(d) (i) Iodine has a large atomic size that is sufficient to accommodate greater
number of O atoms surrounding it in IO53- and IO65- .

(ii) 5-

O
x x
x x
x x x x
x x xx x x

x O
x
x x O x
x

x I
23
x x
x x

x O
x x
x
x
x
x O
x x
x
x
x O x
x
LL

xx

Shape: octahedral

Multiple Choice Questions (Section A)


1. 2. 3. 4. 5. 6. 7. 8. 9. 10. 11. 12. 13. 14. 15. 16. 17. 18. 19. 20.
C C B A D A B D B C D A A C C B B B A C
21. 22. 23. 24. 25. 26. 27. 28. 29. 30.
D D D B A A D D C D

Multiple Choice Questions (Section B)


1. 2. 3. 4. 5. 6. 7. 8. 9. 10.
A C D B B C B C A A

Note:
Do not assume that all the given answers are correct.
Do discuss with your friends and check with your tutor should you think that any given
answer is incorrect.

27
Gaseous State

THE GASEOUS STATE


Structured / Free Response Questions
4
1 [RVHS 2011/III/1]
(a) Rank the four gases of NH3, O2, N2 and H2O in order of increasing deviation from ideal
behaviour. Explain your answer.
[3]

(b) (i) It was found that one molecule of haemoglobin combines with four molecules of
oxygen. If 2.00 g of haemoglobin combines with 3.06 cm3 of oxygen at 37.0 °C
and 98.7 kPa, what is the molar mass of haemoglobin?
State any assumptions you made in your calculations.

(ii) The actual molar mass of haemoglobin is 6.90 x 104 g mol-1. Based on your
working in (ii), suggest, with reasons, why there is a discrepancy in the two
values.
[4]

2 [MJC 2010/II/3(a)]
The plots of PV/RT against P for one mole of an ideal gas and one mole of HCl at 100 K are
given below.
23
LL

(i) Show clearly, on the same axes, how one mole of HF will behave at the same
temperature of 100 K.

(ii) Explain the difference in behaviour between HF and HCl at 100 K.

(iii) Explain what happens when the HCl gas is heated to 400 K.
Illustrate your answer clearly on the same axes in a(i).
[4]

1
Gaseous State

3 a At a given temperature, the fraction of neon molecules with a higher molecular speed
is lesser than that of hydrogen molecules. However, the average kinetic energies of
the sample of the two gases are the same. Explain.

b A 2 dm3 cylinder containing hydrogen at a pressure of 300 kPa is connected to a 5


dm3 cylinder containing neon at a pressure of 400 kPa and sufficient time was allowed
for complete mixing. Calculate the partial pressures of the two gases and the total
pressure in the joined cylinders.

c 2 volumes of hydrogen were mixed with 5 volumes of dry air containing 80% nitrogen
and 20% oxygen at 1000C and at atmospheric pressure. The mixture was sparked so
that the combustion of hydrogen goes to completion. What is the mole fraction of the
water vapour in the resulting mixture?

4 RI CT1 2013/1a modified


a The decomposition of M(NO3)2 may be represented as

2 M(NO3)2(s) ⎯→ 2 MO(s) + 4 NO2(g) + O2(g)

Calculate the mass of Mg(NO3)2 to be used to produce 100 cm3 of gas at 27 oC and
atmospheric pressure = 1.01 x 105 Pa.

b Calculate the partial pressure of each gas collected in the vessel


23
c Which of the two gases would deviate more from ideal gas behavior? Explain your
answer.
LL

5 In an experiment, a student is attempting to collect 60 cm3 of gas over water in the


following set-up.

60 cm3
gas

Mg

Gas collection set-up Adjustment to atmospheric pressure

Using the following considerations,


• Concentration of HCl (aq) = 2.0 mol dm−3
• Temperature = 30 oC
• Vapour pressure of water at 30 oC = 4.23 x 103 Pa
• Atmospheric pressure = 1.01 x 105 Pa

2
Gaseous State

Calculate the mass of magnesium ribbon that the student requires to collect 60 cm3 of
gas at atmospheric pressure.

6 [AJC 2011/II/2]
(a) (i) State two assumptions of the kinetic theory of gases.

(ii) Carbon dioxide is a non–ideal gas. The graph below shows experimental values
of compressibility factor (Z) plot for one mole of carbon dioxide gas. (Z = pV/RT)

Z = pV/RT

p
23
On the plot given, sketch the graph to illustrate the behavior of an ideal gas.
LL

(iii) State the type of hybridisation present for the carbon atom in carbon dioxide
molecule.
[4]

(b) In the industrial liquefaction of air, high–pressure air is pre–cooled in a coil surrounded
by cold water. It is then allowed to expand into a region of low pressure, whereupon it
cools down by a large amount. An ideal gas does not show this behaviour.

(i) Real gases like carbon dioxide can be liquefied at room temperature just by
pressuring them. Explain why the application of pressure causes the gas to
liquefy.

(ii) Suggest why the expansion of air into a region of low pressure is an
endothermic process.
[3]

3
Gaseous State

7 [NYJC 2011/III/4(c)]
Two unknown gases X and Y could be hydrogen fluoride or hydrogen chloride. In an
experiment conducted at 1 atmospheric pressure, the volumes of separate samples
containing equal amounts of gas X and Y were measured at different temperatures and the
results are tabulated below.

Gas X Gas Y
Experiment No. T/K
V / dm3 V / dm3
1 200 20 000 16 500
2 300 22 500 20 000
3 600 24 000 23 500

(i) Sketch and explain the expected variations of V/T with T for a given amount of an
ideal gas at constant pressure.

(ii) Identify gas X and Y. Explain your reasoning.


[4]

Multiple Choice Questions (Section A)


23
1 [ACJC 2011/I/4]
LL

4
Gaseous State

2 [HCI 2010/I/6]

3 [HCI 2011/I/1] 23
LL

4 [ACJC 2010/I/5]

5
Gaseous State

5 [NJC 2010/I/5]
23
LL

6 [NYJC 2010/I/5]

6
Gaseous State

7 [PJC 2010/I/6]

8 [RI 2010/I/4]

23
LL

7
Gaseous State

9 [RI 2010/I/5]

23
LL

10 [RI 2011/I/6]

8
Gaseous State

11 [MJC 2010/I/2]

23
LL

12 [NJC 2011/I/11]

9
Gaseous State

13 [CJC 2010/I/7]

23
14 [RVHS 2011/I/15]
LL

10
Gaseous State

15 [HCI 2011/I/5]

23
LL

16 [AJC 2011/I/7]

17 [NYJC 2011/I/3]

11
Gaseous State

18 [TPJC 2011/I/11]

19 [NYJC 2011/I/4]

23
20 [AJC 2010/I/6]
LL

12
Gaseous State

Multiple Choice Questions (Section B)

1 [TPJC 2011/I/31]

23
2 [AJC 2009/I/32]
LL

3 [ACJC 2010/I/31]

13
Gaseous State

4 [NYJC 2010/I/15]

23
LL

14
Gaseous State

5 [PJC 2009/I/33]

23
LL

15
Gaseous State

GASEOUS STATE SUGGESTED SOLUTIONS

Structured/Free Response Questions

1 (a)
Increasing deviation: N2 < O2 < NH3 < H2O
Deviation from ideal behaviour depends on the strength of intermolecular forces and
number of electrons in the molecule. Weak instantaneous dipole-induced
dipole/van der Waals’ forces exist between nitrogen molecules and between
oxygen molecules. Strong hydrogen bonds exist between ammonia molecules
and water molecules.
Water shows more extensive hydrogen bonding than ammonia while oxygen
has a larger electron cloud, and therefore stronger instantaneous dipole-
induced dipole/van der Waals’ forces than nitrogen.

(b) (i) Assuming oxygen is an ideal gas,


pV = nRT
(98.7 x 103)(3.06 x 10-6) = n(8.314)(37+273)
n = 1.172 x 10-4 mol

Amount of haemoglobin = n/4 = 2.930 x 10-5 mol


23
Molar mass of haemoglobin = 2.00/2.930 x 10-5 = 6.83 x 104 g mol-1

(ii) Oxygen is not an ideal gas.


Intermolecular forces of id-id attraction are present between oxygen
LL

molecules.

2 (i) (iii)

(ii) HF is less ideal than HCl as HF has stronger intermolecular hydrogen bonding
whereas HCl has weaker intermolecular Van der Waals forces of attraction.

(iii) At high temperature, there are negligible forces of attraction between the gas
particles.
Hence, HCl gas will deviate less from ideal gas behaviour.

16
Gaseous State

3 a Molar mass of Ne (28.0 g mol 1) is greater than H2 (2.0 g mol 1), thus, the fraction of
neon molecules with a higher molecular speed is lesser than that of hydrogen
molecules.
1
Average kinetic energy = mv2
2
Where v is the speed of the molecules.
The molar mass of Ne (28.0 g mol 1) is greater than H2 (2.0 g mol 1), but the speed of
Ne molecules is less than H2 molecules. The average kinetic energies of the sample of
the two gases are the same.

b 300 x103 x 2 x10−3


No. of moles of H2 =
8.31xT

400 x103 x5 x10−3


No. of moles of Ne=
8.31xT

300 x103 x 2 x10−3 8.31xT


Partial pressure of H2 = x
8.31xT 7 x10−3
= 85.71 kPa

300 x103 x 2 x10−3 8.31xT


23
Partial pressure of Ne = x
8.31xT 7 x10−3
= 285.7 kPa
LL

Total pressure = 85.71 + 285.7 kPa


= 371 kPa

c 2H2 (g) + O2 (g) → 2H2O (g)

Since air contains 20% O2,


1
Volume of O2 in 5 volumes of dry air = x5
5
=1

Stoichiometric ratio of O2 to H2 is 1:2,

Volume of H2 required = 1 x 2
=2

Thus, the reactants are in exact stoichiometric ratio.

Stoichiometric ratio of H2 to H2O is 1:1

Volume of H2O = 2

Initially, there were 5 volumes of air containing 1 volume of O2 and 4 volumes of N2.
After reaction of the 1 volume of O2 with the 2 volumes of H2, 4 volumes of N2 are left.

17
Gaseous State

Volume of gas at the end of the experiment = Vol. of N2 + Vol. of H2O


=4+2
=6

Volume of H 2 O
Mole fraction of water vapour =
Vol . of N2 +Vol . of H 2 O
2
=
6
1
=
3

4 a Gas collection using gas syringe (NO2 and O2 will be collected)

1.01 105  100  10−6


Total amount of NO2 and O2 = = 4.05  10−3 mol
8.31 300
4.05  10−3
Amount of Mg(NO3 )2 =  2 = 1.62  10−3 mol
5
Mass of Mg(NO3)2 = 148.3  1.62 10−3 = 0.240 g
23
b Total pressure of gases = 1.01 x 105 Pa

4
Mole fraction of NO2 =
5
LL

4
Partial pressure of NO2 = x 1.01 x 105 = 8.08 x 104 Pa
5

1
Mole fraction of NO2 =
5
1
Partial pressure of NO2 = x 1.01 x 105 = 2.02 x 104 Pa
5

c NO2 deviates more from ideal gas behavior. NO2 is a polar molecule with stronger
permanent dipole-permanent dipole forces between molecules as compared to the
non polar O2 molecules which has instantaneous dipole-induced dipole forces
between molecules.

5 Partial pressure of hydrogen collected = 1.01 x 105 − 4.23 x 103


= 9.677 x 104 Pa

9.667 104  60 10−6


No. of moles of hydrogen collected = = 2.304 x 10−3 mol
8.31 303

Mg + 2 HCl → MgCl2 + H2

18
Gaseous State

No. of moles of HCl = 30 x 10−3 x 2 = 0.0600 mol


HCl is in excess.

Mass of Mg = 2.304 x 10−3 x 24.3 = 0.0560 g

6 (a) (i) negligible intermolecular forces of attraction.


volume of the gas molecules is negligible compared to the volume it occupies
or volume of container.

(ii)

23
(iii) sp
LL

(b) (i) When gas is compressed, the molecules are very close together, and the
intermolecular forces of attraction become significant.

(ii) expansion into low pressure requires the molecules to overcome


intermolecular forces of attraction in order to move apart.
hence, heat is absorbed by the molecules from the surroundings, making the
gas cool.

7 (i)

19
Gaseous State

(ii) Gas X is HF while Gas Y is HCl


Work out V/T values.
Experiment Gas X Gas Y
No. T/K V / dm3 V /T V / dm3 V /T
dm3K-1 dm3K-1
1 200 20 000 100 16 500 82.5
2 300 22 500 75 20 000 66.7
3 600 24 000 40 23 500 39.1

The V/T values of Gas X deviates more from a constant value than Gas Y.

Hence Gas X behaves less like an ideal gas than Gas Y.

Gas X is HF which has stronger hydrogen bonds between molecules while HCl has
weaker permanent dipole-permanent dipole interactions.

Multiple Choice Questions (Section A)


1. 2. 3. 4. 5. 6. 7. 8. 9. 10. 11. 12. 13. 14. 15. 16. 17. 18. 19. 20.
D B A D A D B A D D C C C D C D C C D A

Multiple Choice Questions (Section B)


1. 2. 3. 4. 5.
23
B B A B A

Note:
LL

Do not assume that all the given answers are correct.


Do discuss with your friends and check with your tutor should you think that any given
answer is incorrect.

20
Chemical Energetics

CHEMICAL ENERGETICS 5a
Structured / Free Response Questions

1 When planning an energetics experiment, the temperature change to be measured is


usually between 5 – 10 oC to minimise experimental errors.

Zinc reacts with aqueous copper(II) sulfate as shown by the following equation
Zn(s) + CuSO4 (aq) → ZnSO4 (aq) + Cu (s) H  − 200 kJmol−1

Determine the mass of zinc powder required to react with 50 cm 3 of aqueous


copper(II) sulfate (in excess) to ensure that the temperature rise in the reaction is 8
oC.

2 A student is attempting to determine the enthalpy change of combustion of propanone,


CH3COCH3 using the following experimental setup.
23
LL

propanone

(a) The student first calibrated the copper can of water using ethanol. Given the
following information,

Mass of ethanol used = 0.390 g


Temperature rise = 9.6 oC
Enthalpy of combustion of ethanol = –1367 kJmol–1

calculate the heat capacity, C of the copper can containing water.

(b) Using the same copper can of water, the student proceeded to determine the
exact enthalpy of combustion of propanone.
Given the following information and using your answer in (a), determine the

1
Chemical Energetics

volume of propanone burnt to bring about a temperature rise of 10oC for the
copper can containing water.

• Approximate enthalpy of combustion of propanone = –1740 kJmol–1


• Density of propanone = 0.791 g cm−3

(c) Suggest a possible source of error in the experiment.

3 [HCI 2009/III/1(a)]
(a) Liquid phosphorus trichloride is prepared by the reaction of chlorine with white
phosphorus. Some related thermochemical data are shown below.

Enthalpy change of formation of PCl3 (l) –320 kJmol–1

Enthalpy change of vaporization of PCl3 (l) +32 kJmol–1

Enthalpy change of atomisation of phosphorus +314 kJmol–1

Using the data given, and relevant data from the Data Booklet, construct an energy
level diagram to determine the P−Cl bond energy in PCl3.

[3]
23
[VJC 2009/III/3(a)]
(b) Chlorine forms an oxide, Cl2O7, with the structure O3ClOClO3 and a boiling point of
LL

82oC. Its standard enthalpy change of formation is -546 kJ mol-1.

(i) Write the equation which corresponds to the standard enthalpy change of
formation of Cl2O7.

(ii) Given that the bond energy of Cl–O bond is 269 kJ mol-1 and using relevant data
from the Data Booklet, estimate the average bond energy of the Cl=O bond.

(iii) The bond energy calculated in (ii) could have been more accurate if the value of
an additional enthalpy change is known. What process does this enthalpy
change correspond to?
[4]

4 [VJC 2011/II/4(c)]
(a) Tonnes of organic waste are dumped into rivers annually in the rural areas of China.
Environmental scientists have warned of the danger of dead seas forming in a few
decades due to eutrophication (depletion of oxygen in water bodies). To encourage the
reduction of disposal of organic waste into the rivers, the Chinese government built
biogas tanks in the rural areas where the inhabitants can obtain electricity from the
combustion of methane gas generated from the organic waste. One such biogas
tank is shown below:

2
Chemical Energetics

(i) With reference to methane, define standard enthalpy change of combustion.

(ii) It was found that 10 kg of organic waste on the average generates 0.35 kg of
methane gas in a biogas tank Using relevant data from the Data Booklet,
estimate the average amount of energy released from 505 kg of organic waste
deposited into the biogas tank.
[4]

[VJC 2007/III/3(b)]
(b) (i) When burnt in a limited supply of air, ethane, CH3CH3, forms carbon monoxide
and water. The enthalpy change of the reaction is –1004 kJ per mol of ethane.
23
Construct a balanced equation that represents this enthalpy change.

(ii) Ethane can be synthesized through the hydrogenation of ethene as shown in the
following equation:
LL

CH2=CH2(g) + H2(g) → CH3CH3(g)

Calculate the enthalpy change of the hydrogenation reaction, using the


following data in addition to the one given in (b)(i):
Enthalpy change of combustion of CH2=CH2(g) = –1411 kJ mol-1
Enthalpy change of formation of H2O(l) = –286 kJ mol-1
Enthalpy change of combustion of CO(g) = –283 kJ mol-1
[4]

3
Chemical Energetics

5 [TJC 2011/III/2(a)]
(a) Sodium hydroxide is often used for acid-base titrations.
A student prepared a standard solution of sodium hydroxide by dissolving 10.0 g of
sodium hydroxide in 250 cm3 of water. In the process, he found the solution warm to the
touch.
Given the information below, calculate the heat evolved when solid sodium hydroxide is
dissolved in water.

Compound ΔH f / kJ mol−1
NaOH(s) −425
NaOH(aq) −470

Hence, find the increase in temperature of the standard solution.


[3]

(b) [RVHS 2011/II/4(e)]


Draw an appropriate energy level diagram to determine the lattice energy of
chromium(III) nitrate, Cr(NO3)3.
The following enthalpy changes are given:
23
Enthalpy change of hydration of chromium ion = −4563 kJ mol−1

Enthalpy change of hydration of nitrate ion = −314 kJ mol−1


LL

Enthalpy change of solution of chromium(III) nitrate = −35 kJ mol−1


[3]

6 [YJC 2011/II/3(b)]
(a) Magnesium nitride, Mg3N2, is produced when magnesium metal is burnt in pure nitrogen
atmosphere.
The lattice energy of magnesium nitride can be calculated from a Born-Haber cycle
using the following data:
H / kJ mol−1
standard enthalpy change of atomisation of magnesium +148
sum of the first three electron affinities of nitrogen +146
standard enthalpy change of formation of magnesium nitride −461

(i) Define what is meant by the term lattice energy of an ionic compound.

(ii) Using relevant data from the Data Booklet and the information given above,
construct the energy level diagram for magnesium nitride in the axis below, and
use the cycle to calculate the lattice energy of magnesium nitride.

4
Chemical Energetics

(iii) The standard enthalpy change of formation of magnesium oxide is


−601 kJ mol−1. With reference to the energy changes in the Born-Haber cycle,
suggest why the formation of magnesium oxide is more exothermic than
magnesium nitride.

(iv) [MJC 2011/III/3(a)(iii)]


Explain why the first electron affinity of oxygen is exothermic while the second
electron affinity of oxygen is endothermic.
[9]

[RI 2011/III/4(a)]
(b) Aluminium is the most common metallic element in the earth’s crust and occurs in rocks
such as feldspars and micas. Its only oxide is aluminium oxide, Al2O3, commonly known
as alumina.

Use the following data, together with appropriate data from the Data Booklet, to
construct a Born-Haber cycle to determine the lattice energy of aluminum oxide.

enthalpy change of formation of aluminium oxide = −1676 kJ mol−1


enthalpy change of atomisation of aluminium = +322 kJ mol−1
first electron affinity of oxygen = −142 kJ mol−1
second electron affinity of oxygen = +744 kJ mol−1
23
[4]

7 [MJC 2011/III/1(a)]
Lattice energies are often used as a measure of the energetic stability of ionic
LL

compounds. Usually ionic compounds with the same crystal structure can be
compared based on the lattice energy value.

Lattice energies need to be determined experimentally using thermochemical data.


However based on electrostatic considerations (i.e. attraction and repulsion of ions),
theoretical values of the lattice energies can be calculated using equations such as
the Kapustinskii equation:

  z+  z−
H latt = −107.1 kJ mol-1
r+ + r−

where  is the number of ions in the empirical formula


z + , z − is the charge of the cation and anion respectively
r+ , r− is the ionic radius (in nm) of the cation and anion respectively

5
Chemical Energetics

The table below shows the numerical values of lattice energies for a series of magnesium
and other related compounds.

Compound LiCl MgCl2 MgBr2 MgI2 MgO


Experimental - 853 - 2526 - 2440 - 2327 - 3933
value / kJ mol-1

(i) Define, with the aid of an equation, the lattice energy of magnesium
oxide.

(ii) By quoting appropriate data from the Data Booklet, explain why the lattice energies of
the magnesium halides decrease from MgCl2 to MgI2.

(iii) By using appropriate data from the Data Booklet, calculate the theoretical lattice energy
of magnesium chloride.

(iv) Explain the differences between the theoretical and experimental values for lithium
chloride and magnesium chloride respectively, and comment whether the assumption
made in (a)(iv) is a valid one.

Compound LiCl MgCl2


Theoretical - 871 - 2612
value / kJ mol-1
23
Experimental - 853 - 2526
value / kJ mol-1
[10]
LL

8 [SAJC 2008/III/4(a)]
(a) Nitroglycerin, C3H5(NO3)3, is a flammable oil commonly used to manufacture dynamite.
The atomisation of nitroglycerin is represented by the equation:

C3H5(NO3)3(l) → 3C(g) + 5H(g) + 3N(g) + 9O(g)


Given:
Standard enthalpy change of formation of nitroglycerin(l) / kJ mol-1 -364
Standard enthalpy change of atomisation of carbon (graphite) / kJ mol-1 +715
Standard enthalpy change of formation of H2O(g) / kJ mol-1 -242
Standard enthalpy change of formation of CO2(g) / kJ mol-1 -394

Using the data above and relevant values from the Data Booklet, draw a labelled
energy cycle and use it to calculate the standard enthalpy change of atomisation of
nitroglycerin.
[4]

6
Chemical Energetics

[NJC 2011/III/3(b)]
(b) The values for ΔHθ for the reactions of NaOH with HCl and ibuprofen, RCOOH, are
given below:

HCl(aq) + NaOH(aq) → NaCl(aq) + H2O(l) ΔH1θ = –57.3 kJmol–1

RCOOH (aq) + NaOH(aq) → RCOONa(aq) + H2O(l) ΔH2θ = –50.0 kJmol–1

Comment on the values of ΔH1θ and ΔH2θ given and deduce the value of the enthalpy
change of acid dissociation of ibuprofen by drawing a suitable energy cycle.
[4]

9 [Modified DHS 2011/III/2(d)]


When 30 cm3 of 1.0 mol dm–3 CH3NH2, is added to 25 cm3 of 1.0 mol dm–3 sulfuric acid in a
plastic container, the temperature rose by 6.3 C. The process efficiency is expected to be
90%.

(i) Calculate the standard enthalpy change of the reaction assuming that it takes 4.18 J to
increase the temperature of 1 cm3 of the solution by 1.0 C.

(ii) Predict if the enthalpy change of the reaction will be higher, the same or lower than that
calculated in (i) if sulfuric acid is replaced by a weak acid. Explain.
23
[4]

10 [VJC ‘08/III/1(b),(d)]
(a) Calcium reacts with water to form aqueous calcium hydroxide.
LL

H
Ca(s) + 2H2O(l) → Ca2+(aq) + 2OH–(aq) + H2(g)

Some enthalpy changes are listed below.

Enthalpy change of atomisation of calcium = 178 kJ mol–1


Enthalpy change of neutralisation = –58 kJ mol–1
Enthalpy change of hydration of Ca2+(g) = –1577 kJ mol–1
Enthalpy change for 2H+(aq) → H2(g) = –796 kJ mol–1

By using data in this question and relevant data in the Data Booklet, draw an energy
level diagram to determine H for the above reaction.
[5]

(b) The lattice energy of a metal is small compared with that of an ionic compound. The
lattice energy of calcium is –178 kJ mol–1 and that of calcium hydroxide is –2506 kJ
mol–1.

Describe the structure and bonding of Ca and Ca(OH)2. Hence, comment on the
difference between their lattice energies.
[4]

7
Chemical Energetics

11 [Modified VJC 2011/III/2(d)]


Tetraethyl-lead (IV), Pb(C2H5)4, is an example of an organometallic compound. It can be used
as an antiknock agent in petrol. At the high temperature in a car engine, the Pb-C bond in
Pb(C2H5)4, breaks to yield lead atoms and ethyl radicals. Radicals are atoms, molecules, or
ions with unpaired valence electrons, eg. Br• where the dot represent the unpaired electron.

(a) Using only the data provided below, draw an energy level diagram to calculate ΔH1, the
enthalpy change of the following reaction:

2C(g) + 5H(g) → CH3CH2•(g)

Enthalpy change of atomisation of C(s), ΔH2 + 717 kJ mol-1


Enthalpy change of atomisation of H2(g), ΔH3 + 218 kJ mol-1
Enthalpy change of formation of CH3CH3(g), ΔH4 − 85 kJ mol-1
Bond dissociation energy of the first C−H bond in CH3CH3(g), ΔH5 + 435 kJ mol-1

[3]

(b) Hence, with the aid of the data in (ii) and additional data given below, draw an energy
cycle to calculate the Pb-C bond energy in Pb(C2H5)4.

Enthalpy change of atomization of Pb(s), ΔH6 +195 kJ mol-1


23
Enthalpy change of formation of Pb(C2H5)4(g), ΔH7 +264 kJ mol-1
[3]
LL

8
Chemical Energetics

Multiple Choice Questions (Section A)


1 [HCI 2007/I/8]

2 [SAJC 2010/I/7]
23
LL

3 [ACJC 2010/I/8]

9
Chemical Energetics

4 [MJC 2010/I/12]

5 [SAJC 2010/I/8]

23
LL

6 [NJC 2010/I/9]

10
Chemical Energetics

7 [NYJC 2010/I/6]

8 [AJC 2010/I/10]
23
LL

9 [CJC 2010/I/8]

11
Chemical Energetics

10 [IJC 2011/I/10]

23
LL

11 [NJC 2010/I/16]

12
Chemical Energetics

12 [ACJC 2010/I/9]

13 [NYJC 2010/I/7] 23
LL

13
Chemical Energetics

14 [RI 2011/I/8]

23
LL

14
Chemical Energetics

15 [JJC 2011/I/8]

16 [JJC 2011/I/11]
23
LL

15
Chemical Energetics

17 [MI H1 2009/I/12]

18 [HCI H1 2009/I/10]

19 [Nov 2011/I/6]
23
LL

20 [PJC 2009/I/5]

16
Chemical Energetics

Multiple Choice Questions (Section B)

1 [AJC 2011/I/33]

2 [CJC 2011/I/32]
23
LL

3 [DHS 2011/I/34]

17
Chemical Energetics

4 [DHS 2011/I/37]

23
5 [NJC 2011/I/31]
LL

6 [RVHS 2011/I/32]

18
Chemical Energetics

7 [SAJC 2011/I/33]

23
8 [RI 2011/I/32]
LL

9 [IJC 2009/I/37]

19
Chemical Energetics

10 [JJC 2009/I/33]

23
LL

CHEMICAL ENERGETICS SUGGESTED SOLUTIONS

Structured/Free Response Questions

1
Heat released = 50 x 4.18 x 8
= 1672 J mol−1

1672
Amount of Zn powder used = = 8.36 x 10−3 mol
200 103

Mass of Zn = 8.36 x 10−3 x 65.4 = 0.547 g

20
Chemical Energetics

2 (a) 0.390
Amount of ethanol used = = 0.008478 mol
46.0

Amt of heat produced by ethanol = heat absorbed by copper can of water


0.008478 × 1367 = C∆T
where C = Heat capacity of the
copper can of water
0.008478 × 1367 = C(9.6)
C = 1.21 kJ K–1

(b) Amount of heat produced by propanone = heat absorbed by copper can of water
= C∆T
= 1.21 x 10
= 12.1 kJ

12.1
Amount of propanone = = 6.954 x 10−3 mol
1740
Mass of propanone burnt = 6.954 x 10−3 x 58.0 = 0.4033 g

0.4033
Volume of propanone burnt = = 0.51 cm3
0.791
23
(c) There may be incomplete combustion of propanone, leading to a less exothermic
enthalpy of combustion obtained in the experiment.

3 (a) (i)
LL

** State symbols must be included. All intermediates, reactants or products in the energy level
diagram must be balanced in terms of atoms of each element as well as charges.

21
Chemical Energetics

-320 = 314 + 3/2(244) – 3BE(P−Cl) -32


 BE(P−Cl) = + 323 kJ mol–1

(b) (i) 7
Cl2(g) + O2(g) → Cl2O7(l)
2
** State symbols must be included.

(ii) Bond energy of Cl−Cl = 244 kJ mol-1, O=O = 496 kJ mol-1


7
-546 = 244 + (496) – [2269 + 6(Cl =O)]
2
Bond energy of Cl =O = +313 kJ mol−1

(iii) The enthalpy change required to change liquid Cl2O7 into its gaseous form OR
enthalpy change of vaporization of Cl2O7

4 (a) (i) The enthalpy change when 1 mole of methane is completely burnt in excess
oxygen under standard conditions of 298K and 1 atm.

CH4(g) + 2O2(g) → CO2(g) + 2H2O(l)


23
(ii)

Bond Energy / kJ mol-1


C-H 410
LL

O=O 496
C=O 740
O-H 460

Hc (methane) = (4 x 410) + (2 x 496) – (2 x 740) – (4 x 460)


= –688 kJ mol-1

No. of moles of CH4 generated from 505 kg of organic waste


50.5
 0.35 1000
= 10
16.0
= 1105 mol

Average amount of energy released = 1105 x 688 = 7.60 x 105 kJ

(b) (i) 5
C2H6(g) + O2(g) → 2CO(g) + 3H2O(l)
2
**State symbols must be included.

22
Chemical Energetics

(ii) Using the reaction in (b)(i),

By Hess’ Law,
–1411 + (−286) = ∆Hr + (–1004) + 2(–283)
∆Hr = –127 kJ mol-1

**Sign and units of H must be included.

5 (a)
23
NaOH(s) + aq → NaOH(aq)
ΔH sol = ΔH f of NaOH (aq) – ΔH f of NaOH (s) = –470 – (–425) = – 45.0 kJ mol−1
10.0
LL

Number of moles of NaOH = = 0.250 mol


40
Heat evolved = 45 x 0.25 = 11.3 kJ
Q 11.3 1000
Temperature increase = = = 10.8 C
mc (250)(4.18)

(b)

23
Chemical Energetics

** State symbols must be included. All intermediates, reactants or products in the energy level
diagram must be balanced in terms of atoms of each element as well as charges.

LE = ΔHhyd − ΔHsoln
= (− 4563 − 942) − (−35)
= − 5470 kJ mol−1 **Sign and units of H must be included.

6 (a) (i) The lattice energy of an ionic compound is the energy evolved when 1 mol of a pure
ionic solid is formed from its constituent gaseous ions.

(ii)

23
0
LL

** State symbols must be included. All intermediates, reactants or products in the energy level
diagram must be balanced in terms of atoms of each element as well as charges.

−1
 LE = −461 − {2(146) + 3(2186) + 994 + 3(148)} = −8749kJmol
**Sign and units of H must be included.

(iii) Although lattice energy of Mg3N2 more exothermic than MgO,


The bond dissociation energy of the NN bond is more endothermic than the
O=O bond
OR
The sum of the electron affinities of nitrogen is more endothermic than that of
oxygen.

(iv) 1st EA is exothermic because the attraction between the nucleus of O atom and
the added electron is greater than the repulsion between the electrons.

2nd EA is endothermic because energy is required to overcome the repulsion


when electron are added to negatively charged O- ion.

24
Chemical Energetics

(b)

** State symbols must be included. All intermediates, reactants or products in the energy level
23
diagram must be balanced in terms of atoms of each element as well as charges.

By Hess’ law
–1676 = 2(+322) + 2(+577+1820+2740) + 3/2 (+496)+ 3(–142)+ 3(+744) + LE
LL

lattice energy of Al2O3, LE = –15144 = –1.51 x 104 kJ mol-1


**Sign and units of H must be included.

7 (i) Lattice energy of magnesium oxide is the energy released when one mole of
magnesium oxide solid is formed from its constituent gaseous ions at 298K and 1 atm.

Mg2+(g) + O2-(g) → MgO (s) ** State symbols must be included.

(ii) From the Data Booklet, ionic radii in nm of Cl-: 0.181, Br- : 0.195, I- : 0.216
q+ q−
SinceHlatt    , but the cation and charges of the ions are the same, the
r+ + r−
magnitude of lattice energy would decrease as ionic radii of the halide increases down
the group.

(iii)   z+  z−
H latt = −107.1
r+ + r−
H latt = −2612 kJ mol -1 **Sign and units of H must be included.

(iv) There is a discrepancy between the experimental and theoretical lattice energies for ionic
compounds with partial covalent character. Theoretical model assume 100% ionic

25
Chemical Energetics

character.

Hence for LiCl, there is very little covalent character as there is a much smaller
difference between the theoretical and experimental values. Hence, the ionic radius of Li+
is likely to be as that predicted by the assumption.

Conversely the charge density of Mg2+ is relatively high, resulting in greater


polarization of the Cl− electron cloud, hence the larger degree of covalent character
in MgCl2.

8 (a)

** State symbols must be included. All intermediates, reactants or products in the energy cycle must
be balanced in terms of atoms of each element as well as charges.

Hformation + Hatm = 3(+715) + 5/2(+436) + 3/2(+994) + 9/2(+496)


Hatm(nitroglycerin) = +364 + 6958 = + 7322 kJ mol-1
**Sign and units of H must be included.
23
(b) ΔH2θ is less exothermic than ΔH1θ.
ΔH1θ is a reaction between a strong acid and strong base. Ibuprofen is a weak acid. Some
LL

of the energy released by neutralisation/formation of water is used to fully dissociate


the weak acid, ibuprofen. (Thereby reducing the magnitude of the enthalpy change of
neutralisation).

** State symbols must be included. All intermediates, reactants or products in the energy cycle must
be balanced in terms of atoms of each element as well as charges.

ΔHdiss = ΔH1θ – ΔH2θ = –50.0 + 57.3 = +7.3 kJmol-1


**Sign and units of H must be included.

26
Chemical Energetics

9 (i) Apparent amount of heat evolved, Q’


= mc∆T = (30 + 25) × 4.18 × 6.3 = 1448.4 J
90
Q’ = × Q (given 90% efficiency)
100
100
Actual amount of heat evolved, Q = × 1448.4 = 1609.3 J
90
25
No. of mole of H2SO4 = x 1.0 = 0.0250 mol
1000
30
No. of mole of CH3NH2 = x 1.0 = 0.0300 mol
1000
CH3NH2 is the limiting reagent.
Hence, n(H2O) formed = 0.0300 mol

Q 1609.3
∆Hnө = – =– = – 53.6 kJ mol–1 **Sign and units of H must be included.
n(H2 O) 0.0300

(ii) Lower magnitude /Less exothermic. This is so as some amount of energy is


required to cause ionisation of the undissociated weak acid molecules in order for
neutralisation to occur.

10 (a)
23
LL

** State symbols must be included. All intermediates, reactants or products in the energy level
diagram must be balanced in terms of atoms of each element as well as charges.

H = +178 + 1740 – (–116) – 1577 – 796


= –339 kJ mol–1 **Sign and units of H must be included.

27
Chemical Energetics

(b) Ca has a giant metallic structure with metallic bonds between Ca2+ ions and sea of
delocalised electrons.

Ca(OH)2 has a giant ionic structure with ionic bonds between Ca2+ and OH–ions.

The lattice energy of Ca is the reverse of the enthalpy change of atomisation of Ca,
i.e. Ca(g) → Ca(s), H = –178 kJ mol–1.

As the metallic bonds in Ca are weaker than the ionic bonds in Ca(OH)2, the lattice
energy of Ca is much less exothermic that that of Ca(OH)2.

11 (a)

23
LL

** State symbols must be included. All intermediates, reactants or products in the energy level
diagram must be balanced in terms of atoms of each element as well as charges.

By Hess’s Law,
ΔH1 = ΔH4 + ΔH5 − 2ΔH2 − 6ΔH3
= 435 + (−85) −2(717) − 6(218)
= − 2390 kJ mol-1 (3 s.f.) **Sign and units of H must be included.

28
Chemical Energetics

(b)

** State symbols must be included. All intermediates, reactants or products in the energy level
diagram must be balanced in terms of atoms of each element as well as charges.

By Hess’s Law,

4 BE (Pb−C) = 4ΔH1 + ΔH6 + 8ΔH2 + 20ΔH3 − ΔH7


= 4(−2392) + 195 +8(717) + 20(218) − 264
= 459 kJ
Mean Pb-C bond energy = 459/4 = +115 kJ mol-1
**Sign and units of H must be included.

23
Multiple Choice Questions (Section A)
1. 2. 3. 4. 5. 6. 7. 8. 9. 10. 11. 12. 13. 14. 15. 16. 17. 18. 19. 20.
D C B B C B B B C A B D B C C D C D C A
LL

Multiple Choice Questions (Section B)


1. 2. 3. 4. 5. 6. 7. 8. 9. 10.
D C A C D D B C C A

Note:
Do not assume that all the given answers are correct. Do discuss with your friends and check
with your tutor should you think that any given answer is incorrect.

29
5b
Entropy & Gibbs Free Energy Change

ENTROPY & GIBBS FREE ENERGY CHANGE

Structured / Free Response Questions

1 [DHS 2011/II/4(e)]
Different lead compounds can be reduced to the elemental form. One example is the
reaction between lead(II) oxide and lead(II) sulfide.

2PbO (s) + PbS (s) → 3Pb (s) + SO2 (g) (H = +180 kJ mol–1)

(i) State how the entropy of the reaction will most likely change.

(ii) Hence or otherwise, explain why heating is required for the reaction to occur.
[3]

2 [RVHS 2011/III/1(a)]
During the combustion of ammonia, it reacts to form nitrogen gas and water vapour as seen
in the equation below:
4NH3(g) + 3O2(g) → 2N2(g) + 6H2O(g)
Using relevant data from the Data Booklet, calculate the enthalpy change of the
(i)
reaction.
23
(ii) Given that the theoretical value of enthalpy change of reaction is −1267 kJ mol-1,
explain why the calculated value is different.
LL

State, with reason, the sign of the entropy change of the reaction.
(iii)
Hence, explain if the above reaction is spontaneous. [5]

3 [TPJC 2011/III/2(a)]
Often, there is insufficient oxygen inhaled to produce sufficient energy required for a race.
An alternative metabolic pathway which produces energy without oxygen is described as
anaerobic. In this process, glucose is converted into lactic acid, C3H6O3.

(i) Given that the standard enthalpy change of combustion of lactic acid was determined
experimentally to be −1362 kJ mol-1 and the standard entropy change (ΔSo) of the
combustion of lactic acid is +663 J mol-1 K-1, find the temperature range at which the
combustion of lactic acid can occur.

(ii) State one assumption made about the value of ΔSo in the calculation above.
[3]

1
Entropy & Gibbs Free Energy Change

4 [Modified SAJC 2009/II/4]

(a) By considering the structures of diamond and graphite, explain why the entropy
change for the conversion of diamond to graphite is positive.

H = −2 kJ mol−1

[2]

(b) Explain why the conversion of diamond to graphite is spontaneous at all temperatures.
Explain why in reality, this is not absolutely true.
[3]

5 [ACJC 2007/III/3(c)]
The reaction between hydrogen and iodine is carried out at two temperatures, 298K and
500K. Iodine sublimes at 458K.
23
H2 (g) + I2 (solid or gaseous) 2 HI (g)
LL

The entropy changes at 298K and 500K are +165 J mol-1 K-1 and +22 J mol-1 K-1 respectively.

(i) Explain why there is a difference in the entropy values at 298K and 500K.

(ii) The standard enthalpy change of the reaction at 298k and 500K are +53.0 kJ mol-1
and –10.4 kJ mol-1 respectively. Calculate ΔG of the reaction at 298K and 500K.
Comment on the feasibility of the reaction using the calculated values.
[3]

6 [SAJC 2008/III/4(b)]
Nitroglycerin, C3H5(NO3)3, is a flammable oil commonly used to manufacture dynamite.
Upon ignition, nitroglycerin decomposes to produce nitrogen, oxygen, carbon dioxide and
steam.
Given:
Standard enthalpy change of formation of nitroglycerin(l) / kJ mol-1 -364
-1
Standard enthalpy change of atomisation of carbon (graphite) / kJ mol +715
Standard enthalpy change of formation of H2O(g) / kJ mol-1 -242
Standard enthalpy change of formation of CO2(g) / kJ mol-1 -394

2
Entropy & Gibbs Free Energy Change

(i) Write a balanced equation, with state symbols, for the decomposition of nitroglycerin.
(ii) With reference to above data, calculate the standard enthalpy change of
decomposition of nitroglycerin.

(iii) Calculate G at 250C for this decomposition given that S = +208 J K-1mol–1.
Predict the spontaneity of the reaction at 250C.

(iv) Is the reaction spontaneous at all temperatures? Explain. [5]

7 [SRJC 2010/III/5(e)]
Hard water (water containing Ca2+) contributes to costly operation of water-using
appliances. Heated hard water forms a scale of calcium carbonate that can contribute to the
inefficient operation or failure of water-using appliances. Pipes can become clogged with
scale that reduces water flow and ultimately requires pipe replacement.

Data of standard enthalpy changes and Gibb’s free energies of some reactions pertaining to
calcium carbonate are given below.

Reactions HO (kJ mol-1) GO (kJ mol-1)

Ca(s) + C(s) + O2(g) → CaCO3(s) – 1212 – 1133


23
Ca(s) + C(s) + O2(g) + aq → CaCO3(aq) – 1224 – 1086

CaCO3(s) + aq → CaCO3(aq) R + 47
LL

(i) Calculate the value of R, and hence determine the standard entropy change of
solution for calcium carbonate.

(ii) Discuss whether the sign of the entropy change when CaCO3 (s) dissolves in water to
form CaCO3 (aq) obtained in part (i) is as expected.

(iii) Hence, explain why hard water causes pipes to clog more severely in a water heater
system.
[6]

8 [MJC 2009/II/2(c)]
‘Hard’ water also contains magnesium ions which can form a precipitate with the detergent.
For example, magnesium ions form magnesium iodide, MgI2, in the presence of aqueous
potassium iodide.

(i) The lattice energy of MgI2 is –2327 kJ mol-1 while the values of the enthalpy changes
of hydration are listed below:

Ions Hhyd / kJ mol-1


Mg2+ -1920
I- -295
Calculate the enthalpy change of solution of magnesium iodide.

3
Entropy & Gibbs Free Energy Change

(ii) By using your answer from (i), and given the entropy change of solution of magnesium
iodide is positive, predict whether magnesium iodide is soluble in water at room
temperature and pressure. Give your reasoning.
[3]

9 [HCI 2009/II/2(c)]
The Group II compounds, magnesium sulfate and barium sulfate, dissolve in water to
different extent.

(i) Using the data below, calculate the standard Gibbs’ free energy change of solution,
∆Gsoln, of the two compounds:

∆Hsoln / kJ mol–1 ∆Ssoln / J mol–1 K–1


MgSO4 –91.2 –210
BaSO4 +26.3 –103

Use your answers to comment on the relative solubilities of the two compounds.

(ii) Suggest a reason for the sign of ∆Ssoln given in the table.
[3]
23
10 [IJC 2011/II/2(a)]
The diagram below shows an experimental setup when a copper rod is dipped into a silver
nitrate solution. After one hour, the copper rod is coated with a shiny substance and the
LL

solution turned slightly blue.

(a) (i) Write an ionic equation, with state symbols, for the reaction that has occurred.

(ii) Given that


ΔGθ = –nFEθ

where n = the number of moles of electrons transferred in the reaction,


F = Faraday constant, 96500 C mol-1
ΔGθ = Gibbs Free Energy in J mol-1
Eθ is the electromotive force generated,

4
Entropy & Gibbs Free Energy Change

Using this equation, calculate ΔGθ, in kJ mol-1, of the above reaction and deduce
whether the reaction is spontaneous at standard conditions.
You may assume Eθ = +0.46V in this case.

(iii) Suggest and explain if the above reaction has a positive or negative ΔSθ.
[3]

(b) [Modified RI 2011/III/1(b)]


The standard electrode potentials, E, and standard Gibbs free energy changes, G, of
different chlorine-containing species are tabulated below.

Half-equation E / V G / kJ mol−1


1 ClO4− + 2H+ + 2e− ⇌ ClO3− + H2O +1.19 −230
2 2ClO3− + 12H+ + 10e− ⇌ Cl2 + 6H2O +1.47 ?
3 Cl2 + 2e− ⇌ 2Cl− +1.36 −262

These electrode potentials can be summarised using the Latimer diagram shown below. In
a Latimer diagram, the most highly oxidised form of the element is on the left, with
successively lower oxidation states to the right.
23
The electrode potentials are shown on the arrows.
LL

+1.19 V +1.47 V +1.36 V


ClO4− ClO3− Cl2 Cl

+1.45
V
G and E are related by the following equation.

G = −zFE

where G is the standard Gibbs free energy change in joules per mole,
z is the number of moles of electrons transferred during the redox reaction,
standard electrode potentials, E in V and
F is the Faraday constant, 96500 C mol-1.

(i) Calculate G for half-reaction 2.

2ClO3− + 12H+ + 10e− ⇌ Cl2 + 6H2O

5
Entropy & Gibbs Free Energy Change

(ii) The standard electrode potential of converting ClO3− to Cl− is not the summation of
+1.47 V and +1.36 V. This is because the number of electrons transferred in each step
must be taken into account.

 Write a half-equation for the conversion of ClO3− to Cl−.

 Using your knowledge of Hess’ Law for G and your answers to (b)(i), show
with the aid of an energy cycle that the E for the conversion of ClO3− to Cl− is
+1.45 V.

(iii) Given that Ecell of the reaction below is +0.26 V, determine its G and its feasibility
at standard conditions.

4ClO3−(aq) ⎯→ 3ClO4−(aq) + Cl−(aq) [6]

11 [Modified NJC 2010/II/4]


The following plot shows the variation of the Gibbs energy change, G, with temperature,
T, when separate samples of different substances were heated.
23
LL

(i) When solid strontium nitrate, Sr(NO3)2, was heated, it decomposes to form metallic
oxide, brown NO2 gas and a colourless gas that relights a glowing splint.

Construct a balanced equation for thermal decomposition of Sr(NO3)2.

(ii) Which one of the graphs, I or II, shows the variation of G with T when a sample of
solid strontium nitrate, Sr(NO3)2, was heated?
Explain your choice by considering the Gibbs equation.

6
Entropy & Gibbs Free Energy Change

(iii) Use your chosen graph in (i) to find a likely value for
• the enthalpy change of reaction when solid strontium nitrate was heated,
• the temperature at which solid strontium nitrate starts to decompose.
Indicate clearly how you arrive at your answers. [5]

12 [JJC 2007/III/3(c)]
The graph below is an Ellingham diagram, which shows the variation in the standard free
energy change of formation, ∆Gf, with temperature, T, for both carbon monoxide and zinc
oxide.

23
LL

(i) By considering the relationship between the free energy change, ∆G, the enthalpy
change, ∆H, and the entropy change ∆S for a reaction, suggest how you would
obtain a value for entropy change from the Ellingham diagram above.

(ii) Explain why the line for carbon monoxide in the diagram above has a negative
gradient.

(iii) Deduce, from the diagram, which oxide, CO or ZnO, is the stronger oxidising agent at
1800 K. [3]

7
Entropy & Gibbs Free Energy Change

Multiple Choice Questions (Section A)


1 [NJC 2010/I/16]

23
LL

2 [Nov 2011/I/7]

8
Entropy & Gibbs Free Energy Change

3 [AJC 2010/I/11]

4 [HCI 2010/I/7]
23
LL

5 [CJC 2011/I/9]

9
Entropy & Gibbs Free Energy Change

6 [HCI 2010/I/13]

7 [SRJC 2010/I/12]
23
Which graphs below correctly represents how ΔG varies with temperature for the
combustion of octane?
LL

10
Entropy & Gibbs Free Energy Change

8 [HCI 2011/I/6]

9 [RI 2011/I/9]

23
LL

11
Entropy & Gibbs Free Energy Change

10 [TJC 20101/I/14]
The common rubber band has some very interesting thermodynamic properties due to its
randomly coiled long polymeric molecular structure. When the rubber band is stretched, a
slight warming effect is felt.
What are the correct signs of ∆S, ∆H and ∆G if the stretched rubber band is released
quickly?

Multiple Choice Questions (Section B)


23
LL

1 [SAJC 2011/I/34]

12
Entropy & Gibbs Free Energy Change

2 [SRJC 2011/I/34]

3 [NYJC 2011/I/38]
23
LL

4 [SRJC 2009/I/32]

13
Entropy & Gibbs Free Energy Change

5 [SAJC 2010/I/33]

ENTROPY & GIBBS FREE ENERGY SUGGESTED SOLUTIONS

Structured/Free Response Questions

1 (i) The entropy of the reaction will increase as gaseous molecules are produced
from orderly solid reactants or there is increase in amt of gaseous particles in
the system (from 0 moles to 1 mole).

(ii) G = H − T S
Since ΔH > 0, ΔS > 0
23
At high temperatures, H  TΔ S , thus ΔG < 0
The reaction is feasible at high temperatures.

∆Hr = BE(bonds broken) – BE(bonds form)


LL

2 (i)
= [12BE(N-H) + 3BE(O=O)] – [2(NΞN) + 12(O-H)]
= [12(+390) + 3(+496)] − [2(+994)+12(+460)]
= −1340 kJ mol-1

(ii)
The calculated value differs from the theoretical value because the bond energies
given in the Data Booklet are only average energies.

(iii)
∆S is expected to be positive as the reaction proceeds with an increase in
disorder due to increase in the number of gaseous molecules (from 7 moles to
8 moles).
∆G = ∆H − T∆S, since ∆S is positive and ∆H is negative, ∆G will always be a
negative value. Hence we can conclude that the given reaction is spontaneous.

3 (i) Since H is negative and S is positive, G wlll always be negative for all
temperature range.

(ii) Assumption: ΔS is constant at all temperatures.

14
Entropy & Gibbs Free Energy Change

4 (a) The layered structure of graphite has more disorderliness due to the presence
of delocalized electrons as well as mobility of the planes of interconnected
hexagonal rings of carbon atoms held by weak instantaneous dipole-induced
dipole interactions.

Diamond has a giant covalent structure where the carbon atoms are bonded to
one another by network strong covalent bonds that is more orderly and fewer
number of ways of arrangement of the carbon atoms.

(b) Since ∆S> 0, hence −T∆S is always positive.


Since ∆H< 0, ∆G = ∆H- T∆S will always be negative.
Hence, since ∆G <0 at all temperatures, the conversion is spontaneous at all
temperatures.
Diamond is more kinetically stable than graphite and the activation energy
required for the conversion of diamond into graphite is very great, hence the
reaction is very slow at room temperature.

5 (i) At 298K, iodine is a solid. Hence, there is an increase in the number of gaseous
particles during the reaction and the entropy is more positive.

At 500K, iodine is a gas. Hence, there is very little entropy change as the number
of gaseous reactants and products are the same.

At 298K, ΔG = 53.0 – (298)(165 x 10–3) = + 3.83 kJmol–1


23
(ii)
At 500K, ΔG = -10.4 – (500)(22 x 10–3) = -21.4 kJmol–1
The reaction is more feasible at 500K because ΔG is negative at 500K whereas
ΔG is positive at 298K.
LL

6 (i) 5 3 1
C3H5(NO3)3(l) → 3CO2(g) + H2O(g) + N2(g) + O2(g)
2 2 4

(ii) Hdecomposition = (Hformation Products) – (Hformation Reactants)


5
= 3(–394) + (–242) – (–364) = – 1420 kJ mol–1
2

(iii) Gdecomposition = Hdecomposition – T Sdecomposition


208
= – 1423 – [298 (+ )] = – 1490 kJ mol–1
1000
Since G is negative, the reaction is spontaneous at 25C.

(iv) The spontaneity is not affected by changes in temperature because


Hdecomposition is negative and Sdecomposition is positive(due to increase in amt
of gaseous particles (from 0 moles to 7.25 moles)), hence – T Sdecomposition is
negative, Gdecomposition is negative. (since G = H – T S)

Thus, the decomposition reaction of nitroglycerin is spontaneous at all


temperatures.

15
Entropy & Gibbs Free Energy Change

7 (i) Ca(s) + C(s) + O2(g) + aq → CaCO3(aq)

– Ca(s) + C(s) + O2(g) → CaCO3(s)

CaCO3(s) + aq → CaCO3(aq)

By Hess’ Law, R = – 1224 – (–1212) = –12 kJ mol-1


At standard condition, T = 298K
GO = HO – TSO
+47 = –12 – (298) SO
− 12 − 47
SO = = – 0.1980 kJ mol-1
298

(ii) Sign of entropy change of solution of calcium carbonate is negative indicates that
there is a decrease in disorderliness / an increase in orderliness after calcium
carbonate is dissolved.

This is out of expectation because there is a change of state from solid to


aqueous, increase in the randomness / disorderliness in the arrangement of the
particles.

(iii) GO = HO – TSO


23
Since HO and SO have negative values, HO is negative and – TSO is positive.
Assuming HO and SO are constant, the higher the temperature in the water
heater system, the more positive GO is. Calcium carbonate becomes less
LL

soluble and form scale in the water pipes, clogging them.

8 (i) Enthalpy change of solution of magnesium iodide = −(−2327) + (−1920) + 2(−295)


= −183 kJ mol-1

(ii) At lower temperatures, − T ΔS o and ΔHo are both negative


hence G = H – TS is also negative .
Therefore the dissolution process is feasible and hence MgI2 is soluble in
water.

9 (i) ∆Gsoln for MgSO4 = (–91.2) – 298(–0.210) = – 28.5 kJ mol-1


∆Gsoln for BaSO4 = (+26.3) – 298(–0.103) = + 57.0 kJ mol-1

MgSO4 is soluble but BaSO4 is insoluble because ∆Gsoln is negative only for
MgSO4.

(ii) The negative ∆Ssoln indicates that there is a decrease in entropy when the two
compounds dissolve to form aqueous ions. This could be due to a more orderly
arrangement of H2O molecules when the aqueous ions are formed.

16
Entropy & Gibbs Free Energy Change

10(a) (i) Cu(s) +2Ag+(aq) → Cu2+(aq) + 2Ag(s)

(ii) ΔGθ= –nFEθ


= – 2 X 96500X 0.46 = –88.8 kJ mol-1 (spontaneous)

(iii) As the number of free aqueous ions is reduced, the system will be less
disorderly. The sign of ΔSθ should be negative.

(b) (i) 2ClO3− + 12H+ + 10e− ⇌ Cl2 + 6H2O


G = –(10)(1.47)(96500) = –1418.6 = –1420 kJ mol–1 (3 s.f.)

(ii)  2ClO3– + 12H+ + 12 e− ⎯→ 2Cl– + 6H2O


23
Applying Hess Law,
G = –1420 + (–262) = –1682 kJ mol−1

E = – [–1682 x 103 / (12 x 96500)] = +1.45 V (shown)


LL

(iii) Amount of electrons transferred = 6 mol


G = –(6)(96500)(+0.26) = –150 kJ mol–1 (spontaneous at std conditions)

11 (i) Sr(NO3)2(s) → SrO(s) + 2NO2(g) + ½ O2(g)

(ii) Graph II.


Since Sr(NO3)2(s) → SrO(s) + 2NO2(g) + ½ O2(g)
∆S is expected to be positive as the reaction proceeds with an increase in
disorder due to increase in the number of gaseous molecules(from 0 moles to
2½ moles).
Graph of G against T should give a straight line with a negative gradient as
G = H - TS (Gibbs eqn) and gradient of graph is - S.

(iii) From y-intercept of Graph II, H = + 600 kJ mol −1


From Graph II, when G = 0, Decomposition temp = 672 K (read off x-axis)

17
Entropy & Gibbs Free Energy Change

12 (i) Since ∆G = ∆H − T∆S, ∆S is the negative of the gradient of the graph.

(ii) There is an increase in entropy since the amount of gaseous molecules


increased from 1 to 2 mol.

(iii) ZnO is the stronger oxidising agent as it is more likely to undergo reduction
since the ∆G for 2 Zn + O2 → 2 ZnO is more positive and less energetically
feasible. This means that ∆G for 2 ZnO → 2 Zn + O2 is more negative and more
energetically feasible.

Multiple Choice Questions (Section A)


1. 2. 3. 4. 5. 6. 7. 8. 9. 10.
B D B A D C D D A C

Multiple Choice Questions (Section B)


1. 2. 3. 4. 5.
A D A B B

Note:
Do not assume that all the given answers are correct. Do discuss with your friends and check
with your tutor should you think that any given answer is incorrect.
23
LL

18
Chemical Equilibrium

CHEMICAL EQUILIBRIUM 7
Structured / Free Response Questions
1 [AJC 2011/III/2(a)]
The key step in the manufacture of sulfuric acid is the reaction between sulfur dioxide and
oxygen in excess air.

2SO2(g) + O2(g) 2SO3(g) H = –197 kJ mol–1

(i) For the above reaction, write an expression for the equilibrium constant, Kp, stating the
units.

(ii) Explain why excess air is required in the reacting gas mixture.

(iii) Describe and explain the effect of increase in temperature on the


I. rate at which equilibrium is achieved.
II. equilibrium constant Kp. [6]

2 [MI 2011/III/2(d)]
0.500 mole of ethanoic acid and 1.00 mole of ethanol were shaken together. The total
volume of the mixture is 0.5 dm3. The equilibrium mixture required 80 cm3 of 1.00 mol dm-3
of sodium hydroxide for complete reaction.
23
The equation for the reaction between ethanoic acid and ethanol is as follows.
LL

CH3COOH(l) + CH3CH2OH(l) ⇌ CH3COOCH2CH3(l) + H2O(l)

(i) Explain why the reaction mixture has to be titrated with sodium hydroxide rapidly.

(ii) Write the Kc expression for the reaction formed between ethanoic acid and ethanol.

(iii) Calculate the equilibrium constant for the equilibrium.


[6]

3 [MJC 2011/III/2(a)]
A mixture was prepared using 1.00 mol of ethanedioic acid and 2.00 mol of ethanol. At a
given temperature, the mixture was left to reach dynamic equilibrium according to the
following equation.

(COOH)2 + 2C2H5OH (COOC2H5)2 + 2H2O H < 0

The equilibrium mixture contained 36.8 g of ethanol.

(i) What do you understand by the term dynamic equilibrium?

(ii) Write an expression for Kc for this reaction.

(iii) Use your expression in (a)(ii) to calculate the value of Kc.

1
Chemical Equilibrium

(iv) For this equilibrium, predict the effect of an increase in temperature on the value of Kc.
[6]
4 [NJC 2011/II/4(c)]
In Singapore, nearly all the sulfuric acid is manufactured in a series of steps, starting with
sulfur. In one of these steps, sulfur dioxide is oxidised in a reversible reaction.

2SO2(g) + O2(g) 2SO3 (g)

In a research laboratory, a chemist was developing improvements to this process. The


equilibrium partial pressures of SO2(g) and SO3(g) at 27oC are shown in the table below.

Gas SO2(g) SO3(g)


Equilibrium partial pressure/ kPa 32 35

(i) Given that the total pressure is 110 kPa, calculate Kp for this equilibrium, giving your
answer in 3 significant figures.

(ii) Calculate the total volume occupied by the equilibrium mixture when 20.5 mol of SO2
is present at equilibrium.

(iii) Predict and explain how the position of the equilibrium might change if vanadium(V)
oxide catalyst is added into the system.
23
(iv) At 40oC, the chemist continued his investigation on a new mixture of SO2(g) and O2(g)
by monitoring the changes in partial pressures of the individual gases. After the
system reached equilibrium, he compressed the volume of the system at time t.
LL

Sketch on the graph above to show how the partial pressures of individual gases will
change when the chemist compressed the volume of the system at time t, which was
maintained at 40oC. Label the graph extensions for each species till equilibrium is
reached at t2.
[9]

2
Chemical Equilibrium

5 [VJC 2011/II/3]
The two most abundant atmospheric gases react to a small extent at 298 K in the presence
of a catalyst to achieve dynamic equilibrium as shown:

N2(g) + O2(g) 2NO(g) Kp = 0.30

(i) Explain the term dynamic equilibrium.

(ii) Atmospheric oxygen and nitrogen, each at a partial pressure of 0.780 atm, are put into
a 1 dm3 evacuated flask at 298 K with a catalyst and equilibrium is established after 30
minutes. Calculate the equilibrium partial pressure for each of the three components at
298 K.

(iii) What is the total pressure in the container at equilibrium?

(iv) Find Kc for this reaction at 298 K.

(v) On the grid below, sketch pressure versus time curves for N2 and NO under the
conditions as described in (b) at 298 K from 0 to 40 minutes. Label the curves and
indicate significant values on the axes.
23
LL

3
Chemical Equilibrium

(vi) The temperature of the system was raised to 350 K at 40 min.


Given that the reaction is endothermic and equilibrium is re-established at 60 min,
sketch on the axes above, the pressure versus time curves for N2 and NO from 40 min
to 70 min.
[10]

6 [RJC 2007/III/3(b)]
Iron(III) chloride is a dark brown solid which exists as a dimer, Fe2Cl6.
At 306 oC, it starts to sublime and decompose to its gaseous monomer. The decomposition
to its monomer is complete at 400 oC. 0.450 g of Fe2Cl6 was placed in an evacuated vessel
of volume 0.500 dm3. The vessel was heated to 350 oC and the pressure of the mixture at
equilibrium was found to be 2.63 x 104 Pa.

Calculate the degree of dissociation of Fe2Cl6 at 350 oC.


[3]

7 [Modified RVHS 2011/III/3(c)]


The reaction involving ibuprofen and propanol in a homogeneous equilibrium with the
products can be represented as follows.
23
LL

An experiment was set up to further investigate the effect of experimental conditions on the
reaction between ibuprofen and propanol. In this experiment, 0.00500 mol of ibuprofen,
0.500 mol of propanol and 0.25 mol of concentrated sulfuric acid catalyst are mixed together
in a flask at 20 C.

4
Chemical Equilibrium

The following graph shows the proportion of ibuprofen in the flask that had reacted over
time.

(i)
Determine the Kc of the reaction at 19 min.

(ii)
The temperature of the reaction mixture was increased to 45 C at 20 min. Explain
whether the forward reaction is endothermic or exothermic.
23
(iii) In the original experiment, a small volume of dilute potassium hydroxide solution was
added at 10 min. A student repeated the experiment and added the same volume of
water instead of dilute potassium hydroxide solution at 10 min.
LL

Account for the shape of the graph in both the original and the repeated experiment
from 10 to 20 min. [7]
8 [MJC 2009/III/1(a)]
Sulphuryl chloride, SO2Cl2, is often used as a source of Cl2 for various organic reactions.
Sulphuryl chloride is also a highly reactive gaseous compound. When heated, it
decomposes endothermically as follows:

5
Chemical Equilibrium

SO2Cl2(g) SO2(g) + Cl2(g)


A 7:2 mole ratio of SO2Cl2 and Cl2 is placed in an evacuated vessel at 375 K and 6 atm.
After 10 minutes, the mixture reached equilibrium and the partial pressure of SO 2 is found to
be 0.625 atm.

(i) Write an expression for the equilibrium constant, Kp, and calculate its value.

(ii) Suggest, with a reason, how the position of equilibrium might alter when the
temperature is increased at the 20th minute.

Hence, draw labelled sketches on the same graph to show how the number of moles
of sulphuryl chloride and sulphur dioxide gas would change from the start of the
decomposition till the new equilibrium is reached.
[6]

9 [SAJC 2009/III/3(c)]
Methane can react with steam to give carbon monoxide and hydrogen gas in the presence
of a nickel catalyst.
CH4(g) + H2O(g) CO(g) + 3H2(g)
When a 3:1 mole ratio of methane and steam at a total initial pressure of 4 atm is passed
over nickel catalyst at 450 oC, the partial pressure of carbon monoxide at equilibrium is
found to be 0.66 atm.
23
(i) Write an expression for Kp and calculate its value, giving its units.

(ii) Predict and explain the effect of increasing pressure on the equilibrium position and
LL

the value of Kp.

(iii) The graph below shows how the fraction of CO produced varies with temperature.
Fraction of CO in equilibrium

Temperature/K
Deduce whether the reaction of methane with steam is an exothermic or an
endothermic process.
[7]
10 [SAJC 2008/III/2(b)]
An Fe/Fe2O3 catalyst is used in the synthesis of ammonia in the Haber Process

N2 (g) + 2 H2 (g) 2 NH3 (g) ΔHo negative

6
Chemical Equilibrium

But no catalyst was used in the synthesis of nitrogen monoxide

N2 (g) + O2 (g) 2 NO (g) ΔHo positive

Explain the observation of the usage of catalyst in one synthesis and not in the other.

[4]

[3]
11 [RI 2002/III/2(c)]
An experiment was done to determine the equilibrium constant at 25 oC for the reversible
reaction

Ag+(aq) + Fe2+(aq) Fe3+(aq) + Ag(s)

Into a large plastic bottle, 500 cm3 of 1.50 mol dm−3 silver nitrate solution was mixed with
500 cm3 of 1.06 mol dm−3 iron(II) sulphate solution. The stopper of the bottle was screwed
up tightly, and the mixture was left to stand for an hour at 25 oC. The amount of Fe2+(aq)
remaining in the mixture was found by titrating with 0.0850 mol dm−3 acidified potassium
manganate(VII) solution. 25.0 cm3 of the mixture required 25.60 cm3 of acidified potassium
manganate(VII) for reaction.

Given that 5 moles of Fe2+ react with 1 mole of MnO4−, calculate the equilibrium constant,
23
Kc, for the reaction at 25 oC. [3]
LL

7
Chemical Equilibrium

Multiple Choice Questions (Section A)


1 [ACJC 2010/I/11]

23
LL

2 [CJC 2010/I/11]

8
Chemical Equilibrium

3 [AJC 2010/I/13]

4 [AJC 2010/I/14]
23
LL

9
Chemical Equilibrium

5 [MJC 2010/I/8]

6 [SAJC 2010/I/11]
23
LL

7 [PJC 2010/I/9]

10
Chemical Equilibrium

8 [RI 2010/I/9]

9 [RI 2010/I/10]

23
LL

11
Chemical Equilibrium

10 [HCI 2010/I/9]

23
11 [NYJC 2010/I/9]
LL

12 [RVHS 2011/I/7]

12
Chemical Equilibrium

13 [ACJC 2010/I/13]

23
LL

14 [NYJC 2011/I/12]

13
Chemical Equilibrium

15 [MJC 2011/I/8]

23
LL

16 [Nov 2008/I/7]
A pure sample of N2O4(l) is introduced into an evacuated vessel. The vessel, of constant
volume, is heated to a constant temperature such that the equilibrium below is established.

2NO2(g) 2NO(g) + O2(g)

The value of the pressure p is then found to be 20% greater than if only NO2(g) were
present.

What is the mole fraction, x, of oxygen in this equilibrium mixture?

A 0.17 B 0.20 C 0.33 D 0.40

14
Chemical Equilibrium

17 [SAJC 2011/I/14]

18 [IJC 2011/I/8]
23
LL

15
Chemical Equilibrium

19 [RI 2011/I/12]

23
LL

20 [Nov H1 2009/I/11]

16
Chemical Equilibrium

Multiple Choice Questions (Section B)

1 [NJC 2011/I/32]

23
2 [SRJC 2011/I/33]
LL

17
Chemical Equilibrium

3 [TJC 2011/I/33]

4 [RI 2011/I/34]
23
LL

18
Chemical Equilibrium

5 [MJC 2010/I/32]

23
LL

19
Chemical Equilibrium

CHEMICAL EQUILIBRIUM SUGGESTED SOLUTIONS

Structured/Free Response Questions


1 (i) pSO3
2

Kp = 2
atm-1 or Pa-1
pSO2 pO2

(ii) To increase the SO3 yield.


When the no. of moles of O2 gas increases, the position of equilibrium will shift to
the right by Le Chatelier’s principle, and the resultant mixture will have more SO3
and less SO2 and O2.

(iii) I. rate at which equilibrium is achieved.


When temperature is increased, the rate of the forward and backward reaction
will increase. Hence the rate at which equilibrium is achieved will be faster.

II. equilibrium constant Kp


When temperature is increased, the rate of the backward reaction will increase to
a greater extent than the rate of the forward reaction. Hence Kp will decrease.

2 (i) This is to reduce the effects of equilibrium changes as when more ethanoic acid is
23
removed when NaOH is added, the equilibrium mixture may shift to the left to
produce more ethanoic acid.
LL

(ii) [CH 3COOCH 2CH 3 ][ H 2O ]


Kc =
[CH 3COOH ][CH 3CH 2OH ]

(iii) Amount of NaOH used


= Amt of excess CH3COOH at equilibrium
= 80/1000 x 1
= 0.0800 mol ;
CH3COOH + CH3CH2OH ⇌ CH3COOCH2CH3 + H2O
I/mol 0.5 1 0 0
C/mol -x -x +x +x
E/ mol 0.5 -x 1-x +x +x
E/ mol; 0.08 0.58 0.42 0.42
Conc/ 0.16 1.16 0.84 0.84
moldm-3
x = 0.5 – 0.0800 = 0.42
0.42
[CH3COOCH2CH3] = = 0.84 mol dm−3
0.5
[0.84][0.84]
 Kc = = 3.80
[0.16][1.16]

20
Chemical Equilibrium

3 (i) At dynamic chemical equilibrium in a reversible reaction, rate of forward reaction


is equal to rate of backward reaction and the substances are still reacting together
although the concentration of the reactants and products remain constant.

(ii) [(COOC2 H 5 ) 2 ][ H 2O]2


Kc =
[(COOH ) 2 ][C2 H 5OH ]2

(iii) (COOH)2 + 2C2H5OH (COOC2H5)2 + 2H2O


Eqm. mol 0.4 0.8 +0.6 +1.2

Let volume of mixture be V dm3


0.6 1.2 2
[ ][ ]
Kc = V V = 3.38
0.4 0.8 2
[ ][ ]
V V

(iv) When the temperature is increased, equilibrium position shifts to the left towards
endothermic reaction to absorb heat by Le Chatelier’s principle.

The no. of mole of ester and water decreases and no. of mole of ethanedioic
acid and ethanol increases, and hence the value of Kc decreases.
23
4 (i) Partial Pressure of O2(g) = 110-32-35 = 43 kPa
LL

= 0.0278 kPa–1 = 2.78 ×10–5 Pa–1

(ii) pV= nRT

(32×103)Vtotal = (20.5) (8.31)(27+273)


Vtotal = 1.60 m3

OR

ntotal = 70.46 mol


(110×103)Vtotal = (70.46) (8.31)(27+273)
Vtotal = 1.60 m3

(iii) The catalyst increases the rate of the forward reaction and the backward rate of
reaction by the same extent, hence allowing the equilibrium position to be reached
faster but there will not be any change in equilibrium position.

21
Chemical Equilibrium

(iv)

5 (i) It refers to a state of the system where the rate of forward reaction is the same as
the rate of the backward reaction.

(ii)
N2(g) O2(g) 2NO(g)
Initial P/ atm 0.780 0.780 0
Change P/ atm -x -x +2x
Eqm P / atm 0.78-x 0.780-x 2x

Kp = (PNO)2 / (PN2)(PO2)
23
0.30 = 4x2 / (0.78-x)2

x = 12.14 (reject) or 0.1677


LL

PO2 = PN2 = 0.78 – 0.1677 = 0.612 atm

PNO = 2 x 0.1677 = 0.335 atm

(iii) Total pressure = 0.612 + 0.612 + 0.335 = 1.56 atm

(iv) Since total number of moles of gaseous reactants and products is the same, Kc = Kp
= 0.30

22
Chemical Equilibrium

(v) (vi)

23
LL

A 6 Assuming ideal behaviour, pV = ntotal RT


2.63 x 104 x 0.500 x 10−3 = ntotal x 8.31 x (273+350)
2.63x104 x0.500x10−3
ntotal = =0.002540 mol ]
8.31x623
0.450
Initial amount of Fe2Cl6 = = 0.001386 mol
55.8x2 + 35.5x6
Let amount of Fe2Cl6 dissociated at equilibrium be q mol
Fe2Cl6 (g) 2FeCl3 (g)
Initial amount/ mol 0.001386 0
Change in amount/ mol −q +2q
Eqm amount/ mol 0.001386−q 2q

23
Chemical Equilibrium

Total amount of gases at equilibrium = 0.001386−q+2q = 0.001386+q


Hence, 0.001386+q = 0.00254  q = 0.00254−0.001386 = 0.001154 [1]
0.001154
Degree of dissociation = x100% = 83.3 %
0.001386
7 (i)
Ibuprofen + propanol ester + water
Initial amt 0.00500 0.500 0 0
Change - 0.00230 - 0.00230 + 0.00230 + 0.00230
Eqm amt 0.00270 0.498 0.00230 0.00230
0.00230 0.00230
( )( )
Kc = V V
0.00270 0.498
( )( ) .
V V
= 0.00393

(ii)
When temperature is increased, the proportion of ibuprofen reacted decreased. The
equilibrium position shifted to the left to absorb some of the energy.
Since increased in temperature favours endothermic reaction, the forward
reaction is exothermic.
23
(iii)
In the original experiment, the equilibrium position is shift to the left to
consume some of the water added. Therefore, a smaller proportion ibuprofen
LL

reacted in the original experiment.


KOH (aq), an alkali, neutralized some of the sulfuric acid which is the catalyst for
the reaction. Equilibrium is established at a slow rate/less negative gradient than
the original.
8 (i) SO2Cl2 SO2 + Cl2
Initial P/atm 7 0 2
x6 x6
7+2 7+2
=4.667 = 1.333
Change in P /atm −0.625 +0.625 +0.625

Eqm P/ atm 4.042 0.625 1.958

( PSO2 )( PCl2 ) 1.958 x 0.625


Kp = = = 0.303 atm
( PSO2Cl2 ) 4.042

(ii) The equilibrium position will shift right to favour the endothermic reaction to
absorb excess heat.

24
Chemical Equilibrium

9 (i) PCO PH 2 3
KP =
PCH 4 PH 20
CH4 H2O CO H2
Initial 3.00 1.00 0 0
pressure/atm
Change in -0.66 -0.66 +0.66 +1.98
pressure/atm
Partial 2.34 0.34 0.66 1.98
pressure/atm

Kp = 6.44 atm2
23
(ii) Equilibrium position will shift to the left to lower the pressure of the system / to
decrease the number of gaseous particles.
LL

No effect on Kp as Kp is only dependent on temperature.

(iii) From graph, increase in temperature increases fraction of CO indicating forward


reaction favoured.
Reaction is endothermic to absorb the excess heat when temperature is
increased.

10 The use of a low temperature is preferred to keep the yield of ammonia gas high as the
forward exothermic reaction liberates energy for the synthesis of ammonia.

A catalyst is used to speed up the reaction in the synthesis of ammonia as too high
temperature is not favoured and to keep operation costs down.

When nitrogen is converted into nitrogen monoxide, the high temperature used will
favour the forward endothermic reaction which involves the formation of nitrogen
monoxide and gives a high rate of reaction, hence no catalyst is used.

25
Chemical Equilibrium

11 Ag+(aq) + Fe2+(aq) Fe3+(aq) + Ag(s)

Initial [Ag+(aq)] = 500/1000 x 1.50 = 0.75 moldm-3


Initial [Fe2+(aq)] = 500/1000 x 1.06 = 0.53 moldm-3
[Fe2+(aq)] left = 5 x 25.60/25.0 x 0.0850 = 0.4352 moldm-3
[Fe2+(aq)] reacted = 0.53 – 0.4352 = 0.0948 moldm-3

Kc=[Fe3+]/[Ag+][Fe2+] = (0.0948)/{(0.75 – 0.0948)(0.4352)} = 0.332 mol-1dm3

Multiple Choice Questions (Section A)


1. 2. 3. 4. 5. 6. 7. 8. 9. 10. 11. 12. 13. 14. 15. 16. 17. 18. 19. 20.
B A B C B D B B A A C B A D D A C C B D
Multiple Choice Questions (Section B)
1. 2. 3. 4. 5.
C D A C B
23
Note:
Do not assume that all the given answers are correct.
Do discuss with your friends and check with your tutor should you think that any given
LL

answer is incorrect.

26
Acid Base Equilibrium

ACID BASE EQUILIBRIUM 8a


Structured / Free Response Questions

1 Calculate the pH of the following solutions at 25C

(a) 1.0 x 10−2 mol dm−3 nitric acid

(b) 1.0 x 10−10 mol dm−3 hydrochloric acid

(c) 2.0 x 10−2 mol dm−3 sodium hydroxide

(d) 2.00 g of Ba(OH)2 dissolved in 750 cm3 of solution.

(e) 0.10 mol dm−3 HX which is 20% ionized.

(f) 0.15 mol dm−3 solution of phenol. (Ka of C6H5OH = 1.0 x 10−10 mol dm−3)
23
(g) 0.245 mol dm−3 methylamine, CH3NH2.(Kb of CH3NH2 = 5.0 x 10−4 mol dm−3)
Equation: CH3NH2 + H2O CH3NH3+ + OH−
LL

(h) 25.0 cm3 of 0.01 mol dm−3 H2SO4 is mixed with 15.0 cm3 of 0.12 mol dm−3 HNO3.

(i) 50.0 cm3 of 0.10 mol dm−3 ammonia is mixed with 50.0 cm3 of 0.20 mol dm−3 ammonium
chloride.
(Kb of NH3 = 1.8 × 10−5 mol dm−3)

2 N95/III/3

“Acidity regulators” are food additives that have a buffering action on the pH of foodstuffs.
Mixtures of citric acid and its sodium salt are often used for this purpose.

C5H7O4CO2H C5H7O4CO2− + H+ Ka = 7.4 x 10−4 mol dm−3 citric acid

(a) The concentration of citric acid in lemon juice is 0.22 mol dm−3.
Assuming that no other acid is present, calculate the pH of lemon juice.
[3]
(b) Write equations to show how the citric acid/sodium citrate buffer system regulate the acidity
on the addition of
(i) H+ ions (ii) OH− iions
[2]

1
Acid Base Equilibrium

(c) Calculate the pH of a solution containing 0.10 mol dm−3 citric acid and 0.30 mol dm−3
sodium citrate.
[2]
(d) Define the term Kw and explain why at 25oC, water has a pH of 7.
[2]

(e) How would you expect the pH of water to change(if any) when the temperature is
increased? Explain.
[2]

(f) Based on your answer in (e), explain if water is neutral at higher temperatures.
[1]

3 J95/III/3 and J98/II/1 modified

(a) Explain what is meant by a buffer solution.


[1]
23
(b) With the aid of two equations, explain how a solution of ammonium chloride and ammonia
control pH.
[2]
LL

(c) A buffer solution is prepared as follows: 53.5 g of ammonium chloride is dissolved in


400 cm3 of 15.0 mol dm−3 ammonia and the mixture is diluted to 1.00 dm3.
Calculate the concentration, in mol dm−3 in the prepared buffer solution of
(i) [NH4+] (ii) [NH3]

[2]

(d) Given that the Kb of NH3 is 1.67 x 10−5 mol dm−3, calculate the pH of the prepared buffer
solution.

[2]
(e) Calculate the change in pH of the buffer solution when 0.05 mol of solid sodium hydroxide
is added.
[2]

(f) What is the change in pH when 0.05 mol of solid sodium hydroxide is added to 1.00 dm3 of
water?
[1]
(g) Explain the difference in values in (e) and (f).
[1]

2
Acid Base Equilibrium

[AJC2011/3/2b]

4 One of the major industrial uses of sulfuric acid is to convert phosphate rock into dihydrogen
phosphates (H2PO4–), which makes useful buffers for biochemical experiments.

H2PO4– HPO42– + H+ Ka = 6.3 x 10–8 mol dm–3

HPO42– PO43– + H+ Ka = 4.2 x 10–13 mol dm–3

(i) Calculate the pH at which maximum buffering capacity occurs for a solution that
contains NaH2PO4 and Na2HPO4.

(ii) Hence, suggest a suitable indicator for the titration of H2PO4– with aqueous
sodium hydroxide. Explain your answer.
((iii) Hydrogen phosphate, HPO42–, can behave both as an acid and a base.
Using the given Ka values, explain why it is a stronger base than it is an acid.
[5]

[JJC2011/2/3b-e]
5 Phosphoric acid, H3PO4, is often used in teeth whiteners to remove plaque in dentistry.
23
(a) State the oxidation state of phosphorus in H3PO4. [1]

Phosphoric acid is a tribasic acid. The values of the three acid dissociation constants at 25oC
LL

of phosphoric acid are given below.

H3PO4 H2PO4− + H+ Ka1 = 7.50 × 10−3 mol dm−3


H2PO4− HPO42− + H+ Ka2 = 6.20 × 10−8 mol dm−3
HPO42− PO43− + H+ Ka3 = 4.80 × 10−13 mol dm−3

(b) Suggest why there is a decreasing trend in the Ka values. [1]

(c) Identify the conjugate acid of HPO42− and the conjugate base of HPO42−.

conjugate acid ……………… conjugate base ……………… [1]

(d) Calculate the Kb of HPO42−. [1]

A phosphate buffer system (made of H2PO4− and HPO42−) serves to buffer the intracellular
fluid of cells.

(e) (i) Explain what is meant by a buffer solution.

(ii) Write an equation to show how the phosphate buffer system acts as a buffer
when a small amount of base is added.

3
Acid Base Equilibrium

The diagram below shows the pH changes when 0.0305 mol dm−3 HNO3 is added to a
25.0 cm3 solution of sodium phosphate, Na3PO4.

14
A
12
B
10 pH = 9.40
pH
8 C
D
6 pH = 4.92

4 E F pH = 2.71 G

2
10.0 20.0 30.0
Volume of HNO3 added / cm3
23
(f) Calculate the initial concentration of the sodium phosphate solution. [1]
LL

(g) State the volume of HNO3 required to reach the point where the buffer
(made of H2PO4− and HPO42−) is operating at its maximum buffering capacity. [1]

(h) (i) From the table below, select an indicator that is most suitable for determining the
second equivalence point in the titration shown on the graph.

Indicator pKa acid form base form


methyl yellow 3.3 red yellow
bromocresol green 4.9 yellow blue
chlorophenol red 6.0 yellow red
bromothymol blue 7.1 yellow blue
phenolphthalein 9.3 colourless purple

Indicator: ……………………………………..

(ii) Suggest the colour change observed with this indicator at the end point.

…………………………………………………………………………………… [2]

[Total: 12]

4
Acid Base Equilibrium

[NJC2011/3/4a]
6 20.0 cm3 of 0.10 mol dm–3 solution of propylamine, CH3CH2CH2NH2, was titrated against
0.10 mol dm–3 solution of hydrochloric acid. The following titration curve was obtained.

(a) Calculate the base dissociation constant, Kb, of propylamine with the use of initial
pH on the titration curve.

(b) Explain the pH value obtained at equivalence point with the aid of an appropriate
23
equation.

(c) A student used phenolphthalein to carry out the above titration. Comment on the
suitability of this indicator.
LL

(d) An aqueous solution of propylamine, CH3CH2CH2NH2 and CH3CH2CH2NH3Cl can


act as a buffer solution.

I State the volume of hydrochloric acid at which the buffer operates at its
maximum buffer capacity.

II Write suitable equations to illustrate the buffering action of an aqueous


solution of CH3CH2CH2NH2 and CH3CH2CH2NH3Cl when small amounts
of NaOH or HCl is added to it.

[9]

5
Acid Base Equilibrium

[PJC2011/3/3a]
7 Human blood is slightly basic, with a normal pH of approximately 7.4. Its pH must be
maintained for the effective transport of oxygen.

When CO2 is dissolved in the blood plasma, a series of equilibria is established.

I CO2(aq) + H2O(l) ⇌ H2CO3(aq)

II H2CO3(aq) ⇌ H+(aq) + HCO3−(aq)

Equilibrium II has a Ka value of 7.90  10−7 mol dm−3 and is responsible for buffering the
pH of the blood plasma at 7.4.

In the lungs, excess dissolved CO2(aq) is exhaled as CO2(g) according to the following
equilibrium.

III CO2(aq) ⇌ CO2(g)

During heavy exercise, lactic acid is released into the blood and is buffered by the blood
plasma. The buffering action of the blood plasma will eventually lead to an increased
breathing rate.
23
(a) State what is meant by a buffer solution

(b) Use the data above to determine the [HCO3−(aq)]/[H2CO3(aq)] ratio in the blood
LL

plasma.

(c) Write an equation to show how lactic acid is removed from the body. Use RCO 2H
to represent lactic acid.

(d) Hence, by considering the effect on equilibria I and III, explain why the process of
removing lactic acid causes the breathing rate to increase.

(e) A student wishes to simulate the blood plasma buffer by mixing 25.0 cm3 of 0.20
mol dm−3 H2CO3 solution with 0.30 mol dm−3 NaOH solution. The neutralisation
that takes place is represented by the following equation.

NaOH + H2CO3 → NaHCO3 + H2O

Using your answer in (b), calculate the volume of NaOH that should be added.

6
Acid Base Equilibrium

[SRJC2011/3/3b]
8 Propanoic acid, CH3CH2COOH has a pKa value of 4.87.
Buffered propanoic acid mixes have always been used as preservatives in the process
of growing alfalfa-grass. It is by far, the most effective in inhibiting growth of moulds. To
ensure its effectiveness, the propanoic acid/sodium propanoate buffer system is
maintained at pH 6.
(i) With the aid of equation(s) or otherwise, explain how the buffer system helps to
maintain pH.
(ii) Using the pKa value of propanoic acid that you have identified in (a)(ii), calculate the
ratio of propanoate and propanoic acid in the buffer system.
Hence, determine the new pH of a 2 dm3 buffer solution containing 0.05 mol dm-3 of
propanoic acid when 0.03 mol of H+ is added.
[6]

[MJC2011/3/2b]
9 A buffer solution is prepared by adding 10.00 cm3 of 0.100 mol dm−3 potassium
hydroxide to 25.00 cm3 of 0.400 mol dm−3 ethanoic acid. (pKa of ethanoic acid is 4.75)
23
(i) What is meant by the term buffer?

(ii) Explain how the solution above acts as a buffer when H+ ions are added.
LL

(iii) Calculate the pH of the buffer solution formed.

(iv) Suggest why the concentration of potassium hydroxide in a solution slowly


decreases when left open to air. [6]

7
Acid Base Equilibrium

[TJC2011/3/2b]
10 3-nitrophenol and benzoic acid are weak Bronsted acids with pKa values of 8.35 and
4.20 respectively.

(a) Calculate the pH of 0.1 mol dm−3 of 3-nitrophenol.

(b) Phenolpthalein and bromothymol blue are common indicators for acid-base titrations.

OH
OH
OH OH
Br

O O Br
S
O O O

phenolpthalein
Phenolphthalein Bromothymol blue
Bromothymol blue
pKa = 9.3 pKa = 7.1
pKa = 9.3 pKa = 7.1
23
A 25.0 cm3 solution containing both 3-nitrophenol and benzoic acid was titrated against
1 mol dm−3 sodium hydroxide using 1 to 2 drops of phenolphthalein and bromothymol
LL

blue indicator.

(i) Explain why both indicators are added in small amounts.


It was found that 14.40 cm3 was needed for the first colour change and a further
7.60 cm3 was needed for the second colour change.

A sketch (not to scale) of the pH curve of the titration is shown below.

8
Acid Base Equilibrium

(ii) State the values for A and B on the graph.

(iii) Calculate the concentration of benzoic acid and 3-nitrophenol in the solution. [8]

11. Maleic acid and fumaric acid with the molecular formula C4H4O4 are two isomeric
alkenedioic acids.

The following table compares the pKa values of the two acids.

acid structure pK1 pK2


H H

maleic C C 1.90 6.07


HOOC COOH

H COOH
23
fumaric C C 3.03 4.44
HOOC H
LL

(a) Suggest a reason why the pK1 value of fumaric acid is lower than its pK2.

(b) Suggest a reason why the pK1 value of maleic acid is lower than the pK1 of fumaric
acid.

(c) Calculate the pH of a 0.15 mol dm−3 solution of fumaric acid (ignore the effect of pK2
on the pH).

(d) 40 cm3 of 0.15 mol dm−3 NaOH is added to 10 cm3 of 0.15 mol dm−3 fumaric acid.
Sketch a graph to illustrate how the pH would vary with volume of NaOH added.

Show how you have obtained the various key points on the graph.

9
Acid Base Equilibrium

[ACJC2011/3/2a,b,c]
12 Oxalic acid is an organic compound with the formula H2C2O4. This colourless solid is a
dicarboxylic acid. In terms of acid strength, it is about 3,000 times stronger than acetic acid. Its
conjugate base, known as oxalate (C2O42−), is a reducing agent as well as a chelating agent for
metal cations.

Oxalic acid dissociates in water according to the following equations

I: HOOC-COOH + H2O HOOC-COO- + H3O+ Ka1 = 5.6 x 10-2 mol dm-3

II: HOOC-COO- + H2O -


OOC-COO- + H3O+ Ka2 = 5.4 x 10-5 mol dm-3

(a) (i) Explain why the value of Ka1 is larger than Ka2.
[1]

(ii) Write expressions for the acid dissociation constants for equation I and II above. [2]

(b) (i) The neutralisation between oxalic acid and sodium hydroxide corresponds to the two
equations given:

HOOC-COOH + NaOH → HOOC-COONa + H2O step 1

HOOC-COONa + NaOH→ NaOOC-COONa + H2O step 2


23
A 25 cm3 sample of 0.100 mol dm-3 of oxalic acid was titrated with 0.100 mol dm-3 of
sodium hydroxide.
LL

The titration curve for the above titration was given below.

pH

25 50 Vol of NaOH/cm3

At the first equivalent point, X, the species formed is HOOC-COO-(aq), which is both
an acid and a base where the relevant equilibriums are:

10
Acid Base Equilibrium

HOOC-COO-(aq) + H2O(l) H3O+(aq) + (COO)22-(aq)

HOOC-COO-(aq) + H2O(l) OH-(aq) + (COOH)2 (aq)

It can be shown that in such an instance the [H3O+] at the first equivalent point, X,
can be given by expression,

[H3O+] = K a1 K a 2

Using the expression, determine the pH value at point X. [1]

(ii) Calculate the pH value at the second equivalence point, Y, given that the [OH-] can
be assumed to be entirely due to the hydrolysis:

-
OOC-COO- + H2O HOOC-COO- + OH- [3]

(c) The concentration of oxalic acid in a solution can be determined by an acid-base titration.
State another way by which the concentration can be determined volumetrically. [1]
23
LL

11
Acid Base Equilibrium

Multiple Choice Questions (Section A)

[ACJC/11/1/12]
1 The following graph shows the change in pH when 30.0 cm3 of 1.0 mol dm-3 solution of
aqueous NH3 is titrated against 1.0 mol dm-3 solution of dilute aqueous H2SO4.
pH

Volume of acid added / cm3


7.50 15.00
23
Given that Kb of NH3 is 1.8 x 10-5 mol dm-3, what is the pH of the solution at point Z?
A 8.86 B 9.26 C 9.79 D 11.6
LL

[AJC/11/1/13]
2 At body temperature of 37 oC, Kw has a value of 2.4 x 10–14.

What is the concentration of OH– if the pH of blood is 7.4 under these conditions?

A 7.00 x 10–7

B 6.03 x 10–7

C 2.51 x 10–7

D 3.98 x 10–8

[CJC/11/1/12]

12
Acid Base Equilibrium

3 The pH of normal human blood is 7.4. Strenuous exercise can cause the condition called
acidosis in which the pH falls. If the pH drops to 6.8, death may occur.

How many times greater is the hydrogen ion concentration in blood at pH 6.8 compared to
that at pH 7.4?

A 0.3

B 0.6

C 1.1

D 4.0

[IJC/11/1/12]
4 The following graph shows the pH changes when a 0.10 mol dm-3 solution of J is added to
20.0 cm3 of 0.10 mol dm-3 of sulfuric acid.

pH
13 Z
23
7 Y
LL

X
1
Volume of J / cm3
20 40 60

Which of the following statements regarding the titration is correct?

A J is a weak monoprotic base.

B There are no buffer regions in this graph.

C Phenolphthalein is a suitable indicator for this titration, but not methyl orange.

D At point Y, the salt at end point is hydrolysed to give an acidic solution.

[MJC/11/1/9]

13
Acid Base Equilibrium

5 At 298 K, the numerical values for the dissociation constant of the aliphatic carboxylic
acids, RCO2H and R’CO2H in aqueous solution are 2.1 x 10-8 and 2.2 x 10-4 respectively.

Which of the following can be inferred from the given information?

A The volume of 1.0 mol dm-3 NaOH used to neutralize RCO2H completely is lower
than that for R’CO2H.

B The volume of 1.0 mol dm-3 HCl used to neutralize RCO2-Na+ completely is lower
than that for R’CO2-Na+.

C RCO2H is a stronger acid than R’CO2H.

D RCO2- is a stronger base than R’CO2-.

[MJC/11/1/10]
6 20.0 cm3 of 0.10 mol dm−3 sulfuric acid was mixed with 22.0 cm3 of 0.12 mol dm−3
aqueous ammonia. What is the pH of the resulting solution?

A 1.49 C 3.17
23
B 2.87 D 13.08

[MJC/11/1/11]
LL

7 The value of the ionic product of water, Kw varies with temperature.

Temperature / oC Kw / mol2 dm-6

25 1.0 x 10-14

62 1.0 x 10-13

What can be deduced from this information?

A Water is acidic at 62C.

B The pH of water increases as temperature increases.

C The ionic dissociation of water is an exothermic process.

D [OH-] ions in water increases as temperature increases.

[NYJC/11/1/14 (attempt after proteins lectures)]

14
Acid Base Equilibrium

8 Which of the following solutions is unable to maintain a fairly constant pH when a small
amount of acid is added to it?

A 20 cm3 of 0.1 mol dm‾3 NH3 with 10 cm3 of of 0.1 mol dm‾3 H2SO4

B 20 cm3 of 0.1 mol dm‾3 NaOH with 20 cm3 of of 0.2 mol dm‾3 CH3COOH

C 0.25 mol KHCO3 with 0.30 mol K2CO3

D 2-amino-2-methylethanoic acid

[PJC/11/1/10]
9 Which of the following statements about acids, bases and salts is correct?

A 0.10 mol dm-3 of ethanoic acid has higher pH than 0.10 mol dm-3 of aqueous sodium
ethanoate.
B 0.10 mol dm-3 of aqueous sodium ethanoate has higher pH than 0.10 mol dm-3 of
aqueous ammonium chloride.
23
C 10 cm3 of 0.10 mol dm-3 of hydrochloric acid reacts with 5 cm3 of 0.10 mol dm-3 aqueous
ammonia to produce an acidic buffer.
D 10 cm3 of 0.10 mol dm-3 of ethanoic acid reacts with 5 cm3 of 0.10 mol dm-3 sodium
LL

hydroxide to produce an alkaline buffer.

[SAJC/11/1/15]
10 Calculate the pH of the resulting solution when 0.1 mole of aqueous nitric acid is added to a
1 dm3 of solution containing 1.0 mol dm-3 of propanoic acid and 1.0 mol dm-3 of sodium
propanoate.
(Ka propanoic acid = 1 x 10-5 mol dm-3)

A 5.08

B 5.00

C 4.91

D 4.00

[TJC/11/1/6]

15
Acid Base Equilibrium

11 The table below shows the values of the ionic product of water, Kw, at two different
temperatures.

Temperature / oC Kw / mol2 dm−6


25 1.00  10−14
62 1.00  10−13

Which of the following statements is correct for pure water?

A The ionic dissociation of water is an exothermic process.

B At 62 oC, pH = 14 − pOH.

C At 62 oC, pH < pOH.

D At 62 oC, pH < 7.

[MI/11/1/12]
23
12 Which of the following resultant solutions will have the lowest pH?

A 15 cm3 of 0.100 mol dm-3 KOH added to 15 cm3 of 0.050 mol dm-3 benzoic acid
LL

B 50 cm3 of 0.050 mol dm-3 KOH added to 25 cm3 of 0.100 mol dm-3 benzoic acid

C 25 cm3 of 0.100 mol dm-3 KOH added to 50 cm3 of 0.100 mol dm-3 benzoic acid

D 5 cm3 of 0.100 mol dm-3 KOH added to 50 cm3 of 0.100 mol dm-3 benzoic acid

16
Acid Base Equilibrium

[AJC/12/1/11]
13 Water dissociates into H+ and OH– as shown.

H2O H+ + OH–

At 25 °C, the equilibrium [H+] is 10–7 mol dm–3; [H2O] = 55.6 mol dm–3.

What is the order of increasing numerical value of pH, pKa and pKw for this equilibrium at
this temperature?

smallest largest

A pH pKw pKa

B pH pKa pKw

C pKw pKa pH

D pKa pKw pH

[SAJC/12/1/12]
14 Solutions E, F, G and H contain a strong monobasic acid, a weak monobasic acid HX,
23
a strong base and a salt of HX, but not necessarily in the same order. The
concentration and pH for each solution are shown below.
LL

Solution Concentration /mol dm-3 pH

E 1.0 4.0

F 1.0 9.4

G 0.01 12.0

H 0.01 2.0

Which one of the following statements is false?

A E contains a weak acid while H contains a strong acid.

B Mixing 10 cm3 of E and 50 cm3 of G produces a buffer solution.

C Mixing 10 cm3 of E and 50 cm3 of H produces a buffer solution.

D Mixing 10 cm3 of F and 1 dm3 of H produces an acidic solution.

17
Acid Base Equilibrium

[JJC/12/1/7]
15 The equilibrium constant for the reaction represented by the following equation is smaller
than 1.0. Which of the following gives the correct relative strengths of the acids and bases
in the reaction?

HPO42−(aq) + H2BO3−(aq) ⇌ H2PO4− (aq) + HBO32− (aq)

Acids Bases

H2PO4− > H2BO3− and HBO32− > HPO42−


A

B H2BO3− > H2PO4− and HBO32− > HPO42−

C H2PO4− > HPO42− and HBO32− > H2BO3−

D H2BO3− > HPO42− and H2PO4− > HBO32−

[NYJC/12/1/10]
16 Which of the following correctly lists 0.10 mol dm–3 solutions of HCl, KCl, NH4Cl, KOH and
23
KCN in order of increasing pH?
LL

A HCl, KCl, KCN, NH4Cl, KOH


B HCl, NH4Cl, KCl, KCN, KOH
C HCl, KCl, NH4Cl, KCN, KOH
D KCl, KCN, KOH, HCl, NH4Cl

18
Acid Base Equilibrium

Multiple Choice Questions (Section B)

[MI/11/1/33]
1 Based on the Bronsted-Lowry theory of acids and bases, which of the following is/are true
about conjugate acids and conjugate bases?

1 An aqueous solution of C6 H5O-K+ contains the conjugate base of phenol.

2 A conjugate acid of a weak base has a higher Ka than H2O at room temperature and
pressure.
23
3 Cl- is the conjugate acid of HCl.
LL

[PJC/12/1/33]
2 Which of the following could act as acidic buffers?

1 1:2 mixture of NaOH and C6H5COOH

2 1:2 mixture of HCl and NaHCO3

3 1:2 mixture of HCl and CH3CO2Na

19
Acid Base Equilibrium

[NYJC/11/1/35]

3 At 298 K, the numerical values of the dissociation constants for the carboxylic acids are given
below:

RCOOH = 2.5 x 10‾8


R’COOH = 3.0 x 10‾10

It can be inferred that

1 the pH of 1 mol dm‾3 R’COOH is greater than 1 mol dm‾3 RCOOH.

2 the Kb of RCOO− is smaller than that of R’COO‾ .


23
3 the electrical conductivity of a solution of RCOOH is greater than that of R’COOH of
the same concentration.
LL

[PJC/11/1/35]
4 Tropeolin indicator has a pH range of 7.6 to 9.0.
Its colour is yellow at pH < 7.6 and rose-red at pH > 9.0.
Which of the following statements about Tropeolin indicator are correct?

1 The indicator is yellow when 50 cm3 of 0.20 mol dm-3 of aqueous ammonia is mixed
with 50 cm3 of 0.85 mol dm-3 aqueous ammonium chloride (pKa of NH4+ is 9.25).

2 It is a suitable indicator for titration of ethanoic acid with sodium hydroxide.

3 The indicator is rose-red in a solution of 0.01 mol dm-3 of methylamine


(Kb = 4.37 x 10-4 mol dm-3).

20
Acid Base Equilibrium

[SRJC/12/1/34]
5 Which of the following indicators can be used for the titration between ethylamine and
hydrochloric acid?

Indicator pH transition range

1 Naphtholphthalein 7.3 – 8.7

2 Congo red 3.0 – 5.0

3 Azolitmin 4.5 – 8.3

[YJC/11/1/34]
23
6 The table below shows the values of the ionic product of water, Kw, at two different
temperatures.

Temperature / C Kw / mol2 dm−3


LL

25 1.00 x 10−14
62 1.00 x 10−13

What conclusions can be drawn from the Kw values?

1 At 62 C, pH < 7

2 At 62 C, pH = pOH

3 At 62 C, pH = 14 – pOH

21
Acid Base Equilibrium

[YJC/12/1/33]
7 A 0.100 mol dm–3 propanoic acid (Ka = 1.34 x 10-5 mol dm–3) is titrated against a
0.100 mol dm–3 NaOH solution.

Which indicator is suitable for this titration?

Indicator pH range and colour change

1 Bromothymol blue 6.0 7.6


yellow blue
23
2 Phenolphthalein 8.3 10.0
colourless red
LL

3 Thymolphthalein 9.5 10.6


colourless blue

22
Acid Base Equilibrium

ACID BASE EQUILIBRIUM SUGGESTED SOLUTIONS

Structured/Free Response Questions

1 Calculate pH of the following solutions

a) 1.0 x 10−2 mol dm−3 nitric acid.

pH = − lg (1.0 x 10−2) = 2

b) 1.0 x 10−10 mol dm−3 hydrochloric acid.

from HCl from water


[H+] = (1.0 x 10−10) + (1.0 x 10−7)
= 1.001 x 10−7mol dm−3

pH = − lg (1.001 x 10−7)
= 6.9996
= 7.00

c) 2 x 10−2 mol dm−3 NaOH

pOH = −lg (2 x 10−2)


23
= 1.69

pH = 12.3
LL

d) 2.00 g of Ba(OH)2 dissolved in 750 cm3 of solution.

2.00 1000
[Ba(OH)2 ] = x
171 750
= 0.0156 mol dm−3

[OH-] = 0.0312 mol dm−3 ; pOH = 1.51


pH = 12.5

e) −3
0.10 mol dm HX which is 20% ionized.

HX(aq) H+(aq) + X− (aq)

Initial n/mol 0.1 - -

20 20 20
Change in n/mol – x 0.1 x 0.1 x 0.1
100 100 100

Equilibrium n/mol 0.98 0.02 0.02

23
Acid Base Equilibrium

[H+] = 0.02 mol dm−3


pH = 1.70

f) −3 −10 −3
0.15 mol dm solution of phenol. (Ka of C6H5OH = 1.0 x 10 mol dm )

C6H5OH(aq) C6H5O–(aq) + H+ (aq)

Initial [ ] /mol dm–3 0.15 0 0

Equilibrium [ ] /mol dm–3 0.15 – x x x

[C 6H5O − ][H+ ]
Acid dissociation constant, K a = = 1.0 x 10−10
[C 6H5OH ]
x2
= 1.0 x 10−10
0.15 − x

Assume 0.15 – x  0.15 ([HA]eqm ≈ [HA]initial).

x2
= 1.0 x 10−10
0.15
23
x = 1.5  10 −11 = 3.87 x 10−6 mol dm−3
= [H+]
LL

pH = − lg [H+] = 5.41

g) −3 −4 −3
0.245 mol dm methylamine (CH3NH2). (Kb of methylamine= 5.0 x 10 mol dm )

Let x be the amount of CH3NH2 that accepts H+ from H2O.

CH3NH2 (aq) + H2O(l) CH3NH3+(a + OH− (aq)


q)

Initial [ ] /mol dm– 0.245 0 0


3

Equilibrium [ ] 0.245 – x x x
/mol dm–3
+
[CH3NH 3 ][ OH − ]
Kb =
[CH3NH 2 ]

x2
5.0  10− 4 =
0.245 − x

24
Acid Base Equilibrium

Assume [CH3NH2]eqm ≈ [CH3NH2]initial = 0.245

i.e. x = 5.0  10 −4  0.245 = 0.112 moldm−3 = [OH−]

pOH = 1.95
pH = 12.0

h) Amt of H+ = (25 x 10−3 x 0.01 x 2) + (15 x 10−3 x 0.12)


= 2.30 x 10−3 mol
2.30  10 −3
[H+] =  1000 = 0.0575 mol dm−3
40
pH = 1.24
salt
i) pOH = pKb + lg
base
0.2
= 4.74 + lg
0 .1
= 5.04

pH = 8.96
23
2. N95/III/3
LL

[C 5H 7O 4CO 2 − ][H + ]
(a) Ka =
[C 5H 7O 4CO 2 H ]

Let the H+ = x
[H+] = [C5H7O4CO2−] since only the acid is present and it dissociates to give equal amounts
of H+ and C5H7O4CO2−.

−4
x2
7.4 x 10 =
0.22 − x

Given that x is small, 0.22 – x = 0.22

−4
x2
7.4 x 10 =
0.22
x = 0.0128 mol dm−3

pH = −lg[H+]
= 1.89

(b)(i) C5H7O4CO2− + H+ → C5H7O4CO2H

25
Acid Base Equilibrium

(ii) C5H7O4COOH + OH− → C5H7O4CO2− + H2O

NOTE: full arrows since all H+ and OH− is removed so reaction is complete.

[C 5H 7O 4CO 2 − ]
(c) pH = pKa + lg
[C 5H 7O 4CO 2 H ]
0.3
= 3.13 + lg
0.1
= 3.61

(d) Kw is the ionic product of water. At 25oC, [H+] = [OH−] = 1 x 10−7 mol dm−3. Hence, pH =
−lg[H+], thus pH = 7.

(e) H2O H+ + OH−

When temperature increases, dissociation of water increases as the process is


endothermic. Hence, equilibrium position will shift to the right and [H+] increases. Hence,
pH of water will decrease.

(f) Water is still neutral since the [H+] = [OH−] as the concentrations of both increase to the
same extent.
23
3. J95/III/3 and J98/II/1 modified
LL

(a) A buffer solution resists pH change when small amounts of acid or base is added.
(b) NH3 + H+ → NH4+
NH4+ + OH− → NH3 + H2O

53.5
(c) (i) Amount of NH4+ = Amount of NH4Cl = = 1.00 mol
53.5
Since the mixture is diluted from 400 cm3 to 1.00 dm3 and only water is added, no of
moles of NH4Cl in 400 cm3 and 1 dm3 is the same.

[NH4+] = 1.00  1.00


= 1.00 mol dm−3

15.0
(ii) [NH3] = x 400 = 6.00 mol dm−3
1000

[NH 4 + ]
(d) pOH = pKb +lg
[NH 3 ]
1.00
= 4.78 + lg = 4.00
6.00

pH = 10.0

26
Acid Base Equilibrium

(e) NH4+ + OH− → NH3 + H2O


Amount of NH4+ left = 1 – 0.05 = 0.95 mol
Amount of NH3 = 6 + 0.05 = 6.05 mol

[NH 4 + ]
pOH = pKb +lg
[NH 3 ]
0.95
= 4.78 + lg = 3.98
6.05

pH = 10.02

Change in pH = 0.02

(f) pOH = lg 0.05 = 1.30

pH = 12.7

Change in pH = 12.7 – 7 = 5.7 (note that original pH of water is 7)

(g) pH change when NaOH added to buffer is much smaller since buffer resists changes in
pH when small amounts of acid or base is added. However, water does not have a
23
conjugate acid like NH4+ to remove OH- ions, hence, pH changes is more significant.
LL

4 (i) at maximum buffering capacity, pH = pKa


= –lg(6.3 x 10–8)
= 7.20

(ii) equivalence point is between 7.2 and 12.4

phenolphthalein is a suitable indicator as its working range is between pH 8 – 10,


which coincides with the region of rapid pH change.

(iii) Kb = 10–14 / 6.3 x 10–8


= 1.59 x 10–7 mol dm–3

It is a stronger base because the Kb value of HPO42– is greater that the Ka value of
HPO42– hence it has a greater tendency to accept proton than donate proton.

5. (a) +5

(b) There is a decreasing trend as it is more difficult to remove H+ from


increasingly negatively charged species / anions.

(c) conjugate acid: H2PO4 conjugate base :PO43−

27
Acid Base Equilibrium

(d) 1.00 ×10-14


Kb = -8
= 1.61 × 10−7 mol dm−3
6.20 ×10

(e) (i) A buffer solution is one that resists changes in pH due to the addition of
small amount of acid/H+ or base/OH−.

(ii) H2PO4− + OH− → HPO42− + H2O


System: Acidic buffer
(iii)
pH = −lg(6.20 x 10−8) + lg (½)

= 6.91

(f) [PO43−] = (10/0 x 0.0305) / 25.0 = 0.0122 mol dm−3

(g) 15.0 cm3

(h) (i) Bromocresol green

(ii) blue to green


23
6 (a) pOH = 14 – 11.1 = 2.9 [OH-] = 1.26 x 10-5 mol dm-3 Kb = 1.58 x 10-9 mol dm-3
LL

(b)

✓ The salt produced at equivalence point is the strong conjugate acid of a weak
base, or
✓ The hydrolysis of the salt produces protons,
✓ Hence the equivalence pH is below 7.

(c) Not suitable. The indicator would change color before the equivalence point is
reached
OR the equivalence point pH is out of the working range of phenolphthalein.

(d) (I) 10 cm3.

(II) On the addition of a small amt. of alkali and acid respectively,


CH3 (CH 2 ) 2 NH3+ +OH - → H 2O+CH 3 (CH 2 ) 2 NH 2
CH3 (CH 2 ) 2 NH 2 +H + → H 2O+CH3 (CH 2 ) 2 NH 3+
Therefore the pH of the solution remains relatively constant.

28
Acid Base Equilibrium

7. (a) A buffer solution is one that resists changes in pH when a small amount of
acid or alkali is added.

(b) 7.90  10−7 = [H+][HCO3−]/[H2CO3]

7.90  10−7 = 10−7.4 [HCO3−]/[H2CO3]

[HCO3−]/[H2CO3] = 19.8

(c) HCO3− + RCO2H → H2CO3 + RCO2−

(d) As more H2CO3 is formed, position of equilibrium I shifts left.

This increases [CO2(aq)], and in turn causes the position of equilibrium III to
shift right, i.e. breathing rate increases.

(e) Let the volume of NaOH added be V cm3.

Initial amount of H2CO3 = 0.20 x 25.0 x 10–3 = 5.0 x 10–3 mol

Amt of H2CO3 left after neutralisation = 5.0 x 10–3 – 0.30 V x 10–3

= 0.005 – 3 x 10–4 V mol


23
Amt of HCO3– formed = Amt of NaOH used = 3 x 10–4 V mol

3 x 10–4 V / [0.005 – 3 x 10–4 V] = 19.8  V = 15.9 cm3


LL

8 (i) When a small amount of H+ is added,


CH3CH2COO- + H+ → CH3CH2COOH
H+ is removed as CH3CH2COOH.
When a small amount of OH- is added,
CH3CH2COOH + OH- → CH3CH2COO- + H2O
OH- is removed as CH3CH2COO- and H2O.

(ii) pKa = -lg Ka

[CH 3 CH 2 COO − ][H+ ]


4.87= -lg
[CH 3 CH 2COOH ]

[CH 3 CH 2COO − ][H+ ]


=1.35 x 10-5
[CH 3 CH 2 COOH ]

[CH 3 CH 2 COO − ] 1.35  10 −5


= = 13.5
[CH 3 CH 2 COOH ] 10 −6

29
Acid Base Equilibrium

Hence, determine the new pH of a 2 dm3 buffer solution containing 0.05 mol dm-3 of
propanoic acid when 0.03 mol of H+ is added.

[CH 3 CH 2 COO − ]
pH = pKa + lg
[CH 3 CH 2 COOH ]

[CH3CH2COO-] = 13.5 x 0.05 = 0.675 mol dm-3


[CH3CH2COO-]new = [(0.675 x 2) - 0.03] / 2.0 = 0.66
[CH3CH2COOH]new = [(0.05 x 2) + 0.03] / 2.0 = 0.065 New pH= 5.88

9 (i) A buffer solution is one which is capable of maintaining a fairly constant pH (by
resisting pH change) when small amounts of acid or base are added to it.

(ii) When a small amount of H+ is added

CH3COO− + H+ → CH3COOH

The H+ is removed by reacting with CH3COO−.

[H+] is slightly changed and pH of buffer solution remains fairly constant.


23
(iii) No. of mole of KOH = 0.00100

In the buffer solution,


LL

No. of mole of CH3COOK = 0.00100


25
No. of mole of CH3COOH left = (0.400 × 1000) - 0.00100

= 0.00900 mol

[salt ]
pH= pKa + lg
[acid ]

0.001
V)
pH = 4.75 + lg( where V is volume of buffer = 3.80
0.009
V
(iv) Potassium hydroxide reacts with carbon dioxide in the air.

30
Acid Base Equilibrium

110a)
[H+] = √Ka.c = √10-8.35  0.1 = 2.11 x 10−6 mol dm−3
pH = –lg[H+] = 4.68

(b) ● Phenolpthalein and bromothymol blue are both weak acids. Small amounts are used so
(i)
that it will not affect the accuracy of the end-point titre value. (If larger amounts are
used, a larger titre value will be obtained)

(ii) ● A: pH = 4.20 ● B: Volume of titre = 7.20 cm3

(iii) (first end-point with bromothymol blue)


● concentration of benzoic acid = (14.40/1000 x 1) / (25.0/1000) = 0.576 mol dm−3
(second end-point with phenolphthalein)
● concentration of 3-nitrophenol = (7.60/1000 x 1) / (25.0/1000) = 0.304 mol dm−3

11 (a) The pK2 of fumaric acid is higher than pK1 because it is more difficult to remove an H+
23
ion from an already negatively charged species formed after the first H+ dissociate.
LL

(b) A stronger acid has a smaller pK value. The conjugate base of maleic acid formed is
further stabilised by the formation of an intramolecular hydrogen bond between the
unionised –COOH group and the –COO– group which are in close proximity to each
other. Thus, pK1 of maleic acid is smaller as it would dissociate to a greater extent due to
stability of the conjugate base.

31
Acid Base Equilibrium

(c) HO2CCH2CO2H ⇌ HO2CCH2CO2− + H+

Initial conc. 0.10 0 0

Eqm conc. 0.10–x x x

Ka = x2 / (0.10–x) = 1.4125 x 10–3

x = 1.1885 x 10–2 mol dm–3

pH = 1.93

pH

4.44

3.03
23
1.93 Volume of
5 15 NaOH /cm3
40
LL

12 (a) (i) HOOC-COOH is a stronger acid than HOOC-COO- , it is more difficult to


remove a H+ from HOOC-COO-, a negative ion.

(ii) [H 3 O + ][HOOC − COO − ] [ H 3 O + ][ − OOC − COO − ]


Ka1= Ka2 = [2]
[HOOC − COOH] [ HOOC − COO − ]

(b) (i) [H3O+] = 5.6 x10 −2 x 5.4 x10 −5 = 1.74 x 10-3

pH =2.76

(ii)

[OH − ][ HOOC − COO − ]


Kb2 = ,
[ − OOC − COO − ]

32
Acid Base Equilibrium

K w [OH − ][ HOOC − COO − ] [OH − ] 2


= =
K a2 [ − OOC − COO − ] [ − OOC − COO − ]

[−OOC-COO−] = 25/1000 x 0.100 (25+50)/1000 = 0.0333 mol dm-3

-10
[OH − ] 2
1.85 x 10 = , [OH-] = 2.48 x 10-6 mol dm-3 , pOH =5.61,
0.0333
pH =8.39

(c) Another method is to titrate an acidified solution of the oxalic acid with standard
acidified potassium manganate (VII) through redox titration.

Multiple Choice Questions (Section A)

1. 2. 3. 4. 5. 6. 7. 8. 9. 10. 11. 12. 13. 14. 15. 16.


B B D B D A D A B C D D A C A B
23
Multiple Choice Questions (Section B)

1. 2. 3. 4. 5. 6. 7.
LL

B A A C C B C

Note: Do not assume that all the given answers are correct. Do discuss with your friends and
check with your tutor should you think that any given answer is incorrect.

33
Solubility Equilibria

SOLUBILITY EQUILIBRIA 8bb


Structured / Free Response Questions

1 [N95/III/3]
A student on a field trip investigates some disused lead workings which have been
flooded for some time. The presence of lead(II) ions in the water is to be demonstrated by
precipitating yellow lead(II) iodide.

(a) Write an expression for the solubility product, Ksp of lead(II) iodide.
[1]

(b) The solubility of lead(II) iodide in water at 15C is 0.46 g dm–3.


For a saturated solution of lead (II) iodide at 15 C, calculate

(i) the concentration, in mol dm–3 of lead(II) ions,

(ii) the concentration, in mol dm–3 of iodide ions,

(ii) the value of Ksp of lead(II) iodide, stating the units. [4]

(c) If equal volumes of the water from the flooded lead workings and 4 x 10–3 mol dm–3
23
potassium iodide are mixed, calculate the minimum concentration, in mol dm–3, of
lead(II) ions which might be detected in the flood water. [1]
(d) There are reasons why this method of detection is not particularly reliable.
LL

(i) Name another metal ion which might also be precipitated by adding aqueous
potassium iodide.

(ii) When an excess of aqueous potassium iodide is added to the precipitate of


lead(II) iodide, it redissolves.
Suggest a reason why this should be so.
[2]

2 [YJC 2011/II/2(c)]
Sparingly soluble chromium(III) compounds like chromium(III) hydroxide are not
considered health hazards, as only limited amount will be absorbed by cells.

A student investigated the solubility of chromium(III) hydroxide by adding solid sodium


hydroxide to a 100 cm3 sample of chromium(III) chloride.

(a) The solubility of chromium(III) hydroxide in water is 1.27 x 10−6 g dm−3 at 25 C.
Calculate the value of the solubility product, Ksp, of chromium(III) hydroxide, stating
the units.

(b) Given that the concentration of the chromium(III) ions in the 100 cm3 sample is
1.0 x 10−5 mol dm−3, calculate the minimum mass of sodium hydroxide added that

1
Solubility Equilibria

will cause chromium(III) hydroxide to be precipitated.

(c) When an excess of sodium hydroxide was added to the precipitate of chromium(III)
hydroxide, it dissolves. Explain why this is so.
[6]

3 [SRJC 2012/II/3(b)
Strontium compounds such as SrF2 and SrSO4 are sparingly soluble in water. Their
solubility products at 298 K are given in the table below:

Strontium compound Numerical value of Ksp

SrF2 2.5 x 10– 9

SrSO4 3.2 x 10– 7

(a) Suggest, using quantitative calculations, which of the two compounds is less
soluble in water at 298 K.
(b) A sample of strontium(II) fluoride is dissolved in a solution of sodium fluoride. Predict
qualitatively the effect, if any, on the solubility and solubility product of strontium(II)
fluoride. [4]
23
4 [modified RI 2011/II/3(c)
LL

(a) The solubility of magnesium hydroxide at 25°C is 1.5 x 10−4 mol dm−3. Calculate the
solubility product, Ksp, of magnesium hydroxide and state the units.

(b) To a 1.0 dm3 saturated solution of Mg(OH)2, 10.0 cm3 of 1.00 mol dm–3 aqueous
NaOH is added. Find the mass of the precipitate formed.
[4]

2
Solubility Equilibria

5 [PJC 2011 /III/4]


Silver and lead form a series of halides of general formula AgX and PbX2. The halides of
these two cations are sparingly soluble in water at 25oC.

Salt Solubility product / Ksp


AgCl 2.00 x 10-10 mol2 dm-6
PbCl2 1.70 x 10-5 mol3 dm-9
PbBr2 6.60 x 10-6 mol3 dm-9

(a) A solution of 100 cm3 of 0.120 mol dm-3 AgNO3(aq) was accidentally mixed with
50 cm3 of 0.120 mol dm-3 Pb(NO3)2(aq). Two methods were suggested to separate
the cations.

(i) In the first method, 0.180 g of solid NaCl was added to the mixture. Show, by
means of calculation, how the two cations can be separated by selective
precipitation.

(ii) The alternative method of separating the two metal ions is to add excess
aqueous NaCl to the mixture in (a) followed by excess NH3(aq) and then
filtration.
23
Construct ionic equations for
(I) the reactions between the mixture in (a) and excess NaCl(aq).
(II) the reaction that takes place when excess NH3(aq) is added.
[7]
LL

(b) When a precipitate is formed, ∆Gppt, in J mol-1, is given by the following expression.
∆Gppt = 2.303 RT log Ksp

(i) Use the data above to calculate ∆Gppt for lead(II) bromide.

(ii) The standard enthalpy change of precipitation of lead(II) bromide is -


279 kJ mol-1. Hence, calculate ∆Sppt, in J mol-1 K-1, for lead(II) bromide a
298K.

(iii) Explain the significance of the sign of your answer in (b)(ii).

(iv) Assuming ∆Sppt does not change with temperature, state the effect of
increasing temperature on ∆Gppt and hence solubility of lead(II) bromide.
[4]

3
Solubility Equilibria

6 [RI2010/II/5 modified] + [MI2008/II/4]

In the analysis of drinking water, the method known as Mohr’s titration is used to determine
the chloride content present. Chloride ions present in a water sample is titrated against
AgNO3 (aq).

As AgNO3 is slowly added, sparingly soluble AgCl precipitate is formed. The indicator used
in the titration is yellow aqueous K2CrO4. The end-point of the titration is obtained when
almost all the chloride present has been precipitated. Addition of AgNO3 beyond this point
causes the formation of a red-brown precipitate of Ag2CrO4.

This method can be used to determine the chloride ion concentration of water samples
from many sources.

(a) Use the data given in the table below, state what you would see at the
beginning of the titration and at the end-point.

colour Ksp (at 25 °C)


AgCl white 2.02 x 10–10 mol2 dm–6
Ag2CrO4 red 3.01 x 10–12 mol3 dm–9
23
Observations

at the beginning of the titration:


LL

….………………………………………………………

at the end-point:

….……………………………………………………….
[2]

(f) At the end point, the concentration of CrO42− is 0.01 mol dm−3. Show that
Ag2CrO4 can only be precipitated when almost all AgCl has been formed.

4
Solubility Equilibria

7 [RI 2012/CT1/C2(b) modified]


A student misplaced the labels of three reagent bottles, each of which contained a different
solution. Each solution contained a different cation, which could be Al3+, Mg2+ or Zn2+. The
concentration of the cation in each solution was the same.

The student randomly labelled the solutions as X, Y and Z. To a small sample of each of
the three solutions, he added NaOH(aq) and measured the amount of precipitate formed in
each sample. The data was then tabulated and a graph was plotted as follows:

Amount of ppt formed / mol

Solution X

Solution Z
23
Solution Y
LL

1 2 3 4 5 6 7 8
Amount of NaOH added (arbitrary units)

(a) Identify, with reasons, the cation in each solution. Write balanced equations, with
state symbols, for the reactions which involve solutions X and Z.

(b) The Ksp of Mg(OH)2 is 1.5 x 10–11 mol3 dm–9. Calculate [Mg2+(aq)] in a saturated
solution of Mg(OH)2.
[8]

5
Solubility Equilibria

Multiple Choice Questions (Section A)

1 [NYJC 2011/I/13]
The numerical value for the solubility product of calcium phosphate, Ca3(PO4)2, is
2.07×10‾33.

Determine the solubility of Ca3(PO4)2.

A 2.28 x 10‾7 mol dm‾3 B 1.14 x 10‾7 mol dm‾3


C 3.42 x 10‾7 mol dm‾3 D 2.91 x 10‾7 mol dm‾3

2 [NYJC 2011/I/12]
Thorium hydroxide, Th(OH)4, is a sparingly soluble salt. Which of the following shows the
correct expression of its molar solubility s (mol dm–3) in terms of its solubility product Ksp?
A s = (Ksp)1/5
B s = (Ksp/4)1/2
C s = (Ksp)1/5/256
D s = (Ksp/256)1/5
23
3 [SAJC 2011/I/16]
The solubility product of a substance is K. If the concentration of the cation in a saturated
LL

solution is found to be , which of the following is most likely to be the substance?

A Cr(OH)3
B Mg3(PO4)2
C Ga2(SO4)3
D PbBr2

[YJC 2011/I/12]
4 Strontium fluoride, SrF2 has a Ksp of 1.0 x 10–9 mol3 dm–9. To a 5.0 x 10–4 mol dm–3 strontium
fluoride solution, an equal volume of a solution was added. Which of these solutions, when
added, will give a precipitate of strontium fluoride?

A 0.002 mol dm–3 NaF


B 0.002 mol dm–3 BaF2
C 0.002 mol dm–3 SrCl2
D 0.002 mol dm–3 SrCrO4
5 [IJC 2011/I/11]
The solubility products of silver chloride and silver carbonate at 25 oC are 2 x 10–10 mol2 dm-6

6
Solubility Equilibria

and 8 x 10–12 mol3 dm-9 respectively.

Which of the following statements is not true?

A When silver nitrate is added into a 1 dm3 solution containing 0.01 mol of chloride
and 0.01 mol of carbonate ions, silver chloride precipitates out first.
B Silver chloride has a lower molar solubility than silver carbonate.
C Addition of hydrochloric acid increases the solubility of silver carbonate.
D Addition of sodium chloride to a solution containing silver chloride decreases the
solubility product of silver chloride.

6 [JJC 2011/I/13]
Which of the following is the least soluble compound?
Compound Solubility product, Ksp
AgBr 5.35 x 10─13
AgI 1.50 x 10─16
Cu(OH)2 2.60 ×10─19
Pb(OH)2 1.43 ×10─20

A AgBr B AgI C Cu(OH)2 D Pb(OH)2


23
7 [DHS 2011/I/7]
When a KI solution is added to a solution of mercury(II) chloride, a precipitate of mercury(II)
LL

iodide is formed. A student plotted the mass of the precipitate formed versus the volume of
potassium iodide solution added and obtained the following graph.
mass of HgI2
formed / g

0
volume of KI added / cm3

What can be a possible explanation for the shape of the graph?


A After a certain amount of KI added, no more precipitate is formed and the
concentration of HgI2 decreases with more KI added.
B Mercury(II) iodide is soluble in excess KI because of the common ion effect.
C Mercury(II) iodide reacts with excess KI to form a soluble complex.
D When more KI is added, the ionic product of the solution becomes less than the
solubility product and more HgI2 dissolves.
8 [PJC 2012/I/14]
A sample of waste solution at pH 1 is found to contain 0.100 mol dm-3 zinc sulfate and

7
Solubility Equilibria

0.100 mol dm-3 lead(II) sulfate. Solid sodium hydroxide is slowly added until the pH is 5.

The solubility product of zinc hydroxide is 3.00 x 10-17 mol3 dm-9 and that of lead(II)
hydroxide is 1.42 x 10-20 mol3 dm-9.

Which statement describes what happens in the solution?

A No precipitate is formed.
B Only lead(II) hydroxide is precipitated.
C Only zinc hydroxide is precipitated.
D Both zinc hydroxide and lead(II) hydroxide are precipitated.

9 [RI 2011/CT1/3]
Which one of the following affects the value of the solubility product, Ksp, of silver sulfide
when it is precipitated by passing hydrogen sulfide into aqueous silver nitrate?

2AgNO3(aq) + H2S(g) ⎯→ Ag2S(s) + 2HNO3(aq)

A The addition of aqueous sodium sulfide


B The presence of an excess acid
23
C The pressure of hydrogen sulfide
D The use of a higher temperature
LL

10 [SRJC 2011/I/12]
Given that the Ksp of Mg(OH)2 is 1.5 × 10−11 mol3 dm−9 and the Kb of aqueous C2H5NH2 is
5.6 × 10−4 mol dm−3, what is the solubility of Mg(OH)2 in 1.0 mol dm−3 aqueous C2H5NH2?

A 1.64 x 10−4 mol dm−3


B 4.78 x 10−5 mol dm−3
C 2.68 x 10−8 mol dm−3
D 6.34 x 10−10 mol dm−3

8
Solubility Equilibria

11 [HCI 2011/I/11]
The numerical value of the solubility product of calcium fluoride is 3.9  10−11.

Given that HF is a weak acid, which diagram shows how the solubility of CaF2 will vary with
pH at constant temperature?

solubility solubility

A B

pH pH

solubility solubility

C D
23
pH pH
LL

12 [ACJC 2011/I/13]
The solubility products of barium carbonate (BaCO3) and barium molybdate (BaMoO4) are
2.58 x 10-9 mol2 dm-6 and 3.54 x 10-8 mol2 dm-6 at 25 °C respectively.

Which of the following statements is false?

A Barium molybdate has a higher solubility than barium carbonate.

B Addition of nitric acid will increase the solubility of barium carbonate.

C Addition of potassium molybdate to a solution containing barium molybdate will


decrease the solubility product of barium molybdate.

D When barium nitrate is added to a solution containing equal concentrations of


carbonate and molybdate ions, barium carbonate is precipitated first.

9
Solubility Equilibria

13 [VJC 2011/I/8]
A 0.100 mol dm−3 solution of lead(II) nitrate is added, with stirring, into an equal volume of
a solution containing a mixture of Cl−, Br−, and I− ions, each with the same concentration of
1.0 x 10−2 mol dm−3.

Given the following data:


Compound Numerical value of solubility product (at 25oC)
Lead(II) chloride 1.7 x 10−5
Lead(II) bromide 6.6 x 10−6
Lead(II) iodide 9.8 x 10−9

Which one of the following statements is correct?


A No precipitate will form.
B Only PbI2 precipitate will form.
C A mixture of PbI2 and PbBr2 precipitates will form.
D All three precipitates, PbI2, PbBr2, and PbCl2, will form.
23
14 [RI 2012/CT1/3]
In which of the following is lead(II) hydroxide, Pb(OH)2, most soluble?
A pure water
LL

B concentrated NH3
C aqueous Pb(NO3)2
D concentrated NaOH

15 [AJC 2011/I/14]
A solution contains 0.03 mol dm–3 Fe2+ and 0.02 mol dm–3 Ag+ at 25 oC. The Fe2+
and Ag+ can be separated by slowly adding Na2CO3 solid. The numerical value of
the solubility product of FeCO3 at 25 oC is 2.1  10–11 and that of Ag2CO3 is
8.1  10–12.

At which point would the concentration of Fe 2+ in the solution be half that of its
original concentration?
A after the first trace of FeCO3 and before the first trace of Ag2CO3
B after the first trace of Ag2CO3 and before the first trace of FeCO3
C at the first trace of FeCO3
D at the first trace of Ag2CO3

10
Solubility Equilibria

16 [CJC 2011/I/13] [need Gp VII Chemistry]


Which of the following statements explain why silver chloride is soluble in aqueous
ammonia, but silver iodide is not?

A The solubility product of silver iodide is numerically larger than that of silver
chloride.
B The equilibrium constant for AgX(s) + 2 NH3(aq) Ag(NH3)2X(aq) is
numerically larger for X = Cl than that for X = I.
C The lattice energy of silver chloride is numerically larger than that of silver iodide.
D Ammonia is a more powerful ligand that can displace chloride but not iodide ions.

17 [RVHS 2011/1/10] (need concepts from Acid Chlorides and Gp II Chemistry)


The solubility of barium hydroxide in water is least affected by the addition of

A BaSO4 B Br2 C NH3 D CH3COCl

Multiple Choice Questions (Section B)


23
LL

1 [RVHS 2011/I/34]
XY2 and ZY are sparingly soluble salts containing Z+, X2+and Y− ions.
The solubility product for the reaction Ksp of the sparingly soluble salt XY2 varies with
temperature as shown in the diagram below.

Ksp

Which conclusions can be drawn from these information?

1 The reaction is endothermic in the forward direction.


2 Adding Y− to the solution of XY2 decreases the Ksp of XY2.
3 Given two solutions with [X2+] = [Z+], higher concentration of Y− is required to
precipitate X2+ compared to Z+.

11
Solubility Equilibria

2 [PJC 2011/I/34]
The numerical values of the solubility product of calcium carbonate and calcium fluoride are
8.0 x 10-7 and 4.5 x 10-11 respectively at 30ºC.

Which of the following statements are correct?


1 Calcium carbonate is more soluble than calcium fluoride.
2 The solubility product of calcium carbonate decreases in a solution
containing calcium nitrate.
3 Addition of calcium nitrate solution to a solution containing equal
concentration of carbonate and fluoride ions causes calcium carbonate to
precipitate out first.
23
3 [NYJC 2012/I/34]
The numerical values of the solubility product of strontium carbonate and strontium fluoride
are 8.7  10−9 and 4.0  10−11 respectively at 25 oC.
LL

Which of the following statements are true?


1 Addition of potassium fluoride to a solution containing strontium fluoride does not
affect the solubility product of strontium fluoride.
2 Addition of potassium fluoride to a saturated solution of strontium fluoride
decreases the solubility of strontium fluoride.
3 Strontium fluoride has a higher solubility than strontium carbonate.

4 [SAJC 2012/1/33]
When a few drops of calcium nitrate solution is added to a saturated calcium hydroxide
solution, a white precipitate is observed. Which of the following provides the correct
explanation?

1 [Ca2+] and [OH−] are high enough such that the ionic product exceeds the Ksp.
2 The precipitate is formed because of the common ion, NO3− , and its effect on the
equilibrium.
3 The Ksp has decreased due to the addition of calcium nitrate.

12
Solubility Equilibria

5 [R 2012/CT1/12]
In a saturated aqueous solution of lead(II) chromate(VI), the following equilibrium exists.

PbCrO4(s) Pb2+(aq) + CrO42–(aq)

Given the solubility product of PbCrO4 at two different temperatures,

temperature / oC Ksp / mol2 dm–6


15 1.7 x 10–14
25 2.8 x 10–13

which of the following statements are correct?


23
1 The addition of Pb(NO3)2(s) to a saturated solution of PbCrO4 will result in the
formation of a yellow precipitate.
2 The precipitation of PbCrO4 from its aqueous solution releases heat.
LL

3 The solubility of PbCrO4 at 15 oC is 1.3 x 10–7 mol dm–3.

13
Solubility Equilibria

SOLUBILITY EQULIBRIA SUGGESTED SOLUTIONS

Structured/Free Response Questions


1 (a) Ksp = [Pb2+][I−]2

(b) (i) [Pb2+] = 0.46 / 461 = 1.0 x 10–3 mol dm–3 (2 sfg and not 3 sfg)

(ii) [I–] = 2 x [Pb2+] = 2.0 x 10–3 mol dm–3 (2 sfg and not 3 sfg)

(iii) Ksp = (1.0 x 10–3)( 2.0 x 10–3)2 = 4.0 x 10–9 mol3 dm–9

(c) Let [Pb2+]in flood water be x mol dm–3


 [Pb2+] in mixture = (x/2) mol dm–3
(Note: concentration is diluted by half when mixed with equal volume of water)
i.e. [I–] in mixture = 2 x 10–3 mol dm–3

Using IP = Ksp
(x/2)(2 x 10-3)2 = 4.0 x 10–9
x = 2 x 10–3 mol dm–3 (1sfg)
[Pb2+]in flood water is 2 x 10–3 mol dm–3 [1sfg]
23
(d) (i) Silver or copper

(ii) Complex [PbI4]2– ion is formed which is soluble.


LL

Pb2+ (aq) + 4I– (aq) [PbI4]2– (aq)


Eqm for PbI2(s) Pb2+ (aq) + 2I– (aq) will shift right due to a decrease in
2+
[Pb ].

2 (a) 1.27  10 −6 1.27  10 −6


solubility of Cr(OH)3 = = = 1.23  10 −8 moldm −3
52.0 + 3(16.0 + 1.0) 103.0

As Cr(OH)3(s) Cr3+(aq) + 3OH−(aq)


K sp = [Cr 3+ ][OH − ]3 = (1.23  10 −8 )  (3  1.23  10 −8 ) 3 = 6.24  10 −31 mol 4 dm −12

(b) For precipitation, ionic product > Ksp


 (1.0  10 −5 )  [OH − ]3  6.24  10 −31
 [OH − ]  3.97  10 −9 moldm−3
 n NaOH = 3.97  10 −9  100  10 −3 = 3.97  10 −10 mol
 m NaOH = 3.97  10 −10  (23.0 + 16.0 + 1.0) = 3.97  10 −10  40.0 = 1.59  10 −8 g

14
Solubility Equilibria

(c) When an excess of NaOH is added, the complex [Cr(OH)6]3− is formed.


Or
Cr3+(aq) + 6OH−(aq) [Cr(OH)6]3−(aq)
3+
Hence [Cr ] decreases, causing the equilibrium as shown below to shift right,
Cr(OH)3(s) Cr3+(aq) + 3OH−(aq)
 Cr(OH)3(s) dissolves

3 (a) SrF2 (s) Sr2+ (aq) + 2F- (aq)


Let the solubility of SrF2 be s mol dm−3.
Ksp(SrF2) = [Sr2+][F-]2
2.5 x 10-9 = (s)(2s)2
s = 8.55 x 10-4 mol dm-3

SrSO4 (s) Sr2+ (aq) + SO42- (aq)


Ksp(SrSO4) = [Sr ][SO42-]
2+

3.2 x 10-7 = (s)2


s = 5.66 x 10-4 mol dm-3
Based on the calculated solubilities, SrSO4 is less soluble than SrF2.
23
(b) SrF2 (s) Sr2+ (aq) + 2F- (aq)
NaF (aq) → Na+ (aq) + F- (aq)
There will be common ion effect due to the increase in [F-].
By Le Chatelier’s Principle, position of equilibrium will shift to the left to
LL

decrease [F-]. The solubility of SrF2 is reduced.


The solubility product of SrF2 is not affected as it is only dependent on temperature.

4 (a) Mg(OH)2(s) ⇌ Mg2+(aq) + 2OH–(aq)

Ksp = [Mg2+][OH–]2 = (1.5  10–4)(2  1.5  10–4)2 = 1.4 x 10–11 mol3 dm–9 (2sfg)
(b) Let the solubility of Mg(OH)2 in NaOH be s mol dm–3.

[OH–] from NaOH = (10 / 1000  1.00) / 1.010 = 0.00990 mol dm–3

Hence, Ksp = s(2s + 0.00990)2

Since NaOH is a strong base, the contribution to [OH–] from Mg(OH)2, a weak base,
is negligible.

s = [Mg2+] = 1.37  10–7 mol dm–3

amt of Mg2+ in the ppt


= (1.5  10–4 – 1.37  10–7)  1.01
= 1.499  10–4 mol

15
Solubility Equilibria

mass of Mg(OH)2 precipitated = (1.499  10–4) [24.3 + 2(17.0)]


= 8.74  10–3 g

5(a) (i) No. of moles of NaCl added = 0.180 / (23.0+35.5)


= 3.077 x 10-3 mol

[NaCl]after dissolving = 3.077 x 10-3 / (150/1000)


= 0.0205 mol dm-3

[Ag+(aq)]after mixing = no. of moles of Ag+(aq) before mixing  total volume


= (100/1000 x 0.120)  (150/1000)
= 0.0800 mol dm-3

[Pb2+(aq)]after mixing = no. of moles of Pb2+(aq) before mixing  total volume


= (50/1000 x 0.120)  (150/1000)
= 0.0400 mol dm-3

Ionic product of AgCl = [Ag+(aq)]after mixing [Cl-(aq)]after dissolving


= 0.0800 x 0.0205
= 1.64 x 10-3 mol2 dm-6 > Ksp(AgCl)
23
Hence, AgCl(s) will form

Ionic product of PbCl2 = [Pb2+(aq)]after mixing [Cl-(aq)]2after dissolving


LL

= 0.0400 x (0.0205)2
= 1.68 x 10-5 mol3 dm-9 < Ksp(PbCl2)
Hence, no PbCl2(s) will form.

(ii) (I) Ag+(aq) + Cl-(aq) ⎯→ AgCl(s)


Pb2+(aq) + 2Cl-(aq) ⎯→ PbCl2(s)

(II) AgCl(s) + 2NH3(aq) ⎯→ [Ag(NH3)2]+(aq) + Cl-(aq)

(b) (i) ∆Gppt(PbBr2) = 2.303 RT log Ksp(PbBr2)


= 2.303 x 8.31 x (273+25) log (6.60 x 10-6)
= - 29544
= -29500 J mol-1

(ii) ∆Gppt = ∆Hppt - T∆Sppt


- 29544 = (- 279 x 103) – (298) ∆Sppt
∆Sppt = - 837 J K-1 mol-1

(iii) Since ∆Sppt <0, the formation of PbBr2(s) results a decrease in entropy of the
system, i.e. from a more disordered aqueous solution to a less disordered solid
formed.

16
Solubility Equilibria

(iv) Increasing temperature causes magnitude of T∆S to rise above magnitude of ∆H.
This causes ∆G to become positive. Precipitation is non-spontaneous, indicating that
solubility of PbBr2 increases with increasing temperature.

6 (a) At beginning of titration: white ppt


At end point: red ppt;

(b) Method 1: When all AgCl is precipitated, [Ag+] = 1.42 x 10−5 mol dm−3
At this point, [Ag+]2[CrO42−] = (1.42 x 10−5)2 x 0.01 = 2.02 x 10−12
which is just less than Ksp for Ag2CrO4 (3.01 x 10−12)
Method 2:

Indicator CrO42- is only added when almost all AgCl is precipitated. Hence, when first trace of red
ppt of Ag2CrO4 start to appear, it is when ionic product [Ag+]2[CrO42-] is equal to Ksp(or rather, just
begin to exceed Ksp)

But for simplicity, we shall just equate ionic product [Ag+]2[CrO42-] to Ksp of Ag2CrO4

3.10  10 −12
[Ag+] when Ag2CrO4 starts to precipitate = = 1.73x10−5 mol dm−3
0.01
2.02  10−10
[Cl−] when Ag2CrO4 starts to precipitate = = 1.16x10−5 mol dm−3.
1.73  10−5
23
This value is very small, hence most of AgCl would have ppted out by the time Ag2CrO4 starts to
ppt out.
LL

7 (a) For Y and Z, there is a max amt of ppt, followed by a decrease till zero. This indicates the
ppt is soluble in excess reagent thus the ppt falls to zero.

Solution X contains Mg2+ because the ppt remains undissolved in excess NaOH. Hence,
graph X remains constant after max amt of ppt is formed.

Mg2+(aq) + 2OH–(aq) → Mg(OH)2(s)

Solution Z contains Al3+ because the amount of NaOH required to precipitate the ion is 1.5
times of that needed to precipitate solution X and solution Y.

Al3+(aq) + 3OH–(aq) → Al(OH)3(s)


Al(OH)3(s) + OH–(aq) → [Al(OH)4]–(aq)

(both coefficients in front of OH- are 2 for Mg2+ and Zn2+, but coefficient in front of OH- for
Al3+ is 3. Hence, 1.5 times more OH- is needed for Al3+. For graph Z, we can see that 6
units of NaOH is required to form max amt of ppt as compared to 4 units for both X and Y.
Hence, 6/4 = 1.5 times.)

17
Solubility Equilibria

Solution Y contains Zn2+ because the amount of NaOH required to precipitate the ion and
to dissolve the precipitate are equal.
Zn2+ (aq) + 2OH–(aq) → Zn(OH)2(s)
Zn(OH)2(s) + 2 OH–(aq) → [Zn(OH)4]–(aq)

(both coefficients in front of OH- are 2, hence equal amounts are used, this is indicated by
4 units to form the max amt of ppt for solution Y and another 4 units for amount of ppt to
fall to zero)

(b) Let the solubility of Mg(OH)2 be s mol dm−3.


Mg(OH)2(s) Mg2+(aq) + 2OH−(aq)
n −3
Eqm conc / mol dm s 2s

Ksp of Mg(OH)2 = [Mg2+(aq)][OH−(aq)]2 = (s)(2s)2


= 4s3
= 1.5 x 10−11
1.5 x 10−11
s= 3 = [Mg2+(aq)] = 1.55 x 10–4 mol dm–3
4

Multiple Choice Questions (Section A)


23
1. 2. 3. 4. 5. 6. 7. 8. 9. 10. 11. 12. 13. 14. 15. 16. 17.
B D C B D B C B D C B C B D A B A
LL

Multiple Choice Questions (Section B)


1. 2. 3. 4. 5.
D D A D A

18
The Periodic Table

THE PERIODIC TABLE 8


Multiple Choice Questions (Section A)
1 [ACJC 2010/I/6]

2 [CJC 2010/I/17]
23
3 [MJC 2010/I/15]
LL

1
The Periodic Table

4 [HCI 2010/I/12]

23
LL

5 [MJC 2010/I/16]

2
The Periodic Table

6 [AJC 2010/I/18]

23
7 [NYJC 2010/I/17]
LL

8 [PJC 2010/I/14]

3
The Periodic Table

9 [ACJC 2010/I/17]

23
LL

10 [CJC 2011/I/17]

4
The Periodic Table

11 [ACJC 2011/I/7]

23
LL

12 [DHS 2011/I/11]

5
The Periodic Table

13 [HCI 2011/I/13]

23
14 [MJC 2011/I/27]
LL

6
The Periodic Table

15 [TJC 2011/I/13]

23
LL

7
The Periodic Table

16 [MJC H1 2009/I/8]

17 [Nov 2009/I/7]

23
LL

8
The Periodic Table

18 [Nov H1 2009/I/15]

19 [YJC H1 2009/I/17]

23
LL

9
The Periodic Table

20 [RI2011 Lecture test]


The graph below shows the variation in molar enthalpy change of vapourisation, Hv for 8
consecutive elements in the Periodic Table, all with atomic number Z  20.

What can be deduced from the above?


A Element F exists as diatomic molecules.
B Element G forms an oxide which is acidic in aqueous solution.
C Element D is in the same group as nitrogen in the Periodic Table.
D Element A forms a chloride which exists as a dimer in the vapour phase.

Multiple Choice Questions (Section B)


23
LL

1 [CJC H1 2009/I/28]

2 [ACJC 2009/I/36]

10
The Periodic Table

3 [ACJC H1 2009/I/28]

4 [VJC 2009/I/36]
23
LL

11
The Periodic Table

5 [YJC 2011/I/35]

Structured / Free Response Questions

1 [AJC 2011/III/4(d)]
Magnesium chloride, MgCl2, can be contained in supplements used to treat magnesium
deficiency in the body. It dissolves in the water in the blood, which is maintained at a normal
pH of 7.4, and magnesium is absorbed by the body in ionic form.

(i) Describe the reaction of magnesium chloride with water and write equations where
appropriate.

(ii) Phosphorus can be contained in supplements to treat phosphorus deficiency in the


23
body as well.

Explain, with the use of an equation, whether phosphorous(V) chloride is likely to be


contained in such supplements.
LL

[4]

2 [CJC 2011/III/4(a)]
The elements Y and Z are in Period 3 and can be one of the following: Mg, Al, Si and P.
The following graph depicts the variation in first ionization energy (I.E.) of the elements
across Period 3.

Element
Y

(i) Based on the above data, identify element Y. Explain why the first ionization energy of Y
is lower than that of the element preceding it.

12
The Periodic Table

When NaOH is added to a solution containing ions of Y, a white precipitate is formed initially.
When an excess of NaOH is added, the precipitate dissolves to give a colorless solution.
The oxide of Y reacts with both aqueous hydrochloric acid and sodium hydroxide.

Element Z has a chloride and an oxide and both react vigorously with water to form a
solution of pH = 2.

(ii) By identifying the white precipitate, construct ionic equations for the formation of this
precipitate and its subsequent dissolving in NaOH.

(iii) State the identity the element Z and hence give the equation for the reaction between
the chloride of Z and water.
[7]

3 [DHS 2011/II/4(a)-(c)]
Group IV elements progress towards metallic behaviour down the group. The table below
shows selected data for the dioxides from carbon to germanium.

compound formula melting point/ oC acid–base nature

carbon dioxide CO2 –78 acidic


silicon dioxide SiO2 1600 weakly acidic
23
germanium dioxide GeO2 1200

(a) State the valence shell electronic configuration of these elements, and their oxidation
LL

state in the above dioxides.


[2]

(b) Explain the difference in melting points between carbon dioxide and silicon dioxide, with
respect to chemical bonding and structure.
[3]

(c) Using the concept of diagonal relationship as well as information from the trend above,
suggest the acid–base nature of germanium dioxide by completing the table above.
[1]

4 [RI 2011/III/4(b),(d)]
(a) The oxides, MgO, Al2O3 and SiO2, are all used as refractory materials due to their high
melting points. If a sample of one of the oxides was provided as a white powder,
describe the reactions you could carry out on the powder to determine which of the
three oxides it was.
[3]

(b) Compounds of beryllium and aluminium share certain chemical properties. Beryllium
salts, probably from rocks such as feldspars, may sometimes be found in drinking
water.
An aqueous solution of beryllium sulfate is known to irritate the skin. With the aid of
relevant equation(s), explain why that is so. [2]

13
The Periodic Table

5 [MI 2011/III/5(c)]
(a) Describe the reactions of oxides of Period 3 elements (sodium to silicon) with water,
stating their pH values, with reference to their structure and bonding.
[3]

(b) Describe the difference in melting of sodium oxide and oxides of sulphur with reference
to their structure and bonding.
[2]

6 [SRJC 2011/II/5(a)(b)]
Elements E and F are period 3 elements.

(a) E is a white solid with melting point of 317 K. When exposed to air, it reacts
spontaneously, producing a white powder of empirical formula E2O5.
E2O5 readily undergoes neutralisation with aqueous NaOH to form salt and water.

(i) From the observations above, deduce the identity of E.

(ii) Based on your deduced identity of E, describe the above reactions using only equations
with state symbols.

(iii) With the aid of an equation, suggest and account for a pH value for the resultant
solution when the oxide of E dissolves in water.
23
[6]
(b) Element F forms an amphoteric oxide readily in air. Using only equations with state
symbols, explain what is meant by the word in bold.
[2]
LL

7 [Modified TJC 2011/III/3(a)(c)]


(a) State and explain the pH and the colour observed when some universal indicator is
added to the solution formed from AlCl3 dissolving in water.
Write equations where appropriate.
[3]

(b) When liquid SiCl4 is added to water, a white solid of SiO2 is immediately formed.

(i) Explain why SiCl4 can be hydrolysed by limited amount of water to form SiO2.

(ii) The standard entropy change of the above reaction is positive. Write a balanced
equation for the above reaction, with state symbols, and hence explain why the
standard entropy change is positive.

(iii) Unlike the reaction with water, when liquid SiCl4 is reacted with excess methanol,
SiO2 is not formed. Write a balanced equation for this reaction and describe what
you may observe.
[5]

14
The Periodic Table

8 [Modified VJC 2011/III/2(a)(b)]


(a) The elements in Period 3 range from metals on the left of the Periodic Table to non-
metals on the right. By describing one physical property of the element and one
chemical property of its chloride, explain why phosphorus can be regarded as a non-
metal. Write equation(s) where relevant.
[2]

(b) (i) The element aluminium and its compounds have some properties characteristic
of metals, and some of non-metals. Aluminium hydroxide, for example, is known
to be amphoteric.
Explain the meaning of the word in italics.

Aluminium sulfate and calcium oxide are sometimes added to water supplies to co-
precipitate suspended solids and bacteria. A small amount of aluminium-containing ions
remains in solution and its presence in drinking water may contribute to the mental
illness known as Alzheimer’s disease.

(ii) Write a balanced equation for the reaction that occurs when aluminium sulfate
and calcium oxide is added to water, given that aluminium hydroxide is one of the
products formed.

(iii) Explain why adding too much calcium oxide would increase the probability of
23
contracting Alzheimer’s disease. Write equations for all reactions that occur.
[4]

9 [ACJC 2008/III/3(c)]
LL

(a) (i) X and Y are Period III elements.


Element X forms a white oxide that is slightly soluble in cold water. Its chloride
dissolves in water to form a weakly acidic solution.
Element Y forms two oxides. 0.03 mol of one of the two oxides produces 6.99 g
of white precipitate when shaken with excess barium chloride solution. A solution
containing 0.05 mol of the oxide of Element Y forms a neutral solution when the
same amount of the oxide of the Element X is added to it.
Identify the element X, Y and the oxide of Y in the above reaction.
Explain the observations with the help of relevant equations.

(ii) Compare and explain the difference in the melting points of elements X and Y.
[8]
10 [MJC 2009/II/2(c)]
Tellurium resembles silicon in many aspects. For example, both are metalloids and are used
as semiconductors. Tellurium (IV) oxide, TeO2, reacts with concentrated strong bases in the
same way as silicon (IV) oxide, SiO2.

(i) Write a balanced equation for the reaction between TeO2 and NaOH

(ii) Predict the pH of the solution formed when gaseous tellurium tetrachloride, TeCl4, is
bubbled into water. Suggest an equation for this reaction.

15
The Periodic Table

THE PERIODIC TABLE SUGGESTED SOLUTIONS

Multiple Choice Questions (Section A)


1. 2. 3. 4. 5. 6. 7. 8. 9. 10. 11. 12. 13. 14. 15. 16. 17. 18. 19. 20.
B C D A C D D B C D B C A C C D B B A D

Multiple Choice Questions (Section B)


1. 2. 3. 4. 5.
D B D C B

Structured/Free Response Questions

1 (i) MgCl2(s) + 6H2O(l) [Mg(H2O)6]2+(aq) + 2Cl–(aq)


Mg2+ ions are hydrated to [Mg(H2O)6]2+.

[Mg(H2O)6]2+(aq) + H2O(l) [Mg(H2O)5(OH)]+(aq) + H3O+(aq)


2+
Mg has a high charge and small radius, hence, high charge density. It slightly
polarises the O–H bond in water molecules, which is weakened and easily broken to
form a slightly acidic solution (pH  6.5).

(ii) PCl5(s) + 4H2O(l) H3PO4(aq) + 5HCl(aq)


OR PCl5(s) + H2O(l) POCl3(aq) + 2HCl(aq)
23
Not likely, since a strongly acidic solution is formed when PCl5 reacts with water in
the body and the excessive H+ formed may cause gastric problem.
LL

2 (i) Al. The first I.E. is lower since it involves removal of an electron from the 3p orbital
which is of a higher energy or further from nucleus compared to the 3s electron
of Mg.

(ii) Al(OH)3
Al3+ + 3OH- → Al(OH)3 Al(OH)3 + OH- → [Al(OH)4]-

(iii) P
PCl5 + 4H2O → H3PO4 + 5HCl
OR
PCl3 + 3H2O → H3PO3 + 3HCl

3 (a) Valance shell electronic configuration: ns2np2


In the dioxides, the oxidation state is +4.

(b) CO2 has simple covalent structure with weak instantaneous dipole induced
dipole forces (between molecules). SiO2 has giant covalent structure with
strong covalent bonds. Thus a greater amount of energy is required to break the
strong covalent bonds in SiO2

(c) amphoteric

16
The Periodic Table

4 (a) First, add dilute HCl to the sample. If the solid is insoluble in acid, it must be the
acidic oxide SiO2

If the solid dissolves in acid, add NaOH(aq). If the solid is insoluble in NaOH(aq), it
must be the basic oxide MgO. If the solid dissolves in both acid and base, it must
be the amphoteric oxide Al2O3

(b) Solution of beryllium sulfate is acidic (pH  3) thus causing skin irritations.

[Be(H2O)4]2+ + H2O ⇌ [Be(OH)(H2O)3]+ + H3O+

The small, highly polarising beryllium cation weakens the O–H bonds of the water
molecules in its surrounding sphere of coordination and results in the release of
hydrogen ions in solution.

5 (i) Na2O + H2O → 2NaOH pH= 13

MgO + H2O ⇌ Mg(OH)2 pH = 9

MgO dissolves sparingly in water because of its highly exothermic lattice


energy.

Al2O3 and SiO2 do not react with water because the energy evolved from solute-
23
solvent interactions cannot overcome the highly exothermic lattice energy of Al2O3
and the extensive strong covalent bonds in SiO2. pH = 7.

(ii) Sodium oxide is an ionic oxide with strong electrostatic forces of attraction
LL

between the oppositely charged Na+ and O2- ions. Hence, a lot of energy is
required to overcome the strong forces of attraction resulting in its high melting
point. ;

Oxides of sulphur, e.g. SO3 are simple covalent molecules with weak
instantaneous dipole-induced dipole forces between molecules. Hence, less
energy is required to overcome the weaker forces of attraction between the
molecules resulting in its low melting point. ;
(NOTE: SO2 is polar and will have permanent dipole-permanent dipole forces
between molecules).

6 (a) Low melting point hence E must be of simple covalent structure: sulfur or
(i) phosphorus

The oxide of E is a white powder with empirical formula E2O5 hence must be
phosphorus as sulfur is yellow and does not have empirical formula of E2O5

(ii) P4 (s) + 5 O2 (g) P4O10 (s)


P4O10 (s) +12 NaOH(aq) 4 Na3PO4(aq) + 6 H2O(l)

(iii) P4O10(s) + 6H2O(l) 4 H3PO4(aq)


Resulting pH: 2−3

17
The Periodic Table

(b) Al2O3(s) + 6 HCl(aq)  2 AlCl3(aq) + 3 H2O (l)


Al2O3(s) + 2 NaOH(aq) + 3 H2O(l)  2 Na[Al(OH)4] (aq)
Reacts with both acid and base.

7 (a) Solution formed is red (pH ~ 3).


AlCl3 undergoes both hydration and hydrolysis as Al3+ is highly charged and has
a small radius, thus has a high charge density and a high polarizing power. It
draws electrons away from its surrounding water molecules and weakens the O-H
bond.
AlCl3(s) + 6H2O(l)  [Al(H2O)6]3+(aq) + 3Cl-(aq)
[Al(H2O)6]3+(aq) + H2O  [Al(H2O)5OH]2+(aq) + H3O+(aq)

(b) SiCl4 is a simple covalent chloride that undergoes hydrolysis in water. Si has
(i) empty, “low-lying” 3d-orbitals to accept the lone pair of electrons from water.
This facilitates the hydrolysis reaction available for dative bonding with water.

(ii) SiCl4(l) + 2H2O(l) → SiO2(s) + 4HCl(g)


The standard entropy change is positive because 4 moles of gas is being produced
in the reaction, increasing the degree of randomness / disorderliness of the
system.
23
(iii) SiCl4 + 4CH3OH → Si(OCH3)4 + 4HCl
Steamy white fumes will be observed.
LL

8 (a) Physical property of phosphorus:

Phosphorus does not conduct electricity due to absence of delocalised


electrons in its simple covalent structure.
(OR It has low melting point due to weak dispersion forces to be overcome in
its simple molecular solid structure.)

Chemical property of PCl3 or PCl5:

It undergoes complete hydrolysis to form an acidic solution due to its covalent


nature.

PCl3 + 3H2O → H3PO3 + 3HCl or PCl5 + 4H2O → H3PO4 + 5HCl

b(i) An amphoteric compound is one which reacts with both acids and bases.

(ii) Al2(SO4)3 + 3CaO + 3H2O  2Al(OH)3 + 3CaSO4

(iii) CaO dissolves to form an alkaline solution of Ca(OH)2.


CaO + H2O  Ca(OH)2

OH- from dissolved Ca(OH)2 reacts with Al(OH)3 to form soluble Al(OH)4-.
Al(OH)3 + OH-  Al(OH)4-

18
The Periodic Table

9 (a) Element X : Magnesium


(i)
White ppt from addition of BaCl2 is most likely BaSO4, since Ba2+ forms ppt with SO42
Element Y: Sulfur
Formula of the oxide of Element Y: SO3
Upon addition of BaCl2 (aq), water from the aq solution reacts with SO3 to form
H2SO4 (aq)
SO42 from full dissociation of H2SO4 then reacts with Ba2+ from BaCl2 to form the
white ppt BaSO4.
6.99g of white ppt correspond to the mass of 0.03 mol of BaSO4

 MgCl2 hydrolyses slightly to form a weakly acidic solution of pH ~ 6.5


MgO(s) + H2O(l) → Mg(OH)2(aq)-----------(i)
MgCl2(s) + 6H2O  [Mg(H2O)6]2+ (aq) + 2Cl– (aq)--------(ii)

[Mg(H2O)6]2+ (aq) ⇌ [Mg(H2O)5(OH)]+ (aq) + H+(aq)--------(iii)

OR
[Mg(H2O)6]2+ (aq) + H2O(l) ⇌[Mg(H2O)5(OH)]+ (aq)+ H3O+(aq)-------(iii)

SO3(g) + H2O(l) → H2SO4(aq)-------(iv)


23
H2SO4(aq) + BaCl2(aq) → BaSO4(s) + 2HCl(aq)--------(v)
White ppt
LL

H2SO4(aq) + MgO(s) → MgSO4(aq) + H2O(l)-------(vi)

(ii) Mg (Element X) has a higher melting point than S8 (Element Y)


Mg has giant metallic structure while S8 has simple covalent structure.
During melting, more energy is required to overcome
the strong metallic bonds between Mg2+ ions and its sea of delocalised electrons
than the weaker instantaneous dipole induced dipole interactions between S8
molecules.

10 (i) TeO2 + 2NaOH → Na2TeO3 + H2O

(ii) pH of solution : About 2.


Equation: TeCl4 + 2H2O → TeO2 + 4HCl

Note:
Do discuss with your friends and check with your tutor should you think that any given
answer is incorrect.

19
The Periodic Table

GROUP 2

Structured / Free Response Questions

1 [MJC 2011/III/1(b)]
Describe, and suggest an explanation for, the trend observed in the thermal decomposition
of nitrates of the Group 2 elements from magnesium to barium. Write a general equation for
the reaction that occurs on heating.
[4]

2 [MI 2011/III/3(b)]
Beryllium is another element in Group 2. However, the temperature required for thermal
decomposition of BeCO3 is much lower than BaCO3. Explain the observation, quoting
relevant data from the data booklet.
[2]

3 [IJC 2011/III/1(ci)]
Group II carbonates decompose in a similar manner as Group 2 nitrates. The following table
shows the temperature at which Group 2 carbonates decompose.
23
Carbonate Decomposition temperature / K
MgCO3 523
CaCO3 1098
LL

SrCO3 1373
BaCO3 1573

Based on the above data, state and explain the trend of thermal stabilities of Group II
carbonates down the group.
[3]

4 [DHS 2011/III/4(c)]
Calcium ethanoate, one of the Group II salts of carboxylic acids, is also commonly used in
the production of propanone.
Predict, with reasoning, if the decomposition temperature of calcium ethanoate will be
higher or lower than that of beryllium ethanoate.
[2]

5 [JJC 2011/III/2(biii)]
Group 1 metal carbonates, like Group 2 metal carbonates, can undergo thermal
decomposition.
Explain why Group 1 metal carbonates generally decompose at a higher temperature than
Group 2 metal carbonates.
[2]

20
The Periodic Table

6 [MJC 2012/III/1(d)] and [RVHS 2011/II/3c]


Group 2 nitrates and sugars undergo decomposition when heated.

(i) Write an equation for the thermal decomposition of Group 2 nitrates.

(ii) Suggest why the numerical value of the entropy change is very similar for the
decomposition reaction of all the Group 2 nitrates.

(iii) Predict, with reasoning, whether magnesium nitrate or barium nitrate has a higher
decomposition temperature.

(iv) How would you expect the decomposition temperature of aluminium nitrate to
compare with that of magnesium nitrate?
Explain your answer using data from the Data Booklet.

7 [CJC 2011/III/2(c)]
Each Group 2 element forms an iodate(V) with the formula M(IO3)2. The iodates(V) have
many uses, for example, calcium iodate is an oxidant added to lotions and ointments as an
antiseptic and deodorant.

(i) Draw the structural formula of the iodate(V) ion, indicating the likely bond angle.
23
(ii) Draw a dot-and-cross diagram corresponding to this structure
LL

(iii) The iodates(V) readily decompose on heating giving oxygen gas as one of the two
products formed.
(I) Write a likely equation for the decomposition of M(IO3)2.
(II) Predict and explain how the ease of thermal decomposition of these iodates(V)
varies down the group. The decomposition of the iodates(V) is similar to the
decomposition of the nitrates(V).
[6]

21
The Periodic Table

8 [Modified NJC 2011/III/1(d)]


(i) Write an equation, with state symbols, for the thermal decomposition of barium
carbonate.

(ii)

MCO3 (s) Test tube


containing X

heat

When the same amount of MgCO3 and BaCO3 was heated as shown in the above
setup, the following data was obtained.
23
Time taken for white precipitate to be observed in X
MgCO3 3 minutes
BaCO3 9 minutes
LL

State the identity of X and explain the above observations.


[5]

9 [MJC 2011/III/1(c)]
In an experiment, water is added to a test tube containing the solid remaining after
magnesium carbonate has been heated. Dilute sulfuric acid is then added to the test tube.
The procedure is repeated to the solid remaining after barium carbonate has been heated.
Describe and explain what you would see in both experiments.
[3]

10 [RI 2009/II/3(c)]
The graph below shows the change in mass when 1.00 g of calcium carbonate was heated
at 900 oC.

22
The Periodic Table

Mass / g
1.00

Decomposition
curve of CaCO3
x

(i) Calculate the value of x.

(ii) On the same axes above, sketch the graph that illustrates the change in mass when
1.00 g of magnesium carbonate was heated at 900 oC until there was no change in
mass.

(iii) Using relevant data from the Data Booklet, compare the thermal stabilities of the two
23
carbonates to account for your sketch in (c)(ii).
[4]
LL

23
The Periodic Table

Multiple Choice Questions (Section A)


1 [HCI 2011/I/14]

2 [IJC 2011/I/16]

23
LL

3 [PJC 2011/I/14]

24
The Periodic Table

23
LL

25
The Periodic Table

4 [SRJC 2011/I/16]

23
LL

5 [RI 2009/I/16]

26
The Periodic Table

Multiple Choice Questions (Section B)

1 [AJC 2011/I/35]

2 [DHS 2011/I/34] 23
LL

3 [MJC 2011/I/35]

4 [NYJC 2012/I/35]

27
The Periodic Table

5 [DHS 2012/I/37]

6 [CJC 2012/I/35] 23
LL

28
The Periodic Table

GROUP 2 SUGGESTED SOLUTIONS

Multiple Choice Questions (Section A)


1. 2. 3. 4. 5.
C A D D B

Multiple Choice Questions (Section B)


1. 2. 3. 4. 5. 6.
A A D B C C

Structured/Free Response Questions

1 Down the group,


 cationic radii increase
 charge density and polarizing power of cations decrease
 less distortion of the electron cloud of NO32- anion
 N-O bond is weakened to a lesser extent
 less energy required to break the N-O bond
 thermal stability of nitrates increases and higher temperature is required to bring about
decomposition.
23
General equation: M(NO3)2(s)  MO(s) + 2NO2(g) + ½ O2(g)
LL

2 Cationic radius: Be2+ : 0.031nm Ba2+ : 0.135nm

Since Be2+ has a much smaller cationic radius than Ba2+, the charge density and
polarizing power of Be2+ will be much higher. Hence, Be2+ will be able to
polarize/distort the electron cloud of CO32- a greater extent, leading a greater extent
of weakening of the C-O bond and a lower temperature required for the thermal
decomposition.

3 The decomposition temperature increases down the group and hence the thermal
stability increases down the group. Down the group, size of cation increases. This result
in a decrease in its charge density and polarising power, hence extent of polarization
of electron cloud of carbonate ion decreases. The C-O bond is weakened to a lesser
extent and more energy is required to break the C-O bond.

4 Decomposition temperature of calcium ethanoate will be higher than that of beryllium


ethanoate.
Calcium is further down the group than beryllium. Calcium ion has a larger ionic radius and
a lower charge density than beryllium ion. Hence, calcium ion has a lower polarising
power than beryllium ion and results in less distortion of the electron cloud of the
ethanoate ion. Hence, calcium ethanoate is more thermally stable than beryllium
ethanoate.

29
The Periodic Table

5 Since Group I metal ions have a lower charge and bigger size than Group II metal ions,
Group I metal ions have a lower charge density and polarizing power than Group II
metal ions. Thus, Group I metal ions will distort the large electron cloud of CO32 to a
smaller extent. Hence, Group I metal carbonates are more thermally stable and require a
higher decomposition temperature.

6 (i) M(NO3)2  MO + 2NO2 + ½O2

(ii) Upon decomposition, each mole of Group II nitrates release the same amount of
gaseous molecules, hence the numerical value of the entropy change is very
similar.

(iii)  Ionic radius of Mg2+ is smaller than Ba2+


 Charge density and hence polarizing power of Mg2+ is higher than Ba2+
 Extent of distortion of electron cloud of NO3– is greater in Mg(NO3)2 than
Ba(NO3)2
 Hence, Ba(NO3)2 has a higher decomposition temperature.

(iv) Ionic radius of Al3+: 0.050 nm; Mg2+: 0.065 nm

Aluminium nitrate has a lower decomposition temperature.


Al3+ has a smaller ionic radius and a larger charge. Thus Al3+ has a higher
charge density and greater polarising power, distorting the electron cloud of
23
NO3 anion to a larger extent.
LL

7 (i)

Bond angle is likely to be 107o.

(ii)
O
xx
xx

xx

IO x
x
x

xx
x

x x
O x
x

x
x

(iii) (I) M(IO3)2(s)  MI2(s) + 3O2(g)


(II) The ease of thermal decomposition decreases down the group.
Down Group II,
 Size of cation increases
 Charge density and hence polarizing power of cation decreases
 Less distortion of electron cloud of IO3– and the I-O bond is weakened to a
lesser extent
 More energy is required to break the I-O bond
 Thermal stability of the iodates(V) increases

30
The Periodic Table

8 (i) BaCO3(s)  BaO(s) + CO2(g)

(ii)  X is Ca(OH)2(aq).
 Mg2+ has a smaller cationic radius and thus a higher polarising power than
Ba2+.
 Electron cloud of CO32 in MgCO3 is polarized/distorted to a greater extent and
hence the C-O bond is weakened more.
 Less time required to break the C-O bond in CO32 anion of MgCO3 using the
same Bunsen flame, thus lesser time required for ppt to be observed.
 CO2(g) produced forms a white ppt of CaCO3(s) in Ca(OH)2(aq).

9 MgO solid did not dissolve in water. MgO solid reacts with dilute sulfuric acid to form a
colourless solution of MgSO4.
BaO solid dissolves in water to form an alkaline solution of Ba(OH)2. When dilute
sulfuric acid is added, white ppt of BaSO4 is seen.

[Note: MgO is a basic oxide and reacts with dilute sulfuric acid.
MgSO4 is soluble in water whereas BaSO4 is insoluble in water.]

10 (i) CaCO3  CaO + CO2


23
X = mass of CaO = 1.00/100.1 x 56.1 = 0.560 g

(ii)
LL

(iii) From Data Booklet

ion Mg2+ Ca2+


Ionic radius 0.065 0.099

The thermal stability depends on the charge density of the cation. The greater the
charge density, the thermally less stable the carbonate.

Since Mg2+ has a smaller ionic radius than Ca2+, Mg2+ has a higher charge
density and polarising power than Ca2+. Hence, the distortion of the electron

31
The Periodic Table

cloud of the carbonate anion and thus the weakening effect of the C-O bond in
MgCO3 occurs to a greater extent.

Hence MgCO3 is less thermally stable than CaCO3 and should decompose to MgO
and CO2 at a faster rate (since its decomposition temperature is lower than that of
CaCO3.) Also, the mass of MgO obtained is 0.478 g, lower than x (i.e. 0.560 g)

Calculation for mass of MgO obtained:


Molar mass of MgCO3 = 24.3 + 12.0 + 3(16.0) = 84.3 g mol–1
Molar mass of MgO = 24.3 + 16.0 = 40.3 g mol–1
Mass of MgO = 1.00 / 84.3 x 40.3 = 0.478 g

Note: Do not assume that all the given answers are correct. Do discuss with your
friends and check with your tutor should you think that any given answer is incorrect.

23
LL

32
The Periodic Table

GROUP 17

Structured / Free Response Questions

1 [VJC 2011/III/2(c)]
Explain each of the following as fully as you can. Justify your answers with relevant data
from the Data Booklet. Write balanced equations, including state symbols, for any reaction
that occurs.

When a hot glass rod is plunged into a gas jar of gaseous HI, some violet vapour is seen.
On repeating the experiment with HBr, no change is observed.
[2]

2 [YJC 2011/III/1(c)]
Under suitable conditions, SCl2 reacts with water to produce a yellow precipitate and a
solution J. Solution J is a mixture of SO2(aq) and compound K.

(i) Suggest the identity of compound K.

(ii) Write a balanced equation, including state symbols, for the reaction between SCl2 and
water.
23
(iii) Describe what would be observed when each of the following is added to separate
samples of solution J. Write balanced equation for any reaction that occurs.
 AgNO3(aq)
LL

 acidified K2Cr2O7(aq)
[6]

33
The Periodic Table

3 [ACJC 2011/II/4(b)]
Iodine is a Group 17 element. On 11 March 2011, a major earthquake struck Japan.
Subsequently it caused a nuclear meltdown and the Japanese were given potassium iodide
to protect their thyroid glands from radioactive iodine released from nuclear accidents.

(i) State the physical state of iodine under room conditions.


[1]
(ii) Iodine is not soluble in water. An aqueous solution of iodine can only be prepared by
dissolving iodine in excess KI(aq) to form a dark reddish brown solution. Identify the
iodine-containing species that is formed in this reaction.
[1]
(iii) Describe what happens when aqueous silver nitrate is added to aqueous sodium
iodide. Write balanced equation(s) for the reaction, if any.
[1]
(iv) What changes (if any) occur when aqueous ammonia is added slowly to mixture in
b(iii) until in excess?
[1]

4 [AJC 2011/III/4(d)]
Magnesium chloride, MgCl2, can be contained in supplements used to treat magnesium
deficiency in the body. It dissolves in the water in the blood, which is maintained at a normal
pH of 7.4, and magnesium is absorbed by the body in ionic form
23
(i) Describe the reaction of magnesium chloride with water and write equations where
appropriate.
LL

(ii) Phosphorous can be contained in supplements to treat phosphorous deficiency in the


body as well.

Explain, with the use of an equation, whether phosphorous(V) chloride is likely to be


contained in such supplements.
[5]

5 [CJC 2011/II/3(b)-(d)]
The preparation of the acid, HBr in the laboratory can be carried out by the reaction
between bromine, sulfur dioxide and water only. The only by-product formed is also a
strong acid.

(a) (i) Write the equation for the above laboratory preparation of HBr.

(ii) When concentrated solutions of the two products formed in the above preparation
reacts with each other, Br2 is regenerated. State and explain the type of reaction that
is occurring.
[3]

34
The Periodic Table

(b) Intra and intermolecular bondings can explain the trends in the thermal stability and boiling
points of the hydrogen halides.
(i) State and explain the trend in the boiling points of hydrogen halides from HF to HI.

(ii) State and explain the trend in the thermal stability of hydrogen halides from
HCl to HI.
[6]

6 [JJC 2011/III/3(a)]
Halogens A and B were reacted with two different samples of aqueous sodium thiosulfate
(Na2S2O3). Excess halogen was removed by some suitable means.
The resulting solution from halogen A was found
 to react with potassium hydroxide
 to form a white precipitate, C, with silver nitrate
 to give a white precipitate, D, when treated with excess aqueous barium nitrate.
The resulting solution from halogen B, however, was found
 to form a yellow precipitate, E, with silver nitrate
 not to form white precipitate when treated with excess aqueous barium nitrate.
23
(i) Identify the precipitates C, D, and E formed.

(ii) Write a balanced equation for the reaction between each halogen and sodium
LL

thiosulfate.

(iii) With the help of the Data Booklet, explain why the two halogens react differently with
aqueous sodium thiosulfate.
[6]

7 [NYJC 2011/III/1(a)]
Cyanogen, (CN)2 is called a pseudohalogen because several of its reactions are similar to
those shown by halogens Cl2, Br2 and I2. For example, it can be reduced to hydrogen
cyanide, which is a weak acid.

½ (CN)2 + H+ + e HCN EӨ = +0.37 V

(i) By analogy with the reactions of halogens or halides ions and by making use of the
above data and information in the Data Booklet, predict the reactions, if any, that
might occur when the following reagents are mixed.

I. Cl2 + HCN
II. Na2S2O3 + (CN)2
III. K2Cr2O7 + (CN)2

(ii) Draw a dot-and-cross diagram of the cyanogen molecule, suggesting its shape.
[8]

35
The Periodic Table

8 [NYJC 2011/III/4(a)-(b)]
The halogens refer to the series of nonmetallic elements in Group VII of the Periodic
Table. Owning to their high reactivity, they are commonly found in the environment
as part of organic or inorganic compounds. This question looks at the chemistry of
halogen compounds.

(a) The halogens all form binary compounds with hydrogen known as hydrogen halides.
State and explain how the thermal stability of hydrogen halides varies down the
Group.
[2]

9 [HCI 2011/II/2(a)]
Elements and compounds react with each other in numerous ways. Nearly every inorganic
chemical reaction falls into one or more of four broad categories.

23
LL

(i) Write a balanced chemical equation, including state symbols, of a reaction to illustrate
each of the four categories of reactions above. Each equation must include at least
one element or compound from Group II or Group VII.

The first has been done for you.

36
The Periodic Table

(ii) The reaction of aqueous potassium iodide with aqueous copper(II) sulfate does not fall
neatly into any of the four categories above. State the type of reaction and write a
balanced equation, including state symbols, for this reaction
[5]

10 [IJC 2011/II/2(b)]
In the laboratory, there are three bottles labelled N, P and Q. Each bottle contains one of
the following reagents:
Cl2(aq), NaI(aq) and KBr(aq)
Three tests were carried out using the reagents in the bottles. The results are summarised
in the table below:

Test Procedure Observations


1 Mix reagent in bottle N with reagent in bottle P No Change
2 Mix reagent in bottle N with reagent in bottle Q Mixture turn brown
3 Mix reagent in bottle P with reagent in bottle Q Mixture turn brown

(i) Deduce and explain which bottle contains Cl2(aq). Write chemical equations with state
symbols to support your reasoning.

(ii) To determine which bottle contains NaI(aq) and KBr(aq) respectively, tests 2 and 3
23
were repeated. Hexane was added to the resulting reaction mixture after the tests
were conducted. The bottles were then shaken and allowed to stand.

State the observations which will help to identify which bottle contains NaI(aq) and
LL

KBr(aq) respectively.

11 [TJC 2011/II/2]
(a) In the 18th and 19th centuries, iodine was industrially produced from kelp, a large seaweed.
Combustion of the kelp converts the organic substances to ash, and sodium halides (mainly
sodium iodide) are obtained.
In the laboratory, a similar process can be done according to the following procedures

Procedures
B1. Fill a large crucible on a tripod with the seaweed. Heat with a strong Bunsen
flame until all the seaweed has been turned to ash.
B2. Boil the ash with about 20 cm3 of purified water in a beaker, and filter while hot.
Collect the clear filtrate in a second beaker and allow to cool.
B3. Add about 2 cm3 of dilute sulfuric acid to the solution, followed by hydrogen
peroxide solution.
B4. Transfer the mixture to a separating funnel and add 10 - 20 cm3 of
cyclohexane. Stopper the separating funnel and shake vigorously for about 30
s. With the separating funnel inverted, release any pressure that has built by
opening the tap briefly.
B5. Clamp the funnel and allow the layers to separate. The cyclohexane will form a
layer on top of the aqueous layer.
B6. Run off the lower aqueous layer into a 250 cm3 conical flask.

37
The Periodic Table

B7. Run the purple cyclohexane layer into an evaporating basin, and set aside to
evaporate in the fume cupboard to obtain the iodine crystals.

(i) State and explain, with the aid of a relevant equation, what you would observe during
procedure B3.

(ii) With reference to the Data Booklet, explain why hydrochloric acid is not used in the
acidification process in procedure B3.

(iii) Explain why cyclohexane is a suitable solvent for the extraction of iodine and state
what you would observe during procedure B4.

(iv) Suggest a reason why in procedure B7, evaporation is preferred over direct heating
over a strong Bunsen flame.

(v) In order to check if the kelp contained significant amount of chloride anions, a
student transferred 1 cm3 of the filtrate obtained in procedure B2 into a test tube and
added acidified silver nitrate solution. A cream precipitate was obtained.
State and explain what reagent should be added to verify if there was a significant
amount of chloride mixed with the iodide.

(vi) When the student left the test tube from (b)(v) in the sun, the precipitate started to
darken and gave a purplish colouration. A very faint smell of chlorine gas was
23
detected. Given the following data, explain what caused the purplish colouration when
the test tube was left in the sun.

Silver metal deposits are either silvery, gray or black in large amounts. However,
LL

when a solution contains a fine suspension of silver nanoclusters, the colour


varies depending on the size of the nanoclusters. A silver nanocluster is a group
of silver atoms that are nanometres in diameter.

Size of silver
20 30 40 50 60 70
nanocluster / nm

Colour of solution red green blue cyan orange violet


[11]

(b) Ultra-pure elemental iodine is often prepared for research work through the reaction of
potassium iodide with copper(II) sulfate(VI). Precipitation first occurs, but the precipitate
instantaneously decomposes to form a white precipitate and iodine, which turns the pale
blue solution brown.
Write a balanced equation, including state symbols, for the decomposition reaction that
occurs.
[1]

38
The Periodic Table

Multiple Choice Questions (Section A)


1 [HCI 2011/I/15]

2 [ACJC 2011/I/26]
23
LL

3 [ACJC 2011/I/29]

39
The Periodic Table

4 [AJC 2011/I/18]

5 [CJC 2011/I/13]

6 [MJC 2011/I/28]
23
LL

7 [PJC 2011/I/15]

40
The Periodic Table

8 [SRJC 2011/I/17]

9 [SRJC 2011/I/18]

23
LL

10 [TJC 2011/I/16]

41
The Periodic Table

11 [TPJC 2011/I/16]

12 [TPJC 2011/I/17] 23
LL

13 [TPJC 2011/I/19]

42
The Periodic Table

14 [VJC 2011/I/16]

23
Multiple Choice Questions (Section B)
LL

1 [HCI 2011/I/35]

43
The Periodic Table

2 [CJC 2011/I/35]

23
LL

3 [PJC 2011/I/36]

4 [TJC 2011/I/35]

44
The Periodic Table

Group VII SUGGESTED SOLUTIONS

Multiple Choice Questions (Section A)


1. 2. 3. 4. 5. 6. 7. 8. 9. 10. 11. 12. 13. 14.
B A A C B C B D D A C D C C

Multiple Choice Questions (Section B)


1. 2. 3. 4.
B D B D

Structured/Free Response Questions

1 Hot glass rod able to decompose HI to H2 and I2. Purple vapour is iodine.
2HI (g)  H2(g) + I2 (g)
Reaction involves breaking of H-I bond (BE= 299 kJmol-1) which is weaker than the H-Br
bond (BE= 366 kJ mol-1).
[Or HI is easier to decompose as the bond is weaker due to the larger I atom, hence bond
length is longer.]

2 (i) HCl
23
(ii) 2SCl2(l) + 2H2O(l)  SO2(aq) + S(s) + 4HCl(aq)

(iii) with AgNO3(aq)


LL

white precipitate formed


Ag+(aq) + Cl(aq)  AgCl(s) and/or Ag+(aq) + SO32(aq)  Ag2SO3(s)

with acidified K2Cr2O7


orange K2Cr2O7 turns green
Cr2O72 + 3SO2 + 2H+  2Cr3+ + 3SO42 + H2O

3 (i) Solid

(ii) I3

(iii) Yellow precipitate formed.


AgNO3(aq) + NaI(aq) → AgI(s) + NaNO3(aq)

(iv) Precipitate remains insoluble in NH3(aq).

45
The Periodic Table

4 (i) MgCl2(s) + 6H2O(l) [Mg(H2O)6]2+(aq) + 2Cl–(aq)


Mg ions are hydrated to [Mg(H2O)6]2+.
2+

[Mg(H2O)6]2+(aq) + H2O(l) [Mg(H2O)5(OH)]+(aq) + H3O+(aq)


2+
Mg has high charge density and slightly polarises the O–H bond in water
molecules, which is weakened and easily broken to form a slightly acidic solution.

(ii) PCl5(s) + 4H2O(l) H3PO4(aq) + 5HCl(aq)


OR PCl5(s) + H2O(l) POCl3(aq) + 2HCl(aq)

Not likely, since a strongly acidic solution is formed when PCl5 reacts with water in
the body and the excessive H+ formed may cause gastric problem.

5
(a) (i) Br2 + SO2 + 2 H2O  2 HBr+ H2SO4

(ii) Oxidation.
Concentrated sulfuric acid is an oxidizing agent and will oxidize HBr to Br 2.

(b) (i) HCl to HI molecules: Increasing BP from HCl to HBr to HI,


HF molecules: HF boiling point is exceptionally high.
HCl to HI molecules: Increasing size of the halogen atom causes the size of the the
electron cloud of the hydrogen halides to increase down the group, thus the van der
Waals’ forces of attraction increases down the group.
23
HF molecules: Due to hydrogen bonding between HF molecules, HF has an
exceptionally high boiling point.
LL

(ii) Decreasing stability of the hydrogen halides down the group. This is because down
the group, the H-X bond becomes longer, the bond becomes weaker and the bond
energy becomes lower ; the hydrogen halides will be broken down by heat more
readily.

6 (i) From solution formed from halogen A:


White precipitate, C, formed with AgNO3 is AgCl.
White precipitate, D, formed with Ba(NO3)2 is BaSO4.
From solution formed from halogen B:
Yellow precipitate, E, formed with AgNO3 is AgI.

(ii) Halogen A is Cl2.


4Cl2 + S2O32– + 5H2O  8Cl– + 2SO42– + 10H+

Halogen B is I2.
I2 + 2S2O32–  S4O62– + 2I–

(iii) From the Data Booklet: E(Cl2/Cl–) = +1.36 V; E(I2/ I–) = +0.54 V
 
Since E (Cl2/Cl–) is more positive than E (I2/I–), Cl2 is a stronger oxidising
agent than I2.

46
The Periodic Table

7 (i) I. ½ (CN)2 + H+ + e HCN E= +0.37 V


Cl2 + 2e 2Cl E = +1.36V

E cell = +1.36 – (+0.37) = +0.99V
Overall Equation: Cl2 + 2HCN (CN)2 + 2H+ + 2Cl- 

II. ½ (CN)2 + H+ + e HCN E = +0.37 V


2- 2- 
S4O6 + 2e 2S2O3 E = +0.09 V

E cell = +0.37 – (+0.09) = +0.28 V
Overall Equation: (CN)2 + 2H+ + S2O32-  2 HCN + S4O62-

III. ½ (CN)2 + H+ + e HCN E = +0.37 V


Cr2O72- + 14H+ + 6e 2Cr3+ + 7H2O E = +1.33 V
Both are oxidising agents. Hence reaction cannot proceed.

(ii)

linear

8 (a) Down the group, thermal stability of HX decreases


as the H-X bond length increases and it becomes easier to break the H-X bond.
23
9 (i) Examples of possible answers :
Category of Equation from Group II or Group VII
reaction
Decomposition 2Mg(NO3)2 (s)  2MgO(s) + 4NO2(g) + O2(g)
LL

2HCl(g)  H2(g) + Cl2(g)


Single Ca (s) + 2H2O(l)  Ca(OH)2(aq) + H2(g)
displacement Cl2(g) + 2KI(aq)  2KCl(aq) + I2(aq)
Double Na2SO4(aq) + Ba(NO3)2(aq) BaSO4(s) + 2NaNO3(aq)
displacement NaCl(aq) + AgNO3(aq)  AgCl(s) + NaNO3(aq)

(ii) Type of reaction : redox


Equation : 4KI(aq) + 2CuSO4(aq)  2K2SO4(aq) + 2CuI(s) + I2(aq)

10 (i) Bottle Q contains Cl2 (aq) which oxidized / displaced Br- (aq) to Br2 (aq) and I- (aq) to
I2 (aq) which accounts for the brown mixture obtained.

Cl2 (aq) + 2Br- (aq)  2Cl- (aq) + Br2 (aq)

Cl2 (aq) + 2I- (aq)  2Cl- (aq) + I2 (aq)

(ii) The bottle with reddish brown organic layer contains KBr (aq) initially.
The bottle with violet / purple organic layer contains NaI (aq) initially.

11
(a) (i) H2O2 oxidises the I-, to aqueous I2, so a brown solution would be obtained.
H2O2 + 2I- + 2H+ → I2 + 2H2O

47
The Periodic Table

(ii) EθCl - =+1.36V


2 /Cl

EHθ 2O2 /H2O =+1.77V


Since EHθ 2O2 /H2O is more positive than EθCl - , H2O2 can oxidise chloride to chlorine
2 /Cl

while it itself is reduced to H2O. Hence, the oxidation of iodide may not be complete.

(iii) ●Both cyclohexane and iodine have simple molecular structures, are non-polar and
have van der Waals’ (vdW) forces of attraction between their molecules.
●When iodine dissolves in cyclohexane, the energy released from the vdW forces
formed between iodine and cyclohexane is enough to overcome the vdW forces
between iodine molecules and the vdW forces between cyclohexane.
●On shaking with cyclohexane, I2 will dissolve in the colourless cyclohexane to form
a violet organic layer. Brown colour of aqueous layer fades.

(iv) Cyclohexane is flammable, and can cause a fire to break out / iodine may sublime
and escape.

(v) Dilute aqueous NH3 should be added. AgCl is soluble in NH3(aq) but AgI is not. If
there was significant amount of silver chloride in the precipitate obtained, much of the
precipitate dissolved upon adding aqueous NH3.

(vi) ●Sunlight likely catalysed the decomposition of silver chloride / redox reaction
23
between the Ag+ and Cl- ions to form silver metal and chlorine gas.
●Silver metal atoms are likely to be formed in nanoclusters in the range of 70 nm,
producing the purplish colouration.
LL

(c) 2CuI2(s) → 2CuI(s) + I2(aq)

Note: Do not assume that all the given answers are correct. Do discuss with your
friends and check with your tutor should you think that any given answer is incorrect.

48
Reaction Kinetics

REACTION KINETICS 6
Structured / Free Response Questions

1 N88/1/3
The kinetics of the acid catalyzed reaction between iodine and propanone
CH3COCH3 (aq) + I2 (aq) →CH2ICOCH3 (aq) + HI (aq)
can be investigated experimentally by varying the concentrations of the three
substances involved and determining the time for the colour of the iodine to
disappear. In this method, the rate of reaction is measured in terms of the rate at
which the iodine concentration changes.
Volume of iodine used
Rate α
time
Several experiments were carried out to determine the order of reaction with respect
to propanone and iodine and the results were given below:

Expt Volume of Volume of Volume of Volume of Time taken for Rate


propanone / iodine / cm3 sulfuric acid / water / cm3 iodine colour / cm3 s−1
cm3 cm3 to disappear /s
1 8 4 8 0 1
2 4 4 8 4 2
3 8 2 8 2 0.5
23
(a) Complete the table by calculating the rate of reaction for the three experiments.
LL

(b) Hence, determine the order of reaction with respect to propanone and iodine.

(c) A fourth experiment was carried out to determine the order of reaction with
respect to sulfuric acid. The volume of propanone used was 8 cm 3, the volume of
iodine used was 4 cm3 and the time taken for iodine colour to disappear was 4 s.

If the order of reaction with respect to sulfuric acid is one, determine the volume
of sulfuric acid and water used in the fourth experiment.

2 [Modified CJC 2011/III/3b]


Ethanal, CH3CHO, and iodine reacts in an acidic medium according to the mechanism as
shown. Kinetics studies have shown that the reaction is second order overall.

CH3CHO

1
Reaction Kinetics

(i) Suggest the rate equation for the reaction, and hence the units of the rate constant.

(ii) Suggest how the rate of reaction between ethanal, CH3CHO, and iodine can be determined
experimentally.

(iii) Sketch an appropriate graph and show how the order of reaction with respect to ethanal,
CH3CHO, can be obtained experimentally.

(iv) Explain how the rate of reaction would change if bromine is used instead of iodine. [5]

3 [NYJC 2011/II/3(b)]
(a)
One reaction which occurs in air polluted with nitrogen oxides is shown below.

2NO(g) + O2(g) → 2NO2(g)

Five experiments were carried out to find the relationship between the initial concentration of
NO and of O2, and the initial rate of formation of NO2.

23
LL

(i) Deduce the order of reaction with respect to each reactant.

(ii) Hence suggest an equation for the rate-determining step.

(iii) Comment on why this rate-determining step is unusual.

(iv) The actual mechanism for the reaction between NO and O2 is actually bimolecular. By
considering the relative stabilities of the reactant molecules involved, suggest a more
plausible mechanism for the reaction.
[4]

(b) At 700 oC, nitrogen monoxide and hydrogen react as follows:

2NO(g) + 2H2(g) → N2(g) + 2H2O(g)

(i) Explain briefly why the initial reaction rate would be expected to increase by
increasing each of the following:
• The pressure
• The temperature

2
Reaction Kinetics

(ii) Suggest, with reasons, whether you would expect the reaction between nitrogen
monoxide and hydrogen to be endothermic or exothermic.
[4]

4 [VJC 2011/III/5(a)(b)]
Sucrose is the most common natural food sweetener, often known as table sugar. In
acidic solutions, sucrose is readily hydrolysed to a 1:1 mixture of glucose and
fructose. The reaction is catalysed by aqueous H+ ions.
H+(aq)
C12H22O11 + H2O C6H12O6 + C6H12O6
(sucrose) (glucose) (fructose)

A series of experiments was carried out at 25°C to investigate the kinetics of this
reaction, using 0.79 mol dm−3 sucrose solution and 1.25 mol dm−3 hydrochloric acid.
The following data were obtained:
Expt Volume of Volume of Volume of Initial rate of
sucrose HCl water reaction
/cm3 /cm3 /cm3 /mol dm−3 min−1
1 20 20 10 1.25 x 10-3
23
2 20 30 0 1.88 x 10-3
3 10 30 10 9.38 x 10-4
LL

4 40 20 10 ?

(a) (i) Using the data given above, determine the order of reaction with respect to sucrose
and HCl.
Hence, determine the rate equation and calculate a value for the rate constant of the
reaction, stating its units.

(ii) Using your answer in (i), deduce the initial rate of reaction for Experiment 4.
[6]

(b) In bacteria, sucrose is broken down into glucose and fructose by the enzyme, invertase. A few
experiments were carried out to measure the initial rate of the enzyme-catalysed hydrolysis
reaction for different concentrations of sucrose.

(i) Sketch a graph to show how the initial rate of this enzyme-catalysed hydrolysis
reaction varies with the concentration of sucrose. Label your graph as Graph 1.

(ii) Explain the shape of Graph 1, making reference to the order of reaction with respect to
sucrose.
[4]

3
Reaction Kinetics

5 [HCI 2011/III/4]
In the atmosphere, nitrogen monoxide reacts with ozone as shown in the following equation.

(a) NO2 is considered to be an air pollutant. State one undesirable consequence of the presence
of NO2 in the atmosphere.
[1]

(b) The reaction does not take place in a single step. Experimentally it has been found that in
mixtures of NO and O3, free O atoms are present. No other short-lived intermediates can be
detected.

The rate equation for the reaction was also found to be Rate = k [NO][O3].

Outline the simplest mechanism consistent with the above observations, indicating the rate
determining step.
[2]

(c) When NO and O3, both at an initial concentration of 1.0 × 10–6 mol dm–3 were mixed together
at 25 ˚C, [NO2] varies with time, as shown in the table below.

Time / s [NO2] / mol dm–3


23
0.05 0.50 × 10–6
0.10 0.69 × 10–6
LL

0.15 0.77 × 10–6


0.20 0.82 × 10–6
0.25 0.85 × 10–6
0.30 0.87 × 10–6

(i) How is [O3] related to [NO] during the course of the reaction?

(ii) Δ[NO 2 ]
The rate at a particular time can be approximated using rate = .
Δt

Use the information given in the table to produce data suitable for plotting a graph of
rate against [NO2]2 to confirm that the reaction shows overall second order kinetics.

(iii) Use the graph and use it to determine the rate constant k.
[8]

4
Reaction Kinetics

6 [Modified NJC 2011/III/2(b)]


The hydrolysis reaction of aspirin is catalysed by an acid. Its equation is represented below.

Four experiments were carried out to find the relationship between the rate of reaction and the
initial concentrations of aspirin and the acid catalyst. The results are recorded in the table
below.

[aspirin] [H+] Initial rate


Experiment
/mol dm−3 /mol dm−3 / mol dm−3 hr−1
I 0.04 0.08 1.00
II 0.05 0.08 1.25
III 0.04 0.06 0.75
IV 0.05 0.06 0.94
23
(i) Using the data in the table, deduce the order of reaction with respect to aspirin and the acid
catalyst, showing how you arrive at your answers.
LL

Hence determine the rate equation and calculate the rate constant for this reaction, giving its
units.

(ii) Given that the half-life of aspirin in experiment I was 3 hours, predict the half-life of aspirin in
experiment II and in experiment III, giving your reasoning.

(iii) Sketch a graph showing how the concentration of one of the products of the hydrolysis
reaction varies with time, by making use of the data given for experiment I.

[9]

7 [SRJC 2011/III/5]
(a) The following steps in a mechanism are as shown:

K + L ⎯⎯→ KL (slow)

KL + M ⎯⎯→ N (fast)

Given that the activation energy of the reaction is +35 kJ mol-1 and ∆H is -90 kJ mol-1,
sketch the energy profile diagram for the above reaction.

(b) Using an appropriate diagram, explain how higher temperatures affect the rate of reaction.

5
Reaction Kinetics

(c) [RVHS 2011/III/2(c)]


One of the properties of an ideal drug is that the drug can be synthesised quickly with high
yield. Using a Boltzmann distribution curve, explain how the use of catalyst can speed up the
rate of reaction of synthesising a drug molecule.
[6]

8 [Modified RI 2011/II/2(a)−(c)]
The rate of reaction of bromine with bromopropanone was studied in acid solution at 25 C.
The table below gives some results obtained for the reaction

The reaction may be followed by either chemical or physical methods of analysis.


23
Sampling & Titration
In each of the above experiments, the reagents were mixed and then three samples were
successively withdrawn, at appropriate times, as the reaction proceeded. At 1500 s, ice-cold
water was added to the first sample. To this, excess aqueous potassium iodide was added.
LL

The resultant solution was then titrated against standard sodium thiosulfate solution. This
procedure was repeated with the second and third samples, with ice-cold water being added
at 3000 s and 4500 s respectively.

(a) (i) Explain the meaning of the following terms.

• Order of reaction
• Rate constant
• Half-life

(ii) Why was ice-cold water added?


(iii) Write an ionic equation to show the reaction involving potassium iodide.

(iv) Suggest why an indicator was not needed for the titration.
[6]

(b) Suggest how the above reaction could be followed by a physical method of analysis.
[1]

(c) (i) The rate equation for the reaction is

rate = k [Br2] a [CH3COCH2Br] b [H+]

6
Reaction Kinetics

The orders of reaction with respect to Br2 and CH3COCH2Br are to be determined. Use
the data provided to plot suitable graphs on graph paper.

(ii) Determine the values of a and b, showing clearly how you arrived at your answers.

(iii) What are the units of the rate constant for this reaction?
[9]

9 [ACJC 2010/III/5(a)(i)]
The reaction between iodide ions,I-, and persulfate ions, S2O82-, is slow.
The reaction can be speeded up by adding a small amount of Fe2+ ions.

The initial rate of the slow reaction between iodide ions and persulfate ions can be studied by
using thiosulfate ions. The equations for the reactions are as follows.

2I- + S2O82- I2 + 2SO42- (slow) reaction I

I2 + 2S2O32- 2I- + S4O62- ( fast ) reaction II

In the presence of a constant amount of thiosulfate ions, the iodine being slowly produced by
reaction l will immediately react in reaction ll until all the thiosulfate ions has been used up.
At that point, free iodine will be present in the solution, which will cause a sudden appearance
23
of a deep blue colour if starch is present.

A series of experiments was carried out using different volumes of the five reagents. The
following results were obtained.
LL

experiment Volume Volume Volume Volume Volume Time for


number of of of of of the
S2O82- I- S2O32- distilled starch appearance
/cm3 /cm3 /cm3 water /cm3 of deep blue
/cm3 colour/s
1 20 20 10 5 5 30
2 20 15 10 10 5 40
3 5 25 10 15 5 t3
4 10 15 10 20 5 80

If the orders of reaction with respect to persulfate ions and iodide ions are both one
respectively, deduce an expression relating the volume of these two reactants and time taken
for the appearance of deep blue colour. Explain your reasoning.

Predict the time, t3, required for the appearance of deep blue colour in experiment 3.
[3]

10 [RI 2011/III/1(a)]
Peroxodisulphate ions react with iodide ions according to the equation

S2O82-(aq) + 2I−(aq) → 2SO42−(aq) + I2(aq)

7
Reaction Kinetics

The rate of the reaction may be found by mixing the two solutions together, adding water to
keep the total volume constant and measuring the time taken for the iodine concentration to
reach a given value (using a colorimeter or a titration method).

In experiments 1 to 5, various volumes of peroxodisulphate ions and iodide ions, both of initial
concentration 0.050 mol dm−3 were mixed. The time taken for the iodine concentration to
become 0.0010 mol dm−3 was measured. The following results were obtained.

Experiment Volume of S2O82- Volume of Volume of


number (aq)/cm3 I−(aq)/cm3 H2O(l)/ cm3 Time/s

1 5 10 10 56
2 3 10 12 96
3 2 10 13 145
4 5 5 15 120
5 5 3 17 185

The rate equation for the reaction has the form: rate = k[S2O82-]n [I−]m
23
(a) Deduce the values of n and m in the above rate equation. [3]

(b) Find the initial rate of formation of iodine in experiment 1 and


hence find the value of k and state its unit. [4]
LL

(c) Comment on the relatively small value of k in (b). [1]

(d) In another experiment, the initial concentrations of peroxodisulphate ions and iodide
ions used were 1.00 mol dm−3 and 0.0050 mol dm−3 respectively.

Determine the half-life of the reaction and hence calculate the concentration of iodide
ions after 1 second from the start of the experiment.
[2]

[7]
11 [Modified VJC 2005/III/3 either]
At 600 K, SO2Cl2 decomposes according to the following equation:

SO2Cl2(g) → SO2 (g) + Cl2(g)

The rate of the reaction is followed by measuring the total pressure at regular time interval
when 0.10 mol of SO2Cl2(g) is heated in a 1.0 dm3 vessel.

The following results were obtained:


Time/h 0.0 1.0 2.0 4.0 8.0 16.0
Pressure/atm 4.91 5.58 6.32 7.31 8.54 9.54

8
Reaction Kinetics

(a) Copy the table above and determine the partial pressure of SO2 at each time interval as
the reaction proceeds from 0.0 h to 16.0 h.
[2]

(b) Using the values in (a), plot a graph to confirm that the reaction is first order with
respect to SO2. Hence, deduce the rate equation and calculate the rate constant.
[4]

(c) Assuming the volume remains constant, what would be the total gas pressure after 12
hours of decomposition at 600 K.
[1]

(d) Predict what will happen to the initial rates and half-lives if

(i) 0.20 mol of SO2Cl2 is used instead of 0.10 mol

(ii) 2.0 dm3 vessel is used instead of 1.0 dm3


[2]

(e) At time t, the amount of the three gases SO2Cl2, SO2 and Cl2 are the same.
Draw on the same axes, the distribution of the molecular speeds for each of the three
gases.
[2]
23
LL

9
Reaction Kinetics

Multiple Choice Questions (Section A)

1 [Modified SRJC 2011/I/7]

23
LL

2 [RI 2011/I/11]

10
Reaction Kinetics

3 [MJC 2011/I/6]

23
LL

4 [MJC 2010/I/7]

11
Reaction Kinetics

5 [TJC 2011/I/9]

23
6 [CJC 2011/I/11]
LL

12
Reaction Kinetics

7 [RI 2010/I/8]

23
LL

8 [HCI 2011/I/8]

13
Reaction Kinetics

9 [NYJC 2011/I/8]

23
LL

10 [RI 2011/I/10]

14
Reaction Kinetics

11 [ACJC 2011/I/10]

23
LL

12 [NJC 2008/I/12]

15
Reaction Kinetics

13 [CJC 2010/I/9]

23
LL

14 [PJC 2011/I/11]

16
Reaction Kinetics

15 [SAJC 2011/I/10]

23
LL

16 [TJC 2011/I/8]

17
Reaction Kinetics

17 [SRJC 2009/I/7]

23
LL

18 [SAJC 2010/I/10]

18
Reaction Kinetics

19 [NYJC 2010/I/8]

23
LL

20 [RI 2005/I/5]

19
Reaction Kinetics

23
LL

21 [VJC 2005/I/11]

The dependence of rate on HCl for the following reaction was investigated at 298K and
atmospheric pressure using excess Na2CO3 :

Na2CO3(s) + 2HCl(aq) → 2NaCl(aq) + H2O(l) +CO2(g)

The volume of CO2 evolved was plotted against time and the t½ was determined to be
constant at 6 s (represented by Curve I).

The reaction was then repeated under different conditions with the total volume of the
reaction mixture with the total volume of the reaction mixture remaining constant
(represented by Curve II).

Which of the following is correct?

20
Reaction Kinetics

23
LL

Multiple Choice Questions (Section B)

1 [ACJC 2010/I/35]

21
Reaction Kinetics

23
LL

2 [RVHS 2011/I/35]

3 [CJC 2010/I/33]

22
Reaction Kinetics

23
LL

4 [VJC 2011/I/34]

23
Reaction Kinetics

5 [RI 2011/I/33]
23
LL

6 [HCI 2010/I/32]

24
Reaction Kinetics

7 [AJC 2011/I/34]

23
LL

8 [MI 2009/I/38]

25
Reaction Kinetics

9 [RI H1 2009/I/27]

23
LL

10 [NJC 2010/I/32]

26
Reaction Kinetics

11 [VJC 2005/I/31]
Consider the following reaction mechanism that involves only aqueous species:
k1
A+B C Fast
k−1
k2
C P Slow

Which of the following relationships are consistent with the above mechanism?
23
1 rate  [A][B]
LL

d [ P]
2 rate = = k2[C]
dt

3 [C] = Kc[A][B] (where Kc : equilibrium constant)

27
Reaction Kinetics

REACTION KINETICS SUGGESTED SOLUTIONS

Structured/Free Response Questions

1(a) Experiment Volume of Volume Volume Volume Time Rate /


propanone of iodine of of water taken for cm3
/ cm3 / cm3 sulfuric / cm3 iodine s−1
acid / colour to
cm3 disappear
/s
1 8 4 8 0 1 4
2 4 4 8 4 2 2
3 8 2 8 2 0.5 4

1(b) Comparing experiments 1 and 2, when volume of propanone increase 2 x, rate


increase 2 x. Hence, Rate α [propanone]. 1st order with respect to propanone.
Comparing experiments 1 and 3, when volume of iodine increase 2 x, rate
remains the same. Zero order with respect to iodine.
23
1(c) Since volume of propanone and iodine used is similar to experiment 1, compare
experiment 4 with experiment 1.
LL

Experiment Volume of Volume Volume Volume Time Rate /


propanone of iodine of of water taken for cm3
/ cm3 / cm3 sulfuric / cm3 iodine s−1
acid / colour to
cm 3 disappear
/s
1 8 4 8 0 1 4
4 8 4 4 1

Since rate was decreased by 4 x, volume of sulfuric used must be ¼ of


experiment 1.
Hence, volume of sulfuric acid used is 2 cm3. Since total volume must be kept
constant at 20 cm3, volume of water used = 6 cm3

2(i) Rate =k[HCOCH3][H+] Units of rate constant: mol-1dm3s-1

2(ii) Measure time taken for the brown colour of iodine to disappear
OR

28
Reaction Kinetics

Monitor colour intensity of reaction using colourimeter and let concentration of


iodine be directly be proportional to colour intensity. Plot graph of colour intensity
against time. Gradient of tangent to the graph at time= 0 is initial rate of reaction.
OR
Quench the reaction with addition of sodium carbonate (to remove acid catalyst)
or soak in ice bath at regular fixed time intervals and titrate the iodine with sodium
thiosulfate. Plot graph of conc of iodine against time. Gradient of tangent to the
graph at time= 0 is initial rate of reaction.

2(iii)
[I2]

[ethanal] = x mol dm-3

[ethanal] = 2x mol dm-3


Time
If the gradient of the graph doubles when [ethanal] is doubled, order of reaction wrt to ethanal is
23
1.

2(iv) Rate would remain unchanged. Rate is independent of the halogen used.
LL

3(a)(i) Comparing experiment 3 & 4, when conc of NO is doubled while rest remains the
same, initial rate of reaction quadruples. Hence the initial rate of reaction is directly
proportional to square of conc of NO, it is second order with respect to NO.

Comparing experiment 1 & 2, conc of O2 is doubled, initial rate of reaction also


doubles. Hence the initial rate of reaction is directly proportional to conc of O2, it is first
order with respect to O2.

3(a)(ii) 2NO(g) + O2(g) → 2NO2(g)

3(a)(iii) This reaction involves the collision of three particles at the correct orientation and
sufficient energy, which is statistically very highly unlikely.

3(a)(iv)
1st step (fast step): 2NO(g) → N2O2(g)
2nd step (slow step): N2O2(g) + O2(g) → 2NO2

Since cannot have more than 2 molecules in one step, take any 2 out of 3
molecules from 2 NO + O2 to combine and form 1 particle first in a fast step,
e.g. 2 NO to form N2O2, then take that particle, N2O2 to collide with O2 to
form 2 NO2

29
Reaction Kinetics

3(b)(i) When pressure is increased, the particles are brought closer together and hence
frequency of collision of molecules increases. The frequency of effective collision
also increases.
When temperature increases, the number of particles with energy greater than
activation energy increases as the average kinetic energy of particles increases.
The frequency of effective collision therefore increases.

3(b)(ii) Exothermic.
The reaction involves forming of highly stable N2 and H2O molecules (or formation
of strong N≡N bond and O-H bonds) from highly unstable NO molecule.

4(a)(i) Compare Expts 1 and 2:


When VHCl or [HCl ] increases by 1.5 times (VHCl  [HCl ] since total volume of solution
is constant)
0.00188
 rate increases by = 1.5 times.
0.00125
Hence reaction is first order with respect to HCl.
Compare Expts 2 and 3:
When Vsucrose or [sucrose] decreases by 0.5 times (Vsucrose  [sucrose] since total vol
of solution is constant)
23
0.000938
 rate decreases by ( = 0.499 ≈ 0.5 times.
0.00188
Hence reaction is first order with respect to sucrose.
LL

rate = k[sucrose][HCl]
From Expt 1,
[sucrose] = 0.79  20/50 = 0.316 mol dm−3
[HCl] = 1.25  20/50 = 0.500 mol dm−3
0.00125
k= = 7.91 x 10−3 mol−1 dm3 min−1
0.316  0.500
OR
From Expt 2,
[sucrose] = 0.79  20/50 = 0.316 mol dm−3
[HCl] = 1.25  30/50 = 0.750 mol dm−3
0.00188
k= = 7.93 x 10−3 mol−1 dm3 min−1
0.316  0.750
OR
From Expt 3,
[sucrose] = 0.79  10/50 = 0.158 mol dm−3
[HCl] = 1.25  30/50 = 0.750 mol dm−3
0.000938
k= = 7.92 x 10−3 mol−1 dm3 min−1
0.158  0.750

4(a)(ii) From Expt 4,

30
Reaction Kinetics

[sucrose] = 0.79  40/70 = 0.451 mol dm−3


[HCl] = 1.25  20/70 = 0.357 mol dm−3

 Initial rate = 7.91 x 10−3  0.451  0.357


= 1.27 x 10−3 mol dm−3 min−1

4(b)(i)

4(b)(ii) At low [sucrose], reaction is first order wrt sucrose due to availability of active sites
on enzyme for binding.

As [sucrose] increases, more active sites on enzyme molecules are occupied by


sucrose molecules. Hence, reaction is no longer first order wrt sucrose (mixed order).
23
At high [sucrose], all active sites of enzyme molecules will be occupied by sucrose
molecules and become saturated. Further increase in [sucrose] will no longer increase
the reaction rate. Hence, reaction is zero order wrt to sucrose.
LL

5(a) Formation of acid rain / cause respiratory problems / carcinogen / toxicity

5(b) NO + O3 → NO2 + 2O slow


2O → O2 fast

5(c)(i) NO] = [O3] at each timing or


Concentration of both reactants fall at the same rate

31
Reaction Kinetics

5(c)(ii) Since the reaction is overall second order, the following rate equations may be possible:
Rate = k [NO]2 or Rate = k [NO][O3]
As such, a graph of rate against [NO] 2 or a graph of rate against [NO][O3] would give a
straight line. Values for [NO]2 and [NO][O3] should be identical since [NO] = [O3] at every
timing.

Time / s [NO2] / Δ[NO 2 ] [NO] / [NO]2 /


mol dm–3 / mol dm–3 mol2 dm–6
Δt
mol dm–3 s–1
0.05 0.50 × 10–6 1.00× 10–5 5.00 × 10–7 2.50 × 10–13
0.10 0.69 × 10–6 3.80× 10–6 3.10 × 10–7 9.61 × 10–14
0.15 0.77 × 10–6 1.60× 10–6 2.30 × 10–7 5.29 × 10–14
0.20 0.82 × 10–6 1.00× 10–6 1.80 × 10–7 3.24 × 10–14
0.25 0.85 × 10–6 6.00× 10–7 1.50 × 10–7 2.25 × 10–14
0.30 0.87 × 10–6 4.00× 10–7 1.30 × 10–7 1.69 × 10–14
Calculate values for:
Δ[NO2 ]
• Reaction rate, using
Δt
• Concentration of NO at each timing, since [NO] = 1.0 × 10–6 – [NO2]
• [NO]2 or [NO][O3]
5(c)(iii)
23
LL

• Correct axes
• Correct shape
• Cut through origin

Rate constant k = gradient of graph = 4 × 107 mol–1 dm3 s–1

32
Reaction Kinetics

6(i) Comparing experiment III & IV, when conc of aspirin is increased by 25% from 0.04
mol dm-3 to 0.05 mol dm-3, initial rate of reaction also increases by 25% from 0.75 mol
dm-3 hr-1 to 0.94 mol dm-3 hr-1. Hence the initial rate of reaction is directly proportional to
conc of aspirin, it is first order with respect to aspirin.

Comparing experiment I & II, conc of H+ is increased by 33% from 0.06 mol dm-3 to
0.08 mol dm-3, initial rate of reaction also increases by 33% from 0.75 mol dm-3 hr-1 to
1.00 mol dm-3 hr-1. Hence the initial rate of reaction is directly proportional to conc of H+, it
is first order with respect to H+.

Rate = k [aspirin][H+]

Using data from experiment I, k = 313 mol-1dm3 hr-1

6(ii) Since H+ is acting as a catalyst, the concentration of H+ does not change and remains
constant.
Hence the rate equation becomes:
Rate = k‟ [aspirin] where k‟ = k [H+]
i.e. 1st order wrt aspirin only
23
Expt II : Half life of aspirin = 3 hours since conc of H+ is the same.
8
Expt III : Half life of aspirin = 3 × = 4 hours.
LL

6
6(iii)

**At least 2 successive half


lives must be indicated
correctly.

33
Reaction Kinetics

7(a)

7(b)

Diagram
• labeled axes;
• correct shape
• shaded region for higher
23
and lower temperature;
• legend (accept if mentioned
in explanation that “ area
under the graph represents
LL

the no. of particles)

When temperature increases, average kinetic energy of particles increases. Greater


proportion of particles will possess energy greater than or equal to the minimum
amount of energy, Ea, for a successful collision to occur.
Frequency of effective collisions increases , rate of reaction increases.

7(c)

In the presence of a catalyst, presence of an alternate reaction pathway with lower


Ea. Greater proportion of particles will possess energy greater than or equal to the
minimum amount of energy, Ea, for a successful collision to occur. Frequency of
effective collisions increases , rate of reaction increases.

34
Reaction Kinetics

8(a)(i) The order of reaction with respect to a particular reactant is the power to which the
concentration of that reactant is raised in the rate equation.
OR
The overall order of reaction is the sum of the exponents in the rate equation. (If rate =
k[A]m[B]n, overall order of reaction = m + n)

The rate constant, k, is a constant of proportionality in the rate equation. It is


constant for a particular reaction at a given temperature.

The half-life of a reaction is the time taken for the concentration of a reactant to fall
to half its initial value.

8(a)(ii) To quench the reaction at the specified times by rapidly lowering the temperature of
the reaction mixture.

8(a)(iii) 2I – + Br2 ⎯→ 2Br – + I2

8(a)(iv) At the end point, the yellow solution of I2 decolourises.


Hence there is no need for any indicator.

8(b) By measuring the change in the colour intensity of the orange Br2(aq). Let
concentration of Br be directly proportional to colour intensity. Plot a graph of colour
23
intensity against time. Gradient of graph at any instant is the rate of reaction.

8(c)(i)
LL

35
Reaction Kinetics

8(c)(ii) From Expt 1,


initial rate = – [(0.020 – 0.050) / (2500 – 0)] = 1.20  10–5 mol dm–3 s–1

From Expt 2,
initial rate = – [(0.070 – 0.100) / (2500 – 0)] = 1.20  10–5 mol dm–3 s–1

Comparing Expt 1 and 2, when [Br2] is doubled, there is no change in the initial rate
 reaction is zero order w.r.t. Br2.
 a=0

From Expt 3,
initial rate = – [(0.030 – 0.050) / (4100 – 0)]
= 4.88  10–6 mol dm–3 s–1

Compare Expt 1 and Expt 3.


When [CH3COCH2Br] is increased 2.5 times,
the initial rate increases 1.20  10–5 / 4.88  10–6 = 2.5 times.
 reaction is first order w.r.t. CH3COCH2Br.
 b=1
OR
Compare Expt 1 and Expt 3.
23
From rate = k [Br2] a [CH3COCH2Br] b [H+]
1.20  10–5 = (0.050) b
4.88  10–6 (0.020) b
LL

 b=1

8(c)(iii) mol–1 dm3 s–1

9 Rate = k [S2O82-][ I-]


 Since total volume of solution is constant,
concentration of a reactant  volume of reactant
 rate  1/time
1/time = k (volume of S2O82-)(volume of I-)

(volume of S2O82-)(volume of I-) x time = constant

time for expt 3 = 96 s

10 (a) For a constant total volume, [A]  V, the volume of reactant used.
Therefore, [A] = V where  is the proportionality constant.

The rate of reaction, Rate  1/t, where t is the time.


Therefore, Rate = /t where  is the proportionality constant.

Since Rate = k[A]n where k is the rate constant and n is the order of reaction.

Therefore, /t = k(V)n  Vnt = /kn = constant

36
Reaction Kinetics

Expt. V(S2O82-)/ V( I -)/cm3 V(H2O)/cm3


No. cm3 Time/s V(S2O82-).t

1 5 10 10 56 280
2 3 10 12 96 288
3 2 10 13 145 290
4 5 5 15 120 -
5 5 3 17 185 -
Expt. V(S2O82-)/ V(I -)/cm3 V(H2O)/cm3
No. cm3 Time/s V(I -).t

1 5 10 10 56 560
2 3 10 12 96 -
3 2 10 13 145 -
4 5 5 15 120 600
5 5 3 17 185 555

Since V(S2O82-).t = constant  n=1


23
Since V(I -).t = constant  m=1

10(b) Concentration of I2 formed = 0.0010 moldm-3 and time taken = 56 sec


Hence initial rate of I2 formation = 0.0010/ 56 = 1.79x10-5 mol dm-3 s-1
LL

Since Rate = k[S2O82-][I -]


Initial concentration of I - = 10/25 x 0.050 = 0.020 moldm-3
Initial concentration of S2O8 2- = 5/25 x 0.050 = 0.010 moldm-3

Hence k = 1.79x10-5/(0.020x0.010) = 8.93x10-2 mol-1dm3s-1

11(c) The small value of k is due to a high activation energy which arises due to the
collision between 2 negatively charged particles.

10(d) Due to [S2O82-]>>[I -], rate  k’[I-] where k’=k[S2O82-]


Therefore t1/2 = ln 2/k’ = ln 2/(k[S2O82-]) = ln 2/(8.93x10-2 x 1.00) = 7.76 s
Hence n x t1/2= 1  n= 0.13
After 1 second, [I-] left = (1/2)0.13 x 0.005  4.57x10-7 moldm-3

11(a)

37
Reaction Kinetics

11(b)

23
Graph with working on graph to indicate at least 2 successive half-lives.
t½ = 4.1 hr (approximately constant for at least 2 successive half lives.
LL

Therefore, order of reaction wrt SO2Cl2 is 1.


rate = k (PSO2Cl2) **Note that rate = k [PSO2Cl2] or rate = k[SO2Cl2] are wrong
k = ln 2/ t½ = ln 2/ 4.1 hr = 0.169 h−1

11(c) Total pressure = 4.35 + 4.91 = 9.26 atm

11(d)(i) Initial rate doubles but half-life remains unchanged.

11(d)(ii) Initial rate i but half-life remains unchanged.

11(e)

38
Reaction Kinetics

Multiple Choice Questions (Section A)


1. 2. 3. 4. 5. 6. 7. 8. 9. 10. 11. 12. 13. 14. 15. 16. 17. 18. 19. 20. 21.
D A C B C B C B C B D C D D C B A C B A D

Multiple Choice Questions (Section B)


1. 2. 3. 4. 5. 6. 7. 8. 9. 10. 11.
A C B B B A A D B B A

Note: Do not assume that all the given answers are correct. Do discuss with your friends and
check with your tutor should you think that any given answer is incorrect.
23
LL

39
Introduction to Organic Chemistry

INTRODUCTION TO ORGANIC CHEMISTRY 1


Structured / Free Response Questions
1 [DHS 2011/II/5]
In mammals, tyrosine forms adrenaline, a ‘fight or flight’ hormone.

Draw a pair of enantiomers for adrenaline.


[2]

2 [HCI 2011/II/5]
Pseudohalogen is a class of compounds that show a chemical reactivity similar to halogens.
An example is thiocyanogen, (SCN)2.

NC S S CN
Thiocyanogen
23
(i) Draw the shape of the hybrid orbitals around one of the carbon atoms in
thiocyanogen.
LL

(ii) Draw a diagram to illustrate the shape of thiocyanogen about the two sulfur atoms.

[4]

3 [RI 2011/II/4(d)]
Some alkenes, such as hex-3-ene, occur in two isomeric forms but both compounds P and Q
each occur in only one form.

Explain why P and Q do not exhibit the isomerism that hex-3-ene displays.
[2]
4 [RVHS 2011/III/1(c)]
Valine,H2NCH(COOH)CH(CH3)2 is able to exhibit stereoisomerism.

State the type of stereoisomerism which occurs in valine and draw two structures which
illustrate this isomerism.
[3]

1
Introduction to Organic Chemistry

5 [RVHS 2011/III/4(c)]
3-chloropropanoic acid is a drug used in scientific research due to its structural resemblance
to 4-hydroxybutanoic acid, a naturally occurring substance found in the central nervous
system.
O

Cl OH
3-chloropropanoic acid

A positional isomer of 3-chloropropanoic acid is 2-chloropropanoic acid. Draw the displayed


formula of the optically active isomer.
[2]

6 [TPJC 2011/III/4(a),(d)]
In 2002, Cargill Dow won the Greener Reaction Conditions Award for devising an efficient
way to use lactide (shown below) to make polymers, with applications in areas such as
textile, cutlery and food package, replacing the traditional petroleum polymers.
23
Lactide
LL

(a) Copy the molecule and circle all chiral carbons present in the molecule.
[1]

(b) 20 g of lactide is completely combusted in 50 dm3 of oxygen at 500 K and 101 kPa.

(i) Write a balanced equation for the combustion of lactide. Include state symbols.

(ii) What is the volume of the gases remaining under these conditions?
[4]

2
Introduction to Organic Chemistry

7 [NYJC 2011/III/5(a)]
In May 2011, the illegal use of the plasticiser DEHP in food and beverages was reported in
Taiwan. As more food products were tested, inspectors found extensive use DEHP and
DINP. This incident caused shock and panic among the citizens and had attracted
international attention. The structures of DEHP and DINP are given below.

O O

C9H19
O O
O
O
C9H19
O
O
2-ethylhexyl phthalate, DEHP Diisononyl phthalate, DINP

(i) Name the functional group, other than phenyl, which is common to both DEHP and
DINP.

(ii) Copy the structure of DEHP onto your writing paper, label clearly (with an *) the chiral
carbon(s) in the structure and determine the total number of  electrons that are
present in one molecule of DEHP. [3]
23
8 [SRJC 2011/III4(c)]
A common household weed killer, mecoprop has the following structure:
LL

O
OC(CH3)2COOH

Cl CH=CHCH3
(i) What type of isomerism is displayed by mecoprop?

(ii) [YJC 2011/II4(c)]


Draw the displayed structural formulae of the two isomers of 1-phenylpropene, to
illustrate the type of stereoisomerism involved.
[3]

3
Introduction to Organic Chemistry

9 [RI 2008/II/6(a)]

10 [Modified Nov 2011/II/5(c)]


Draw the displayed formulae of all the possible stereoisomers of K which have the following
features.
 They have molecular formulae C4H8O2 and have one C=C bond and two –OH groups.
 No oxygen atom is attached to any carbon atom which is involved in a double bond.
 No carbon atom has more than one oxygen atom joined to it.

For each structure you draw, state the type of stereoisomerism it shows and give one further
23
piece of relevant information about it.
You are not required to label functional groups.
[7]
LL

4
Introduction to Organic Chemistry

Multiple Choice Questions


1 [Nov 2011/I/19]

2 [ACJC 2010/I/22]
23
LL

3 [Modified HCI Promos 2003]

Which of the following molecules is not planar?

A Benzene
B Ethene
C Cyclohexene
D Ethyne

5
Introduction to Organic Chemistry

4 [NYJC 2010/I/20]

23
5 [SAJC 2010/I/22]
LL

6
Introduction to Organic Chemistry

6 [MJC 2010/I/24]

7 [PJC 2011/I/18]
23
LL

8 [CJC 2011/I/19]

9 [TPJC 2009/I/18]

7
Introduction to Organic Chemistry

10 [JJC 2011/I/21]

11 [MJC 2011/I/16]
23
LL

12 [NYJC 2011/I/21]

8
Introduction to Organic Chemistry

13 [RVHS 2011/I/18]

14 [RVHS 2011/I/19]
23
LL

9
Introduction to Organic Chemistry

15 [RVHS 2011/I/20]

16 [SRJC 2011/I/20]

23
LL

17 [TPJC 2010/I/28]

10
Introduction to Organic Chemistry

18 [SRJC 2011/I/21]

19 [VJC 2011/I/22]
23
LL

11
Introduction to Organic Chemistry

20 [YJC 2010/I/17]

21 [YJC 2011/I/19]

23
LL

22 [Nov 2010/I/2]
Dipyridamole is a drug that is used to treat recovering stroke patients.

Dipyridamole

What is the empirical formula of this drug?

A C6H9N2O B C6H10N2O C C11H20N4O2 D C24H40N8O4

12
Introduction to Organic Chemistry

Multiple Choice Questions (Section B)

1 [Nov 2011/I/33]

23
LL

2 [AJC 2010/I/36]

13
Introduction to Organic Chemistry

3 [NYJC 2010/I/37]

23
LL

4 [RI 2010/I/40]

14
Introduction to Organic Chemistry

5 [Nov 2011/I/37]

23
LL

15
Introduction to Organic Chemistry

INTRODUCTION TO ORGANIC CHEMISTRY SUGGESTED SOLUTIONS

Structured/Free Response Questions


1

**Tetrahedral arrangement abt chiral C is required.

2 (i)

(ii)

3 Compound P does not exhibit geometric isomerism because the trans-isomer is too
unstable to exist due to ring strain.
Compound Q does not exhibit geometric isomerism because one of the carbon involved
23
in the C=C bond has two identical groups attached to it.

4
Valine exhibits optical isomerism.
LL

**All covalent bonds must be shown clearly

6 (a) O O

O O

(b) (i) C6H8O4 (s) + 6O2 (g) -> 6CO2 (g) + 4H2O (g)
Balanced Equation AND all state symbols correct

16
Introduction to Organic Chemistry

(ii) Mr of lactide = 144.0


Amount of lactide used = 20/144.0 = 0.139mol
Amount of Oxygen added = n = PV/RT
= (101000)(0.050) / (8.31)(500) = 1.22mol
Oxygen used = 6 x 0.139mol = 0.834mol
Gases (CO2 and H2O) produced = 10 x 0.139mol = 1.39mol
Final amount of gases = 1.22 -0.834 + 1.39 = 1.78mol
Pressure of gas = 1.78/1.22 x 50 = 73.0 dm3

7 (i) ester
(ii) O *

O
* 10  electrons
8 (i) Geometric isomerism

(ii)
H H
23
C C
H
C H
C C
H H H C H
LL

H H

9 (i) Optical activity is the ability to rotate the plane of plane-polarised light.

(ii) Pair of enantiomers of any of the isomers below.


Draw 3-D structure of isomer and its corresponding mirror image

(iii)

17
Introduction to Organic Chemistry

10 Geometric isomers. There is restricted rotation about the C=C bond.


H
H
H O
O O H
H
H C
Note that ALL covalent bonds
H
C C
H
must be shown for displayed
C C
H
C C
H H formulae of organic
H C
O compounds.
H H
trans H
cis H
Optical isomers.
There is a chiral C atom with 4 different groups attached to it and so is optically active.

Note that ALL covalent bonds


must be shown for displayed
formulae of organic
compounds.

Multiple Choice Questions (Section A)


1. 2. 3. 4. 5. 6. 7. 8. 9. 10. 11. 12. 13. 14. 15. 16. 17. 18. 19. 20. 21. 22.
23
C B C D C C C C B B B C B D D B D C D A D B

Multiple Choice Questions (Section B)


LL

1. 2. 3. 4. 5.
A B D C D

Note: Do not assume that all the given answers are correct. Do discuss with your friends and check with your tutor
should you think that any given answer is incorrect.

18
Hydrocarbons

HYDROCARBONS 2
Structured / Free Response Questions
ALKANES

1 [VJC 2010/III/2(e)]
Tetraethyl-lead (IV), Pb(C2H5)4, is an example of an organometallic compound. It can be used
as an antiknock agent in petrol. At the high temperature in a car engine, the Pb-C bond in
Pb(C2H5)4, breaks to yield lead atoms and ethyl radicals.

The ethyl radical formed in this reaction reacts readily with chlorine gas to give chloroethane.
Another possible product is 1, 2-dichloroethane.

Outline the steps in the mechanism to show the formation of 1, 2-dichloroethane, starting from
chlorine gas and ethyl radical.
[2]

2 [MJC 2011/II/2]
Name and outline the mechanism in Step I to form A.
23
LL

[3]

3 [NYJC 2011/III/2(a)]
The term “octane rating” is often used in the petroleum industry for rating the ability of octane’s various
branched isomers in reducing engine knock in vehicles. Compounds A, B, C and D are isomers of
the hydrocarbon octane, C8H18.

The table below shows the boiling points and data relating to the optical activity of the compounds.

Structure Boiling Points / oC

Isomer A
CH3
119
H3C CH2 CH CH2 CH2 CH2 CH3
Isomer B
CH3 CH3
118
H3C CH2 CH CH CH2 CH3

1
Hydrocarbons

Isomer C
CH3 CH3
H3C C CH CH2 CH3 110

CH3
Isomer D
CH3 CH3
H3C C C CH3 107

CH3 CH
3

(i) Suggest why Isomer B has no optical activity even though it has 2 chiral centres.

(ii) Explain in terms of structure and bonding, why isomers A to D have different boiling
points.

(iii) Controlled chlorination of Isomer B in the presence of UV light produces 4 different


isomers of formula C8H17Cl.
Draw the structural formula of each of the 4 isomers and give the mole ratio in which they
are formed.
[8]

4 [RI 2011/III/3(d)]
23
A cycloalkane, C5H10, gives only two mono-brominated products (ignoring any stereoisomers)
with Br2 under uv light.
LL

(i) Give the structural formulae of the cycloalkane and the two mono-brominated products
formed.

(ii) Suggest the ratio in which the two mono-brominated products will be formed.
[4]

5 [RI 2011/III/3(b)(iv)]
For 2-methylbutane, the order of reactivity of tertiary, secondary and primary hydrogen atoms
follows a 5:4:1 ratio. Draw all the monochlorinated products formed when 2-methylbutane
reacts with chlorine and predict the ratio in which these products are formed.
[3]

2
Hydrocarbons

6 [TJC 2011/II/5(d)]
‘When a mixture of 2,4-dimethylpentane and chlorine is irradiated, reaction starts immediately.
However, the reaction does not stop immediately when irradiation stops’.

(i) Account for the above observation.

(ii) State the possible monochlorinated products and predict their mole ratio.

(iii) The hydrogen in an organic molecule can be classified as primary, secondary or tertiary
hydrogen as illustrated below.

H R’ R’
| | |
R–C–H R–C–H R – C – R’’
| | |
H H H

Primary H Secondary H Tertiary H

You are provided with the following bond dissociation enthalpy data:
Bond Bond dissociation enthalpy
/ kJ mol−1
23
H–Cl +432
Cl–Cl +243
H–CH2CH3 +420
LL

H–CH(CH3)2 +401
H–C(CH3)3 +390
Cl–CH2CH3 +338
Cl–CH(CH3)2 +339
Cl–C(CH3)3 +330

Using the equation for the monochlorination of an alkane below, estimate the respective
enthalpy changes of the monochlorination of primary, secondary and tertiary hydrogen
atoms in 2,4-dimethylpentane.

RH + Cl–Cl RCl + H–Cl

(iv) From your answer in (iii), suggest a factor that could have affected the yield of the
monochlorinated products from 2,4-dimethylpentane. Explain your answer.

(v) The mole ratio of 1°, 2° and 3° monochlorinated products was found to be 1.71 : 1: 1.43
experimentally.
Suggest a reason why there is a higher fraction of the 3° monochlorinated product
compared to that of the secondary monochlorinated product.
[10]

3
Hydrocarbons

7 [MJC 2009/III/1(d)]
Sulphuryl chloride is a very good source of Cl2. Propene undergoes free radical substitution with
sulphuryl chloride to give 3-chloroprop-1-ene.

475 K, uv light
CH2=CXHCH3 + SO2Cl 2 CH2=CHCH2Cl + SO 2 + HCl
Trace of hydrogen peroxide

(i) Name the type of hybridisation that is present in the carbon atom, Cx, of propene as
shown above. Sketch the shape of the hybrid orbitals of Cx.

(ii) The initiation stage of this mechanism is unique as it consists of two steps:

Step 1 Homolytic fission of hydrogen peroxide results in the formation of two hydroxyl
radicals.

Step 2 The hydroxyl radical will then combine with one of the Cl atoms of the sulphuryl
chloride to form two stable compounds and a chlorine radical.

1 Define the term homolytic fission.

2 Using the information given, deduce the equation for Step 2 of the initiation stage.
23
[4]

8 [HCI 2011/II/5(b)]
Thiocyanogen can react with hydrocarbons such as methylbenzene via free radical substitution.
LL

N
C
S
CH3 uv CH2
+ (SCN)2

(i) Write equations that describe the following steps.


• Initiation
• Propagation
• Termination

[3]

4
Hydrocarbons

ALKENES

1 [AJC 2011/III/3(a)]
Predict the organic products of the following reactions with KMnO4(aq):
(i) OH
hot acidified KMnO4

(ii) OH cold acidified KMnO4

(iii)
hot alkaline KMnO4

[3]

2 [SRJC 2011/II/3(a)(b)]
(a) A hydrocarbon burns in excess oxygen forming 480 cm3 of carbon dioxide gas and 0.36g
23
of water at room temperature and pressure. The hydrocarbon reacts with chlorine
dissolved in tetrachloromethane to form a compound with only 1 chiral centre.

(i) Deduce the structure of a hydrocarbon containing four carbon atoms which
LL

fulfils the above requirements.

(ii) Name the type of reaction and describe the mechanism of the reaction between
the hydrocarbon and chlorine.

(iii) The product from (a)(ii) is then heated with ethanolic sodium hydroxide. Three
different structural isomers of monochloroalkenes are obtained. Draw the
structures of any two monochloroalkenes.
[7]

(b) Aldehydes and ketones are produced industrially by the catalytic oxidation of alkenes, e.g.
ethanal is manufactured from ethene as shown below.

CH2=CH2 + ½ O2  CH3CHO

This process is also used industrially with but-2-ene.


Draw the displayed formula of the product which would be produced from but-2-ene.
[2]

5
Hydrocarbons

3 [MI 2011/II/3(a)]
The number of double bonds in a fat can be determined by its iodine number. The iodine number
is defined by the mass (in grams) of iodine that has reacted with 100 g of fats. As iodine is added
to the fat, the colour of iodine will not appear until all C=C double bonds are saturated.

Linolenic acid is a carboxylic acid containing 18 carbon atoms with an iodine number of 261.

(i) Write a balanced equation to show how CH3(CH2)14CHCHCOOH reacts with iodine.

(ii) Calculate the number of C=C double bonds in linolenic acid. Hence, deduce the
molecular formula of linolenic acid.

(iii) While the iodine number indicates the number of C=C bonds, it does not indicate the
positions of the C=C bond. Suggest a method that could allow the position of the C=C to
be determined in linolenic acid.
[5]

4 [YJC 2011/III/2(d)]
Cervonic acid is a polyunsaturated monobasic acid commonly found in fatty fish oil. The number
of carbon-carbon double bonds in a molecule of cervonic acid can be determined by titration with
iodine, I2, solution.

20.00 cm3 of 0.300 mol dm−3 iodine solution reacted with 0.328 g of cervonic acid. The molar
23
mass of cervonic acid is 328.0 g mol−1.

(i) Calculate the number of carbon-carbon double bonds in a molecule of cervonic acid.
LL

(ii) Given that there are 22 carbon atoms in a molecule of cervonic acid, deduce the molecular
formula of cervonic acid.
[3]

5 [SAJC 2011/III/2(b)]
Outline the mechanism for the reaction of 2-methylpropene and liquid iodine monochloride, ICl.

6 [TJC 2011/II/6(a)]
3-bromopropene can undergo reaction readily with HBr under room conditions to produce a
dibromo-compound.

(i) Draw the possible optical isomers of the dibromo-compound produced.

(ii) Explain why the resultant mixture formed by the reaction of 3-bromopropene and HBr is
optically inactive.

[4]

7 [IJC 2011/II/3(a)]
Alkenes are very reactive due to the presence of unsaturation.

6
Hydrocarbons

In the Koch reaction, alkenes can react with carbon monoxide and water to form carboxylic acids.
The reaction scheme below shows propene undergoing the Koch reaction.

CH3 + H3C O
H CO H2O
CH2 step I step II H
H CH3
carbocation intermediate final
organic product

(i) Alkenes undergo electrophilic addition in the presence of electrophiles such as H+ due to
the electron-rich π bond. Draw in the space provided above, the structure of the carbocation
intermediate formed from propene.

(ii) Draw in the space provided above, the structure of the final organic product.

(iii) Suggest a simple chemical test you would use to distinguish between the two types
of alkenes shown below.
23
LL

methylenecyclopentane cyclopentene
[4]

8 [RJC 2007/III/4]
Chloric(I) acid, HOCl, has the structure as shown:

Both chloric(I) acid and hydrogen chloride react with propene. In the reaction between chloric(I)
acid and propene, chlorine is added to carbon−1 of propene in the major product. However, in
the reaction between hydrogen chloride and propene, chlorine is added to carbon−2 of propene
in the major product.

(i) Draw the displayed formula of the major product formed between chloric(I) acid and
propene.
[1]

(ii) Explain briefly why chlorine from chloric(I) acid and from hydrogen chloride add to
different carbon atoms in their reactions with propene.
[3]
9 [RI 2011/II/4(a)-(e)]

7
Hydrocarbons

Compound P, known commonly as 2-(1-cyclohexenyl)ethylamine, is a clear, slightly yellow liquid


which is used as an intermediate reagent in the pharmaceutical industry. It is a member of a
large family of isomeric compounds with the formula, C8H15N.
NH2

compound P

Compound Q, N–allylpiperidine, is one such isomer.

N
compound Q

(i) At 298 K, the vapour pressure of P is 0.050 kPa while that for Q is 0.473 kPa.
Explain why P has a lower vapour pressure.

(ii) Give the structural formula of the major product formed when P is reacted with
23
• aqueous bromine
• a dilute solution of bromine in methanol
LL

(iii) Describe what you might observe when Q is heated with acidified potassium
manganate(VII), and give the structural formula of the organic product.

[7]

10 [HCI 2011/III/1(c)]
The bicyclic product B is a precursor for the synthesis of certain types of novel non-ionic
surfactants that help to improve the solubility, foaming action and conditioning action of a
shampoo formulation.

Describe the mechanism for the formation of the bicyclic product B formed in the following
reaction.
N
Br
NH 2

Br

B
[4]

8
Hydrocarbons

ARENES
1 [DHS 2011/III/1(c)]
Ferric chloride is an industrial scale commodity inorganic compound which is often used as
catalyst in organic synthesis. One example is its use as a Lewis acid for catalysing
the alkylation reaction of benzene by chloroethane to form ethylbenzene. This reaction is
similar to the reaction between benzene and chlorine.

(i) Write a balanced equation for the overall reaction of chloroethane and benzene.

(ii) State and outline the mechanism, with equations only, for the above reaction using ferric
chloride as a catalyst.

[3]

2 [IJC 2011/III/1(d)]
(i) Cl2 reacts with benzene in the presence of a suitable catalyst.
Describe the mechanism and suggest the product obtained.

(ii) The reaction in (i) can only take place under anhydrous conditions.
Suggest a reason to explain why it is so.
[5]
3 [JJC 2011/II/5(b)]
23
The reaction below is an example of an aromatic sulfonation organic reaction in which a
hydrogen atom on benzene is replaced by a sulfonic functional group.
LL

SO3H
concentrated H2SO4
+
SO3
heat under reflux
for several hours

In one proposed mechanism for the above reaction, concentrated H2SO4 acts as a catalyst. In
the first step of the reaction, concentrated H2SO4 reacts with SO3 to generate a cation SO3H+.

(i) Write a balanced equation for the first step of the reaction.

(ii) Complete the mechanism below to describe the subsequent step(s) in the above reaction.
Include curly arrows to show the movement of electrons and all charges.

+ SO H+
3

[3]

4 [MI 2011/III/5(b)]
Benzene is a common starting reagent for many reactions.

9
Hydrocarbons

(i) Describe structural isomerism.

(ii) Describe and explain the shape of benzene in relation to sigma and pi bonds.

(iii) Ethylbenzene reacts with chlorine in the presence of UV light. Draw the displayed formula
of the products that can be formed during the reaction. Describe the type of isomerism
involved.

[5]

5 [TJC 2011/II/3(b)]
In electrophilic substitution reactions, AlCl3 can function as a Lewis acid catalyst to generate the
electrophile from heterolytic fission.

(i) Deduce whether Al2Cl6 can also function as a catalyst in electrophilic substitution reactions.

(ii) Describe the mechanism of the reaction when benzene is reacted with ethanoyl chloride,
CH3COCl, in the presence of AlCl3.
[4]

6 [RI 2011/III/4(e)]
Another aluminium compound that has important uses in organic chemistry is aluminium
chloride. AlCl3 is produced when dry chlorine is passed over heated aluminium foil in a hard
23
glass tube.
LL

(i) With the aid of an equation, explain why aluminium chloride can act as a Lewis acid catalyst
in step IV.

(ii) Using your answer to (e)(ii), describe the mechanism of the reaction in step IV.

(iii) Suggest why the yield might be low in step IV.


[5]

7 [2011 HCI Promos/II/C3]


(a) Ethylbenzene is an example of an aromatic hydrocarbon.

10
Hydrocarbons

(i) Write a balanced equation for the oxidation of ethylbenzene using hot, acidified
KMnO4.

(ii) Given that the percentage yield is 80%, what is the mass of the organic product
formed when 1.00 g of ethylbenzene undergoes the oxidation reaction in (a)(i)?

(iii) Ethylbenzene can be prepared by the reaction of benzene with chloroethane,


CH3CH2Cl. This reaction is Friedel-Crafts alkylation and it occurs via a mechanism
similar to that of the chlorination of benzene.

AlCl3
+ CH3CH2Cl CH2CH3 + HCl

ethylbenzene

I Explain why the reaction above would not occur in the absence of AlCl3.

II Describe the mechanism of this reaction.


[7]

(b) Deuterium, D, is a heavy isotope of hydrogen.


23
When deuteriobenzene undergoes Friedel-Crafts alkylation as in (a)(iii), a mixture
containing ethylbenzene and 3-ethyldeuteriobenzene is produced.

D
LL

CH2CH3

deuteriobenzene 3-ethyldeuteriobenzene
Under the conditions used for alkylation, the carbon-deuterium bond is broken as often as
a carbon-hydrogen bond.

Suggest why the molar ratio of ethylbenzene to 3-ethyldeuteriobenzene produced is 1:2.


[2]

(c) The C-D bond actually has a higher bond strength than a C-H bond, yet predeuterated
benzene, C6D6, is alkylated at the same rate as normal benzene, C6H6.

What conclusion can you draw about the mechanism of the alkylation reaction?
[1]
(d) Suggest how you would distinguish the following compounds by carrying out only
two simple chemical tests. You should state the reagents and conditions for each test, and describe
the expected observations for each compound.

11
Hydrocarbons

CH3

Cyclohexene benzene methylbenzene


[4]

(e) (i) The enthalpy change of hydrogenation of an unsaturated hydrocarbon is the


enthalpy change when one mole of the compound reacts completely with hydrogen
to form a saturated hydrocarbon. The enthalpy change of hydrogenation of
cyclohexene, C6H10(l), is −120 kJ mol−1.

Write an equation of the reaction that represents the enthalpy change of


hydrogenation of cyclohexene.

(ii) Naphthalene, C10H8(s), is another example of an aromatic hydrocarbon. When


naphthalene is fully hydrogenated, decalin, C10H18(s), is formed. The enthalpy
change of hydrogenation of naphthalene is −345 kJ mol−1.

hydrogenation
23
naphthalene decalin

The resonance energy of naphthalene can be defined as the energy difference


between the actual structure of naphthalene (I) and the hypothetical polyalkene
LL

structure (II).

actual structure hypothetical polyalkene


of naphthalene structure
(I) (II)
Calculate the resonance energy of naphthalene, by drawing a suitable energy level
diagram, and incorporating relevant data from (e).

(iii) Explain the significance of the resonance energy.

(e) (iv) Using the information from above, calculate an approximate value for the enthalpy
change of hydrogenation for the actual structure of anthracene.

12
Hydrocarbons

actual structure hypothetical polyalkene


of anthracene structure
(III) (IV)

[6]
[Total: 20 marks]

8 [RI 2001 Promos/C3]


The molecular formula of organic compound A is C9H8. B, C9H10O2, is obtained when cold
alkaline potassium manganate(VII) is added to A. However, when hot acidified potassium
manganate(VII) is added to A, the product is C, C8H6O4, instead of B. Mononitration of C gives
a mixture of products. C can also be synthesized via the following reaction scheme:
CH3

CH3Cl [O]
C
AlCl3

A, when treated with bromine under different conditions, decolourises bromine and yields D,
C9H8Br2 and E, C9H7Br3, respectively.
23
(a) Suggest possible structural formulae for A – E and explain the reactions involved.

(b) Explain whether A exhibits stereoisomerism.


LL

[10]

13
Hydrocarbons

Multiple Choice Questions (Section A)

ALKANES

1 [YJC 2011/I/23]

2 [YJC 2011/I/20]
23
LL

3 [HCI 2010/I/21]

14
Hydrocarbons

4 [AJC 2010/I/22]

5 [MJC 2010/I/20]
23
LL

6 [NYJC 2010/I/23]

7 [PJC 2010/I/19]

15
Hydrocarbons

8 [HCI 2011/I/20]

9 [HCI 2012/I/20] 23
LL

10 [TJC 2011/I/29]

16
Hydrocarbons

11 [PJC 2011/I/22]

12 [Nov 2008/I/24]
Non-ionic detergents can be made by reaction of epoxyethane, in an excess, with a C11
alcohol.

A possible mechanism involves homolytic fission of a C−O bond in epoxyethane giving rise to a
‘double-ended’ free radical that initiates a chain reaction.
The first propagation step is as follows.
23
After termination of the reaction with an alcohol, what is a possible formula of such a non-ionic
LL

detergent?

13 [RVHS 2011/I/22]

17
Hydrocarbons

14 [Modified SRJC 2011/I/24]

CH(Br)CH3 CH(CH3) 2

23
LL

15 [JJC 2011/I/22]

18
Hydrocarbons

ALKENES

16 [IJC 2011/I/25]

17 [MJC 2011/I/17]
23
LL

18 [NYJC 2011/I/23]

19
Hydrocarbons

19 [CJC 2010/I/21]

20 [PJC 2011/I/19]

23
LL

21 [RVHS 2011/I/23]

20
Hydrocarbons

22 [RVHS 2011/I/21]

23
LL

23 [SAJC 2011/I/22]

24 [SRJC 2011/I/23]

21
Hydrocarbons

25 [TJC 2011/I/25]

23
LL

26 [PJC 2010/I/20]

22
Hydrocarbons

27 [JJC 2011/I/30]

23
LL

28 [RI 2009/I/21]

23
Hydrocarbons

29 [ACJC 2010/I/26]

23
30 [SAJC 2010/I/28]
LL

31 [AJC 2009/I/25]

24
Hydrocarbons

32 [ACJC 2011/I/20]

23
LL

25
Hydrocarbons

33 [VJC 2011/I/24]

23
LL

26
Hydrocarbons

34 [TJC 2011/I/30]

23
LL

27
Hydrocarbons

35 [NJC 2010/I/30]

23
LL

36 [VJC 2011/I/19]

28
Hydrocarbons

37 [HCI 2011/I/27]

[HCI 2011/I/21]
38
23
39 [HCI 2011/I/22]
LL

29
Hydrocarbons

ARENES

40 [HCI 2010/I/20]

41 [CJC 2011/I/23]
23
LL

42 [HCI 2012/I/19]

30
Hydrocarbons

43 [NYJC 2010/I/25]

44 [TJC 2011/I/19]
23
LL

45 [DHS 2011/I/21]

31
Hydrocarbons

Multiple Choice Questions (Section B)

ALKANES

1 [NYJC 2011/I/37]

23
2 [TPJC 2011/I/39]
LL

32
Hydrocarbons

3 [JJC 2009/I/37]

23
LL

4 [NJC 2009/I/37]

33
Hydrocarbons

ALKENES

5 [HCI 2010/I/36]

23
6 [SAJC 2010/I/39]
LL

1 CH3 CH CH2

Br Br

2 CH3 CH CH2

ONO2 Cl

3 CH3 CH CH2

OH Cl

34
Hydrocarbons

7 [NJC 2011/I/36]

23
LL

8 [SAJC 2010/I/32]

9 [SAJC 2010/I/38]

35
Hydrocarbons

10 [TJC 2011/I/37]

11 [H1 HCI 2009/I/29]


23
LL

12 [AJC 2009/I/39]

36
Hydrocarbons

ARENES

13 [RI 2009/I/38]

23
LL

14 [AJC 2009 H1/I/27]

37
Hydrocarbons

15 [ACJC 2011/I/38]

23
LL

38
Hydrocarbons

HYDROCARBONS SUGGESTED SOLUTIONS

Multiple Choice Questions (Section A)


1. 2. 3. 4. 5. 6. 7. 8. 9. 10. 11. 12. 13. 14. 15. 16. 17. 18. 19. 20.
A B B D C D C A C C C C A C B B A C C B
21. 22. 23. 24. 25. 26. 27. 28. 29. 30. 31. 32. 33. 34. 35. 36. 37. 38. 39. 40.
B C C B D D A A D A A C A B A D B D D C
41 42 43 44 45
B B B A D

Multiple Choice Questions (Section B)


1. 2. 3. 4. 5. 6. 7. 8. 9. 10. 11. 12. 13. 14. 15.
C A A A B D B B A D B D B C B

Structured/Free Response Questions


ALKANE

1 •CH2CH3 + Cl2 → CH3CH2Cl + Cl•


CH3CH2Cl + Cl• → •CH2CH2Cl + HCl
•CH2CH2Cl + Cl2 → ClCH2CH2Cl + Cl•

2 Free radical substitution


23
u.v. light
Cl  Cl
LL

(a)

(b)

then (a), (b), (a), (b)…….

39
Hydrocarbons

3 (i) (There is a plane of symmetry in the molecule or both chiral centres contain the
same groups attached to it), the two chiral centres rotate the plane of polarised
light to the same extent but in the opposite direction hence cancelling out the optical
activity.

(ii) Isomers A to D are simple molecular compounds with weak van der Waals
forces between molecules. However, as we move down the table from A to D, the
molecules become increasingly branched, hence they have less surface area of
interaction with their neighbouring molecules resulting in weaker vdw forces
between molecules. Hence less energy is required to break the vdw bonds,
resulting in lower b.p. down the table.

(iii) Structural formula mole ratio


CH3 CH3 6 3
Cl CH2 CH2 CH CH CH2 CH3
Cl CH3 CH3 4 2
H3C CH CH CH CH2 CH3
CH3 CH3 2 1
H3C CH2 C CH CH2 CH3

Cl
Cl 6 3
23
CH2 CH3
H3C CH2 CH CH CH2 CH3
LL

4 (i) Br

Two mono-brominated products: Br

(ii) Ratio of the above products = 6:4 = 3:2

40
Hydrocarbons

5 H Cl
H H H H H H

Cl C C C C H C C C H
H C
H H H H H
H
H C H H C H
H H
6x1=6 1x5=5
H H
H H H H H H

H C C C C H C H
H C C C
H Cl H Cl
H H
H C H H C H
H
H
2x4=8 3x1=3
Expected ratio = 6:5:8:3

6 (i) Irradiation of Cl2 results in homolytic cleavage to give Cl radicals which are highly
reactive. Reaction starts immediately. When irradiation stops, the radicals are still
23
being regenerated and the reaction continues.
(ii) Mole ratio of 1°, 2° and 3° monochlorinated products 6 : 1 : 1
(iii) ∆H = −∆HRCl − ∆HHCl + ∆HRH + ∆HCl-Cl
LL

For 1° Hydrogen ∆H = 243 + 420 − 338 − 432 = −107 kJ mol−1


For 2° Hydrogen ∆H = 243 + 401 − 339 − 432 = −127 kJ mol−1
For 3° Hydrogen ∆H = 243 + 390 − 330 − 432 = −129 kJ mol−1

(iv) From (iii), the enthalpy change becomes more negative for the substitution of 1° to
3° hydrogen implying that the formation of 2-chloro-2,4-dimethylpentane is more
spontaneous. A factor that could give rise to this result is the strength of C-H
bonds/relative stability of the alkyl radicals.

(v) The tertiary radical formed is more stable than the secondary radical as there are
more electron donating alkyl groups attached to the electron deficient carbon with the
lone electron.

7 (i) sp2 hydrisation

three sp2

41
Hydrocarbons

(ii) Homolytic fission is the breaking of a covalent bond such that each atom retains only
one of the shared electrons.

HO• + SO2Cl2  HOCl + SO2 + Cl•

8 (i)
uv
Initiation (SCN)2 SCN

Propagation CH3 CH2


(a) + SCN + HSCN

CH2
H2C SCN
(b) + (SCN)2
+ SCN

Then, (a), (b), (a), (b)……


23
Termination SCN + SCN (SCN)2
LL

CH2 H2C SCN

+ SCN

CH2 CH2
+

ALKENES

42
Hydrocarbons

1 OH
HO2C CO2H
HO -
O2C
HO2C OH CO2-
(i) (ii) (iii)

2 (a) (i) 480


Amount of CO2 = = 0.02 mol
24000
Amount of C = 0.02 mol
0.36
Amount of H2O = = 0.02 mol
18.0
Amount of H = 0.04 mol

Mole ratio of C: H = 1:2


∴ Empirical formula of the hydrocarbon is CH2.

Since it has 4 carbon atoms, molecular formula = C4H8


H H
Structure of hydrocarbon is
C C
H CH2CH3
23
(ii)
LL

2 (a) (iii)

(b)

ALL BONDS MUST BE SHOWN FOR DISPLAYED FORMULAE

43
Hydrocarbons

Follow pattern
For ethene:
=O forms
here
H H
C C
H H
One H
moves to
adjacent C

Hence, for but-2-ene:

=O forms
here
CH3
CH3 CH3 CH3
23
C C H C C
H H O
H
One H
LL

moves to
adjacent C

3 (i) CH3(CH2)14CHCHCOOH + I2  CH3(CH2)14CHICHICOOH

(ii) Assume molecular formula of linolenic acid be C17H36COOH,


261
Amount of I2 = = 1.027
127 × 2
100
Amount of linolenic acid ≈ = 0.350
(17 × 12) + 36 + 45
1.0275
No. of C=C = ≈ 2.92 = 3 (3 double bonds)
0.350
Molecular formula is C18H30O2

(iii) Add hot, concentrated acidified KMnO4 to linolenic acid. Analyse the oxidation
products. The position of C=C could be determined by determining the location of
C=O.

44
Hydrocarbons

4 (i) Amount of I2 = 0.300 × (20.00 × 10−3) = 6.00 × 10−3 mol


Amount of cervonic acid = 0.328 ÷ 328 = 1.00 × 10−3 mol
n(I2) : n(cervonic acid) = 6 : 1
6 C=C double bonds
4 (ii) C22H32O2 or C21H31COOH

5 Electrophilic addition

6 (i)
23
CH2Br CH2Br
• 1 mark for correct product
• 1 mark for optical isomers
C C
LL

H3C H H CH3
Br Br Br

(ii) 3-bromopropene undergoes electrophilic addition with HBr.


In the first step, a carbocation and Br- ion is formed. The carbocation (CH2BrC+HCH3)
has a trigonal planar shape with respect to the positively charged carbon atom.
In the second step, the Br- ion can attack the positively charged carbon atom from
both sides of the plane with equal probability.

A racemic mixture that does not rotate the plane of polarized light is formed as the
optical activity cancels out..

7 (i) CH3
CH3
H

(ii) H3C O

H
CH3 OH

45
Hydrocarbons

(iii) Reagent(s): KMnO4


Condition(s): dilute H2SO4, heat

Observation for methylenecyclopentane:


Purple KMnO4 decolourises , effervescence observed, gas evolved forms
white ppt with Ca(OH)2 (aq)
Observation for cyclopentene:
Purple KMnO4 decolourises, no effervescence observed

8 (i)

(ii) Propene undergoes electrophilic addition and the first step involves the addition of
the electrophile to carbon−1 as the more stable secondary carbocation is formed:
23
LL

The more stable carbocation is the one that has more electron donating alkyl groups
bonded to the C with the positive charge. This helps to decrease the intensity of the
positive charge to a greater extent.

In HOCl, Cl is partially positive so that Cl+ is the electrophile and Cl+ adds to the
carbon−1 to form the more stable carbocation.
Subsequent addition of OH− results in the formation of the major product with
Cl added to carbon−1.

In HCl, Cl is partially negative so that the electrophile is H+ rather than Cl+.


Hence H+ adds to carbon−1 to form the more stable carbocation.
Subsequent addition of Cl− to carbon−2 results in the formation of the major product
with Cl added to carbon−2.

9 (i) P has a lower vapour pressure due to intermolecular hydrogen bonding (in addition
to its van der Waals’ forces) but there are only van der Waals’ forces between the
molecules of Q. There is stronger attraction between P molecules.

46
Hydrocarbons

(ii) NH2 NH2

OH OCH3
Br Br

(iii) Purple KMnO4 decolourises.


Effervescence of colourless gas / gas evolved that gives a white ppt with
Ca(OH)2(aq).

O
N+
OH
H

10 Electrophilic Addition
23
LL

ARENES

47
Hydrocarbons

1 (i) CH2CH3
FeCl3
+ CH CH Cl + HCl
3 2

(ii) Electrophilic substitution.


+ CH + CH +
FeCl3 3CH2Cl FeCl4 3CH2

H3CH2C H
slow
+ CH CH +
3 2

H3CH2C H CH2CH3

fast
+ FeCl + HCl
-
+ FeCl
4 3
23
2 (i) Product
Cl
LL

Mechanism: Electrophilic Substitution


Step 1: Generation of electrophile
Cl2 + AlCl3 Cl + + [AlCl4] -
catalyst electrophile nucleophile

Step 2: Electrophilic attack on benzene ring


H Cl
slow
Cl+ +

Step 3: Loss of proton


H Cl Cl

fast
+ + H+

Step 4: Regeneration of catalyst


H+ + [AlCl4] - AlCl3 + HCl
nucleophile catalyst

48
Hydrocarbons

(ii) In the presence of water, AlCl3 hydrolyses in water to form acidic solution OR

Hence under hydrated conditions, AlCl3 cannot accept a lone pair of electrons from
Cl2,thus electrophile Cl+ cannot be generated. Thus the condition need to be
anhydrous.

3 (i) SO3 + H2SO4 → SO3H+ + HSO4−

(ii) slow H
+ SO3H
SO3H

-
fast HSO4

SO3H + H SO
2 4
23
4 (i) Structural isomerism arises when molecules have the same molecular formula but
the atoms are arranged differently to form molecules.

(ii) Planar. Each carbon atoms forms 3 sigma bonds with other atoms. Hence, each
LL

carbon atom undergoes sp2 hybridisation leading to its planar shape.;

(iii) Cl H H H
H C C H H C C Cl
H H

Positional isomerism

5 (i) No, because Al2Cl6 is not electron-deficient and is unable to accept a lone pair to form
a dative bond.

49
Hydrocarbons

(ii) Step 1: Generation of electrophile


AlCl3 + CH3COCl +
COCH3 + AlCl4−

Step 2: H
+ +COCH slow
3
+ COCH3

COCH3
Step 3: H
fast
+ COCH3 + AlCl3
+ AlCl4- + HCl
6 (i) OCH2CH2Cl OCH2CH2+

+ AlCl3 23 + AlCl4

NH2 NH2

Al is electron deficient/ has vacant low-lying orbitals to act as electron pair acceptor,
thus acting as a Lewis acid catalyst.
LL

(ii) Name: Electrophilic


+
aromatic substitution
OCH2CH2 O

slow
+ H

NH2 NH2

O O

Cl AlCl3
fast
+ HCl
+ H
+ AlCl3

NH2 NH2
(iii) AlCl3, being a Lewis acid, can react with the basic phenylamine / intermolecular
electrophilic substitution may take place instead of intramolecular substitution.

7 (a) (i) for correct products formed and balancing equation [1]

50
Hydrocarbons

CH2CH3 CO2H

+ 6[O] + CO2 +
heat 2H2O

(ii) No. of moles of ethylbenzene is 1.00/106 = 0.00943 mol


mass of organic product is = 0.00943 x 122 x 0.8 = 0.920 g [1]

(iii) I AlCl3 act as catalyst/electron acceptor to form CH3CH2+ which is a strong


electrophile. A stronger electrophile needs to be generated due to the
stability of benzene. [1]

II Name of mechanism: Electrophilic substitution [1]

CH3CH2Cl + AlCl3  CH3CH2+ + AlCl4- [1]

H CH2CH3
slow
23
+ CH CH +
3 2
electrophile carbocation
intermediate
LL

(b) To form ethylbenzene, the ethyl cation can only substitute the deuterium atom.

To form 3-ethyldeuteriobenzene the ethyl cation can substitute the H atom from
two possible identical positions. Statistically twice the amount of products are
3-ethyldeuteriobenzene.Thus the ratio 1:2

(c) The rate determining step does not involve the breaking of C-D bond.

(d) Any two suitable reagents and observations.

51
Hydrocarbons

Reagents

CH3
Adding liquid bromine reddish brown no no decolourisation
(room temperature; Br2 is decolourisation
absence of light) decolourised
Adding KMnO4 / purple KMnO4 no purple KMnO4 is
H2SO4(aq)heat is decolourised decolourisation decolourised;
formation of white ppt
Concentrated HNO3 No yellow oil yellow oil of yellow oil
and concentrated nitrobenzene
H2SO4 (heat)

(e) (i)
(l) + H2(g) (l)

(ii) Resonance energy = − 600 − (−345) = 255 kJ mol−1


(ii) Energy

-----------------------------------------------------------------------------------------------------
-600-(-345) =
23
-255 kJ mol-1
stabilisation/ resonance energy

-120(5)
LL

---------------------------------------------------------------
5H2
5H2(g)
-345 kJ mol-1

stabilisation/ resonance energy is 255 kJ mol-1

(iii) Naphthalene is more stable than its hypothetical structure


due to the delocalization of the π electrons throughout the ring.
OR therefore naphthalene undergoes substitution rather than addition.

(iv) Heat of hydrogenation = 7(-120) + 3/2(255) = -457.5 kJ mol-1

(multiply by 3/2 as anthracene has 3 rings but naphthalene 2 rings)


OR Heat of hydrogenation = 7(-120) + 7/5(255) = -483 kJ mol-1

(multiply by 7/5 as anthracene has 7 C=C but naphthalene has 5 C=C)

52
Hydrocarbons

8 (a)

COOH
COOH
NO2
23
NOTE: C cannot be COOH , as upon mono-nitration, only COOH will be
LL

obtained, since substitution at any of the 4 remaining sites on the ring gives
the same product.

53
Hydrocarbons

23
LL

(b) No. Absence of chiral center and the C=C double bond in the ring does not exhibit
geometric isomerism due to ring strain.

Note:
Do not assume that all the given answers are correct.
Do discuss with your friends and check with your tutor should you think that any given
answer is incorrect.

54
Halogen Derivatives

HALOGEN DERIVATIVES 3
Structured / Free Response Questions

1 [PJC 2011/II/7(a)]

The reaction scheme shown below outlines how compounds A, B and C can be formed
from (1-chloroethyl)cyclohexane.

Cl CH3
O CH3 HO
step I step II step III
Compound A
NaOH(aq), heat K2Cr2O7(aq),
H2SO4(aq), heat

2-chloroethylhexane

Excess concentrated s
o
H2SO4, 170 C
NaOH in ethanol, heat
23
CH3 CH2NH2

Compound
Compound B + Compound C (minor)
B (major) step V
Comp
LL

(i) Suggest the structural formulae for the compounds A, B and C.


Compound D
(ii) When optically active 2-chloroethylhexane reacted with NaOH(aq) in step I, the
chiral centre of the product, compound A, is inverted. Describe the mechanism in
step I
[6]

1
Halogen Derivatives

2 [SAJC 2011/III/4(a)]

An experiment was set up to determine the mechanism of 2-bromobutane, FA3, of


concentration 0.01 mol dm-3 and sodium hydroxide, FA4, of concentration 0.01 mol
dm-3.
The results were tabulated in the following table:

Experiment Vol. of FA3 Vol. of FA4 Vol. of ether used Rate /


number used / cm3 used / cm3 / cm3 mol dm-3 s-1
1 1.00 29.00 20.00 8.17 x 10-7
2 1.00 35.00 14.00 9.86 x 10-7
3 7.50 2.50 40.00 5.28 x 10-7

(i) Use the data in the table to deduce the order of reaction with respect to
2-bromobutane and sodium hydroxide.

(ii) Write an overall rate equation for the reaction between 2-bromobutane
and sodium hydroxide.

(iii) Hence, outline the mechanism between 2-bromobutane and sodium


hydroxide.
23
[7]
3 [PJC 2011/III/4(c)]
LL

Student A from Pioneer Junior College found a bottle of a liquid substance that is labeled,
C6H9Br. He suggested that the unknown has the following structure.

CH3

Br
(i) Plan two chemical reactions that can positively confirm the suggested structure.
Include your observations and deductions for each test.

(ii) Compound Z can be formed by the reaction sequence below.

CH3 CH3

step I step II
Y

Br
Z
Suggest the structural formula of compound Y. State the reagents and conditions
in each of the steps.

2
Halogen Derivatives

4 [JJC 2011/III/2(c)]

Lithium aluminium hydride, LiAlH4, can reduce epoxides to alcohols. An example


is shown below.

Epoxy resins are polymers which are used as adhesives. One of the monomers
used in their manufacture has the formula:

i) Write a balanced equation for the reaction that occurs when LiAlH4 is added
to the monomer above.

The same monomer can be produced from propene via the following sequence:
23
LL

Na metal

ii) State the reagents and conditions required for steps I and II.
iii) Name the type of reaction that occurs in step IV.
[4]

3
Halogen Derivatives

5 [IJC 2011/III/3(b)]

CH3 CH3
1. NaOH(aq)

Route 1: 2. CH3Cl
HO CH3 O

CH3 - + CH3
CH3O Na
Route 2:

Cl CH3 O
Part of the reaction scheme for the formation of Atenolol involves the synthesis
of the ether functional group via two possible routes:

Which route will give a greater yield of the product? Explain your
reasoning.
23
NOTE: Route 1 step 1 takes place as follows:

+
+ NaOH Na-O
LL

HO CH3 CH3 + H2O

4
Halogen Derivatives

6 [IJC 2011/II/6(ai)-(iii)]

Compound W can be synthesised from compound T via a series of steps.

23
LL

(i) State the reagents and conditions for step I and step II.

(ii) Draw the structures of compound U.

(iii) All 3 steps are the same type of reaction. State the type of reaction for step I to
step III.

[5]

5
Halogen Derivatives

7 [MJC 2011/II/2]

The flow scheme below involves the reaction of D.

Step 1 Br2 in CCl 4


E CH2=CHCH2CH2CH2OH F
D
Step 2 excess CH 3NH2
heat

CH2(COOH)2
G

(i) Draw the structures for E, F and G in the boxes provided below.

(ii) State the reagents and conditions for Step 1 and 2 in the spaces provided.

(iii) G contains a chiral carbon but the eventual reaction mixture was found to be optically
inactive. Suggest a reason for this observation.

(iv) State the type of hybridisation with respect to the carbon atom indicated below by the
asterisk * in compound D.
23
CH2=*CHCH2CH2CH2OH

Sketch the shape of the hybrid orbitals of this carbon and indicate its bond angle.
LL

[9]

8 [TPJC 2011/II/4]
Phentermine is a central nervous system stimulant that is used most frequently in the
treatment of obesity.
NH2

Phentermine

(a) Draw the structural formula of the tertiary alkyl chloride, RX, which can react with
ammonia, NH3, to synthesise Phentermine.

(b) Using the structural formula drawn in (a), write a balanced equation for the reaction
between RX and NH3.

(c) In a series of experiments run with different concentrations of reactants, the initial

6
Halogen Derivatives

rates of the formation of Phentermine were determined. The results are shown in the
table.

Experiment Initial concentration/ Initial rate/

(moldm-3 x 10-3) (moldm-3s-1 x 10-7)

[RX] [NH3]

1 5.0 5.0 2.8

2 5.0 10.0 2.8

3 10.0 5.0 5.6


[9]
(i) State the order of the reaction with respect to RX and NH3 respectively.

(ii) Sketch the graphs of [reactant] against time for RX and NH3 separately.

(iii) Using the data given, derive the rate law for the synthesis of Phentermine.
Hence, state the type of mechanism for the reaction.
23
(iv) Illustrate the mechanism stated in c(iii) by writing suitable equations.

(d) Suggest the type of mechanism of the reaction between chloroethane and NH3.
LL

Explain the difference in the mechanisms for the tertiary chloroalkane, RX, and
chloroethane.
[10]

7
Halogen Derivatives

9 [SRJC 2011/III/4(a)]

Herbicides, also known as weedkillers are widely used in agriculture.

2,4,5-Trichlorophenoxyacetic acid (2,4,5-T) was a widely used herbicide until being phased
out starting in the late 1970s due to health reasons.

2,4,5-T can be synthesised by the following process:

Cl Cl Cl Cl NaOH Cl Cl
NaOH, in excess

High Temperarure
Cl Cl Cl OH Cl O-Na+
I

II ClCH2COOH

Cl Cl

Cl O CH2COOH
(i) What is unusual about reaction I? Explain your answer.
(ii) What type of reaction is reaction II? Describe the mechanism involved.
[5]
23
LL

10 [DHS 2011/III/4(b)]

When sodium ethanoate, CH3COONa+ is heated with 2–chloro–2–methylpropane,


(CH3)3CCl, forming a sweet–smelling liquid, K is formed.

(i) Suggest the structural formula of K.

(ii) Suggest the type of reaction when sodium ethanoate reacts with (CH3)3CCl.

(iii) Propose, with the aid of appropriate equations, the reaction mechanism for the
organic synthesis in part (ii).

(iv) Predict, with reasoning, the solubility of 2–chloro–2–methylpropane, (CH3)3CCl in


water.
[7]

8
Halogen Derivatives

Multiple Choice Questions (Section A)

REACTIONS

1 [AJC 2011/I/23 modified]

Chlorine, in the presence of aluminium chloride, undergoes a substitution reaction with


benzene forming chlorobenzene. When (1chloroethyl)benzene reacts with iodine
monochloride, ICl, in the presence of aluminium chloride, a similar reaction occurs and
compound U is formed.

CHClCH3

(1chloroethyl)benzene

Which product will be present in the greatest yield when U is heated under reflux with aqueous
sodium hydroxide?

A Cl CH(OH)CH3
23
B HO CH(OH)CH3
LL

C I CH(OH)CH3

D HO CHClCH3

9
Halogen Derivatives

2 [CJC 2011/I/25 modified]

1 mole of an organic compound P undergoes a reaction with alcoholic sodium hydroxide to


form 2 moles of HBr.

Which of the following could be P?

A Br B Br
CHBrCH3 CH2Br

C Br D
CH2Br

3 [CJC 2011/I/26]

Which pair of the following reactions could have the same reaction intermediate?
23
W CH3CH(OH)CH3  intermediate  CH3CH(CN)CH3
X CH3CH2CH2OH  intermediate  CH3CHBrCH3
LL

Y CH3CH=CH2  intermediate  CH3CH(OH)CH3


Z CH3CHBrCH3  intermediate  CH3CH(COOH)CH3

A W and X C X and Z
B W and Y D Y and Z

10
Halogen Derivatives

4 [IJC 2011/I/22 modified]

Organic compound T underwent the following successive reactions:

I reaction with thionyl chloride


II reaction with hot alcoholic potassium hydroxide
III reaction with hydrogen chloride

The final organic product was 2-chloropropane.

Which of the following could most likely be compound T?


A propene
B propan-1-ol
C 1-chloropropane
D propanoic acid

5 [NYJC 2011/I/23]

Which of the chloroalkanes, with formula C5H11Cl, gives the largest number of isomeric
products when heated with ethanolic potassium hydroxide?
23
CH3
CH3
LL

A B CH3 C CH2Cl
CH3 CH2 CH CH2Cl
CH3
CH3
Cl
C D CH3 CH2 C Cl
CH3 CH2 CH2 CH CH3
CH3

6 [NYJC 2010/I/26]

What is the product of a nucleophilic substitution reaction between 2-iodobutane and


sodium ethoxide (CH3CH2O‾Na+)?

A CH3CH=CHCH3 B CH3CH2CH(OH)CH2CH3
C CH3CH2CH(CH3)OCH2CH3 D (CH3 )2CHCH2OCH2CH3

11
Halogen Derivatives

7 [YJC 2011/I/22]

Four drops of 1-chlorobutane, 1-bromobutane and 1-iodobutane were put separately into
three test-tubes containing 1.0 cm3 of ethanolic silver nitrate at 60 C.

The rate of formation of cloudiness in the tubes was in the order C4H9Cl < C4H9Br < C4H9I.

Why is this so?


A The C-X bond polarity decreases from C–Cl to C–I.

B The solubility of AgX(s) decreases from AgCl to AgI.

C The ionisation energy of the halogen decreases from Cl to I.

D The bond energy of C-X decreases from C−Cl to C−I.

8 [TPJC 2011/I/27]

2,2–dimethylpropylamine can be produced by this 2-step reaction scheme starting with


compound E.
23
KCN (ethanol) H2(g) CH C(CH ) CH NH
E
reflux Pt 3 3 2 2 2

2,2–dimethylpropylamine
LL

Which one of the following structures represents E?

A CH C(Br)(CH ) B BrCH CH(CH )


3 3 2 2 3 2

C CH (CH ) CH Br D CH CH(CH Br)CH


3 2 2 2 3 2 3

MECHANISMS

9 [HCI 2011/1/24]
Which compound undergoes an SN1 substitution reaction?

A CH3CH2CH2Br
B (CH3)3CCH2I
C Cl CH3

D CH2=CHCl

12
Halogen Derivatives

10 [HCI 2011/I/25]

An optically pure enantiomer of butan-2-ol was found to rotate plane-polarised light clockwise
by 13.5 o.

When an optically pure enantiomer of 2-bromobutane was reacted with aqueous sodium
hydroxide under heating, the purified product obtained was found to rotate plane-polarised light
clockwise by 5.5 o.

Which of the following mechanisms could have taken place, assuming complete reaction?

A SN1 only B SN2 only


C Both SN1 and SN2 D Elimination only

11 [AJC 2011/1/24]

Ethers are cleaved by HI via an SN2 mechanism.

O HI OH I
+
23
Which of the following is the most likely product for the following reaction?

O
2 mol HI
LL

O
A
I OH

I HO
B
OH I

I HO
C
I OH

HO I
D
I OH

13
Halogen Derivatives

12 [NJC 2011/I/26]

When an optically active sample of 2-chlorobutane is heated with aqueous KOH,


the product, 2-butanol, the chiral centre is inverted.
CH3 CH3

CH3CH2 C Cl CH3CH2 C OH

H H
Which diagram represents the reaction profile for this mechanism?

A C
energy energy

Reaction path Reaction path

B D
energy energy
23
LL

Reaction path Reaction path

14
Halogen Derivatives

Multiple Choice Questions (Section B)

REACTIONS

1 [SRJC 2011/1/38 modified]

Chloroethane reacts with excess alcoholic ammonia when heated to form a


quaternary ammonium salt in four steps.

Which of the following statement(s) about the reaction are correct?


23
1 In each step, the attacking nucleophile is more nucleophilic than that in the
previous step.
LL

2 Only the final product will give an immediate white precipitate with aqueous
silver nitrate.
3 In each step, an intermediate with a sp2 hybridised carbon atom is formed.

2 [CJC 2010/I/40 modified]

NaOD in D2O
1 CH3CH2CH2Br
reflux
NaOD in D2O
2 CH3CH2CN
reflux
NaOD in CH3CH2OD
3 CH3CH2Cl
reflux

15
Halogen Derivatives

3 [RI 2009/I/39]

When a halogen-containing compound is hydrolysed with aqueous NaOH, the


corresponding sodium halide, NaX, is produced.

A student investigated the number of moles of NaX produced by hydrolysing two


halogen-containing compounds, P and Q, separately. The experimental results
obtained are shown graphically below.
23
LL

Which compounds could be P and Q?

P Q

1 CH3CH2Cl CH3CH2Br

2 CH3CH2Br BrCH2CH2Br

3 C6H5CH2Cl CH2=CHBr

16
Halogen Derivatives

MECHANISMS

4 [MJC 2009/I/37]

23
LL

17
Halogen Derivatives

HALOGEN DERIVATIVES SUGGESTED SOLUTIONS

Multiple Choice Questions (Section A)

1. 2. 3. 4. 5. 6. 7. 8. 9. 10. 11. 12.


C C B B C C D A C C B A

Multiple Choice Questions (Section B)

1. 2. 3. 4.
B D B B

Structured/Free Response Questions

1 (i) Compound A Compound B Compound C

HO CH3 H CH3
23
LL

Refer to notes (Saytzeff’s rule to decide if B or C is major product)

(ii) Nucleophilic substitution (SN2)


CH3 CH3
CH3 CH3 CH
3
- - HO C HO C
HO : C Cl HO : HO CC Cl
Cl+ Cl - + Cl -
H  H 
H H
H

2 (i) Using experiments 1 and 2,


as the overall volume is the same, concentration is directly proportional to the volume
used,
[FA3] is constant, [FA4] increases by 1.2069 times, rate increases by 1.2069 times.
Since the increase in [FA4] and rate are approximately the same, the order of reaction
with respect to FA4 is 1 (order of reaction with respect to OH- is 1).

18
Halogen Derivatives

Using experiments 1 and 3,


[FA3] increases by 7.5 times, [FA4] decreases by 11.6 times, rate decreases by 1.55
(original rate x 7.5 / 11.6) times.
Hence the order of reaction with respect to FA3 is 1 (order of reaction with respect to
2-bromobutane is 1).

(ii) Rate = k[2-bromobutane][sodium hydroxide]

(iii) Nucleophilic Substitution, SN2

CH3 CH3 CH3


 
C HO C Br C H Br
HO H  Br HO
 C2H5 H C2H5
C2H5

3 (i) Test 1:
Add NaOH(aq), heat. Cool the mixture. Acidify with HNO3(aq). Add
AgNO3(aq)

Cream precipitate of AgBr formed. It contains bromoalkane.


23
Test 2:
Add Br2(aq) in the dark.
LL

Decolourisation of reddish brown solution. It contains C=C double bond.

(ii) CH3 CH3 CH3


OH excess
Br2(aq) concentrated H2SO4

170oC
Br Br

4 (i)

(ii) Step I : limited Cl2(g), uv light


Step II : Cl2(aq), room temperature

(iii) Step IV: Nucleophilic Substitution

5 Route 1 will give greater yield.

Route 2 has a lower yield, because the lone pair of electrons on Cl atom

19
Halogen Derivatives

interacts with the  electrons of the benzene ring, hence strengthening the C –
Cl bond. Thus hydrolysis of the C – Cl bond is slower in Route 2.

6 (i) Step I
Reagent(s): limited CH3Cl
Condition(s): heat with reflux

Step II
Reagent(s): excess ammonia
Condition(s): ethanol, heat in sealed tube

(ii)

23
Compound U
(iii) Nucleophilic substitution
LL

7 (i)

Br

CH2CHCH2CH2CH2OH
CH2=CHCH2CH=CH2
Br
E F

NHCH3

CH2CHCH2CH2CH2OH

NHCH3

20
Halogen Derivatives

(ii)
Reagents Conditions
Step 1 concentrated H2SO4 170oC
Step 2 KMnO4, dilute H2SO4 Heat

(iii) F undergoes SN1 mechanism to form G. Hence, there is an equal


probability of the nucleophile, CH3NH2, attacking from either plane of
the carbocation intermediate, which is trigonal planar in shape.

23
This will lead to the formation of a racemic mixture. Hence, G is optically
inactive.
LL

(iv) Type of Hybridization = sp2


Top View

1200

8 (a) Cl

(b) Cl NH2

+ NH3 + HCl

(c) (i) Order of reaction with respect to RX is 1.


Order of reaction with respect to NH3 is 0.

21
Halogen Derivatives
Nucleophilic Substitution, SN1

(ii) [RX]/moldm-3 [NH3]/moldm-3

Time/s Time/s

(iii) rate = k [RX]


(SN1) Nucleophilic Substitution
(iv)
Nu

+ -
CH2 CH2
 slow
H3C C Cl  H 3C C + Cl
CH3
CH3
23
LL

CH2 fast CH2



C OH C
H3 C H 3C OH
CH3
CH3

(d) SN2
The primary carbocation formed from SN1 reaction with chloroethane is less
stable than the tertiary carbocation formed with tertiary chloroalkane, hence SN1 is
less likel to occur for chloroethane.
OR
In the tertiary chloroalkane, RX, the alkyl groups poses steric hindrance for the
nucleophile, NH3, to approach the carbon directly bonded to chlorine. As a result,
SN2 is less likely to occur for the tertiary chloroalkane.

9 (i) Chlorobenzene does not usually undergo nucleophilic substitution. This


is because the p-orbital of the chloro atom in E overlaps with the p-orbitals of the
benzene ring. Thus, the lone pair of electrons of Cl is delocalised into the
benzene ring. This strengthens the carbon chlorine (C-Cl/ partial double bond)
bond. More energy required to break the stronger CCl bond.

22
Halogen Derivatives

(ii) Nucleophilic substitution , SN2


H

Cl O C Cl
Cl O- H 

C Cl H COOH
Cl Cl H  COOH Cl Cl

Cl O C H
COOH
Cl Cl
+ Cl
10 (i) O
CH3C
O C(CH3)3
K
23
(ii) Nucleophilic substitution.

(iii) Nucleophilic Substitution,SN1


LL

CH3
CH3

 slow 
H3C C Cl H3C C  Cl

CH3 CH3

CH3
O
CH3

H3C
H3C C
C C
O CH3
CH3
H3C
O

C -
H3C O

(iv) (CH3)3CCl will not be soluble in water as it cannot form hydrogen bonding
with water molecules.

23
Halogen Derivatives

23
LL

24
Hydroxy Compounds

HYDROXY COMPOUNDS 4
Structured / Free Response Questions

1 [JJC 2011/III/3(c)(i)&(iii)]
Pharmaceutical companies produce and market the drugs, salbutamol hydrochloride and
ephedrine hydrochloride, for the treatment of asthmatic attacks. The active ingredients are
salbutamol and ephedrine respectively.

(i) Suggest a simple chemical test to distinguish between salbutamol and ephedrine.

(ii) An isomer of salbutamol, compound G, has the structure shown.


23
LL

Predict, with reasoning, whether salbutamol or compound G has a lower melting


point.
[5]

2 [Modified RVHS 2011/III/5(f) & SRJC 2011/III/3(a)(iii)]


Describe how you could distinguish the following pairs of organic compounds by means of a
simple chemical test. Write balanced equations for the reactions that occur.

(i) CH3CH2CH2CH2OH and CH3CH(OH)CH2CH3

(ii) OH

OH
and
[4]

1
Hydroxy Compounds

3 [DHS 2011/II/3(b)]
Alkynes are also found in many naturally occurring organic molecules, such as the hormone
molecules shown below.

Complete the following sequence of chemical tests by stating suitable reagents, conditions
and observations at each step, to identify the stated hormone molecules.

step reagents, conditions and observations molecule


reagent and conditions:
1 G
observation:
reagent and conditions:
2 H
observation:
23
reagent and conditions:
3 F
observation:
reagent and conditions:
4 E
LL

observation:
[6]

4 [MJC 2011/III/5(a) & (c)]


The compound chloroxylenol and Triclosan are two chlorinated phenol compounds which
has been featured for its anti-microbial action, and used in soaps and detergents. While
these compounds are relatively safe for humans to handle, they may have a tendency to
persist in the environment for a long period of time, where in turn, they can be converted to
other more toxic compounds, causing environmental problems.

chloroxylenol Triclosan

2
Hydroxy Compounds

(i) By considering their solubility and reaction with water, suggest two reasons why such
compounds are likely to persist in the environment.

(ii) Triclosan can be synthesised from 2,4-dichlorophenol.

Compare the acidity of 2,4-dichlorophenol with chloroxylenol.


[4]

5 [Modified HCI 2011/II/3(d)]


23
(a) Give the structural formulae of the organic products formed when compound A
reacts with the following reagents.
LL

Compound A

(i) hot alkaline KMnO4

(ii) dilute HNO3

(iii) hot acidified K2Cr2O7

(iv) aqueous bromine

(v) PCl5

(v) CH3CO2H and a small amount of concentrated H2SO4, heat


[9]

(b) Explain why compound A is more soluble in aqueous sodium hydroxide than in water.

3
Hydroxy Compounds

[2]
6 [Modified PJC 2011/II/7(a)]
Amines are commonly found in polymers used in plastics or textiles.
Compound D is an amine-containing monomer which is used in polymerisation to form
plastic materials. The reaction scheme shown below outlines how compound D can be
formed from 2-chloroethylhexane.

Cl CH3
O CH3 HO CH3
step I step II step III CN
Compound A
NaOH(aq), heat K2Cr2O7(aq), HCN
in trace KCN
H2SO4(aq), heat 10 -20 °C

2-chloroethylhexane

Excess concentrated step IV


o
H2SO4, 170 C
NaOH in ethanol, heat
CH3 CH2NH2

Compound B step V
Compound C
23
LL

Compound D

(i) Suggest the structural formulae for the compounds A, B and C.

(ii) Name the type of reactions in steps I to V.

(iii) Suggest suitable reagents and conditions in steps I, II, IV and V.


[12]

4
Hydroxy Compounds

7 [YJC 2011/III/2(c)(iii) & (iv) & RI 2011/III/4(c)(ii) & RI 2009/III/1(c)]


The following table lists the boiling points of some unbranched alkanes and alcohols.

formula boiling points / C


C2H6 −88
C3H8 −42
C4H10 0
CH3OH 65
C2H5OH 78
C3H7OH 97
C4H9OH 118
C5H11OH 133

(a) (i) Explain why the boiling points of the alcohols are much higher than those of the
alkanes.

(ii) A mixture containing two of the alcohols listed in the table above, is to be
23
separated using fractional distillation. The mixture is placed in the distillation
apparatus at room temperature and then gently heated. The first fraction is
collected at 97.2 oC.
LL

I Identify one alcohol from the table above that could not be present in the
mixture.
II By specifically referring to this experiment, explain why the alcohol
identified in (I) could not be present.
III Give a reason why the distillation flask should not be heated using a
bunsen burner.
[5]

(b) Dehydration occurs when alcohols are heated over Al2O3.

(i) Compound N is obtained when propan–2–ol is heated over Al2O3.


Identify compound N.

(ii) Suggest reagents and conditions to convert N into 2-hydroxypropanoic acid


(commonly known as lactic acid) in no more than 3 steps.
[4]

(c) When lactic acid is refluxed with a catalytic amount of concentrated sulfuric acid,
compound Z with the molecular formula of C6H8O4 is formed. It does not react with
sodium. Deduce the structural formula of Z, giving reasons for your answer. [3]

5
Hydroxy Compounds

[3]
8 [SRJC 2011/III/5(c)]
Organometallic compounds, usually a metal attached to an R group, can be used to convert
carbonyl compounds to alcohols. An example is shown below:

H2O
R−CH=O + R’−Li  R−CH(OH)−R’ + LiOH

2-bromobutane can be converted to 2-methylbutan-2-ol in 3 steps. Write the reagents and


conditions required for the conversion, including intermediate compounds.
[5]

9 [CJC 2011/III/4(c)]
A hydrocarbon A has the molecular formula of C10H14. When A is treated with hot aqueous
acidified KMnO4, benzoic acid is formed.

When reacted with bromine in the presence of ultraviolet light, A produced four isomeric
monobromo compounds with the formula C10H13Br.

Upon heating with alcoholic NaOH, two of the monobromo compounds B and C gave the
same compound which can exist as a pair of geometric isomers, D and E.
23
Heating B with aqueous NaOH produces compound F which does not react with acidified
potassium dichromate(VI).
LL

Suggest the structures of A-F, explaining your reasoning.


[9]

10 [PJC 2011/III/2(c)]
A recently discovered organic compound L possesses a pleasant floral scent and is
therefore being used as a component of many essential oils and perfumes. A molecule of L
has the molecular formula, C10H18O.

A sample of L has the ability to rotate plane-polarised light.

No reaction occurs between L and 2,4-dinitrophenylhydrazine. However, L produces M,


C10H22O, with hydrogen with nickel catalyst.

No change is observed when L is heated with acidified potassium dichromate(VI).

Heating L with aluminium oxide produces a mixture of two products. N, with molecular
formula C10H16, is present as the major product.

N reacts with hot concentrated acidified potassium manganate(VII) to produce propanone


and P, C5H6O5. A colourless gas, which forms a white precipitate in limewater, is liberated.

6
Hydroxy Compounds

P can also be produced by the reaction of the following compound with hot concentrated
acidified potassium manganate(VII).

Deduce the structures of compounds L, M, N, and P. Explain the chemistry of the reactions
described.
[9]

11 [IJC 2011/III/5(d)]
α-Terpineol, C10H18O, has a pleasant smell similar to lilac and is a common ingredient in
perfumes, cosmetics, and flavours.

α-Terpineol contains a cyclohexene ring and an alcohol group which is not readily oxidised.

Heating α-Terpineol with concentrated acidified KMnO4 gives H, C10H18O4, which gives a
23
yellow precipitate on warming with alkaline aqueous iodine. On hydrogenation in the
presence of palladium catalyst, α-Terpineol gives J, C10H20O.
LL

J reacts readily in the presence of a few drops of concentrated sulfuric acid to give K,
C10H18. K is oxidised by warm acidified KMnO4 to give M, C9H16O along with the evolution of
CO2 gas.

Deduce the structural formulae for H, J, K and α-Terpineol. Explain your reasoning.
[8]

7
Hydroxy Compounds

Multiple Choice Questions (Section A)

1 [CJC 2011/I/24]
Which of the following reagents may be used to distinguish between 1–bromobutane and 2–
bromobutane?

A alkaline aqueous iodine


B sodium hydroxide
C acidified potassium manganate(VII)
D aqueous silver nitrate

2 [SAJC 2011/I/24]
Which of the following reagent should not be used to distinguish the following two
compounds?
23
LL

3 [MJC 2011/I/17]

8
Hydroxy Compounds

4 [AJC 2011/I/22]

23
LL

5 [NYJC 2011/I/24]

9
Hydroxy Compounds

6 [CJC 2011/I/20]

23
LL

7 [TPJC 2011/I/29]

10
Hydroxy Compounds

8 [RI 2011/I/20]

23
9 [HCI 2011/I/19]
LL

11
Hydroxy Compounds

10 [HCI 2011/I/26]

23
11 [RI 2011/I/26]
LL

12
Hydroxy Compounds

12 [SRJC 2011/I/21]

23
13 [RI 2009/I/27]
LL

13
Hydroxy Compounds

14 [PJC 2011/I/25]

23
LL

14
Hydroxy Compounds

15 [Modified RI 2009/I/28]
Which of the following shows the given ions arranged in the order of increasing pKa values?

A 2–chloroethoxide ion, ethoxide ion, 4-nitrophenoxide ion, phenoxide ion


B ethoxide ion, 2–chloroethoxide ion, phenoxide ion, 4-nitrophenoxide ion
C phenoxide ion, 4-nitrophenoxide ion, ethoxide ion, 2–chloroethoxide ion
D 4-nitrophenoxide ion, phenoxide ion, 2–chloroethoxide ion, ethoxide ion

Multiple Choice Questions (Section B)

1 [NYJC 2011/I/39]
23
Which of the following reagents will have different reactions with the two alcohols,
(CH3)3CCH2OH and (CH3)2CHCH2CH2OH?
LL

1 concentrated sulfuric acid


2 phosphorus pentachloride
3 acidified aqueous potassium manganate(VII)

2 [SAJC 2011/I/38]
Which of the following pairs will give the same organic product when treated with hot
acidified KMnO4?

1 (COOH)2 and CH2=CH2


2 CH3CH=CHCH3 and CH3CH2OH
3 C6H5CH2CH2OH and C6H5CH=CH2

15
Hydroxy Compounds

3 [NJC 2011/I/36]

23
LL

4 [PJC 2011/I/39]

16
Hydroxy Compounds

5 [YJC 2011/I/38]

23
6 [DHS 2011/I/38]
LL

HYDROXY COMPOUNDS SUGGESTED SOLUTIONS

Multiple Choice Questions (Section A)


1. 2. 3. 4. 5. 6. 7. 8. 9. 10. 11. 12. 13. 14. 15.
A C A A B A D B A D D C D D B

Multiple Choice Questions (Section B)


1. 2. 3. 4. 5. 6.
D A B D D C

Structured/Free Response Questions

17
Hydroxy Compounds

1 (i) Test: Add Br2(aq) to both samples separately.


Observations:
Salbutamol: Brown Br2 decolourises.
Ephedrine: Brown Br2 remains.

OR
Test: Add neutral FeCl3(aq) to both samples separately.
Observations:
Salbutamol: Violet colouration observed.
Ephedrine: No violet colouration.

(ii) Salbutamol has a lower melting point than compound G.

Both compounds have simple molecular structures with hydrogen bonds between
their respective molecules.

However, for Salbutamol, it is possible to form intramolecular hydrogen bonds


between the phenol group and the primary alcohol group.

Thus, less energy is required to overcome the less extensive intermolecular


23
hydrogen bonds between salbutamol molecules as compared to that in G.
LL

2 (i) Test: Add I2(aq), NaOH(aq) and heat or warm.


Observations:
CH3CH(OH)CH2CH3: Pale yellow ppt formed
CH3CH(OH)CH2CH3 + 4I2 + 6NaOH  CHI3 + CH3CH2COO-Na+ + 5NaI + 5H2O
CH3CH2CH2CH2OH: No yellow ppt formed

(ii) Test: Add neutral FeCl3 (aq) to both compounds separately.


Observation:
Phenol: Violet complex seen
Cyclohexanol: No violet complex seen

18
Hydroxy Compounds

Or
Test: Add conc. HNO3 to both compounds separately.
Observation:
Phenol: Yellow precipitate formed

Cyclohexanol: No yellow precipitate formed

Or
Test: Add PCl5 to both compounds separately.
Observation:
Phenol: No white fumes evolved
Cyclohexanol: White fumes of HCl evolved
23
LL

Or
Test: Add dil H2SO4/KMnO4 to both compounds separately. Heat.
Observation:
Phenol: Purple KMnO4 remains.
Cyclohexanol: Purple KMnO4 decolourises.

Or
Test: Add Br2(aq) to both compounds separately.
Observation:
Phenol: Brown Br2(aq) decolourises. White ppt formed.

19
Hydroxy Compounds

Cyclohexanol: Brown Br2(aq) remains.

3 Step reagents, conditions and observations molecule

1 Reagent and conditions: K2Cr2O7(aq), H2SO4(aq), Heat under


reflux G
Observation: Orange solution turns green
2 Reagent and conditions: 2,4–dinitrophenylhydrazine, warm
H
Observation: Orange ppt formed
3 Reagent and conditions: PCl5
F
Observation: White fumes produced.
4 Reagent and conditions: Aqueous bromine / bromine in CCl4 in
23
the dark / Neutral FeCl3
Observation: Decolourisation of brown aqueous bromine and E
white ppt formed / Decolourisation of brown bromine in CCl4 and
LL

white ppt formed / Formation of purple complex

4 (i) 1. The presence of the bulky hydrophobic benzene ring interferes with the
formation of hydrogen bonding between the compounds and water. Hence the
compounds are likely to be insoluble in water.

2. The p orbital of chlorine atom overlaps with the  electron cloud of the benzene
ring. giving rise to the partial double bond character of the C–Cl bond. This
makes halogenoarenes less susceptible to nucleophilic substitutions. Hence the
compounds cannot be easily hydrolysed by water.

As such they are difficult to remove by natural processes in the environment.

(ii) There are more electron–withdrawing –Cl groups attached to the benzene ring in
2,4–dichlorophenol.

This disperses the negative charge on the oxygen atom more, thus, further
stabilising the phenoxide ion from 2,4-dichlorophenol with respect to that from
chloroxylenol.

20
Hydroxy Compounds

Therefore, 2,4–dichlorophenol is a stronger acid than chloroxylenol.

5 (a) (i)

(ii)

23
(iii)
LL

(iv)

21
Hydroxy Compounds

(v)

(vi)

(b) The presence of the bulky hydrophobic group in compound A prevents it from
dissolving well in water despite being able to form some hydrogen bonds with
water.

(Note: Energy evolved from formation of solute – solvent interactions is insufficient


23
to compensate for the energy required to overcome the solute – solute interactions
and solven – solvent interactions.)
LL

In the presence of sodium hydroxide, compound A reacts to form a sodium salt


which dissociates completely. Strong ion-dipole interactions between water
molecules and the oppositely charged ions allow compound A to dissolve in sodium
hydroxide.

6 (i) Compound A Compound B Compound C


HO CH3 H CH3 NC CH3

(ii) Step I Step II Step III Step IV Step V


Nucleophilic Oxidation Nucleophilic Elimination Reduction
Substitution Addition

22
Hydroxy Compounds

(iii) Step I NaOH(aq), Heat under reflux

Step II K2Cr2O7(aq), H2SO4(aq), Heat under reflux

Step IV Excess concentrated H2SO4, 170C


Step V LiAlH4 in dry ether

7 (a) (i) The intermolecular hydrogen bonding between alcohol molecules is stronger
than the instantaneous-dipole induced-dipole forces of attraction between
alkane molecules.
Hence greater amount of energy is required to overcome the hydrogen bonds
in alcohols.

(ii) I methanol or ethanol


II If methanol (ethanol) had been present in the mixture it would have
condensed as the first fraction at 65 °C (78 °C);
Or
The boiling temperature of methanol (ethanol) is below 97 °C. Hence the
23
methanol (ethanol) could have boiled off before the first fraction is
collected.
III Flammability of alcohol vapours or risk of fire or explosion
LL

(b) (i) OH Al2O3

heat Compound N

23
Hydroxy Compounds

(ii) OH
cold KMnO4
HO
NaOH(aq)
Compound N
K2Cr2O7
H2SO4(aq)
heat

OH O

HO NaBH4 HO

O O
2-hydroxypropanoic acid

LiAlH4 not accepted for last step as it will reduce –COOH too.
(c) O
C H3 C
C O
H
C H
23
O
C C H3
O
Compound D is .
LL

D does not contain OH nor COOH since it does not react with sodium.

It is a diester formed when OH and COOH groups of one lactic acid molecule
reacts with the COOH and OH groups respectively of another lactic acid
molecule.

8 CH3CH2CH(Br)CH3 CH3CH2C(CH3)(OH)CH3
NaOH(aq),
Heat under CH3 −Li,
reflux H2O

KMnO4/H2SO4,
CH3CH2CH(OH)CH3 Heat under reflux CH3CH2COCH3

1st step: NaOH(aq), Heat under reflux


2nd step: H2SO4(aq), KMnO4(aq), Heat under reflux
3rd step: CH3-Li (and H2O)

24
Hydroxy Compounds

9 Evidence Deduction

When A is treated with hot aqueous Side–chain oxidation occurred.


acidifed KMnO4, benzoic acid is formed.
A has only 1 alkyl side–chain on the
benzene ring.
A reacts with bromine in the presence of uv Free radical substitution occurred on the
light, producing 4 isomeric monobromo alkyl side–chain in A
compounds.
Upon heating with alcoholic NaOH, both B Elimination occurred.
and C gave the same products, D and E, D and E contain C=C.
which are geometric isomers.
Heating B with NaOH(aq) gives F which Nucleophilic substitution occurred.
does not react with acidified K2Cr2O7. F is an alcohol.

F does not undergo oxidation.


F is a tertiary alcohol.
A: B: 23 C:

D and E

F:
LL

10 Evidence Deduction

L rotates plane–polarised light. L exhibits optical isomerism and contains at


least 1 chiral carbon
No reaction occurs between L and 2,4– No condensation reaction occurred.
dinitrophenylhydrazine. No aldehyde or ketone functional group in L.
L produces M, C10H22O Reduction occurred.
L is unsaturated. Addition of 4 H atoms
indicate presence of 2 C=C bonds .
No change is observed when L is heated L does not undergo oxidation.
with acidified potassium dichromate(VI). L contains a 3o alcohol group.
Heating L with aluminium oxide forms N as L contains alcohol that undergoes
major product. elimination.

25
Hydroxy Compounds

N contains an additional C=C bond upon


elimination.
N reacts with hot concentrated acidified Oxidative cleavage of all three C=C
KMnO4 to produce propanone, P and a gas occurred.
which gives a white precipitate with Gas evolved is CO2. The white ppt formed is
limewater. CaCO3.
Presence of 2 terminal C=C bond due to 2
moles of CO2 produced during oxidative
cleavage. (loss of 2 C from N to propanone
& P).
P can also be produced by oxidation of the O O O
given compound with concentrated
C CH2 CH2 C C
acidified KMnO4 and heating.
P is HO OH
L:
CH3 CH3
CH3 C OH CH3 C H
CH2 CH CH CH2 CH2 CH CH2 or CH2 CH C CH2 CH2 CH CH2
OH
23
(Note: The 2 structures below are not accepted as they will not lead to formation of the
correct structure for N)
LL

CH3 CH3
CH3 CH3
C CH C CH2 CH2 CH CH2 or C CH CH2 CH2 C CH CH2

CH3 OH CH3 OH
CH3 CH3
CH3 C OH CH3 C H
CH3 CH2 CH CH2 CH2 CH2 CH3 or CH3 CH2 C CH2 CH2 CH2 CH3

M: OH
CH3 CH3
C

N: CH2 CH C CH2 CH2 CH CH2

11 Evidence Deduction

α-Terpineol has an alcohol group which is α-Terpineol has a tertiary alcohol.

26
Hydroxy Compounds

not readily oxidised.

Heating α-Terpineol with concentrated Oxidative cleavage occurred.


acidified KMnO4 gives H. C=C is broken.

H gives yellow ppt when warmed with H contains –CH3CO.


alkaline I2(aq) H undergoes mild oxidation with alkaline
aqueous iodine to give CHI3, a yellow ppt.
On hydrogenation, α-Terpineol gives J Reduction occurred.
α-Terpineol is unsaturated. Additional of 2
H atoms indicate presence of 1 C=C bonds.
(recall: α-Terpineol has a cyclohexene ring)
J reacts with a few drops of conc H2SO4 to J contains alcohol that undergoes
give K. elimination.
K contains an additional C=C bond upon
elimination.
K is oxidised to give M Oxidative cleavage occurred.
C=C is broken.

Presence of terminal C=C bond due to one


23
mole of CO2 produced during oxidative
cleavage. (loss of 1 C from K to M).
LL

27
Hydroxy Compounds

23
LL

Note: Do not assume that all the given answers are correct. Do discuss with your
friends and check with your tutor should you think that any given answer is incorrect.

28
5
Carbonyl Compounds

CARBONYL COMPOUNDS

Structured / Free Response Questions

1 [ACJC 2011/II/2(a)(b)]
(a) A hydrocarbon W contains 92.3% carbon. On complete combustion, 0.005 mol of W
produces 1.76 g of CO2.

(i) Determine the molecular formula of W.

(ii) Given that 1 mol of W reacts with 1 mol of bromine in tetrachloromethane, draw the
displayed formula of W.
[3]
(b) Apart from using aqueous bromine or bromine in an inert solvent, describe one other
simple chemical test you could carry out to distinguish

State clearly how each compound behaves in the test and write an equation for the
reaction involved.

Reagent(s) and conditions:


Observations:
Equation:
[3]
23
LL

1
Carbonyl Compounds

2 [RVHS 2011/II/3(c)]
Aluminium containing compounds such as lithium aluminium hydroxide and aluminium oxide
are useful in organic synthesis. Consider the following series of reactions.
CO2H CO2H

H3C C C CH3 C
OH OH

REACTION II

O O CN CN
REACTION I
H3C C C CH3 H3C C C CH3

OH OH

LiAlH4 , dry ether LiAlH4 , dry ether


23
LL

A B

(i) State the reagents and conditions for:

Reaction I:

Reaction II:

(ii) Draw the structures of compound A and B in the space provided.

[4]

2
Carbonyl Compounds

3 [YJC 2011/II/4(a)]
Phenylpropanone, C6H5CH2COCH3, is used in the manufacture of amphetamine, a stimulant
commonly used to treat Attention Deficit Hyperactivity Disorder (ADHD). Due to the illicit use
in clandestine chemistry, it was made a controlled substance in 1979 in the United States.

A reaction scheme beginning with phenylpropanone is shown below.


A

O 2,4-dinitrophenylhydrazine
C
CH3

Step I

OH Step II Br2(aq)
23
CH3
C C C
CH3 H
H H
LL

Step III
C

Step IV
CH3
c. HNO3, C O and O C
c. H2SO4, HO OH
90oC

(i) Draw the structures compounds A, B and C.

(ii) Suggest reagents and conditions for steps I to III.


[6]

3
Carbonyl Compounds

4 [VJC 2011/III/4(a)]
Suggest methods by which the following pairs of compounds could be distinguished from
each other by simple chemical tests.

For each case, no more than two steps should be used.

(i)

(ii)

[6]
5 [JJC 2011/III/4 modified]
23
Compounds P and Q are branched isomers with molecular formula of C4H7OCl.
Both P and Q form a red-brown precipitate on heating with Fehling’s solution. On heating
with dilute sodium hydroxide, P and Q produced R and S respectively with molecular
LL

formula of C4H8O2.
A series of two sets of experiments involving P and Q and sodium hydroxide were carried
out at a constant pressure of 101 kPa and temperature of 80°C. The results obtained are
tabulated below.

experiment initial [P] initial [OH] initial rate of formation of R


number / mol dm3 / mol dm3 / mol dm3 s1
1 0.30 0.40 0.0150
2 0.30 0.60 x
3 0.60 0.20 y

experiment initial [Q] initial [OH] initial rate of formation of S


number / mol dm3 / mol dm3 / mol dm3 s1
4 0.30 0.40 0 0240

5 0.30 0.60 0.0360


6 0.60 0.20 0.0240

4
Carbonyl Compounds

(a) The rate equations when P and Q react with NaOH are as shown below.
Rate = k1 [P]
Rate = k2 [Q][OH]
(i) Deduce the values of x and y in experiments 2 and 3.

(ii) Calculate the rate constant k2, stating the units of the rate constant.

(iii) Given that the half-life of Q in experiment 4 is z seconds, deduce the half-life of Q
in experiments 5 and 6.
[4]
(b) (i) Suggest the structural formulae of compounds P, Q, R and S.

(ii) Based on the rate equation given and using your structure deduced in (c)(i),
describe the mechanism when P reacts with NaOH.

(iii) Assuming the reaction is exothermic, sketch a labeled energy profile diagram for
your proposed mechanism.

You are only required to label the reactants, products and activation energy on
your diagram.
[6]
23
6 [AJC 2011/II/4(b) modified]
(i) The Kiliani–Fischer synthesis lengthens the aldose chain by one carbon atom and can
be used to unravel the stereochemical relationships among simple carbohydrates.
LL

The synthesis is carried out using the following steps.


H O HO O
C Step 1 C
Step 2
H C OH F H C OH G
R H C OH
R

H O
C
H C OH
H C OH
R
Suggest the reagents and conditions used in steps 1 and 2. Give the structural
formulae of the intermediates F and G.

5
Carbonyl Compounds

Regents and conditions:


Step 1

Step 2

F G

(ii) Draw the structure of the product formed when glucose, shown below, is subjected to
the Kiiani-Fischer synthesis.

H O
C
H C OH
HO C H
23
H C OH
H C OH
LL

CH2OH
glucose

(iii) Glucose may be converted to the cyclic form which is known as a hemiacetal. The
reaction may be represented using a simpler hydroxyaldehyde as follows:

O O OH
HO
H H
hydroxyaldehyde hemiacetal
State the type of reaction for the forward reaction.
[8]

6
Carbonyl Compounds

7 [CJC 2011/II/5(b)(c)]
Adrenaline is a natural hormone and acts as a stimulant when secreted directly into the
bloodstream. It has the following structure:

(a) Reactions involving the formation of a hydroxynitrile (cyanohydrin) as an intermediate


are useful because of the further reactions that can be carried out. More importantly, it
increases the number of carbon atoms in a chain as a C-C is formed.

Adrenaline can be synthesised from 3,4-dihydroxybenzaldehyde via the following


reaction pathway with a hydroxynitrile being the intermediate.

23
LL

(i) Draw the displayed formula of compound A in the box provided above.

(ii) State the reagents and conditions required for Steps I-III by completing the table
below.
[4]

7
Carbonyl Compounds

(b) The Grignard reaction is another versatile tool for forming C-C bonds. In 1900, Victor
Grignard reported that a warm solution of bromoethane can react with magnesium in a
dry, non-polar solvent to give an organomagnesium compound with formula of
CH3CH2MgBr. The carbon-magnesium bond is polarised and the carbon directly
bonded to magnesium is basic.

Grignard reagents, with different alkyl or aryl groups can be prepared and are widely
used in organic synthesis.

R–X + Mg → R–MgX
Grignard reagent

(where R is an alkyl or aryl group and X can be Cl, Br or I)

The Grignard reagent formed can then react with aldehydes and ketones to yield
secondary and tertiary alcohols respectively.

An example of the use of Grignard reagent is the two-step reaction of pentan-2-one


with CH3MgBr.
23
LL

(i) Identify the types of reactions that occur in Steps I and II.

(ii) Draw the structure of the final organic compound formed when benzaldehyde
reacts with CH3CH2MgBr.

(iii) Suggest why this method of using the Grignard reagent is not suitable for the
preparation of Adrenaline from 3,4-dihydroxybenzaldehyde.

[4]

8
Carbonyl Compounds

8 [SRJC 2011/II/6(b)]
A compound L, C9H10Br2, undergoes reaction with aqueous sodium hydroxide under reflux
to give compound M, C9H12O2. Compound M then undergoes nucleophilic substitution with
ClCOCH2COCl to form a cyclic compound N. Compound M gives a yellow precipitate when
it undergoes oxidation with warm alkaline aqueous iodine.

Compound M can also undergo a series of reactions as shown below to yield Compound Q.

Compound M Compound O Compound P

CH3 H
HO C C COOH
OH
COOH
Compound Q
(i) Draw the structural formulae of compounds M, N, O and P.

(ii) State the reagents and conditions for converting compound O to P.


23
[5]

9 [HCI 2011/III/5(b)]
LL

The compound 4-ethylphenol, P, and chlorinated products derived from it have useful
antiseptic properties.

Under certain conditions, compound P reacts with chlorine to give compound Q, C8H8Cl2O.

(i) Compound Q reacts with NaOH (aq), followed by acidification, to give R, C8H10O3,
which immediately loses water to give T, C8H8O2. Compound T is insoluble in water
but it dissolves in NaOH (aq). It reacts positively with both 2,4-dinitrophenylhydrazine
and alkaline aqueous iodine.

Deduce the structures of compounds Q, R and T. Explain the chemistry of the


reactions described.

9
Carbonyl Compounds

(ii) The compound 2-phenylethanol, V, is an isomer of P. Compound V is responsible for


the fragrance of roses.

Describe a simple chemical test by which P can be distinguished from V. You should
state the reagents and conditions for the test, and describe how each of the isomers
behaves.

[8]

10 [TJC 2011/III/3(e)]
PCl5 is often used as a chemical test as white fumes of HCl are liberated from organic
compounds with specific functional groups.

(i) An organic compound Y, with molecular formula C4H8O2, liberates white fumes when
reacted with solid PCl5 in the cold. It also forms an orange precipitate with 2,4-
dinitrophenylhydrazine and does not react with Tollen’s reagent. When heated with
aqueous alkaline iodine, a bright yellow precipitate is obtained.
23
The organic compound formed by heating compound Y with acidified potassium
manganate(VII) is able to liberate carbon dioxide when added to sodium carbonate
solution.
LL

Deduce the structure of Y, explaining your reasoning.

[5]

11 [ACJC 2011/III/1(b)]
Lactic acid, CH3CH(OH)CO2H, is another common ingredient used in cosmetic products. In
nature, (–)–Lactic acid is produced from the fermentation of lactose. Lactose exists in two
forms, the α–form which has a specific rotation of +92.6° and the β–form which has a
specific rotation of +34° at 20 oC.

(i) If an aqueous solution containing both forms of lactose is allowed to stand, the optical
rotation changes until it reaches +52.3o.

Determine the percentage composition of the α–form in the equilibrium mixture.

(ii) Lactic acid can also be chemically synthesized from ethanal, CH3CHO in a 2–stage
synthesis.

Suggest reagents and conditions for each step and draw the structural formula of the
intermediate compound.

10
Carbonyl Compounds

(iii) Briefly outline the mechanism of the reaction involved in the first step of the synthesis
in (ii).

With reference to your mechanism, explain why lactic acid synthesized by this method
is optically inactive.
(iv) The optically inactive lactic acid obtained in (iii) is reacted with (+)–coniine.

(+)–coniine

Draw a displayed formula of one of the products formed, indicating each chiral centre
with a (*).
[9]
23
Multiple Choice Questions (Section A)
1 [ACJC 2011/I/26]
LL

An imine can be prepared by reacting benzaldehyde with ammonia in methanol:

CHO NH

+ NH3 + H2O

Which of the following statements about the above reaction sequence is correct?

A The value of Kc will increase upon the addition of ammonia.

B The backward reaction is favoured by the addition of benzyldehyde.

C The forward reaction is favoured by the addition of dilute aqueous sulfuric acid.

D The preparation of imine proceeds via nucleophilic addition mechanism, followed

by elimination mechanism.

11
Carbonyl Compounds

2 [ACJC 2011/I/24]
The mechanism below shows how butanone reacts with hydrogen cyanide.

CH 2CH 3
CN - CH2 CH 3
+ slow
- H 3C C O-
C O

H3 C CN

CH2 CH3 CH 2CH3

+ - H 3C C OH + CN -
H 3C C O- + H CN

CN CN

Which of the following statements is true?

A The product mixture is optically active.

B The rate of reaction of hydrogen cyanide with butanal is faster than that with

butanone.
23
C Cyanohydrin produced can react with ammonia at high pressure to form amine.

D Cyanohydrin produced required potassium manganate(VII) to form carboxylic


LL

acid.

3 [AJC 2011/I/26]
Compounds V and W both give an orange precipitate with 2, 4-dinitrophenylhydrazine. Only
one of the compounds gives a yellow precipitate with alkaline aqueous iodine and also
decolourises hot acidifies potassium manganate(VII).

Which of the following pairs could V and W be?

V W

A CH3COCH3 CH3CH2CHO

B CH3CH(OH)CH3 CH3CH2CHO

C CH3CH(OH)COCH3 CH2=CHCOCH3

D CH3(CH2)2CHO CH2=CHCOCH3

12
Carbonyl Compounds

4* [AJC 2011/I/27]
An organic compound X has the following properties:

(i) It has molecular formula C9H10O2.


(ii) It gives a silver mirror with Tollens’ reagent.
(iii) It gives yellow crystals with alkaline aqueous iodine.
(iv) Heating X with Al2O3 at 300 oC, produces a compound Y which gives an
effervescence with hot acidified KMnO4.

Which of the following is formed when Y is oxidised by acidified K2Cr2O7?

A B
COOH CH CH2

COOH COCH3

C D
CH CH2 COCH3

COOH COOH
23
5 [CJC 2011/I/27]
Which of the following syntheses will produce compound Y, 2-methylpropanoic acid?
LL

13
Carbonyl Compounds

6 [CJC 2011/I/28]
Tetrangomycin is one of the first member of the class of antibiotics under the angucycline
group. Which of the following reagents will react with tetrangomycin?

A acidified potassium manganate(VII)

B alkaline copper(II) complex

C alkaline aqueous iodine

D sodium hydroxide
23
7 [DHS 2011/I/24]
The first stage in the synthesis of antipyrine, a drug used in reducing fever, is the reaction
LL

between compound T and phenylhydrazine.

CH3COCH2CO2CH2CH3 + NHNH2 S
T
What will be the product S?

A NHCOCH2COCH3

B NHCH2CO 2CH2CH3

C CH3COCH2CO NHNH2

NH N C CH2CO2CH2CH3
D
CH3

14
Carbonyl Compounds

8 [JJC 2011/I/28]
Smoke from a bonfire contains a compound that causes irritation to the eyes. This
compound produces CO2 when oxidised by KMnO4 and produces a red brown precipitate
when bubbled into Fehling’s solution.

What is a possible structure of the compound?

A CH2=CHCOCH3

B CH2=CHCHO

C
C
CH3

CH2CH3
D
CHO
23
9 [NYJC 2011/I/22]
LL

The compound C3H7Br undergoes a sequence of reactions as follows:

What could be the formulae for R, S and T?

R S T

A CH3CH2CH2OH CH3CH2CHO CH3CO2‾

B CH3CH2CH2OH CH3CH2CHO CH3CH2CO2‾

C CH3CH2CH2OH CH3CH2CO2H CH3CH2CHO

D CH3CH(OH)CH3 CH3COCH3 CH3CO2‾

15
Carbonyl Compounds

10 [PJC 2011/I/27]
Which of the following reagents cannot be used to distinguish separate samples of
methanoic acid and ethanol?

A Tollens’ reagent

B alkaline aqueous iodine

C 2,4-dinitrophenylhydrazine

D aqueous sodium carbonate

11 [RI 2011/I/23]
Compound X was subjected to the following sequence of reactions.

23
LL

Which of the following could be the structure of X?


A
B

C D

16
Carbonyl Compounds

12* [RI 2011/I/25]


Compound F is an isomer of Citral.

The reaction of F with hot acidified potassium manganate(VII) yields three products, G, H
and I.

G can be converted to H with the use of aqueous alkaline iodine followed by acidification
with aqueous sulfuric acid.

Which of the following shows the structure of compound F?


A
B

C D

The following information is relevant to questions 13 and 14.


23
The Wittig reaction involves the reaction of a carbonyl compound with a phosphorus ylide to
give an alkene. The reaction occurs because of an important oxaphosphetane intermediate.
LL

17
Carbonyl Compounds

[RI 2011/I/27]
13 Which one of the following structure shows the product of the reaction between the following
compound and the phosphorus ylide shown?

A
B

C D
23
14 [RI 2011/I/28]
Reagents similar to the phosphorus ylide have been investigated to carry out the same
LL

reaction.

Which of the following atoms can be used in place of phosphorus to carry out the reaction?

A N

B O

C B

D Si

18
Carbonyl Compounds

15 [SAJC 2011/I/27]
Which of the following will give an immediate precipitate with aqueous silver nitrate is added
to the resultant solutions of the following reactions?

A CH3COCH2CH3 + alkaline aqueous I2

C CH3CH2OH + PBr3

D CH3CH2CH=CH2 + I2

16 [TJC 2011/I/18]
In which of the following mechanism does the donation of an electron pair not take place?
A Electrophilic substitution

B Nucleophilic addition

C Electrophilic addition
23
D Free radial substitution
LL

19
Carbonyl Compounds

17 [TPJC 2011/I/23]
Carvone is the main active ingredient of the flavouring agent oil of spearmint.

It is sibjected to reaction with LiAlH4 in dry ether to form Q, and with hydrogen gas in
the presence of nickel to form R.

How many chiral centers do the molecules of Q and R have?

Q R
23
A 4 4

B 3 3
LL

C 2 4

D 2 3

18 [TPJC 2011/I/24]
Ethanal reacts with CN- form HCN in the presence of a weak base as shown below:
OH
O
CH3C CH3C CN
+ HCN 
H H
-
In a similar reaction, CH2COCH3 ions are generated when CH3COCH3 reacts with a strong
base. Which one of the following compound is the product when ethanal reacts with,
-
CH2COCH3?
A CH3CH(OH)CH2COCH3

B (CH3)2C(OH)CH2CHO

C (CH3)2C(CHO)CH2OH

D (CH3)2C(OH)CH2COCH3

20
Carbonyl Compounds

19* [VJC H1 2011/I/26]


0.010 mol of an organic compound was warmed with excess alkaline iodine. The mixture
was then filtered, and the filtrate was washed with tetrachloromethane to remove unreacted
iodine. The aqueous layer was treated with a dilute acid, followed by aqueous silver nitrate,
and 0.030 mol of yellow precipitate was collected.

Which one of the following is a possible structure of the organic compound?

A B

C D

23
LL

21
Carbonyl Compounds

20* [YJC 2011 /I/25]


Aldehydes can undergo addition reactions with a variety of compound of the form HX
according to the following equation.
R R OH
catalyst
C O + HX
H
C
X
H
An example of such a reaction is the formation of cyanohydrin, where X=CN.

Which of the following compounds cannot be obtained by such an addition reaction to


aldehyde, followed by dehydration?

A B
H
CH 3 CH NNH
CH3
O
O

C H D CH3
C C
CH3 CH3
23
LL

22
Carbonyl Compounds

Multiple Choice Questions (Section B)

1 [ACJC 2011/I/39]
Carbonyl compounds which have an α-carbon undergo a rapid interconversion with its
corresponding enol. This unique isomerism is known as tautomerism.

The mechanism for tautomerism of 2-chloropropanal is:


O -
OH
H H
O O H
Cl O O
C C Cl
H C H C C Cl C Cl
H3 C H H C H C
23
CH3
- CH 3 CH3
OH
Keto form Enol f orm
LL

Two resonance f orms of the enolate intermediate


99.99% 0.01%
Which of the following statements can be deduced from the mechanism given above?

1 The mechanism for tautomerism is base catalysed.

2 The enolate intermediate is planar about the α-carbon.

3 Optically pure 2-choropropanal becomes a racemic mixture upon standing in


aqueous sodium hydroxide solution.

23
Carbonyl Compounds

2 [CJC 2011/I/38]
There is a range of reactions involving ketones which have the pattern

of which the formation of a hydroxynitrile (where X = CN) is one such product.

Which compounds could be obtained by such an addition to a ketone, followed by


dehydration?
1

2
23
LL

3 [JJC 2011/I/38]
Which of the following statements about the reaction between carbonyl compounds and
hydrogen cyanide are correct?

1 The slow step involves the formation of a C-C bond.

2 HCN acts as a Bronsted acid.

3 The reaction is catalysed by a small amount of H+ ion.

24
Carbonyl Compounds

4 [SAJC 2011/I/39]
Compounds A and B are both hormones found in the human body. They have the following
structures.

Which of the following statements about the two compounds are true?
23
1 Both decolourises Br2.

2 Both will react with hot acidified KMnO4.


LL

3 Both have 5 chiral carbons each.

5 [TJC 2011/I/38]
Ethylvanilin is a synthetic flavorant used in the manufacture of chocolate and is
approximately three times as potent as vanillin which is found in the extract of vanilla beans.
O H

O CH3

OH
Ethylvanilin

Which of the following statements about ethylvanilin are correct?


1 It can form hydrogen bonds.

2 It gives a red precipitate with Fehling’s solution

3 It reacts with PCl5 to produce white fumes of HCl

25
Carbonyl Compounds

CARBONYL COMPOUNDS SUGGESTED SOLUTIONS

Multiple Choice Questions (Section A)


1. 2. 3. 4. 5. 6. 7. 8. 9. 10. 11. 12. 13. 14. 15. 16. 17. 18. 19. 20.
D B D C B D D B B A C C A D A D D A C D

Multiple Choice Questions (Section B)


1. 2. 3. 4.
A C B D

Structured/Free Response Questions


1 [ACJC 2011/II/2(a)(b)]
(ai) mole ratio C : H = 1 : 1
 empirical formula of W is CH.

Amount of CO2 = 0.0400 mol


 mole ratio W : CO2 = 0.005 : 0.0400
= 1 : 8
 Molecular formula of W is C8H8.

(aii)
23
[3]
LL

(b)

Reagent(s) and conditions: 2,4-DNPH, rt

Alternative answers: (1) - Na(s) or Na2CO3(aq) or PCl5


(2) – H+/KMnO4, heat

Observations: Yellow/Orange ppt obtained for COOHbut no yellow/orange ppt


for W.

Equation: COOH

26
Carbonyl Compounds

N N NO2 + H2O

COOH NO2

[3]

2 [RVHS 2011/II/3(c)]
(i)
Reaction I: HCN, NaCN / trace amounts of base

Reaction II: dilute H2SO4, heat

(ii)
OH OH NH2 NH2

CH2 CH2
H3C C C CH3
23
H 3C C C CH3
H H

A
OH OH B
LL

[4]

3 [YJC 2011/II/4(a)]
(i)
H
H3C
C N N NO2

O2N

A
H H
C C CH3 NO2
OH Br

or
H H
C O
C C CH3 HO

Br OH
B C

27
Carbonyl Compounds

(ii) Steps Reagents and conditions


LiAlH4 in dry ether, or NaBH4, or H2(g) with Ni
I
catalyst
II concentrated H2SO4, 170 C
III KMnO4(aq), H2SO4(aq), reflux
[6]

4 [VJC 2011/III/4(a)]
(i) Add Fehling’s solution and warm.
The 1st compound gives a brick red ppt while the 2nd compound gives no ppt.

(ii) Add aqueous I2 and NaOH. Warm to <70oC.


2nd compound gives a pale yellow ppt while the 1st compound gives no ppt.

[6]

5 [JJC 2011/III/4 modified]


(ai) x = 0.0150 mol dm3 s1
23
y = 0.0300 mol dm3 s1

(ii) Rate = k2 [Q][OH]


LL

Rate 0.0240
 k2  
 = 0.200 mol1 dm3 s1
[Q][OH ] 0.30  0.40

(iii) From Rate = k2 [Q][OH]


 Rate = k’’ [Q] where k’’ = k2 [OH]
and t1/2 of Q = ln 2/k’’ = z seconds in expt 4

Comparing expt 4 & 5


In expt 5, [OH] is increased to 3/2 times of original value in expt 4,
k’’ is increased to 3/2 times and thus t1/2 of Q is decreased to ⅔ of original
value.
 Half-life of Q in expt 5 = z × 0.4/0.6 = ⅔ z seconds

Comparing expt 4 & 6


In expt 6, [OH] is half of that in expt 4, k’’ is halved and thus t1/2 of Q is
doubled.
 Half-life of Q in expt 6 = 2 z seconds
(Although [Q] is doubled in expt 6, it has no effect on the half-life of Q as
half-life of Q is independent of [Q] since rxn is first order with respect to Q.)

[4]

28
Carbonyl Compounds

(bi)  Compounds P & Q are branched  P and Q are either primary or tertiary
alkyl halides
 Compounds P & Q reduced Fehling’s solution  P and Q are aldehydes
 P and Q, both alkyl halides, undergo nucleophilic substitution with NaOH
to form R and S, both alcohols, respectively.
 Since the reaction is first order with respect to P but zero order with
respect to NaOH, the reaction is an SN1 reaction  P is a tertiary alkyl
halide
 Since the reaction is first order with respect to Q and with respect to
NaOH, the reaction is an SN2 reaction  Q is a primary alkyl halide

23
.
LL

(ii) Nucleophilic substitution – SN2


Step 1:
CH3 CH3 CH3
H3C
slow
C Cl C+ + Cl
+ -
CHO CHO

Step 2: CH3 CH3 CH


3 CH3
HO:¯ fast
C+ HO C
+
CHO
CHO

29
Carbonyl Compounds

(iii)

Energy/kJ mol1

Ea

P + OH
R + Cl

[6]

6 [AJC 2011/II/4(b) modified]


23
(i)
LL

Regents and conditions:


HCN, NaCN catalyst
Step 1
dilute H2SO4, heat under reflux
Step 2

Intermediate F: RCH(OH)CH(OH)CN
Intermediate G: RCH(OH)CH(OH)CH2OH

(ii) H O
C
H C OH
H C OH
HO C H
H C OH
H C OH
CH2OH
(iii) Type of reaction: nucleophilic addition
[8]

30
Carbonyl Compounds

7 [CJC 2011/II/5(b)(c)]
(ai)

(ii) Steps Reagents and conditions

I
HCN, trace amount of KCN or NaOH, 10-20˚C

II H2, Ni catalyst, heat or LiAlH4 in dry ether

III Ethanolic CH3Br, heat


23
[4]
(bi)
Step I: nucleophilic addition
LL

Step II: hydrolysis

(ii)

(iii) 3,4-dihydroxybenzaldehyde contains acidic phenolic groups which will react


with the basic Grignard reagent.
[4]

31
Carbonyl Compounds

8 [SRJC 2011/II/6(b)]

(i) Compound N

CH3
Compound M
H C CH2
CH3
O O
H C CH2OH C CH2 C
OH O O

Compound P
Compound O
CH3 OH
CH3 H
HO C C CN
O C C O
CN H

(ii) HCN with trace amt of KCN/aqNaOH, cold (10 – 20oC)


23
9 [HCI 2011/III/5(b)]
(i) Q undergoes hydrolysis (SN) reaction with aq NaOH to give R (which is an alcohol).
LL

T (non-polar, hydrophobic benzene ring hence insoluble in water) has phenol group which is
acidic enough to react with aq NaOH to give soluble ionic salt. [or: acid-base reaction, phenol
group present]

T undergoes condensation with 2,4-DNPH and gives positive triiodomethane tests, hence is a
methyl ketone.

T R Q
[8]

(ii) Add aqueous neutral FeCl3 to both compounds. P will turn yellow FeCl3 violet but FeCl3
remains yellow for V.
(Can also use aqueous Br2)

32
Carbonyl Compounds

10 [TJC 2011/III/3(e)]

(i) Y liberates white fumes with PCl5, therefore it contains –OH group.
 Y forms orange ppt with Brady’s reagent, therefore it contains a carbonyl
group. Y does not react with Tollen’s reagent, therefore it can only be a
ketone.
 Y forms a yellow ppt in the tri-iodomethane test, hence it contains either
CH3CO-R or CH3CH(OH)-R where R is H or an alkyl chain.
 Y is oxidised to form a carboxylic acid, hence it must contain a primary
alcohol.

Therefore, Y must be
 CH3COCH2CH2OH

[5]

11 [ACJC 2011/III/1(b)]
(i) Let the % of α–form in the equilibrium mixture be x.
x(92.6)  (100  x)(34)
23
 52.3
100
x = 31.2%
LL

(ii) HCN,NaCN (or trace NaOH) dilute HCl, heat


CH3CHO CH3CH(OH)CN CH3CH(OH)CO2H

(iii) Nucleophilic addition

NaCN Na+ + CN–


– + +

+

 electron flow
 partial charge
 correct intermediate
 slow and fast steps

Electron deficient carbonyl carbon in ethanal is planar, hence it is equally susceptible to


nucleophilic attack from either direction, resulting in a racemic mixture.

33
Carbonyl Compounds

(iv)

[9]
Note: Do not assume that all the given answers are correct. Do discuss with your friends and
check with your tutor should you think that any given answer is incorrect.

23
LL

34
Carboxylic Acids and Derivatives

CARBOXYLIC ACIDS AND DERIVATIVES 6


Structured / Free Response Questions

1 [RVHS 2011/III/4(c)(i)]
3-chloropropanoic acid is a drug used in scientific research due to its structural resemblance
to 4-hydroxybutanoic acid, a naturally occurring substance found in the central nervous
system.
O

Cl OH
3-chloropropanoic acid
The pKa of propanoic acid is 4.87. Predict, with reasons, how the pKa of 3-chloropropanoic
acid would differ from propanoic acid.
[2]
23
2 [NJC 2011/11/2(c)]
When C was added to an aqueous solution of silver nitrate, a cream precipitate was
observed immediately.
LL

(i) Write equations to show the formation of the cream precipitate.


[4]
(ii) However, no precipitate was observed when compound H was heated with an
aqueous solution of silver nitrate.

O OH

Br

Suggest an explanation for the unreactivity of H.


[4]

1
Carboxylic Acids and Derivatives

3 [TJC 2011/II/5]
Ascorbic acid, also known as Vitamin C is required for the synthesis of collagen in humans.
A vitamin C deficient diet leads to a disease called scurvy. Ascorbic acid is known to be
water soluble and is commonly used as food additives.

The building block for ascorbic acid is the glucose molecule. The following synthetic
pathway was proposed.
H OH
O HO O CH2 OH
C C CH2
H
O C H
HO C H HO C H O OH O C
Step I Step II C C O OH
HO C H controlled cyclic H Step III C C
oxidation HO C H formation H
H C C H
H C OH H C OH C C
HO OH
H C OH H C OH HO OH
H C H H C H
OH OH

Glucose Intermediate E Intermediate F Ascorbic Acid

(a) State the functional group(s) present in Intermediate F.


23
(b)
(i) State the type of reaction found for step II and that for step III.
LL

(ii) Circle the functional group(s) present in the Intermediate E that are involved in Step
II.

(c) Ascorbic acid, thiols and polyphenols are often reducing agents known as
antioxidants. Antioxidants prevent the oxidation of other molecules by being oxidised
themselves. In doing so, they inhibit other oxidation reactions in the body.
An example of a polyphenol is chlorogenic acid.
HO COOH

HO O

OH
OH

Chlorogenic acid OH

Chlorogenic acid is treated with the following reagents. Draw the structural formulae
of all the organic compounds formed in the reactions.

2
Carboxylic Acids and Derivatives

Reagent Structural formula of organic compounds

Solid Na2CO3

aq NaOH, heat

PCl5
aq Br2

(d) 2-oxopropanedioic acid is also known as mesoxalic acid. It has been used as an
antidote to cyanide poisoning.
O

O C O
C C

OH OH
Mesoxalic acid

(i) State the reagents and conditions required for the conversion of compound G below
into mesoxalic acid, and draw the structures of the intermediate J in the spaces
23
provided.
H2 C CH2 CH CH 2 CH2 Step I H2 C CH CH CH CH 2
LL

OH Br OH Br

G H

O Step II

O C O
C C Step III H2 C CH CH CH CH 2

OH OH OH
Mesoxalic acid Intermediate J

Step Reagents and conditions


I
II Intermediate J
III

(ii) Suggest a chemical test that could be used to distinguish G from H. Write a balanced
equation for the reaction that occurred.

[16]

3
Carboxylic Acids and Derivatives

4 [RI JC2 CT1 2004/II/3]


Warts and verrucas are viral growths on dead skin. They are often treated by application of
an acid. Preparations containing 2-hydroxypropanoic acid (lactic acid), are applied to warts
and verrucas because the value of its acid dissociation constant, Ka, is just high enough for
the acid to kill the infection without damaging the live skin underneath.

(a) Explain whether 2-hydroxypropanoic acid has a larger or smaller Ka than propanoic
acid.

(b) Give the structures of the organic products obtained when 2-hydroxypropanoic acid is
treated with each of the following reagents:

(i) phosphorus (ii) ethanoyl choride (iii) ethylamine


pentachloride

(c) When 2-hydroxypropanoic acid is warmed with concentrated sulphuric acid, a


compound of molecular formula C6H8O4 can be isolated. Suggest a structural formula
for C6H8O4.
[7]
23
5 [HCI 2011/III/3(d)]
The quinone product from the above reaction can undergo the following reaction.
LL

HO2C(CH2)2CO2H
Stage 1 Concentrated H2SO4, reflux
Z
K
Step 2
O O HO OH

(i) State the reagents and conditions needed for Stage 1.

(ii) Draw a possible structure for product K.

(iii) Suggest, in no more than 3 steps, how HO2C(CH2)2CO2H can be obtained

from .

Identify the reagents and conditions for each step, and draw the structural
formulae of all the intermediates.
[5]

4
Carboxylic Acids and Derivatives

6 [MI 2011/II/3(d)(iii)]
Suggest and explain one simple chemical method for distinguishing between the members
of each of the following pair, making clear observations you would expect to make for each
substance.

HCO2H and CH3COOH


[2]

7 [YJC 2011/III/5(d)]
A compound E has the molecular formula, C3H4OCl2. Compound E is a colourless liquid
which fumes in moist air. It reacts with cold water to form compound F, C3H5O2Cl, which is
optically active. When compound E is heated under reflux with aqueous sodium hydroxide
and then acidified, compound G, C3H6O3, is obtained.
(i) Deduce the structural formulae of compounds E, F and G and explain the reactions
that occurred.

(ii) Starting from ethanol, devise a synthesis pathway to prepare compound G. You
should state the reagents and the reaction conditions that are needed for each stage.
[8]

8 [HCI 2011/III/2(d)]
23
The distinctive scent of rose oil comes mainly from a family of closely related chemicals
called the rose ketones. Compounds C, D and E belong to this family.
C and D have the same molecular formula, C13H20O, while E has molecular formula,
LL

C13H18O. C can exist as a pair of enantiomers, but D and E do not show any optical activity.
1 mol of C and D each reacts with 2 mol of liquid bromine at room temperature. 1 mol of E,
however, reacts with 3 mol of liquid bromine.
C, D and E react with hydrogen in the presence of nickel to give the same compound,
C13H26O:
OH

C and D each undergo oxidative cleavage with acidified KMnO4 to give ethanoic acid as one
of the two organic products. In addition, C also gives compound C10H14O4, while D gives
compound C11H16O5 respectively.
E, on the other hand, gives three organic products on reaction with KMnO4: ethanoic acid,
CH3COCO2H and compound F, C8H10O6. 1 mol of F reacts with 1 mol of Na2CO3 to produce
CO2 gas. 1 mol of F also reacts with 2 mol of HCN under cold conditions in the presence of
trace base.
Deduce the structures of compounds C, D, E, and F, giving explanations of the reactions
that occurred.
[8]

5
Carboxylic Acids and Derivatives

9 [VJC 2011/III/4(b)]
D (C10H8O2) is a neutral compound. On heating with aqueous NaOH followed by
acidification, D yields only one organic product, E (C10H10O3), which is soluble in aqueous
Na2CO3. When E is treated with H2 and Ni, F (C10H12O3) is formed. Unlike E, F does not
display cis-trans isomerism.

1 mol of F is found to decolorize exactly 2 mol of aqueous Br2. Upon reaction with LiAlH4 in
dry ether, F forms the compound G. G is no longer soluble in Na2CO3. When G is passed
over hot Al2O3, H is produced.

H reacts with gaseous HCl to give the major product, I, which exists as optical isomers. On
warming I with aqueous AgNO3, a white precipitate is formed together with an organic
product which does not change the colour of acidified K2Cr2O7 solution.

Deduce the structures of all the lettered compounds (D to I) and explain the reactions
involved.

[13]

10 [RVHS 2011/III/2(a)]
Compound A, C16H30O4, is believed to be an anti-cancer drug found in one medicinal plant
extract in the jungles of Southeast Asia. On testing, it was found that:
23
- Compound A is insoluble in both water and dilute aqueous HCl
- Upon heating with dilute HCl, compound A gives two compounds: compound B,
C6H14O and compound C, C4H6O4.
LL

- Compound B can rotate the plane of polarised light and reaction of compound B with
hot acidified potassium permanganate (VII) produces compound D, C6H12O, which
gives a yellow precipitate with iodine in aqueous sodium hydroxide.
- Heating compound B over Al2O3 produces compound E, C6H12, and vigorous oxidation
of compound E produces butanoic acid as one of the products.
- Compound C can be synthesised by reacting 1,2-diiodoethane with hot ethanolic
NaCN followed by warming with dilute sulfuric acid.

Deduce the structures of compounds A, B, C, D and E. Explain the chemistry of the


reactions described.
[10]

6
Carboxylic Acids and Derivatives

11 [DHS 2011/III/3(a)-(b)]
(a) An optically active ester, L, (11.6 g) with the molecular formula of C6H12O2, is
hydrolysed by heating with an excess of aqueous sodium hydroxide solution. After
terminating the hydrolysis, the alkaline reaction mixture is extracted several times with
an organic solvent.

The aqueous and organic layers are then collected separately. The aqueous solution
is found to be optically inactive.

Anhydrous magnesium sulfate is added to the organic layer. The mixture is filtered
and the filtrate distilled. 7.4 g of a liquid J is obtained (assume 100 % purity and yield).

(i) Suggest the role of anhydrous magnesium sulfate in the procedure above.

(ii) Calculate the number of moles of ester used and hence calculate the Mr of liquid J.

(iii) Propose a possible structural formula of the ester.

(iv) What would be the structure of an isomeric ester that would produce an optically
active aqueous layer after hydrolysis?

(v) Write an equation for the alkaline hydrolysis of L with sodium hydroxide.
[6]
23
(b) A food chemist decides to synthesise the ester as shown below, for possible use as a
flavouring agent.
LL

The only organic compound available is phenylethanal (C6H5CH2CHO).

Outline how the food chemist is able to synthesise this ester using only phenylethanal
and other appropriate inorganic reagents.

[2]

7
Carboxylic Acids and Derivatives

Multiple Choice Questions (Section A)


1 [RI JC2 CT1 2011/1/8]

2 [RI JC2 CT2 2008/1/8]

23
LL

3 [CJC 2011/I/21]

4 [RI 2011/1/29]

8
Carboxylic Acids and Derivatives

5 [DHS 2011/I/19]

6 [MJC 2011/I/19]

23
LL

7 [ACJC 2011/I/18]

9
Carboxylic Acids and Derivatives

8 [JJC 2011/I/29]

9 [ACJC 2011/I/25] 23
LL

10
Carboxylic Acids and Derivatives

10 [HCI 2011/I/29]

11 [RI JC2 CT2 2008/1/7]

12 [DHS 2011/I/26]
23
LL

13 [RI JC2 CT2 2009/1/15]

11
Carboxylic Acids and Derivatives

14 [DHS 2011/I/29]

15 [DHS 2011/I/28]
23
LL

16 [MJC 2011/I/23]

12
Carboxylic Acids and Derivatives

17 [ACJC 2011/I/16]

18 [RI JC2 CT1 2003/1/6]


23
LL

19 [HCI 2011/I/28]

13
Carboxylic Acids and Derivatives

20 [MJC 2011/I/20]

21 [ACJC 2011/I/23]
23
LL

22 [RI 2011/1/19]

14
Carboxylic Acids and Derivatives

23 [AJC 2011/I/28]

24 [NJC 2011/I/25]
23
LL

25 [RI 2011/1/22]

15
Carboxylic Acids and Derivatives

26 [JJC 2011/I/27]

23
LL

27 [NJC 2011/I/29]

28 [PJC 2011/I/28]

16
Carboxylic Acids and Derivatives

23
29 [VJC 2011/1/20]
LL

30 [TJC 2011/1/20]

17
Carboxylic Acids and Derivatives

23
LL

18
Carboxylic Acids and Derivatives

Multiple Choice Questions (Section B)

1 [HCI 2011/1/39]

23
2 [ACJC 2011/I/36]
LL

3 [CJC 2011/I/40]

19
Carboxylic Acids and Derivatives

4 [NJC 2011/I/39]

23
LL

5 [RI 2011/1/39]

20
Carboxylic Acids and Derivatives

CARBOXYLIC ACIDS AND DERIVATIVES SUGGESTED SOLUTIONS

Multiple Choice Questions (Section A)


1. 2. 3. 4. 5. 6. 7. 8. 9. 10. 11. 12. 13. 14. 15. 16. 17. 18. 19. 20.
B C D A A B C B D A D D C D B B B C D D
21. 22. 23. 24. 25. 26. 27. 28. 29. 30.
B D C D A C D B A D

Multiple Choice Questions (Section B)


1. 2. 3. 4. 5.
A D D B D

Structured/Free Response Questions

1
pKa of 3-chloropropanoic acid will be lower than 4.87/that of propanoic acid.
3-chloropropanoic acid is a stronger acid [] than propanoic acid because the electron-
withdrawing Cl atom [] disperses the negative charge on the COO– group [], making the 3-
chloropropanoate ion more stable [[] than the propanoate ion.
23
2 (i)
LL

O O O O

+ H2O
+ HBr
Br OH

Br(aq) + Ag(aq) AgBr(s)


Cream ppt

(ii) The lone pair on Br is delocalized with the pi electrons on the adjacent C=C bond/ p-
orbital on Br overlaps with the pi-orbitals of adjacent C=C bond/ resonance,
 strengthening the C-Br bond in H/ resulting in partial double bond character in C-Br
bond in H. [1]

C-Br bond in H is not easily broken,


 no free Br– ions available to react with Ag+ to form AgBr, cream ppt. [1]

Other possible reasons:


1. Extra resonance stability for H due to extended conjugation/ delocalization of
electrons from benzene ring, C=O, C=C, oxygen atom of –OH group to Br.
Hence H will not undergo nucleophilic substitution even with heating as the
activation energy is very high.

21
Carboxylic Acids and Derivatives

2. The lone pair on the oxygen atom of –OH group is delocalized into the C=C pi
electron cloud. This decreases the partial positive charge on the carbon atom
next to Br, hence H is less susceptible towards nucleophilic substitution.

3 (a) Primary alcohol, secondary alcohol, ester

(bi)
●Step II: Condensation (Nucleophilic Substitution)
●Step III: Elimination / Oxidation

(bii)

Intermediate J
23
(c)
Reagent Structural formula of organic compounds
LL

Solid Na2CO3 HO COO- Na +

HO O
OH
OH

OH

aq aOH, heat HO COO- Na +


O

+
Na- O
HO OH

OH O -Na +

O- Na +

22
Carboxylic Acids and Derivatives

PCl5 Cl COCl

Cl O

Cl
OH

OH

aq Br2 HO COOH

O Br Br

Br
HO O

OH OH
Br OH

OH
OR
HO COOH
23
O OH Br

Br
HO O
LL

OH Br
Br OH

OH

(di)
Step Reagents and conditions

I  excess concentrated H2SO4, 180 °C or Al2O3, 400 oC

II  NaOH(aq), heat/reflux

III  KMnO4(aq), dilute H2SO4, heat/reflux

(dii)
●Add Br2 dissolved in CCl4 under room conditions each to compound G and H
separately.
With compound H, decolourisation of Br2 and no decolourisation for G

23
Carboxylic Acids and Derivatives

H 2C CH CH CH CH 2 H 2C CH CH CH CH2
+ 2Br2 (CCl4)

● Br Br Br Br Br Br

(other acceptable answers include:


Br2(aq), PCl5, Na metal)

4 (a) 2-Hydroxypropanoic acid is a stronger acid than propanoic acid and hence has
a larger Ka than propanoic acid.

Consider the following equilibria.

CH3 CH COOH CH3 CH COO


+ H+
OH OH
………(1)

CH3 CH2 COOH CH3 CH2 COO + H+


……….(2)

The –OH group is electron-withdrawing due to the presence of the


23
electronegative oxygen atom. It helps to disperse the negative charge in the
2-hydroxypropanoate ion and consequently stabilises the ion.
LL

Such a stabilising effect is absent in the propanoate ion.

Consequently, the equilibrium position of reaction (1) is more to the right with
correspondingly more H+ ions produced. Hence 2-hydroxypropanoic acid is a
stronger acid than propanoic acid.

(b) (i) phosphorus (ii) ethanoyl choride (iii) ethylamine


pentachloride +
CH3 CH COOH CH3 CH COO H3N CH2CH3
CH3 CH COCl
O C CH3 OH
Cl
Note: An acid-base reaction has
O occurred.

(c)
O

H3C C
CH O

O CH
C CH3
O 24
Carboxylic Acids and Derivatives

5 (i) LiAlH4 in dry ether, room temperature OR NaBH4 in (alkaline) methanol, room
temperature

(ii)

O
O O C (CH2)2C
O O
O
O O or n

(iii) KMnO4,
dilute H2SO4, heat O O

Step 1

I2(aq), NaOH(aq)
Step 2
warm
23
O O Dilute H2SO4 O O
HO OH O O
LL

Step 3
.

6 Reagents and conditions:


Add KMnO4(aq) acidified with dilute H2SO4 to each sample and heat each mixture.

Observations:
For HCOOH, there would be decolourisation of purple KMnO4 and also the evolution of CO2
gas which formed a white precipitate when bubbled into limewater/ Ca(OH)2(aq).
For CH3COOH, there would be no reaction and hence no decolourisation of purple KMnO4.

7 (i) structures of unknowns

E F G
Cl O Cl O OH O

CH3 C C Cl CH3 C C OH CH3 C C OH

H H H

25
Carboxylic Acids and Derivatives

Explanations:
 E hydrolyses in moist air to form white fumes of hydrogen chloride and reacts with
water to form carboxylic acid. Thus E is an acid chloride.
 F has a chiral carbon atom since it is optically inactive.
 When reflux with NaOH(aq), E undergoes nucleophilic substitution reactions to form
a hydroxycarboxylate anion, which upon acidification forms a hydroxycarboxylic
acid.

(ii) K2Cr2O7, H2SO4


CH3CH2OH CH3CHO
immediate distillation
HCN OH
traces of NaCN(s) or NaOH(aq)
CH3CHO CH3 C CN
10 oC - 20 oC
H
OH OH O
H2SO4(aq)
CH3 C CN CH3 C C OH
heat
H H
23
8
Summary of the reaction scheme:
LL

26
Carboxylic Acids and Derivatives

Observations Deductions
C can exist as a pair of enantiomers but D C contains a chiral carbon whereas D and E
and E does not show any optical activity. does not contain chiral carbon (or are
symmetrical).
1 mol of C and D reacts with 2 mol of liquid Electrophilic addition. C and D have 2 C=C,
bromine. 1 mol of E reacts with 3 mol of R has 3 C=C.
liquid bromine.
C, D and E react with hydrogen in the (Catalytic) reduction/hydrogenation occurs.
presence of nickel to give the same
compound.
C undergoes oxidative cleavage with C contains a terminal alkene as there is loss
acidified KMnO4 to give ethanoic acid and a of CO2.
compound containing 10C. D does not contain terminal alkene.
D undergoes oxidative cleavage with
acidified KMnO4 to give ethanoic acid and a C and D contains the CH3CH=C structure.
compound containing 11C.
F reacts with 1 mole of Na2CO3 to produce Acid-base reaction occurs and F is dibasic
CO2. (dicarboxylic acid).
F reacts with 2 moles of HCN under cold Nucleophilic addition occurs and F contains 2
condition with base as the catalyst. carbonyl groups.
23
Structures of unknown:

O O O O
LL

OH

O O
O OH
P Q R S

C D E F

27
Carboxylic Acids and Derivatives

9 Evidence Deduction
D (C10H8O2) High C:H ratio shows that D has a benzene ring.
D + aq NaOH  E D is hydrolysed.
Since only E is formed, D is a cyclic ester.
E soluble in Na2CO3. E undergoes acid-base reaction and contains –CO2H.
E displays cis-trans E has C=C with 2 different groups attached to each C of
isomerism whereas F the alkene group.
does not. Alkene in E is reduced to alkane in F.
1 mol F reacts with 2 F contains phenol group which activates the benzene ring.
mol aq Br2 Hence, it undergoes electrophilic substitution readily.
2 mol of Br2 are required since a cyclic phenolic ester
forms a 1, 2-disubstituted ring.
F + LiAlH4  G G does not contain –CO2H.
G not soluble in Hence –CO2H in F is reduced to give primary alcohol in G.
Na2CO3
G + Al2O3  H Elimination / dehydration takes place.
23
H is an alkene.
H + HCl  I Electrophilic addition takes place.
LL

I  optical isomers I possesses a chiral centre.


I + aq AgNO3 White ppt is AgCl. I is confirmed to be a halogenoalkane.
 white ppt + org I undergoes nucleophilic substitution with water to form
product an alcohol.
Since the organic product is not oxidized, alcohol formed
is tertiary OR I is a tertiary halogenoalkane.

Structures [1m each]

28
Carboxylic Acids and Derivatives

23
10
A is not soluble in acid : A is not basic. []
Acid hydrolysis [] of A took place to give B and C. A is likely to be an ester [] while B is
likely to be an alcohol. []
LL

B rotates plane-polarized light. B has at least 1 chiral C. []


B is oxidized [] by hot acidified KMnO4 to give D, with the removal of 2 H in D. D is a ketone
[] while B is a secondary alcohol. []
D gives a positive iodoform test [] with alkaline I2: D is a methyl ketone [] while B is a
methyl alcohol. []
B undergoes dehydration [] with Al2O3 to give alkene E. []
E undergoes oxidative cleavage to give butanoic acid. E contains a fragment with structure.

[]
I I CN CN
ethanol
H2C CH2 + 2NaCN
heat H2C CH2 + 2 NaI
[]
CN CN dil H2SO4
HOOC CH2 CH2 COOH
H2C CH2 + 4H2O + 2H+ heat
CR []
29
Carboxylic Acids and Derivatives

A B

C D
23
LL

30
Carboxylic Acids and Derivatives

11a (i) The anhydrous magnesium sulfate is a drying agent; it removes any remaining
traces of water from the product.
(ii) n(ester) = 11.6/116 = 0.100 mol
n(liquid J) = 0.100 mol
Mr (liquid J) = 7.4/0.100 = 74.0
(iii)

(iv)

(v)

OR 23
b 1. Reflux phenylethanal with KMnO4/H+ to get benzoic acid.
2. Reduce phenylethanal with LiAlH4 in dry ether to get 2–phenylethanol, C6H5(CH2)2OH.
Reflux benzoic acid and 2–phenylethanol with conc. H2SO4 to get the ester.
LL

Note: Do not assume that all the given answers are correct. Do discuss with your
friends and check with your tutor should you think that any given answer is incorrect.

31
NITROGEN COMPOUNDS

Structured and Free-Response Questions


7
1 [IJC 2011/III/3(a)]
Atenolol is a drug used mainly to prevent hypertension and coronary heart disease.

Atenolol
[Assume that the ether functional group is inert and does not undergo any reaction.]

Draw the structural formula of the organic product formed when Atenolol is treated with the
following reagents:
(a) hot aqueous hydrochloric acid
(b) ethanoyl chloride
23
[4]

2 [PJC 2011/II/5(b)]
LL

Tamiflu is an antiviral drug which has been used to treat and prevent influenza A and
influenza B virus infection since 1999. The structure of a derivative of Tamiflu is shown
below.

(a) Name the functional groups present in the Tamiflu derivative.


[2]

(b) Draw the structural formula of each of the organic products formed when the Tamiflu
derivative is treated with the following reagents:
(i) cold alkaline potassium manganate(VII)
(ii) hot acidified potassium dichromate(VI)
(iii) limited chloromethane with heating
[5]
1
3 [IJC 2011/III/4(d)]
The following reaction scheme shows the formation of compound D, benzene-1,2-
dicarboxylic acid.

reaction II

reaction III

(a) State the type of reactions for reaction I and III.


(b) Suggest reagent(s) and condition(s) for reaction II and III.
[3]

4 PJC 2011/II/7(b)]
A method to synthesise amines directly from primary amides is the Hofmann
rearrangement reaction, which was discovered by August Hofmann in 1882. Upon
treatment with bromine in aqueous alkali, a primary amine is formed, as exemplified by the
following:
23
CH3 O CH3
Br2 + NaOH(aq)
CH3 C C NH2 CH3 C NH2
LL

CH3 CH3

Phenylamine can be formed from ethylbenzene in a four-step reaction sequence given


below. Draw the structural formulae of the intermediate compounds X, Y and Z and give
the reagents and conditions for step I to IV. You are required to make use of Hofmann
reaction in one of the steps.
step I step II
CH2CH3 X Y

ethylbenzene step III

step IV

NH2

phenylamine
[5]
2
5 [CJC 2011/II/5(a), 5(b)]
Adrenaline is a natural hormone and acts as a stimulant when secreted directly into the
bloodstream. It has the following structure:

(a) Give the structural formulae of the organic compounds formed when Adrenaline
reacts with each of the following reagents and under the stated conditions.

(i) SOCl2
(ii) CH3COOH (at room temperature)
(iii) Na
[4]

(b) Reactions involving the formation of a hydroxynitrile (cyanohydrin) as an intermediate


are useful because of the further reactions that can be carried out. More importantly, it
increases the number of carbon atoms in a chain as a C-C bond is formed.

Adrenaline can be synthesised from 3,4-dihydroxybenzaldehyde via the following


reaction pathway with a hydroxynitrile being the intermediate.
23
LL

compound A

(i) Draw the displayed formula of compound A.

(i) State the reagents and conditions required for Steps I – III.
[4]

3
6 [DHS 2011/II/5(b)]
The natural occurring amino acids phenylalanine and tyrosine are important building blocks
for many molecules in plants and animals.

(a) Phenylalanine, together with aspartic acid and methanol, is used to make aspartame,
an artificial sweetener. The main synthetic steps are shown below.

(i) Draw a possible structure for X used in Step II.

(ii) Suggest a reason for not mixing X, phenylalanine and methanol together in a
one–step synthesis of aspartame.
[2]
23
(b) In mammals, tyrosine forms adrenaline, a ‘fight or flight’ hormone.
LL

(i) Draw a pair of enantiomers for adrenaline in the space below.


(ii) Tyrosine and adrenaline are separately reacted with aqueous bromine in a cold,
dark room. State and explain which molecule would decolourise bromine at a
faster rate.
(iii) Tyrosine is soluble in water, while adrenaline is not. Explain this difference in
their solubilities.
[4]

4
7 [NYJC 2011/II/5] & [MI 2011/III/3(d)(ii)

(a) Phenylamine can be synthesized from nitrobenzene in a two-step process shown.

(i) Write the formula of the intermediate P.

In the redox reaction in step 1, tin reacts with hydrochloric acid to generate tin (II) ions
which acts as a reducing agent. In reducing nitrobenzene, tin (II) ions are oxidized to
tin (IV) ions.

(ii) Write an equation to show the reaction between tin and hydrochloric acid.

(iii) By means of balanced half-equations, write an equation to show the redox


reaction for step 1.
[5]
23
LL

(b) Phenylamine undergoes the following reaction scheme.


Br

Step 1 Step 2
NH2 NHCOCH3 Br2, under
suitable NHCOCH3
Q conditions
R
(i) Suggest the reagents and conditions for step 1.

(ii) Another possible product besides R is formed. Suggest its structual formula.

(iii) Intermediate Q has three other isomers, X, Y and Z. Identify the other isomers
given the following information:
 Isomer X gives ammonia on reaction with hot aqueous sodium hydroxide.
On reaction with hot acidified potassium manganate (VII), 1,4-dibenzoic
acid is formed.
 Isomer Y gives methanoic acid on reaction with hot aqueous hydrochloric
acid.
 Isomer Y can be used to form the following polymer:

5
O O

N C N C N

CH3 CH3
CH3

 Both X and Y do not react with aquoeus bromine, but reacts with bromine
in the presence of iron (III) bromide.
 Isomer Z does not give methanoic acid or ammonia on hydrolysis.
 1 mol of Z reacts with 3 mol of aqueous bromine to give a substituted
product.

Suggest structures for X, Y and Z.


[5]
(c) Suggest and explain one simple chemical test for distinguishing between the following
pair of substances, stating clearly the observations you would expect to make for
each substance.
23

[2]
LL

8 [IJC 2011/II/2(c)] & [IJC 2011/III/4(e)]


(a) Arrange the following compounds in order of decreasing pKb, giving reasons for your
answer.
NH2 NH2 NHCOCH3

[3]

(b) Compound E, C10H7NO can exhibit geometric isomerism. E can react with
diammine silver(I) but not Fehling’s solution. On reacting E with lithium aluminium
hydride, F, C10H13NO is formed. F is readily soluble in acid. 1 mole of F reacts with 2
moles of ethanoyl chloride to give the compound below:

When E is heated with dilute sodium hydroxide, the dried product is a white crystalline
solid, G, C10H7O3Na.

Deduce the structural formula of E, F and G, explaining your reasoning. [7]

6
9 [JJC 2011/II/5]
Sulfanilamide belongs to a class of drugs known as sulfonamides, which are widely used
as antibiotics and in the treatment of hair-loss. It can be synthesised from phenylamine via
the following synthesis route.

23
(a) (i) State the reagents and conditions required to convert nitrobenzene to
phenylamine.
LL

(ii) State the reagents and conditions for reaction I.


(ii) Name the type of reaction occurring in reaction II.
[3]

(b) Draw the structural formula of compound A in the box provided above.
[1]

(c) Suggest why the acetyl group (COCH3) was introduced in reaction I and then
removed to regenerate the amine group in sulfanilamide.
[1]
(d) Explain why sulfanilamide is insoluble in water.
[1]
(e) The basicities of phenylamine, methylamine and trimethylamine are different.

Compound C6H5NH2 CH3NH2 (CH3)3N


pKb 9.37 3.43 4.20

(i) Arrange the three amines in increasing order of basic strength.


(ii) Explain the relative basic strength of the three amines in terms of their molecular
structures
[3]

7
10 [NJC 2011/III/2(a)]
Aspirin and paracetamol are examples of drugs that have gained world-wide recognition for
their medicinal properties. Aspirin and paracetamol are known as analgesics which is a
group of drugs used to relieve pain.

(a) Devise a three-step synthesis, starting with phenol, for the manufacture of
paracetamol.

(b) Suggest a method by which aspirin can be distinguished from paracetamol.

(c) Predict, with a reason, whether aspirin or paracetamol has a lower pH.

(d) Give the displayed formula of an isomer of aspirin which is not soluble in both
hydrochloric acid and sodium hydroxide.
[10]
23
11 [YJC 2011/III/5(c)]
Suggest a chemical test, stating the reagents and observations that enable the compounds
below to be distinguished from each other.
LL

and
N O

[2]

12 [TJC 2011/III/2(c)]
Methyl orange is another acid-base indicator. It can be synthesised from
N,N-dimethylphenylamine and diazonium salt.

CH3
N
CH3
N,N-dimethylphenylamine

Suggest a 3-step synthesis of N,N-dimethylphenylamine from benzene.


[3]

8
13 [PJC 2011/III/5(c)]
Compound H has the molecular formula, C9H9NO.
 H is insoluble in aqueous sodium hydroxide and hydrochloric acid at room temperature.
 When H is heated with acidified potassium manganate(VII), a compound with the
following structure is obtained.
CO2H

+
NH3

Compound H can be prepared by a series of reactions starting from optically active


compound J, C9H12O.
 Compound J, C9H12O, reacts with concentrated nitric acid and sulfuric acid to give K,
C9H11NO3.
 Treating compound K with hot acidified potassium dichromate(VI) gives compound L,
C9H9NO4.
 Compound L then reacts with phosphorus pentachloride to give M, C9H8NO3Cl.
 Reduction of M under suitable conditions yields N, C9H10NOCl, which undergoes
immediate reaction by eliminating a molecule of hydrogen chloride to give H.
23
Deduce the structures of H, J, K, L, M and N. Explain the chemistry of the reactions
described. [9]
LL

14 [TPJC 2011/III/5(c)]
Compound M (C10H13O3N) reacts fully with Br2 (aq) in the ratio 1:2, but it does not
decolourise cold alkaline KMnO4. M also shows no reaction with PCl5. When warmed in
aqueous alkaline and subsequently neutralised, M produces P (C8H11O2N) and Q
(C2H4O2). P changes acidified K2Cr2O7 from orange to green, and forms R (C8H9O3N) in
the process. Both Q and R react with Na2CO3 solution to give effervescence. R also reacts
fully with Br2 (aq) in the ratio 1:2. Incidentally, R can be found naturally as a building block
for proteins. It was noted that R does not have intramolecular hydrogen bonding.
Suggest structures for M, P, Q and R, explaining how the identities you suggest fit in with
the experimental observations described above.
[10]

9
15 [HCI 2011/III/4(e)]
N2O4 can be used to nitrate aromatic hydrocarbons in the presence of acid catalysts. Nitro
compounds are of particular interest due to their potential use as explosives.

One such compound is picric acid. Under suitable conditions, picric acid can be formed
from 2,4,6-trinitrochlorobenzene, as shown in Step III.

23
LL

(a) State the reagents and conditions for Step I.

(b) Suggest the type of reaction that occurs in Step III.

(c) The reaction in Step III involves the formation of an anionic intermediate:

Suggest a reason why this anion intermediate can be formed.

(d) State the reagents and conditions for Step IV.

(e) Deduce a structure of compound M.


[5]

10
Multiple Choice Questions (Section A)

1 [NJC 2011/I/24]

2 [NYJC 2011/I/29]

3 [CJC 2012/I/29]
23
LL

4 [HCI 2011/I/30]

5 [JJC 2011/I/25]

11
6 [MJC 2011/I/24]

7 [VJC 2011/I/27]

8 [RI 2011/1/21]
23
LL

12
9 [ACJC 2011/I/17]

10 [TPJC 2011/I/27]
23
LL

11 [SAJC 2011/I/29]

13
12 [AJC 2011/I/29]

13 [ACJC 2011/I/19]
23
LL

14
14 [VJC 2011/I/29]

23
15 [MJC 2011/I/21]
LL

15
16 [HCI 2011/I/23]

17 [DHS 2011/I/23]
23
LL

18 [PJC 2011/I/29]

16
19 [YJC 2011/I/29]

23
LL

20 [SAJC 2011/I/23]

17
21 [YJC 2011/I/28]

22 [PJC 2011/I/30]
23
LL

23 [PJC 2011/I/26]

18
24 [MJC 2011/I/25]

25 [NJC 2011/I/27]
23
LL

26 [DHS 2011/I/25]

19
27 [SAJC 2011/I/28]

23
LL

28 [JJC 2011/I/24]

20
29 [TPJC 2011/I/23]

30 [VJC 2011/I/Q25]
23
LL

31 [AJC 2011/I/30]

21
Multiple Choice Questions (Section B)

1 [YJC 2011/I/31]

23
LL

2 [JJC 2011/I/32]

3 [ACJC 2011/I/37]

22
4 [AJC 2011/I/40]

23
5 [MJC 2011/I/36]
LL

23
6 [YJC 2011/I/40]

23
LL

7 [NJC 2011/I/40]

24
8 [SRJC 2011/I/38]

9 [RI 2011/1/37]
23
LL

10 [NJC 2011/I/38]

25
11 [NJC 2011/I/37]

23
LL

12 [RI 2011/1/40]

26
13 [VJC 2011/I/40]

14 [SRJC 2011/I/39]

23
LL

15 [TPJC 2011/I/36]

27
16 [SAJC 2011/I/40]

17 [TJC 2011/I/40]
23
LL

28
NITROGEN COMPOUNDS – SUGGESTED SOLUTIONS

 Note:
Do not assume that all the given answers are correct. Do discuss with your friends and check
with your tutor should you think that any given answer is incorrect.

Multiple Choice Questions (Section A)

1 2 3 4 5 6 7 8 9 10 11 12 13 14 15 16 17 18 19 20
C A B C D D B D C A C C B C B B B B D C
21 22 23 24 25 26 27 28 29 30 31
C A C B B B C A D C B

Multiple Choice Questions (Section B)

1 2 3 4 5 6 7 8 9 10 11 12 13 14 15 16 17
A A A D A C C B D A B A A B D C B

Structured and Free-Response Questions

1 [IJC 2011/III/3(a)]
(a) (b)
23
LL

2 [PJC 2011/II/5(b)]
(a) secondary amide, primary amine, alkene, ester
(b) (i) OH O (iii) O
OH
O O
O O
CH2CH3
CH2CH3 CH3 NH
CH3 NH
NH
NH2 CH3
(ii) O

OH

+
H3N
+
NH3 + CH3CO2H

29
3 [IJC 2011/III/4(d)]
(a) Reaction I: Nucleophilic Substitution , SN2
Reaction III: Side-chain Oxidation(Accept oxidation)

(b) Reaction II
Reagent and condition: Dilute H2SO4 or HCl or aq NaOH, heat under reflux
Reaction III
Reagent and condition: KMnO4, dilute H2SO4, heat under reflux

4 PJC 2011/II/7(b)]

Compound X Compound Y Compound Z


O O O
C C C
OH Cl NH2

Step I: KMnO4, dilute or aqueous H2SO4, heat


Step II: PCl5
Step III: NH3(g)
Step IV: Br2, NaOH(aq)
23

5 [CJC 2011/II/5(a), 5(b)]


LL

(a) (i) (iii)

(ii)

(b) (i)

A:
(ii)

30
6 [DHS 2011/II/5(b)]
(a) (i)

(ii) X contains two functional groups / two carboxylic acids which can react
with methanol or phenylalanine, forming unwanted side products.
(b) (i)

(ii) Adrenaline contains two electron donating hydroxyl groups, while tyrosine
only has one. The benzene ring in adrenaline is more activated / reactive.
Thus, adrenaline would react at a faster rate.
Or
The presence of more electron donating hydroxyl groups in adrenaline
compared to tyrosine makes the ring more electron–rich. Thus, adrenaline
would react at a faster rate.
(iii) Tyrosine exists as a zwitterion / ionic compound, thus can form favourable
ion–dipole interactions with water. Adrenaline can form hydrogen bonds
23
with water molecules but the energy released is insufficient to overcome
the hydrogen bonds between water molecules / adrenaline molecules.
LL

7 (a) (i)

(ii) Sn + 2HCl  Sn2+ + 2Cl- + H2


(iii) reduction half-equation :

+ 7H+ + 6e 2 H2O +

NO2 NH3+ +

oxidation half-equation : Sn2+  Sn4+ + 2e


Overall equation:

+ 3Sn2+ + 7H+ 2 H2O + + 3Sn4+

NO2 NH3+
+

31
(b) (i) CH3COCl, room temperature
(ii)

Br
NHCOCH3
(iii)
O
NH2
O
C NH2
HN

CH3

CH3 CH3 O
X: Y: Z:

Other possible answers:


NH2 NH2

H3C
23
O
O
LL

(c) Reagents and conditions: aqueous bromine solution.


O NH2
NH2

Observations: will decolourise brown aqueous bromine but will not.

8 [IJC 2011/II/2(c)] & [IJC 2011/III/4(e)]


(a)

Highest pKb Lowest pKb

For , the lone pair of electrons on N atom is delocalized over the carbonyl
group and is not available to accept a proton. Hence it is neutral and least basic
among the three compounds.

32
Phenylamine is less basic than as the lone pair on N atom is delocalized into
the benzene ring and is thus less available to accept a proton.

is the most basic as the alkyl group increases the electron density on the N
atom, making the lone pair of electrons more available to accept a proton than in
ammonia.

[IJC 2011/II/2(c)] & [IJC 2011/III/4(e)]


(b) E: B:
O H H

H C C C CN

F: C:
OH H H
23
H C C C CH2NH2

H
E:
G:
LL

O H H

H C C C COO Na

E has a relatively high carbon to hydrogen ratio and hence has a benzene ring.
E has C=C bond / alkene since it can display geometric isomerism.
E undergoes mild oxidation in presence of Tollen’s reagent and has an aromatic
aldehyde functional group.
E undergoes reduction to form F which has a primary alcohol functional group.
F has an amine functional group since it is soluble in acid.
F undergoes nucleophilic substitution to form G.
E undergoes alkaline hydrolysis to form E which is a sodium carboxylate salt.

9 [JJC 2011/II/5]
(a) (i) 1. Concentrated HCl, Sn, heat
2. NaOH/NaHCO3/Na2CO3

(ii) CH3COCl

(iii) electrophilic substitution

33
(b)

(c) It serves as a protecting group to protect the amine functional group.


(d) Sulfanilamide is insoluble in water due to the presence of bulky/large non polar
benzene ring, which hinders the formation of effective hydrogen bonding with water
molecules.
(e) (i) C6H5NH2 < (CH3)3N < CH3NH2

(ii)
C6H5NH2 vs CH3NH2 & (CH3)3N:
For C6H5NH2 or phenylamine, the lone pair of electrons on N atom is delocalised into
the benzene ring, making it less available for protonation.
Thus, C6H5NH2 or phenylamine is a weaker base than CH3NH2 & (CH3)3N.
CH3NH2 vs (CH3)3N:
(CH3)3N or trimethylamine is a weaker base than CH3NH2 or methylamine because
the protonated trimethylamine is less stabilised due to weaker extent of hydration.
23

10 [NJC 2011/III/2(a)]
LL

(a)

(b) Reagent and condition: 1) Neutral FeCl3


2) Na2CO3 (aq)
3) Br2(aq)
4) PCl5
Observation(s):
1) Intense purple complex for paracetamol; yellow solution for aspirin
2) Effervescence for aspirin; no gas released for paracetamol
3) Yellow brown solution decolourised for paracetamol;
yellow brown solution remains for aspirin
4) White fumes for aspirin, no white fumes for paracetamol
(c) Aspirin
Negative charge in the anion of aspirin is delocalised over an extra electronegative O
atom compared that of paracetamol.
Hence the anion of aspirin is stablised to a greater extent, and aspirin is more acidic,
i.e. lower pH.

34
(d) H O
C O CH3
C
O
O

11 [YJC 2011/III/5(e)]

Distinguishing Test N
N O H

H O

warm with NaOH(aq) brown colour of Br2(aq)


no decolourisation and no
followed by addition of decolourised and appearance
precipitate
Br2(aq) of a white precipitate

12 [TJC 2011/III/2(c)]
conc H2SO4 Sn/conc HCl
+ conc HNO3 CH3I CH3
NO2 heat NH2 N
60oC followed by heat CH3
NaOH

13 [PJC 2011/III/5(c)]
23
Information Deduction and explanation.
H is insoluble in aqueous sodium hydroxide  H neutral; H is a amide
and hydrochloric acid at room temperature.
LL

Compound H can be prepared by reacting  J has a chiral centre


optically active compound J, C9H12O, with
concentrated nitric acid and sulfuric acid to  J is not phenol, J is aldehyde/alcohol
give K, C9H11NO3.  J undergo electrophilic substitution;
-NO2 group substituted at relative
postion 2

Compound K reacts with acidified potassium  K undergoes oxidation to give


dichromate(VI) to give compound L, carboxylic acid
C9H9NO4.  K is a 1o alcohol
 L is carboxylic acid

Compound L is then reacted with  L undergoes nucleophilic substitution


phosphorus pentachloride to give M, to give M
C9H8NO3Cl.
 M is acyl chloride.
Under suitable conditions, compound M is  -NO2 reduced to –NH2
reduced to give N, C9H10NOCl, which  Acyl chloride and –NH2 react to
undergoes immediate reaction by eliminate a molecule of HCl, giving
eliminating a molecule of hydrogen chloride cyclic compound H amide present in H.
to give H.

35
CH3
CH3 CH3
* *
* CH CH2OH CH CH2OH CH COOH

NO2 NO2

J, C9H12O K, C9H11NO3 L, C9H9NO4

CH3 CH3 CH3


* * *
CH CH CH
C O C O
C O
NO2 Cl NH2 Cl N
H

M, C9H8NO3Cl N, C9H10NOCl H, C9H9NO

14 [TPJC 2011/II/5(c)]
M, P and R have high C:H ratio  M, P and R contain benzene rings
23
M and R react with Br2(aq)  M and R contain phenol or alkene
M does not react with cold alkaline KMnO4  M is not alkene
M does not react with PCl5  M is not alcohol or carboxylic acid
LL

M undergoes alkaline hydrolysis  M has ester or amide linkage


OR M has ester since primary alcohol in P is used for linkage
P reacts with K2Cr2O7 to give R (+1O -2H)  P is primary alcohol
Q and R react with Na2CO3  Q and R is carboxylic acid.
R is alpha amino acid  R contains -CH(NH2)COOH
R has no intramolecular hydrogen bonding  R must be 1,4-isomer
OR disubstituition of Br on phenol, thus R must be 1,2 or 1,4-isomer

O O
H2N H 2N H2N
O OH OH

M is OH P is OH R is OH Q is CH3COOH

36
15 [HCI 2011/III/4(e)]
(a) Cl2 (g), AlCl3 catalyst, heat
(b) Nucleophilic substitution
(c) The anionic intermediate can be formed because it is stabilised as its negative charge
is dispersed by the electron-withdrawing –NO2 groups attached to the benzene ring.
(d) Limited Sn, conc HCl, heat, followed by NaOH (aq)
(e)

OR

23
LL

37
Electrochemistry

ELECTROCHEMISTRY 9a
Structured / Free Response Questions

1. [AJC 2011/III/5(c), DHS 2011/II/6(d), MJC 2011/III/4(a](i), IJC 2011/III/5(a)]

Ethanol fuel cells are a more practical and viable source of energy as compared to hydrogen
fuel cells.

wire connecting a pair of


graphite electrodes

ethanol air (contains O2 gas)

CO2 H2O

cathode
anode
H2SO4 (aq)
(i) Write balanced half-equations, with state symbols, for the reactions taking place
at the anode and cathode when the fuel cell is in operation.
23
(ii)Use appropriate data from the Data Booklet to explain why an acidic electrolyte
is often preferred over an alkaline or neutral electrolyte.
[4]
LL

2. [DHS 2011/III/1(a)-(b), JJC 2011/II/4(d)-(e), HCI 2011/II/3(c)]


(a) Iron is the cheapest and one of the most abundant of all metals, comprising
nearly 5.6% of the earth's crust and nearly the earth’s entire core. It exists in a wide
range of oxidation states, from −2 to +6, although ferrous (Fe2+) and ferric (Fe3+)
compounds are more common.

(i) Acidic solutions containing ferrous ions are oxidised to ferric ions in air, with no
precipitation seen. On the other hand, ferrous ions give a precipitate in alkaline
solutions and the precipitate turns reddish–brown in air.

With reference to the Data Booklet, explain the two reactions using relevant E
values, writing equations where appropriate.
[4]
(ii) Ferric ions can catalyse the reaction between I–(aq) and S2O82–(aq).
By considering relevant E values, describe and explain the role of the ferric ions
in this reaction, writing equations where appropriate.
[3]

(b) The presence of CN or OH significantly affects the oxidising ability of iron(III) ions.
Illustrate this effect by quoting appropriate data from the Data Booklet and arrange the
following three iron(III) compounds in increasing order of oxidising ability.
Fe(NO3)3, Fe(OH)3, K3Fe(CN)6

1
Electrochemistry

[2]

(c) Potassium ferrate(VI), K2FeO4, is a rare example of an Fe(VI) compound. Reflecting


its high oxidation state, FeO42– is a powerful oxidant.

The standard electrode potential for the following half reaction is +2.00 V.

FeO42–(aq) + 8H+(aq) + 3e Fe3+(aq) + 4H2O(l)

Using relevant data from the Data Booklet, predict whether a reaction will occur when
K2FeO4 crystals are warmed with an aqueous solution of chromium(III) sulfate.

Describe the expected appearance of the resultant solution and write an equation for
any reaction that occurs.
[3]

3. [IJC 2011/III/1(a)]
(a) (i) Manganese(IV) oxide, MnO2 is the most common starting material for the
production of compounds of manganese of other oxidation states.

With reference to the Data Booklet, state which of the following ions would
convert manganese(IV) oxide to manganese(II) sulfate in an acidic solution.
23
Pb4+(aq) / Pb2+(aq) E = +1.69V
Co3+(aq) / Co2+(aq) E = +1.82V
Fe3+(aq) / Fe2+(aq) E = +0.77V
LL

(ii) Calculate the Ecell value to prove that the reaction between MnO2 and the ion
that you have chosen in a(i) is feasible and write a balanced equation for the
reaction.

(iii) What would be the colour of the final solution observed?


[5]

4. [MJC 2011/II/3(a)-(b)+(g)]
Old mine workings are often the cause of the pollution of mountain streams. One of the main
chemical culprits of such pollution is iron pyrites, FeS2, in which iron has an oxidation state
of +2.

The early miners unwittingly accelerated the chemical processes by grinding up unwanted
rock containing some ore and dumping it in waste heaps.

(a) Oxygen in the air and rain water combined to oxidise the sulfide ions in solid iron
pyrites to form an acidic solution of iron(II) sulfates.

Write a balanced ionic equation for this reaction.


[1]

2
Electrochemistry

(b) The resulting ions then run off into the local streams where the iron(II) ions are
oxidised.

(i) By considering the appropriate Eo values in the Data Booklet, explain why the
water in the streams must be acidic for oxygen to oxidise the iron(II) ions.

(ii) Hence, write an equation for the oxidation of iron(II) ions in the stream water
under acidic conditions.
[4]

(c) When potassium cyanide is added to stream water, iron(III) ions form a orange red
complex, [Fe(CN)6]3-.

In an experiment, the standard electrode potential of the [Fe(CN)6]3- | [Fe(CN)6]4-


system is measured.

(i) By means of a fully labelled diagram, show how the standard electrode
potential of the [Fe(CN)6]3- | [Fe(CN)6]4- system can be measured.

(ii) The presence of CN- significantly decrease the oxidising ability of an acidic
solution of Fe(III) ions. Illustrate this effect by selecting appropriate Eo values
23
from the Data Booklet.

(iii) From the list of standard electrode potentials in the booklet, identify a gas that
would reduce [Fe(CN)6]3- to [Fe(CN)6]4-.
LL

Explain your answer.


[6]

5. [PJC 2011/II/3(b), SRJC 2011/III/2(c)]


(a) When SO2 gas is bubbled into an orange acidified chromium-containing solution, a
green solution is formed. When drops of NaOH(aq) are added to this green solution, a
grey green precipitate is formed. This precipitate dissolves when an excess of
NaOH(aq) is added, forming a dark green solution, containing [Cr(OH)6]3-.

When excess aqueous acidified hydrogen peroxide is added to the dark green
solution, the original orange solution is restored.

(i) Use E values from the Data Booklet to explain why the orange acidified
chromium-containing solution turns green, when SO2 gas is bubbled into it.

Write a balanced equation for the reaction that occurs.

(ii) Give the formula of the grey-green precipitate.

(iii) Suggest the role of aqueous hydrogen peroxide.


[4]

3
Electrochemistry

(b) Vanabin in sea squirts accumulates vadadium in the blood cell by using nicotinamide
adenine dinucleotide phosphate (or NADPH) as a reducing agent to give the sea
squirts their distinct green colouration.

Use the standard redox potential data in Data Booklet and writing equations wherever
possible to predict the outcome of the reactions of vanadate (V) ions with excess
NADPH. Hence, determine the ion that is responsible for the green colouration in sea
squirts.
[4]

6 TJC 2011/III/4(a)-(b)
Copper and its compounds can be found everywhere in our daily life. Copper is abundant on
Earth. Known worldwide resources of copper are estimated at nearly 5.8 trillion pounds of
which only about 0.7 trillion have been mined throughout history.

(a) Researchers in Japan have recently developed a low-cost lithium-copper air fuel cell
consisting of a copper cathode immersed in an aqueous alkaline electrolyte and a
lithium anode immersed in an organic electrolyte. Mixing of the two electrolyte
solutions is prevented by using a solid electrolyte separator where only lithium ions
can pass through the separator.
23
LL

Aqueous
alkaline
electrolyte

The copper electrode is oxidised by oxygen in the air to generate copper(I) oxide.
During discharge, copper(I) oxide will be reduced to copper solid at the cathode.
Similarly, during discharge, lithium will be oxidised to give lithium ions at the anode

4
Electrochemistry

and pass through the separator into the aqueous alkaline electrolyte.

(i) Write half-equations, occurring at each electrode, when the battery discharges.

(ii) The voltage of this cell is found to be +2.30 V. With reference to the Data
Booklet, estimate the E  of the reduction of copper(I) oxide to copper half cell
and state any assumptions you make.

[5]

(b) Copper is also widely used in other voltaic cells. An example of another voltaic cell is
when a copper metal plate was dipped into an aqueous solution of 2.0 mol dm –3
copper(II) sulphate solution and this half-cell was connected via a salt bridge, to the
following half-cell in standard conditions,
2CO2 + 2H+ + 2e H2C2O4 E = 0.49 V
A student calculated the overall cell e.m.f to be +0.83V but the overall cell e.m.f was
measured to be +0.86 V.

(i) State the cell-diagram for the above voltaic cell.


23
(ii) Describe as far as possible what you would expect to see when the two half-cells
are connected together via a salt-bridge.

(iii) Account for the difference in the calculated and measured overall cell e.m.f in the
LL

above voltaic cell.


[5]

7 [ACJC 2011/III/4(a)-(b)] (integrated with Ksp)


A chemist design an ion-specific probe for measuring [Ag +] in a NaCl solution saturated with
AgCl using the following set- up.

V
voltmeter Pt wire
Ag wire

Paste of Hg2Cl2 in
Salt Bridge Hg

Saturated KCl
solution

NaCl solution
saturated with AgCl 5
Electrochemistry

(a) (i) Given the following standard half reactions.

Hg2Cl2(s) + 2e ⇌ 2Hg(s) + 2Cl-(aq) Eøred = +0.24 V


Ag+(aq) + e ⇌ Ag(s) Eøred = +0.80 V

Obtain an overall balanced equation, including state symbols for the cell above
and state the direction of the flow of electron in the cell. [2]

(ii) An engineer wish to use the probe to analyse an ore sample. After pretreating
the sample, the chemist measured the cell voltage, Ecell as 0.53V.

The Nernst equation can be used to measure the concentration of silver ions
using the probe,

0.0592
Ecell = Eøcell – log10 K
n

Where n is the no of moles of electrons transferred in the overall reaction and K


is the equilibrium constant for the overall equation in a(i).
23
Assuming the concentration of Cl- is so high that it is essentially constant,
use the Nernst equation to calculate the concentration of silver ions in the ore
sample.
[3]
LL

(iii) Given that,


AgCl(s) + e Ag(s) + Cl-(aq) Eøred = +0.22 V

Using the half equation above and any other relevant data from the Data
Booklet, derive the Eøcell for the overall reaction: Ag+(aq) + Cl-(aq) ⇌ AgCl(s)
and use the equation given in a(ii), calculate a value of the Ksp for silver chloride
at equilibrium, if Ecell of any process at equilibrium is 0V. [3]

6
Electrochemistry

12 [NJC 2009/CT/2]
When a standard hydrogen half cell was connected to a X2+/X half cell, electrons flowed
from the X2+/X half cell to the standard hydrogen half cell. On connecting the same half cell
to the Fe3+/Fe2+ half cell, the e.m.f. was found to be 0.91 V.

(a) (i) Calculate the standard reduction potential of X2+/X half cell and hence identify
X.

[2]
(ii) State the polarity of the electrodes and label the direction of electron flow on the
diagram provided above.
23
Polarity at X2+/X half cell: ………………….

Polarity at Fe3+/Fe2+ half cell: ………………….


LL

[2]

(b) The Fe3+/Fe2+ half cell was made up of 20 cm3 of a solution containing 1 mol dm–3
Fe2+ and Fe3+.

(i) 5.00 cm3 of acidified KMnO4 at a concentration of 0.4 mol dm–3 was added to
the Fe2+/Fe3+ half cell. Write an equation to represent the reaction.

[1]
(ii) Calculate the new concentration of Fe2+ and Fe3+ after the reaction.

[3]
(iii) Hence, predict how the new Ecell value would change from the original E0cell
value.
[2]
(iv) Explain why the new Ecell value would increase drastically when excess
acidified KMnO4 was added to the Fe3+/Fe2+ half cell.

[2]

7
Electrochemistry

9 [PJC 2011/III/5(a)-(b)]
(a) The standard redox potential of manganate(VII) ions is given below:

MnO4- + 8H+ + 5e ⇌ Mn2+ + 4H2O EӨ = +1.52V

(i) What do you understand by the term standard electrode potential?

(ii) Draw a fully labeled sketch of how you would determine the standard electrode
potential of the MnO4-(aq)  Mn2+(aq) electrode system.
[3]

(b) Potassium manganate(VII) is also frequently used in titrimetric analysis due to its
strong oxidising property.
A student was tasked to determine the amount of iron(II) salts present in a nutritional
supplement pill. He decided to carry out a titration with a standard solution of acidified
KMnO4 that he prepared.

(i) Write a balanced equation for the reaction of MnO4- and Fe2+ in acid solution.

(ii) The acid that was used to prepare the standard solution was sulfuric acid.
Using appropriate data from the Data Booklet, explain why hydrochloric acid
could not be used.
23
(iii) Suggest why potassium manganate(VII) rather than potassium dichromate(VI)
was used in the titration.
LL

One such nutritional supplement pill containing Fe2+ was dissolved in a volume of
dilute sulfuric acid and titrated against 0.0200 mol dm-3 KMnO4. The Ecell was
measured against a standard hydrogen electrode and the following graph was
obtained.
Ecell / V
U

0 volume of KMnO4 added / cm3


(iv) Using suitable data listed in the Data Booklet, suggest approximate E values, in
volts, for the points S and U on the graph.

(v) State an important assumption that was made to estimate the answers in part
(b)(iv).

(vi) State the colour of the solution at point T.


8
Electrochemistry

[8]

10 [RI 2011/III/3(a)]
(a) The Nernst equation was developed by the German chemist, Walter Hermann Nernst,
to calculate the cell potential under non-standard conditions.

Two identical concentration half-cells, each consisting of a copper electrode immersed


in 90 cm3 of 0.0100 mol dm3 of copper(II) nitrate, are prepared.

10 cm3
0.500 mol C C
dm3 NH3(aq) u salt bridge u

Cu2+ Cu2+

90 cm3, 0.0100 mol dm3 90 cm3, 0.0100 mol dm3


Cu(NO3)2(aq) Cu(NO3)2(aq)

Half-cell 1 Half-cell 2
23
10 cm3 of 0.500 mol dm3 NH3(aq) is added to half-cell 1 and the colour of the solution
changes from blue to deep blue. The Ecell measured is +0.129 V.
LL

In your calculations, you may apply the following Nernst equation which has been
derived for the above electrochemical cell.

0.0592 [Cu2+ ]half-cell 1


Ecell = Ecell  lg
2 [Cu2+ ]half-cell 2
where
Ecell is the cell potential under non-standard conditions,

Ecell is the standard cell potential, taken to be 0.00 V before the addition of
NH3(aq),

[Cu2+]half-cell 1 is the concentration of copper ions left in the solution of half-


cell 1 after the addition of NH3(aq),

[Cu2+]half-cell 2 is the original concentration of copper ions in half-cell 2.

(i) Write an equation to account for the formation of the deep blue solution.

(ii) Deduce and explain which half-cell contains the negative electrode.

(iii) Give the expression, including its units, for the equilibrium constant, Kf, for the

9
Electrochemistry

formation of the complex ion formed in (a)(i).

(iv) Using the Nernst equation given, calculate the [Cu2+] in half-cell 1 after the
addition of NH3(aq).

(v) Hence, calculate Kf for the formation of the complex ion in half-cell 1.
[8]

Multiple Choice Questions (Section A)


1 [CJC 2011/I/15]
Two separate students accidentally spilled chemicals on their clothes in the laboratory which
resulted in brown MnO2 stains and I2 stains. Their teacher suggested the use of bleaching
agent (H2O2) for removal of these stains.

The relevant standard redox potentials are listed below:

I 2 + 2 e- ⇌ 2 I - Eo= +0.54 V
MnO2(s) + 4 H+(aq) + 2 e- ⇌ Mn2+(aq) + 2 H2O(l) Eo= +1.23 V
H2O2 + 2 H+ + 2 e- ⇌ 2 H2O Eo= +1.77 V
23
O2 + 2 H+ + 2 e- ⇌ H2O2 Eo= +0.68 V

Which of the following is true regarding the removal of the stains?


LL

A Only I2 will be successfully removed.


B Only MnO2 will be successfully removed.
C Both MnO2 and I2 will be successfully removed.
D Both stains cannot be removed.
2 [HCI 2011/I/12]
Use of the Data Booklet is relevant to this question.

The AgCl / Ag reference electrode is based on the following reaction:

AgCl + e ⇌ Ag + Cl

The reduction potential of the Fe2+ / Fe couple was found to be 0.64 V when measured
using the AgCl / Ag reference electrode.

What is the reduction potential of the AgCl / Ag electrode with respect to the standard
hydrogen electrode?

A 0.20 V B +0.20 V C 1.10 V D +1.10 V

10
Electrochemistry

3 [MJC 2011/I/14]
A student set up 4 standard half-cells each containing one of the metals, S, T, V and W
immersed in a solution of its metallic salts. These were then used to make different
electrochemical cells. The table below shows the standard electrode cell potential, Eөcell and
the negative terminal of each electrochemical cell.

Cell Metals used Eөcell / V Negative terminal


1 S and T +1.10 T
2 T and V +0.46 V
3 T and W +0.47 T

Which one of the following statement is correct?


A V is produced in Cell 2.
B S has a s tronger reducing power than W.
C V can reduce all the cations of S, T and W.
D Eөcell for metals S and W is less than +0.47V.

4 [NJC 2011/I/15]
In an aluminium-air battery, one of the electrodes is a carbon rod exposed to air while the
23
other electrode is an aluminium rod. During discharge, aluminium is converted to insoluble
aluminium hydroxide at the aluminium electrode and oxygen in the air is converted to
hydroxide ions at the carbon electrode.
LL

What is the mass of aluminium hydroxide precipitated if 0.030 moles of oxygen is used by
the battery?

A 0.51 g
B 0.68 g
C 2.04 g
D 3.12 g

5 [PJC 2011/I/7]
A voltaic cell is made up of the Mg2+/Mg half-cell and the Fe3+/Fe2+ half-cell.

Which one of the following statements is correct?

A The Mg2+/ Mg half-cell is the positive electrode.


B Increasing the temperature has no effect on the e.m.f. of the cell.
C Addition of water to the Fe3+/Fe2+ half-cell decreases the e.m.f. of the cell.
D Addition of aqueous sodium hydroxide to the Mg2+/Mg half-cell increases the e.m.f.
of the cell.

11
Electrochemistry

6 [NYJC 2011/I/11]
The diagram below is a hydrogen-oxygen fuel cell.

Predict the changes in the pH of the solution around each of the electrode when a current
is flowing.

Porous electrode

 
O2 H2

1.00 mol dm-3 NaOH


23
Cathode Anode
A increase decrease
B decrease decrease
LL

C decrease increase
D increase increase

7 [PJC 2011/I/17]
The use of Data Booklet is relevant to this question.

Peroxodisulfate, S2O82-, is capable of oxidising the tartrate ion, C4H4O62-, to carbon


dioxide and methanoate as shown in the following reaction.

C4H4O62- + 3S2O82- + 3H2O → 2CO2 + 2HCO2- + 6H+ + 6SO42-

The reaction can be catalysed by a homogeneous catalyst. Given that the electrode
potential for the tartrate ion is

2CO2 + 2HCO2- + 6H+ + 6e → C4H4O62- + 2H2O Eθ = +0.56 V

Which metal ion is not a suitable catalyst for this reaction?


A V3+ B Mn3+ C Fe3+ D Co3+

12
Electrochemistry

8 [RVHS 2011/I/12]
The use of Data Booklet is relevant to this question.

In many areas, tap water becomes slightly acidic due to dissolved carbon dioxide.
By considering the relevant E values, which of the following metals will not be dissolved
by tap water containing carbon dioxide?

A chromium B copper
C iron D lead

9 [VJC 2011/I/11]
Use of the Data Booklet is relevant to this question.

The half-cells of four metals Pb, X, Y and Z were connected in pairs and the voltage was
recorded.
V
salt bridge
Metal Metal
electrode electrode
23
LL

The results obtained are set out in the table below.

Negative electrode Positive electrode Voltage /V


Pb X 0.35
Z Pb 2.60
Y Pb 1.10

Which one of the following statements is correct?

A Anions will flow from the salt bridge to the Xn+(aq) | X(s) half-cell when it is
connected to Zp+(aq) | Z(s) half-cell.
B Pb has a greater tendency than Y to form positively charged ions.
C The colour of aqueous CuSO4 solution will become lighter when Y is added to it.
D Electrons will flow from electrode Y to electrode Z when the two half-cells of
Zp+(aq) | Z(s) and Ym+(aq) | Y(s) are connected together.

13
Electrochemistry

10 [SRJC 2011/I/11]
An experiment is set up as shown below:

The Ecell of the cell was monitored as time progressed. When a change was
made continuously to the set-up, the graph below was obtained.

Ecell / V

23
Time / s
What was the continuous change made?
LL

A Add nickel (II) chloride to the nickel half cell


B Add NaCN to the iron half cell
C Add water to the nickel half-cell
D Increase the surface area of iron immersed in the solution.

11 [SAJC 2011/I/13]

14
Electrochemistry

12 [DHS 2011/I/26]

23
LL

13 [TJC 2011/I/11]
Given these standard electrode potentials

Cu2+(aq) + 2e– Cu(s) +0.34 V


MnO4-(aq) + 8H+(aq) + 5e– ⇌ Mn2+(aq) + 4H2O(l) +1.52V

what would be the standard e.m.f. of the cell involving these two half-reactions and the
polarity of the copper electrode?

EӨcell / V Polarity of the copper electrode


A 1.86 +
B 1.86 –
C 1.18 +
D 1.18 –

15
Electrochemistry

14 [VJC 2010/I/6]
The table below contains the reduction potential values for various copper and mercury
species.
half-equations Eo/ V

Cu2+ (aq) + e ⇌ Cu+ (aq) +0.15

Cu+(aq) + e ⇌ Cu (s) +0.52

Hg2+(aq) + e ⇌ Hg+(aq) +0.91

Hg+(aq) + e ⇌ Hg(l) +0.80

Using these data, which one of the following is a correct prediction?

A Both Cu+ and Hg+ undergo disproportionation.


B Only Cu+ undergoes disproportionation.
C Only Hg+ undergoes disproportionation.
D Neither Cu+ nor Hg+ undergoes disproportionation.

15 [HCI 2010/I/11]
A fuel cell is illustrated in the diagram below. There is a common electrolyte of aqueous
23
sodium hydroxide. One electrode in the fuel cell becomes the anode and the other the
cathode.
LL

Electrode A Electrode B

Which of the following statements about the above fuel cell is true?

A Electrode A is the positive electrode.


B Electrons flow from electrode A to electrode B.
C The cell e.m.f. under standard conditions is +0.40 V.
D The electrode potential of B becomes more positive as the concentration of NaOH
increases.

16
Electrochemistry

16 [MJC 2010/I/13]
Which of the following can be deduced from the reduction potentials listed below?

Ag+ + e  Ag Eө = +0.80 V
Co3+ + e  Co2+ Eө = +1.82 V
Co2+ + 2e  Co Eө = 0.28 V
Ni2+ + 2e  Ni Eө = 0.25 V
Pb4+ + 2e  Pb2+ Eө = +1.69 V
Pb2+ + 2e  Pb Eө = 0.13 V

A Ag is a better reducing agent than Pb2+.


B Co3+ is the strongest reducing agent.
C Ni2+ is a better oxidising agent than Pb2+.
D Ni2+ can be reduced by Co2+.

17 [MJC 2010/I/14]
Silver oxide cells are used in small-scale applications such as cameras and calculators.
They contain powdered zinc metal, an alkaline electrolyte and silver oxide encased by a
nickel or steel container, as shown.

Powdered
Ag2O zinc
23
Nickel
casing
The cell reaction is:
LL

Ag2O (s) + Zn (s)  2Ag (s) + ZnO (s)

What is the equation for the reaction at the negative electrode of the cell?

A Zn (s) + 2H+ (aq)  Zn2+ (aq) + H2 (g)


B Zn (s) + 2OH (aq)  ZnO (s) + H2O (l) + 2e
C ZnO (s) + H2O (l) + 2e  Zn (s) + 2OH (aq)
D Ag2O (s) + H2O (l) + 2e  2Ag (s) + 2OH (aq)

18 [NJC 2010/I/12]

17
Electrochemistry

Use of the Data booklet may be relevant to this question.


Three electrochemical cells are set up as shown below. The e.m.f in volts is shown on
each voltmeter.

The e.m.f indicates the order of reactivity of the metals. The order of the weakest to the
strongest reducing agent is
A X, Cu, Y, Z
B Cu, Z, X, Y
C Z, Cu, X , Y
D Y, X, Z, Cu

19 [JJC 2009/I/19]
The use of the Data Booklet is relevant to this question.
The colours of various vanadium ions in aqueous solution are given in the table below.
23
Oxidation state V IV III II
Ion VO2+ VO2+ V3+ V2+
LL

Colour yellow blue green violet

What is the final colour of the solution when excess zinc powder is added to an acidified
solution containing VO2+ ions?

A Yellow
B Blue
C green
D Violet

18
Electrochemistry

20 [VJC 2009/I/12]
Use of the Data Booklet is relevant to this question.

The half-cells for four metals: Mg, A, B and C were in turn connected in pairs and the
potential difference was recorded.

Temperature = 25oC

V
electron flow
23
salt bridge
LL

metal metal

1 mol dm-3 of the metal ion 1 mol dm-3 of the metal ion
The results obtained are as shown in the table below.

Positive electrode Negative electrode e.m.f /V


A Mg +2.10
B Mg +2.72
Mg C +0.33

Rank the four metals in the order of decreasing reducing power.

A B > A > Mg > C


B A > B > Mg > C
C C > Mg > A > B

19
Electrochemistry

D C > Mg > B > A

Multiple Choice Questions (Section B)

The responses A to D should be selected on the basis of

A B C D
1, 2 and 3 are
1 and 2 are correct 2 and 3 are correct Only 1 is correct
correct

No other combination of statements is used as a correct response.


23
1 [CJC 2011/1/34]
A hydrogen fuel cell as illustrated below has a typical e.m.f. of 1.23 V.
LL

Platinum electrodes (inert)


V

Cathode Anode

O2 gas → ←H gas2

Proton exchange membrane

Which of the following is true regarding the operation of this fuel cell?

1 The e.m.f can be increased by increasing the pressure of oxygen gas to 2 atm.
2 The electrode is platinised to increase the rate of reaction, but e.m.f. is not affected.
3 The proton exchange membrane allows the passage of H+ ions in order to complete
the circuit and maintains electrical neutrality.

20
Electrochemistry

2 [JJC 2011/I/34]
Pain is often felt when a piece of aluminium foil touches a dental amalgam filling in a tooth
because an electric current momentarily flows. The amalgam contains tin. The standard
electrode potentials are as follows:
Al3+(aq)/Al (s) Eo = −1.66 V
Sn2+(aq)/Amalgam Eo = −0.13 V
Which of the following are the features of the cell obtained?

1 The aluminium foil acts as the negative electrode.


2 The e.m.f of the cell is +1.79 V.
3 Sn2+ ions are momentarily released into the saliva in the mouth.

3 [MJC 2011/I/34]
Use of Data Booklet is relevant to this question.

The diagram represents an experiment to confirm the value of Eo (Cr3+ / Cr2+).

Hydrogen gas
23
Solution
containing H+ Solution containing
ions Cr3+cand Cr2+ ions
LL

The emf of the cell was found to be 0.45 V rather than the expected 0.41 V.

Which statement(s) is / are correct to explain the above observation?

1 [H+(aq)] is greater than 1.00 moldm-3.


2 [Cr3+(aq)] is smaller than 1.00 moldm-3.
3 The electrode on the left in the set-up is a positive electrode.

4 [NYJC 2011/I/34]
Given the following electrode potentials,

V2+ + 2e ⇌ V Eθ = − 1.20 V
V3+ + e ⇌ V2+ Eθ = − 0.26 V
VO2+ + 2H+ + e ⇌ V3+ + H2O Eθ = + 0.34 V
2H+ + 2e ⇌ H2 Eθ = 0.00 V

It can be inferred that the vanadium dissolves in 1 mol dm‾3 hydrochloric acid to form

1 H2 2 V3+ 3 VO2+

21
Electrochemistry

5 [NJC 2011/I/33]
Use of the Data Booklet is relevant to this question.

Cl2 (g) at 1 atm

[Cl – (aq)]
= 1.0 mol dm–3
23
.
Which of the following will reduce the cell potential?
LL

1 Increasing the pressure of Cl2 in the Cl2 (g)/ Cl – (aq) half cell.
2 Adding water into the S4O62–-(aq)/ S2O32– (aq) half cell.
3 Adding iodine crystals into the anodic half cell.

6 [PJC 2011/ I/33]


A domestic carbon monoxide detector is based on an electrochemical cell
consisting of the electrodes:

CO2(g) + 2H+(aq) + 2e ⇌ CO(g) + H2O(l) Eθ = -0.10 V


½O2(g) + 2H+(aq) + 2e ⇌ H2O(l) Eθ = +1.23 V

Which statements about the cell are true?

1 In the cell, oxygen is reduced.


2 Under standard conditions, the cell potential is +1.33 V.
3 As the concentration of carbon monoxide in the detector increases, the cell
potential becomes more positive.

7 [RI 2011/ I/36]


Use of the Data Booklet is relevant to this question.

22
Electrochemistry

Which of the following are chemically stable when left to stand in the atmosphere?

1 a solution of potassium hexacyanoferrate(III)


2 a solution of chromium(II) chloride
3 a mixture of aqueous sodium hydroxide and iron(II) sulfate

8 [TJC 2011/ I/34]


With reference to the Data Booklet and the data below, deduce which of the following
statements are correct.

Half-equation E/V

VO2+(aq) + 2H+(aq) + e-  VO2+(aq) + H2O(l) +1.00

VO2+(aq) + 2H+(aq) + e-  V3+(aq) + H2O(l) +0.34


23
V3+(aq) + e-  V2+(aq) + H2O(l) -0.26
Xx
LL

1 An excess of zinc is capable of reducing VO2+(aq) to V2+(aq) in acidic conditions.

2 Fe3+(aq) is capable of oxidising V2+(aq) to VO2+ (aq) in acidic conditions.

3 Zn2+(aq) ions will oxidise V2+(aq) ions to V3+(aq) ions.

9 [Modified from SAJC 2011/ I/35]

23
Electrochemistry

Using the above data, which of the following statements are incorrect / cannot be
concluded?

Chlorate(VII) is a stronger oxidizing agent than chlorate(V)


23
LL

10 [SRJC 2011/ I/36]

24
Electrochemistry

23
LL

25
Electrochemistry

ELECTROCHEMISTRY SUGGESTED SOLUTIONS

Multiple Choice Questions (Section A)


1. 2. 3. 4. 5. 6. 7. 8. 9. 10. 11. 12. 13. 14. 15. 16. 17. 18. 19. 20.
B B C D D A A B C C C A D B B A B C D C

Multiple Choice Questions (Section B)


1. 2. 3. 4. 5. 6. 7. 8. 9. 10.
A D B B C A D D A C

Structured/Free Response Questions

1 (i) Anode: CH3CH2OH (l) + 3H2O (l)  2CO2 (g) + 12H+ (aq) + 12e-

Cathode: O2 (g) + 4H+ (aq) + 4e-  2H2O (l)

(ii) At the cathode,


O2 (g) + 4H+ (aq) + 4e 2H2O (l) +1.23V

O2 (g) + 2H2O (l) + 4e 4OH-(aq) +0.40V

In acidic medium, O2 is more easily reduced as it has a more positive standard


23
reduction potential or EO2/H2O. Thus the E cell generated is larger.

2a (i) In acidic solution


LL

O2 + 4H+ + 4e 2H2O Eo = +1.23 V


Fe3+ + e Fe2+ Eo = +0.77 V

4Fe2+ + O2 + 4H+  4Fe3+ + 2H2O

Eocell = +1.23 – (+0.77) = + 0.46 V


In alkaline solution
Fe2+ (aq) + OH– (aq)  Fe(OH)2 (s)
OR Ferrous ion react with hydroxide ions to form the ppt, Fe(OH)2.

O2 + 2H2O + 4e 4OH– Eo = +0.40 V


Fe(OH)3 + e Fe(OH)2 + OH– Eo = –0.56 V

Eocell = +0.40 – (–0.56) = + 0.96 V

4Fe(OH)2 + O2 + 2H2O  4Fe(OH)3


OR The ppt will be oxidised in air to reddish–brown Fe(OH)3.

(ii) Fe3+ acts as a catalyst.

Fe3+ first oxidizes I– to I2 and becomes reduced to the intermediate Fe2+.

Fe3+ + e Fe2+ Eo = +0.77 V

26
Electrochemistry

I2 + 2e 2I– Eo = +0.54 V

2I– (aq) + 2 Fe3+ (aq)  I2 (aq) + 2Fe2+ (aq)


Ecell = +0.77 – (+ 0.54) = + 0.23 V >0. Reaction is feasible.

Fe2+ intermediate reacts with S2O82–. Fe3+ catalyst is regenerated.

Fe3+ + e Fe2+ Eo = +0.77 V


S2O82– + 2e 2SO4– Eo = +2.01 V

S2O82– (aq) + 2Fe2+(aq)  2SO42– (aq) + 2Fe3+(aq)

Ecell = +2.01 – (+ 0.77) = + 1.24 V >0. Reaction is feasible.

b E(Fe3+/Fe2+)= +0.77 V ;
E(Fe(CN)63/Fe(CN)64)= +0.36 V
E(Fe(OH)3/Fe(OH)2)= 0.56 V

E(Fe3+/Fe2+) is more positive than E(Fe(CN)63/ Fe(CN)64) which is more positive than
E(Fe(OH)3/Fe(OH)2)

Oxidising Ability: Fe(OH)3 , K3Fe(CN)6 , Fe(NO3)3


23
c [R] FeO42-(aq) + 8H+(aq) + 3e ⇌ Fe3+(aq) + 4H2O(l) +2.00 V
[O] Cr2O72- + 14H+ + 6e ⇌ 2Cr3+ + 7H2O +1.33 V
Ecell = +2.00 – (+1.33) = +0.67 V > 0
LL

Overall equation:
2FeO42- + 2Cr3+ + 2H+  2Fe3+ + Cr2O72- + H2O

Observation:
The resultant solution should be yellowish-orange

3a (i) Co3+(aq) / Co2+(aq) E = + 1.82V


Pb4+(aq) / Pb2+(aq) E = + 1.69V
MnO2(s) / Mn2+(aq) E = + 1.23V
Fe3+(aq) / Fe2+(aq) E = + 0.77V

Only can choose Fe2+(aq)

(ii) Eq(1) MnO2 + 4H+ + 2e Mn2+ + 2H2O +1.23


3+ 2+
Eq(2) Fe + e Fe +0.77
Overall:
MnO2 (s) + 4H+ (aq) + 2Fe2+ (aq)  Mn2+ (aq) + 2H2O (aq) + 2Fe3+ (aq)

Eθcell = (+1.23) – (+0.77) = +0.46V

Since Eθcell > 0, reaction is feasible.

(iii) Orange (Pale pink + Yellow)

27
Electrochemistry

Also accept yellow

4a 2FeS2 (s) + 7O2 (g) + 2H2O(l) → 2Fe2+ (aq) + 4SO42-(aq) + 4H+ (aq)

b (i) From the Data Booklet


Eo / v
O2 + 2H2O + 4e 4OH– +0.40
Fe3+ + e Fe2+ +0.77
O2 + 4H+ + 4e 2H2O +1.23

If solution is acidic,
Eo cell = Eo red - Eo oxd
= 1.23 – 0.77
= +0.46V
Since Eo cell > 0, reaction is feasible.

If solution is not acidic,


Eo cell = Eo red - Eo oxd
= 0.40 – 0.77
= -0.37 V
Since Eo cell < 0, reaction is not feasible.

(ii) 4Fe2+ (aq) + O2 (g) + 4H+ (aq)  4 Fe3+(aq) + 2H2O (l)


23
c (i)
high-resistance voltmeter
LL

V
salt bridge
H2  g
Pt  s 
at 1 atm,
25 ºC
3
H  aq Fe  CN6   aq , 1 mol dm–3
4
1 mol dm–3 Fe  CN6   aq , 1 mol dm–3
(ii) In water, the more positive Eo (Fe3+/Fe2+) = +0.77 V indicates that Fe3+ is unstable with
respect to Fe2+ and should be spontaneously reduced to Fe2+ (e.g. by water itself).

On the other hand, in the presence of CN- ligands, the reduction potential decreases
to Eo ([Fe(CN)6]3-|[Fe(CN)6]4- = +0.36V, means that CN- ligand stabilises the Fe3+ with
respect to Fe2+.

Smaller E means that [Fe(CN)6]3- is less readily reduced compared to Fe3+,


rendering it less oxidising.

28
Electrochemistry

(iii) SO42- + 4H+ 2e SO2 + 2H2O Eo = +0.17V


[Fe(CN)6]3- + e [Fe(CN)6]4- Eo = +0.36V

Eo cell = Eo red - Eo oxd


= 0.36 – 0.17
= +0.19V

Since Eo cell > 0, reaction is feasible.

The gas is SO2.

5a (i) Cr2O72 + 14H+ +6e 3Cr3+ + 7H2O Ered = +1.33 V


SO2 + 2H2O SO4 + 4H+ +2e
2
Eox= -0.17 V

E cell = 1.33 + (0.17) = +1.16V

3SO2 + Cr2O72 + 2H+ → 3SO42 + 2Cr3+ + H2O

(ii) Cr(OH)3
23
(iii) Oxidising agent

b 2 VO3– + 7 H+ + NADPH  2 VO2+ + 4 H2O + NADP+


LL

Eo = 1.00 – (–0.32) = +1.32 V


2 VO2+ + 3 H+ + NADPH  2 V3+ + 2 H2O + NADP+
Eo = 0.34 – (–0.32) = +0.66 V
2 V3+ + NADPH  2 V2+ + H+ + NADP+
Eo = –0.26 – (–0.32) = +0.06 V
Since E cell is ≈ 0 V, reaction is at equilibrium.
Hence final reduced vanadium species is V2+ or V3+.

29
Electrochemistry

6a (i) ● Anode : Li  Li+ + e-


● Cathode : Cu2O + H2O + 2e-  2Cu + 2OH-

(ii) ELi /Li = -3.04V


Ecell = Ered – Eoxid
● +2.30 = ECu2O/Cu – (-3.04)
ECu2O/Cu = 2.30 – 3.04 = -0.74V
Assumptions made:
● 298K (25oC) & concentration of aqueous alkaline electrolyte = 1.0 moldm-3

(iii) ● Li, being a Group I metal is reactive and would react with water in the aqueous
electrolyte. Hence the Li anode has to be immersed in an organic electrolyte.

b (i) Pt(s) CO2(g) H2C2O4(aq) Cu2+(aq) Cu(s)

(ii) 1) Effervescence/ Bubbles of CO2 gas will be seen around the anode (Pt electrode)
2) Reddish-brown Cu electrode will increase in size (grow thicker)
3) Blue colour in the aqueous solution will fade as [Cu2+] decreases.
23
3 points – 2 marks; 2 points – 1mark; 0 -1 points – 0 mark.

(iii) Cu2+ + 2e- Cu


LL

E oCu2+/Cu = +0.34 V (when [Cu2+] = 1 mol dm3)

● As the concentration of Cu2+ is at 2.0 mol dm3, by Le Chatelier’s Principle, the


system will try to decrease the concentration of Cu2+ by shifting the position of the
above equilibrium to the right. The tendency of Cu2+ to be reduced is increased
and the actual ECu2+/Cu is more positive than +0.34 V.

● Since measured Ecell = ECu2+/Cu – ECO2/H2C2O4, the value of Ecell would be more
positive than +0.83 V.

7a (i) 2Ag+(aq) + 2Cl-(aq) + 2 Hg(s) → Hg2Cl2(s) + 2Ag(s)


From Pt to Ag.

(ii) K = 1/ [Ag+]2 since [Cl-] is high and effectively constant


n=2
0.0592
0.53 = 0.56 – log10 1/ [Ag+]2
n
[Ag+] = 0.311 mol dm-3 [1]

(iii) Ag+(aq) + Cl-(aq) AgCl(s) Eøcell = +0.80 – 0.22 = 0.58V

30
Electrochemistry

Given that for a process at equilibrium, the Ecell = 0V

K = 1 / Ksp(AgCl) and n = 1
0.0592
0 = 0.58 - log10 1/Ksp
n
1/Ksp = 6.27 x 109
Ksp = 1.59 x10-10 mol2dm-6

8 a (i) Electrons flow from X2+/X half cell to Fe3+/Fe2+ half cell, hence, [R] half cell and
cathode is Fe3+/Fe2+ and anode is X2+/X half cell where [O] occur.
Eocell = Eocathode  Eoanode
0.91 = 0.77  Eoanode
Eoanode =  0.14 V

Using data booklet, X2+/X is Sn2+/Sn half cell.


Hence, X is Sn

(ii) Polarity at X2+/X half cell:Negative

Polarity at Fe3+/Fe2+ half cell: Positive


23
b (i) No. of moles of Fe2+ = 20 x 103 x 1.0 = 0.0200 mol
No. of moles of Fe3+ = 0.0200 mol

No. of moles of MnO4 = 5 x 103 x 0.4 = 0.0020 mol


LL

MnO4 + 8 H+ + 5 Fe2+  Mn2+ + 4 H2O + 5 Fe3+

No. of moles of Fe2+ reacted = 5 x 0.0020 = 0.0100 mol


No. of moles of Fe2+ left = 0.0200 – 0.0100 = 0.0100 mol
[Fe2+] = [0.0100 ÷ (20+5)] x 1000 = 0.400 mol dm3

No. of moles of Fe3+ formed = 0.0100 mol


No of moles of Fe3+ present = 0.0020 + 0.0100 = 0.0300 mol
[Fe2+] = [0.0300 ÷ (20+5)] x 1000 = 1.20 mol dm3

(ii) Fe3+ + e ⇌ Fe2+ Eocathode = + 0.77 V

Since [Fe2+] decrease and [Fe3+] increase, eqm shift right, thus, Ecathode
become more positive than +0.77V, thus, Ecell increases.

(iii) Excess MnO4 added, the species present are:


unreacted MnO4, Mn2+ (formed when MnO4 oxidized Fe2+), Fe3+ and acid.

All the Fe2+ has been used up. A new half-cell MnO4/Mn2+ is generated due to
excess MnO4. Ered of MnO4/Mn2+ is much larger (+1.52V), thus, Ecell increases

31
Electrochemistry

drastically.

9a (i) The potential difference between a standard hydrogen electrode and a half cell under
standard conditions of 25C, 1 atm and concentration of any ion is 1 mol dm-3.

(ii) voltmeter
electron flow
V
H2, 1 atm
25oC
Salt
Bridge

Pt Pt

° °
°
H+ 1 mol dm-3
[H+] = [Mn2+] = [MnO4-] =1
23
mol dm -3

(b) (i) MnO4- + 8H+ + 5Fe2+ → Mn2+ + 4H2O + 5Fe3+


LL

(ii) MnO4- + 8H+ + 5e- Mn2+ + 4H2O Eθred = +1.52 V


Cl- Cl2 + 2e- Eθox = -1.36 V

Eθcell = +0.16 V

Cl- ions present would be oxidised to Cl2 by MnO4-. This would cause the titre value to
be higher than expected since MnO4ˉ oxidizes both Clˉ and Fe2+.

(iii) Fe2+ Fe3+ + e-


pale green pale yellow

MnO4- + 8H+ + 5e- Mn2+ + 4H2O


purple pale pink / colourless

Cr2O72- + 14H+ + 6e- 3Cr3+ + 7H2O


orange green

The end point colour change will be more distinct / sharp colour change and hence
can be detected more easily when KMnO4(aq) is used.

(iv) At point S, the mixture contains the same concentration of Fe2+ and Fe3+ ions, along
with the Mn2+ ion.

32
Electrochemistry

An equilibrium exists between Fe2+ and Fe3+ ion and since [Fe2+] = [Fe3+] at S

Fe3+ + e- ⇌ Fe2+ E at S = Eθ = + 0.77 V

At point U, the mixture contains an equal concentration of Mn2+ and MnO4-, along with
Fe3+ ions.

An equilibrium exists between Mn2+ and MnO4- ions and since [Mn2+] = [MnO4ˉ].

MnO4- + 8H+ + 5e- ⇌ Mn2+ + 4H2O E at U = Eθ = +1.52 V

(v) 1) At point S, [Fe3+] = [Fe2+] and at point U, [MnO4-] = [Mn2+]. OR

2) The temperature of the mixture remains constant throughout the entire


titration process.

(vi) Fe2+ Fe3+ + e-


pale green pale yellow

MnO4- + 8H+ + 5e- Mn2+ + 4H2O


purple pale pink / colourless
23
Colour of solution at point T is orange.
LL

10 (i) Cu2+(aq) + 4NH3(aq) ⇌ [Cu(NH3)4]2+(aq)

(ii) The copper electrode in half-cell 1 is the negative electrode.

When aqueous ammonia is added to the half-cell 1, complex [Cu(NH3)4]2+ is formed


and the concentration of Cu2+ ions decreases , causing the equilibrium position of
Cu2+ + 2e ⇌ Cu to shift to the left, favouring oxidation (loss of electrons).

Or

If student says “dilution occurs on addition of ammonia, hence [Cu2+] drops”…

(iii) [Cu NH3 4 ]


2+

Kf = mol4 dm12
[Cu2+ ][NH3 ]4

(iv) 0.0592 [Cu2+ ]half-cell 1


Ecell = Ecell  lg
2 [Cu2+ ]half-cell 2
0.0592 [Cu2+ ]half-cell 1
0.129 = 0.00  lg
2 0.0100

33
Electrochemistry

[Cu2+]half-cell 1 = 4.38 x 10-7 mol dm3

(v) Initial amount of Cu2+ = 90/1000  0.0100 = 9.00  104 mol


Initial amount of NH3 = 10/1000  0.500 = 5.00  103 mol

New volume of solution in half-cell 1 = 10 + 90 = 100 cm3


Eqm. amount of Cu2+ = 100/1000  4.384  107 = 4.384  108 mol
Amount of NH3 reacted = 4(9.00  104 – 4.384  108) = 3.60  103 mol

Cu2+(aq) + 4NH3(aq) ⇌ [Cu(NH3)4]2+(aq)


Initial amt 9.00  104 5.00  103 –
Eqm amt 4.384  108 5.00  103 – 9.00  104 -
3.60  103 4.384  108
= 1.40 x 10-3 = 8.9996  104
 9.00 x 104

 0.100 4.384  107 1.40  102 9.00  103


Eqm [ ]

Kf = (9.00  103) / (4.384  107) (1.40  102)4


= 5.34  1011 mol4 dm12
23
Note: Do not assume that all the given answers are correct. Do discuss with your
friends and check with your tutor should you think that any given answer is incorrect.
LL

34
Electrolysis

ELECTROLYSIS 9b
Structured / Free Response Questions

1 [NJC 2011/III/5(a) modified]


(a) Use of the Data Booklet is relevant to this question.

(i) Predict the products formed at the cathode, and at the anode, when the following are
electrolysed.

(I) K2O(l) using graphite electrodes.


(II) CuSO4(aq) using copper electrodes.

In each case explain your reasoning.

(ii) The electrolysis of K2O(l) is carried out by passing a current of 5A for 30 minutes.
Calculate the volume of gas collected at r.t.p.
[7]

b 100 cm3 of 1.0 mol dm3 aqueous sodium bromide was electrolysed for two minutes
using a current of 1.5 A using graphite electrodes.
23
(i) Write the equations occurring at the anode and cathode.
(ii) Calculate the pH of the solution after electrolysis.
LL

2 [IJC 2011/III/4(a)]
Since its invention in 1950 by John Hopps, the heart pacemaker has saved the lives of many
cardiac patients by regulating their heartbeat.

In this device, electrical energy is generated by implanting zinc and platinum electrode in
body tissues. These electrodes in the oxygen-containing body fluid will form a bioelectric
battery in which zinc is oxidized and oxygen is reduced to water.

(i) Write balanced half equations, with state symbols, for the reactions that occur at the
anode and cathode of the bioelectrical battery.

(ii) Calculate the time taken for a 6.5 g of zinc electrode to be used up, given that a current
of 4 x10-5 A is generated.
[4]

1
Electrolysis

3 [SRJC 2011/II/2(a)]
This is a question on the halogens and their compounds.

Chlorine can be manufactured by the electrolysis of brine, concentrated NaCl, using a


diaphragm cell. During the electrolysis process, a steady current of 3.0 A was passed through
the cell. 500 cm3 of chlorine gas was collected at 30 oC and 1.5 atm.

(i) Write ionic equations for the reactions occurring at the anode and at the cathode.
23
(ii) Determine the amount of chlorine produced, stating any assumptions that you have
made.
(iii) Hence, calculate the time taken for the process.
LL

(iv) If the process was carried out at 5 oC, will your assumption in (a)(ii) still be valid? Why?
(v) With reference to relevant electrode potentials in the Data Booklet, explain why fluorine
gas cannot be prepared by electrolysis of aqueous sodium fluoride, NaF(aq).
[11]

2
Electrolysis

4 [PJC 2011/III/2(a)]
The anodised aluminium layer is grown by passing a direct current through dilute sulfuric
acid, with the aluminium object serving as the anode.

When current is passed through the circuit, the oxygen liberated at the anode will react with
aluminium to create a build-up of aluminium oxide.

3.0 A of current is passed through the circuit and the volume of the oxygen collected at the
anode after 10 minutes is 40 cm3.

platinium electrode

dilute sulfuric acid

aluminium anode
23
(i) Write ion-electron equations to account for the liberation of the two gases at each
electrode.
LL

(ii) Write an equation to show the formation of aluminium oxide.


(iii) Calculate the volume of oxygen you would expect to be produced at the anode using
inert electrodes at room temperature and pressure.
(iv) Using your answer in (a)(iii), calculate the mass of aluminium oxide produced at the
anode.
[7]

5 [RVHS 2011/II/4(a),(b)]
Over one million tonnes of chromium are produced in the world each year and about 30 % of
it is used in electroplating. In an electroplating experiment, an aqueous solution containing
chromium(III) sulfate was electrolysed using a piece of pure chromium and the metal object to
be electroplated as the electrodes.

A current was passed for 1.5 hours and the mass of the metal object was found to increase
by 3.7 g. Calculate the current used.
[2]
[RVHS 2011/II/4(b)]
The experiment was repeated using aqueous aluminium sulfate instead of chromium(III)
sulfate, and a piece of pure aluminium instead of chromium.

Explain why aluminium will not be electroplated on the metal object. Hence, write the
equations, including state symbols, for the reactions which occur at the anode and cathode.
[3]

3
Electrolysis

6 [YJC 2011/II/2(b)
Chromium plating is an important industrial application of electrolysis, where a steel object is
plated with a thin layer of chromium to make it resistant to rusting and scratches.
In a typical process, one electrode is usually made of graphite while the other is the steel
object to be electroplated, and the electrolyte used is chromium(III) sulphate, Cr2(SO4)3.

(i) State whether the steel object should be connected to the positive or negative pole of
the battery.
(ii) Calculate the maximum mass of chromium that will be plated onto the steel object if a
10 A current is passed through the cell for 30 minutes.
(iii) In this method of electroplating, the graphite electrode has to be replaced from time to
time. Suggest a reason for this.
[4]

7 [TJC 2011/II/3]
Electroplating is an important industrial process where it produces a thin coating of precious
metals such as gold or silver onto other metals.
Traditionally in silver plating, the electrolytic cell consists of a silver electrode connected to
one end of the battery and the object to be plated will be connected to the other end. The
electrolyte used in silver plating industrially is an aqueous solution of silver cyanide.
23
(a) Write the appropriate ion-electron half-equations, with state symbols, for the changes which
takes place at each electrode.
LL

[2]

(b) Explain why the concentration of silver ions remains constant during the process of silver
plating.
[1]
A research student tries to plate silver onto a metal coin in his lab, changing the following
laboratory procedures:
I. Electrolyte used: An aqueous solution of silver nitrate
II. Graphite electrode was used instead of silver electrode

(c) (i) Describe any observation by the research student as electric current was passed
through the electrolytic cell.

(ii) Construct an equation for the reaction occurring at the positive electrode, and
hence construct the equation for the overall reaction, showing your working.

(iii) Calculate the length of time in hours that a current of 5.0 A has to be passed through
the above electrolysis setup in order to plate 10 g of silver onto the electrode.

(iv) Suggest a reason why the graphite electrode needs to be replaced after some time.
[10]

4
Electrolysis

8 [JJC 2011/III/2(a)]
Use of the Data Booklet is relevant to the question.
Lithium-air battery is one of the promising air-fuelled batteries currently being studied to
power the next generation of emission-free electric vehicles.
The lithium-air battery consists of a lithium electrode and a porous carbon electrode
separated by an electrolyte. When the battery discharges, lithium atoms at the lithium
electrode are stripped of their outermost electrons. The electrons flow through an external
circuit while the lithium ions travel through the ion-conducting polymer electrolyte to the
porous carbon electrode where it combines with electrons and oxygen from air to produce a
white powder, lithium peroxide, Li2O2.
The discharging and the charging processes of the battery are catalysed by gold and
platinum respectively.
(i) Construct a half-equation for the reaction that occurs at each of the electrodes of this
lithium-air battery during discharge.
(ii) Given that the battery provides a voltage of +3.1 V, calculate E(O2/Li2O2).
(iii) In order for the lithium-air battery to last for a few hundred miles, 37.8 g of lithium
peroxide (Mr = 45.8) has to be deposited on the electrode. How long would it take, in
hours, to perform this charging process at a current of 10.0 A?
(iv) State the polarity of each of the electrodes during the charging process.
(v) Suggest a reason to explain why the lithium-air battery is potentially dangerous as a
23
battery in vehicles.
[8]
LL

9 [MI 2011/III/1(a)]
Ethanol is a renewable energy source because the energy is generated by using a resource,
sunlight, which cannot be depleted. Creation of ethanol starts with photosynthesis causing a
feedstock, such as sugar cane or corn, to grow. These feedstocks are processed into
ethanol.
Ethanol can be used in fuel cells, in the presence of aqueous potassium hydroxide electrolyte
to produce electricity. During the reaction, ethanol is oxidised to ethanal.

(i) Write ionic equations to illustrate the reactions that are happening at the cathode and
anode respectively.
(ii) Write an equation to represent the overall equation
(iii) An ethanol fuel cell is said to provide an e.m.f of 1.43V. Determine the reduction
potential of the ethanol half cell.
(iv) Assume that the current that is drawn by the ethanol fuel cell is 0.2 amperes per 10
hours, what is the mass of ethanol consumed by the ethanol fuel cell in a day?
[7]

10 An aqueous solution of potassium iodide was electrolysed with a copper anode and a
graphite cathode. After some time, a cream precipitate was found coated on the copper
electrode and the solution surrounding the anode turns brown. At the cathode, effervescence
was observed.
With the aid of relevant equations, explain the observations at each electrode.

5
Electrolysis

11 N98/II/3
(a) Give the ion-electron equations for the electrode reactions in the electrolysis of
aqueous sulphuric acid using inert electrodes.

(b) When a graphite anode is used in the electrolysis, the gas liberated is a mixture of
oxygen, carbon monoxide and carbon dioxide. In the experiment illustrated, 30 cm 3 of
gas formed above the cathode and 17 cm3 of gas above the anode. The anode gas
was collected and its volume was reduced to 9 cm3 when shaken with aqueous
sodium hydroxide.

23
LL

(i) Explain why oxides of carbon are produced at the anode.


(ii) The volume of gas collected at the anode is larger than that expected. Explain,
with the aid of an equation, why this is so.

(iii) What volume of carbon dioxide was present in the 17 cm3 of gas from the
anode?
(c) (i) How much oxygen would you expect to be produced at the anode using an
inert electrode?
(ii) What was the volume of each of the following gases, carbon monoxide and
oxygen?

6
Electrolysis

12 [VJC 2011/II/4(a)+(b) modified]


The electrolysis of organic compounds has been well studied since the 1970s. For instance,
Kolbe studied the electrolysis of a series of aromatic organic compounds. Compound E with
the molecular formula, C6H7NO is a 1,4-disubstituted aromatic compound studied using
the experimental set-up as shown.

graphite electrode coated graphite electrode coated


with compound E with compound E
aqueous buffer at pH 6.0

(a) (i) Compound E dissolves in both aqueous hydrochloric acid and aqueous sodium
hydroxide. However, it does not dissolve in aqueous sodium carbonate. Deduce the
structure of compound E.

(ii) Compound E is oxidised to compound F, C6H5NO at the anode.


A series of chemical tests was performed on compound F. When 2,4-
dinitrophenylhydrazine was added, an orange precipitate was observed. No silver
mirror was obtained when warmed with aqueous Ag(NH3)2+ solution. One mole of
compound F reacts with four moles of hydrogen gas in the presence of platinum
23
catalyst to yield compound G with molecular formula C6H13NO.
(hint: there is a C=NH bond in compound F.)
LL

Deduce the structures of compounds F and G.

(b) (i) Compound F is reduced at the cathode back to compound E. Write the half
equation for this reduction using the given molecular formulae.

(ii) A current of 2.0 A is supplied to the experimental set-up. Using the half-equation in
(b)(i), calculate, in mg/min, the average rate of mass gain at the cathode, given that
the molecular mass of compound E is 109.0.
[8]

7
Electrolysis

Multiple Choice Questions (Section A)

1 [CJC 2011/I/14]
During electroplating, a current is passed through a cell containing aqueous silver nitrate
using inert electrodes. After a certain time, 2.70 g of silver was deposited at one electrode.
What volume of gas would be produced at the other electrode at r.t.p.?

A 70 cm3
B 75 cm3
C 140 cm3
D 150 cm3

2 [MJC 2011/1/15]
Two electrolytic cells containing X2(SO4)m(aq) and Y2(SO4)n(aq) are connected in series,
where X and Y are both metals. The relative atomic masses of X to Y is 2 : 3. When a
current is passed through the set-up, the ratio of the masses of X : Y liberated is 1 : 2.

What are the values of m and n?

m n
A 1 3
B 2 6
23
C 3 1
D 4 3
LL

3 [AJC 2011/I/11]
A molten chloride of a Group I metal, QCl, and a molten chloride of a Group II metal, RCl2,
is separately electrolysed using the same current for the same period of time.

Which of the following statements about the experiment is correct?

A An equal number of moles of Q and R are deposited.


B The mass of elemental Q and R that are deposited are independent of the
electrolysis time.
C The number of moles of R deposited is half the number of moles of Q deposited.
D RCl2 gives off twice the volume of chlorine gas as QCl, measured under the same
conditions of temperature and time.

8
Electrolysis

4 [PJC 2011/I/8]
The following aqueous solutions were connected in series as shown below:

Which of the following correctly represents how the mass of each electrode changes as the
reactions proceed?

Electrode 1 Electrode 3 Electrode 4


23
A no change no change decreases
B decreases no change decreases
C decreases decreases no change
LL

D no change decreases no change

5 [VJC 2011/I/12]
Two electrolytic cells containing aqueous X(NO3)m in cell 1 and molten YCln in cell 2 are
connected in series. When a current was passed through the set-up, metals X and Y are
produced at the cathode of cell 1 and cell 2 respectively. The volume of the gas produced at
the anode of cell 1 and cell 2 is p cm3 and q cm3 respectively.

What is the ratio of p : q?

A 1:2
B 2:1
C 2:3
D 3:2

9
Electrolysis

6 [RVHS 2011/I/11]
A direct current is passed through the apparatus shown in the diagram below.
+ 

carbon electrodes
neutral litmus paper
soaked in concentrated
potassium chloride

X Y Z

What are the colours at positions X, Y and Z?

X Y Z
A red purple blue
B blue purple red
C white purple blue
D blue purple white
23
7 [SAJC 2011/I/11]
Use of the Data Booklet is relevant to this question.
LL

An antique pendant is to be chrome-plated. The pendant is dipped into a basic chromate


(VI) solution where it serves as an electrode in an electrolytic cell. Oxygen is formed on the
other electrode. Which one of the following statements is correct?

A Oxidation of water occurs to form oxygen.


B The pendant is the anode of the electrolytic cell.
C It takes about 16 hours to plate 52 g of chromium, if the current used is 1A.
D For every 104 g of chromium plated, 3 moles of oxygen gas is evolved.

8 [TJC 2011/I/12]
When 190 C of electricity is passed through a molten compound, about 0.21 g of a metal is
deposited at the cathode. What could this metal be?

A Magnesium C Copper
B Silver D Iron

10
Electrolysis

9 [TJC 2011/I/10]
Two cells are connected in series as shown in the diagram where L, M, N and O are
electrodes. Which of the following correctly shows the products formed at each electrode?

L M N O
lead platinum

1 mol dm-3 1 mol dm-3


HNO3(aq) FeCl3(aq)

L M N O

A O2 H2 O2 Fe2+

B O2 Pb Cl2 H2
23
C Pb2+ H2 Cl2 H2

D Pb2+ H2 O2 Fe2+
LL

10 [VJC 2011/I/7]
A cell is constructed with zinc and carbon electrodes, each weighing 50 g, partly
immersed in aqueous KOH.

When connected in a circuit, some zinc passes into the solution as zincate(II) ions,
ZnO22-(aq). The cell delivers a current of 1.68  10-2 A. The zinc electrode is
replaced once 60% of it is used up.

What is the time taken, in seconds, before replacement of zinc electrode becomes
necessary?

A 2.63  106 C 5.27 x 106


B 4.39  106 D 8.78 x 106

11
Electrolysis

11 [SAJS 2010/I/15]
Aluminum can be obtained from electrolysis of aluminum oxide. A current of 350 kA was
passed through the electrolytic cell for 8 hours. What is the mass of aluminum obtained?

A 9.68 kg C 28.2 kg
B 940 kg D 2820 kg

12 [RI 2011/I/14]
In an experiment, a cell was set up to obtain pure copper from a copper-silver alloy as shown
below.

electrode P electrode Q

CuSO4(aq)

When a current of 40.0 A flows through the electrolyte for 26.8 minutes, the mass of the
anode changes by 26.47 g.
Which of the following statements is correct?
23
A Electrode P is pure copper and electrode Q is the copper-silver alloy.
B The concentration of CuSO4(aq) decreases during the experiment.
C The mass of the cathode changes by 26.47 g during the experiment.
LL

D The copper-silver alloy contains 20% silver by mass.

13 [NJC 2010/I/13]
The same quantity of electricity that caused the deposition of 5.0 g of silver from a silver
nitrate solution liberated 3.02 g of an element B from a solution containing Bn+ ions. If the
relative atomic mass of B is 197, the value of n is

A 1
B 2
C 3
D 4

14 [JJC 2011/I/12]
A current is passed through two cells connected in series. The first cell contains XCl2(aq)
while the second cell contains YCl(aq). The mass of X and Y deposited at the respective
electrodes is in the ratio of 1:3.
What is the ratio of the relative atomic mass of X : relative atomic mass of Y?

A 1:6 B 2:3 C 3:2 D 6:1

12
Electrolysis

15 [SRJC 2011/I/12]
Aluminium alloys usually undergo a process known as anodisation to increase corrosion
resistance and surface hardness. In this process, oxygen is formed and the gas reacts with
the aluminium object to form a thick layer of aluminium oxide on the metal surface that
protects the metal.

A possible set up for this process is shown as below:

Gases liberated

Battery

H2SO4 (aq)

Aluminium
object
Graphite
23
Which of the following statements about the anodisation of aluminium is true?

A Hydrogen gas and sulfur dioxide gas are liberated.


LL

B Replacing the graphite electrode with copper will cause the reaction to cease.
C Anodisation is also possible when the aluminium object is connected to the
negative terminal
D Water is oxidised at the anode to produce oxygen gas.

Multiple Choice Questions (Section B)

1 [JJC 2011/I/35]
Which of the following can take place when a hot solution of brine is electrolysed using
carbon electrodes in the absence of a diaphragm?
1 The main product is sodium chlorate(V).
2 Oxygen gas is evolved at the anode.
3 The pH of the electrolyte decreases.

13
Electrolysis

2 [DHS 2011/I/35]
Which of the following would yield either hydrogen, oxygen or both gases upon electrolysis?
1 CuSO4(aq) using Cu electrodes
2 MgBr2(aq) using C electrodes
3 Na2SO4(aq) using Pt electrodes

3 [HCI 2011/I/34]
Use of the Data Booklet is relevant to this question.

An electrolytic cell containing Mn2+ and another metal ion, Y3+, is connected to Mn and Pt
electrodes. The reactions that took place are
23
Mn  Mn2+ + 2e

Y3+ + 3e  Y
LL

Which of the following statements are correct?


1 The Mn electrode is the anode.
2 A possible identity of Y is aluminium.
3 The number of moles of Y deposited is 1.5 times the number of moles of Mn 2+
formed.

4 [IJC 2011/I/35]
When a solution of concentrated sodium ethanoate is electrolysed, the equation for the
reaction is
2 CH3CO2Na + 2 H2O CH3CH3 + 2 CO2 + 2 NaOH + H2

Which statement(s) regarding the electrolysis is/are correct?


1 Hydrogen is liberated at the cathode.
2 Carbon dioxide is liberated at the anode.
3 Ethane is liberated at the cathode.

14
Electrolysis

ELECTROLYSIS SUGGESTED SOLUTIONS

Multiple Choice Questions (Section A)


1. 2. 3. 4. 5. 6. 7. 8. 9. 10. 11. 12. 13. 14. 15.
D D C C A C D B D C B D C B D

Multiple Choice Questions (Section B)


1. 2. 3. 4.
D C D B

Structured/Free Response Questions

1a (i) (I) Anode: 2O2O2(g) + 4e


Cathode: K+ + e  K

(II) Anode: Cu(s)  Cu2+ + 2e


Cathode: Cu2+ + 2e  Cu

(ii) ne × 96500 = I × t
ne = 5 × 30 × 60 / 96500 = 0.09326 moles
No. of moles of O2 formed = ¼ × (0.09326)
= 0.02332 moles.
Volume of O2 formed at r.t.p = 0.02332 × 24 dm3
23
= 0.560 dm3 (560 cm3)

(b) (i) Anode: 2 Br  Br2 + 2 e


LL

(Note: Eo of Br2/Br is less positive than Eo of O2/H2O, hence, bromide is


preferentially undergo [O] as compared to H2O)

Cathode: 2 H2O + 2 e  2 OH + H2

(ii) As there is production of OH, pH will increase over time.


Q=Ixt
= 1.5 x 2 x 60
= 180 C
180
No. of moles of electrons = = 1.865 x 103 mol
96500
No. of moles of OH = 1.865 x 103 mol
1.865 ×10−3
[OH] = 100
× 1000 = 0.0187 mol dm3
pH = 14  (lg0.0187) = 12.3

15
Electrolysis

2 (i) Anode: Zn(s)  Zn2+(aq) + 2e


Cathode: O2 (g) + 4H+(aq) + 4e  2H2O(l)

(ii) No of moles of zinc = 6.5 / 65.4 = 0.09939 mol


No of moles of electrons given out = 0.09939 x 2 = 0.1988 mol
neF = It
0.1988 x 96500 = 4 x 10-5 x t
t = 480 000000 s OR 4.80 x 108 sec OR 15.2 years OR 1.33 x 105 hours OR time in
other units (weeks, days, etc)

3 (i) Anode: 2Cl- (aq)  Cl2 (g) + 2e


Cathode : 2H2O (l) + 2e  2OH- (aq) + H2 (g)

(ii) Assume Chlorine behaves ideally.


Using pV = nRT,
(1.5 1.01105 )(500 106 )
Amount of Cl2 = = 0.0301 mol
(8.31)(273  30)
23
(iii) Since 2Cl- (aq)  Cl2 (g) + 2e
Q
Amount of Cl2 =
LL

ne F
It
0.03008 =
ne F
3t
0.03008 =
2  96500
5805.44
t= = 1940 s
3
(iv) No.
This is because at low temperature, there is significant intermolecular forces of
attraction. We can no longer assume that it is an ideal gas.

(v) F2 + 2e 2F– +2.87 V


+
O2 + 4H + 4e 2 H2O +1.23 V
Ered (F2 / F−) is much more positive / bigger / higher than Ered (O2 / H2O )

O2 will always be preferentially discharged (rather than F2 gas).


Or F- has less tendency to undergo oxidation compared to H2O

16
Electrolysis

4 (i) Cathode: 2H+(aq) + 2e  H2(g)


Anode: 2H2O(l)  O2(g) + 4H+(aq) + 4e
(ii) 2Al(s) + 3/2O2(g)  Al2O3(s)
(iii) Q = 3.0 x 600 = 1800 C
No. of moles of electrons = 1800/96500 = 0.01865 moles
No. of moles of O2 produced = 0.01865/4 = 4.663 x 10-3 moles
Volume of O2 produced = 4.663 x 10-3 x 24 = 0.112 dm3
(iv) Volume of O2 reacted with aluminium = 111.9 – 40 = 71.9 cm3
No. of moles of O2 reacted = 0.0719/24 = 3.329 x 10-3 moles
Mass of Al2O3 produced = 3.329 x 10-3 x 2/3 x 102 = 0.226 g

5 (a) Amount of Cr deposited on metal object = 3.7/52 = 7.115  102 mol


Cr3+(aq) + 3e  Cr(s)
Amount of e = 7.115 x 102  3 = 0.2135 mol
Q=It
0.2135  96500 = I  1.5  60  60
I = 3.82 A
(b) The reduction potential for the reduction of H2O is more positive/less
negative than that for Al3+.
Therefore, H2O will be preferentially reduced at the cathode to form H2 and
23
aluminium will not be plated on the metal object.
Anode: Al(s)  Al3+(aq) + 3e
Cathode: 2H2O(l) + 2e  H2(g) + 2OH(aq)
LL

6 (i) negative

(ii) Q I  t 10  30  60 18000
ne       0.187mol
F F 96500 96500
1 1
mCr  nCr  M Cr   ne   M Cr   0.187  52.0  3.23g
3 3

(iii) The graphite electrode (anode) will be oxidized by the O2(g) produced to form CO2(g)

7a) Anode: Ag(s)  Ag+(aq) + e-


Cathode: Ag+(aq) + e-  Ag(s)

(b) For every Ag+ ion reduced and plated onto the object, one Ag+ ion will be released into
the electrolyte as the silver electrode is oxidized to release Ag+ ion.

(c) (i) Effervescence of oxygen gas at the positive electrode.

(ii) 2H2O  O2 + 4H+ + 4e-


Ag+ + e-  Ag (x4)
Overall: 2H2O + 4Ag+  O2 + 4H+ + 4Ag

17
Electrolysis

(iii) No. of mol of Ag = 10/107 = 0.0935mol


No. of mol of e- transferred = 0.0935mol
Total Quantity of charge = 0.0935 x 96500 = 9018.7C
Q = It
9018.7 = 5(t)
t= 1803.7s = 0.5hrs

(iv) C(s) + O2(g)  CO2(g)


The graphite electrode will react with O2 gas/ be oxidized by O2 gas to form CO2
and thus needs to be replaced after some time.

8 (i) Lithium electrode: Li  Li+ + e


Carbon electrode: 2Li+ + 2e + O2  Li2O2

(ii) Ecell = E(O2/Li2O2)  E(Li+/Li)


E(O2/Li2O2) = Ecell + E(Li+/Li) = +3.1 + (3.04) = +0.06 V

(iii) Amt of Li2O2 = 37.8 / 45.8 = 0.825 mol


Since Li2O2  2e, amt of e, n = 2  0.825 = 1.65 mol
Using Q=It= nF, t = nF/I = (1.65)(96500)/10.0 = 15844 s = 4.42 h OR 4.40 h
23
(iv) Lithium electrode (cathode): negative
Carbon electrode (anode): positive
LL

(v) Any one of the following:


 Lithium is very reactive. It is burns explosively with oxygen or water.
 Lithium is a fire hazardous material which will further ignite in times of car
accidents/overheated.
 Lithium peroxide is a very dangerous fire hazardous material as it is a strong
oxidizing agent.
Lithium peroxide gives off a lot of heat (or highly exothermic) with reacted with
water or steam.

9 (i) Anode: CH3CH2OH + 2OH-  CH3CHO + 2 H2O + 2e ;


Cathode: O2+ 2H2O + 4e  4OH-

(ii) 2CH3CH2OH + O2  2 CH3CHO + 2 H2O

(iii) [O]: CH3CH2OH + 2OH-  CH3CHO + 2 H2O + 2e E =a


[R]: O2+ 2H2O + 4e  4OH- E = 0.40V

a+ 0.40 = 1.43V
a = 1.03 V

Reduction potential = -1.03V ; (Reject if the answer is +1.03V)

18
Electrolysis

(iv) Q = It = 0.2 x 10 x 60 x 60 = 7200 C


Amt of electrons consumed in 10 hours = 7200 ÷ 96500 = 0.07461 mol.
Amt of ethanol consumed in 10 hours = 0.07461 x 0.5 = 0.03731 mol. ;
Mass of ethanol consumed in 10 hours
= 0.03731 x (2 x 12.0 + 6 x 1.0 + 16.0) = 1.72g
Mass of ethanol consumed in a day = 1.72 x 24 ÷ 10= 4.12g (3sf)

10 Copper anode [O]:


Cu  Cu2+ + 2 e
E Cu2+/Cu is +0.34 V, which is less positive than E I2/I which is + 0.54 V and
E(O2/H2O) which is + 1.23 V, hence, Cu will most likely undergo [O] to form Cu2+
I ions will migrate to the copper anode and will react with the Cu2+ produced at the
anode as shown below:

2 Cu2+ (aq) + 4 I (aq)  2 CuI (s) + I2 (aq): Found on p 21 of lecture notes for
Electrochemistry 1.

CuI is the cream ppt coated on the copper anode.

Graphite cathode:
As the Cu2+ has reacted with I ions at the anode to form CuI, it will not migrate to the
23
cathode.
Hence, at the cathode, the following occurs:
H2O + 2 e  2 OH + H2
Effervescence of H2 would be observed.
LL

11 (a) Cathode: 2 H+ + 2 e  H2
Anode: 2 H2O  O2 + 4 H+ + 4 e

(b) (i) The O2 produced at the anode reacts with the carbon electrode to form CO
and CO2.

(ii) C (s) + O2 (g)  CO2 (g)


When one volume of CO2 is produced, one volume of O2 is consumed, hence,
this process would NOT lead to increase in volume of gas at anode.

2 C (s) + O2 (g)  2 CO (g)


When one volume of O2 is consumed, two volumes of CO is formed, thus,
there will be a net increase in one volume of gas when CO is produced.
Hence, this process leads to an increase in volume of gas at the anode.

(c) (i) Since 9 cm3 of gas is left after 17 cm3 of anode gas is passed through NaOH,
Volume of CO2 = 17 – 9 cm3
= 8 cm3
(ii) Since volume of H2 = 30 cm3, volume of O2 expected = 15 cm3
(NOTE: Vol of H2: O2 is 2:1. Referring to equations in (a), production of H2
requires 2 e, and O2 is 4 e, hence, to balance electrons, the eqn at the cathode
must be multiplied by 2, thus cathode reaction is 4 H+ + 4 e  2 H2. )

19
Electrolysis

(iii) Long method:


Volume of CO2 + unreacted O2 + CO = 17 cm3
Volume of CO2 = 8 cm3
Hence, volume of unreacted O2 + CO = 9 cm3

2 C (s) + O2 (g)  2 CO (g)


Let volume of O2 reacted to form CO = y
Hence, volume of CO produced = 2 y

C (s) + O2 (g)  CO2 (g)


Volume of CO2 produced = 8 cm3
Hence, volume of O2 reacted to form CO2 = 8 cm3

Volume of O2 used in total = 8 + y


Volume of O2 expected = 15 cm3
Hence, volume of O2 unreacted = 15  (8+y)
Since volume of CO = 2 y and volume of unreacted O2 + CO = 9
15  (8+y) + 2 y = 9
y = 2 cm3

Hence, Volume of CO = 2 x 2 = 4 cm3


Volume of unreacted O2 = 15  (8+2) = 5 cm3
23
Short method:
Net increase in volume at anode is 17 – 15 cm3 = 2 cm3
As mentioned, net increase is due to CO production.
LL

2 C (s) + O2 (g)  2 CO (g)

When one volume of O2 is consumed, two volumes of CO is formed, thus,


there will be a net increase in one volume of gas when CO is produced.
Hence, this process leads to an increase in volume of gas at the anode.

Since there is a net increase in 2 cm3, this implies that 2 cm3 of O2 was
consumed and 4 cm3 of CO was formed, leading to a net increase in 2 cm3.

Hence, volume of CO is 4 cm3, Volume of unreacted O2 = 9 – 4 cm3 = 5 cm3

12 (a) (i) Compound E could contain basic –NH2 group which can react with HCl.
Compound E has acidic phenolic –OH group which is capable of reacting with
NaOH.
Phenol is not sufficiently acidic to react with Na2CO3, unlike carboxylic acid.
Structure of compound E

20
Electrolysis

(ii) Orange ppt with 2,4 DNPH implies that carbonyl (aldehyde or ketone) is
present.
No silver mirror with aqueous Ag(NH3)2+ implies that aldehyde is absent (or
ketone is present).
Catalytic reduction adds 8 hydrogen atoms to 1 molecule of compound F.
1 molecule of H2 is used to reduce the ketone.
The remaining 3 molecules of H2 must be used to reduce 3 double bonds.

Structure of compound F

Structure of compound G

(b) (i) Cathode: C6H5NO + 2H+ + 2e  C6H7NO

(ii) Quantity of charge per minute = 2 x 1 x 60 = 120 C


23
No. of moles of compound E formed at cathode per minute
= 0.5 x no of moles of electrons
= 0.5 x (120 / 96500) = 6.22 x 10-4 mol
LL

Average rate of mass gain = 6.22 x 10-4 x 109.0 x 1000


= 67.8 mg/min

21
Transition Elements

TRANSITION ELEMENTS 10
Structured / Free Response Questions
1 [DHS 2011/II/6(a)-(b)]

Rhodium is a second row transition element with a melting point of 1964 oC.

(a) Explain why rhodium has a much higher melting point than strontium, which is also found in
the same period.
[2]
(b) Although the rhodium atom has more electrons than the strontium atom, the atomic radius
of rhodium is smaller than that of strontium.
Provide an explanation for this.
[2]
[Total: 4]

2 [MJC 2011/II/3 modified]

Old mine workings are often the cause of the pollution of mountain streams. One of the
main chemical culprits of such pollution is iron pyrites, FeS2, in which iron has an oxidation
state of +2.
23
The early miners unwittingly accelerated the chemical processes by grinding up unwanted
rock containing some ore and dumping it in waste heaps. Oxygen in the air and rain water
combined to oxidise the sulfide ions in solid iron pyrites to form an acidic solution of iron(II)
sulfates. The resulting ions then run off into the local streams where the iron(II) ions are
LL

oxidised.

(a) Write an equation for the oxidation of iron(II) ions in the stream water under acidic
conditions.
[1]

(b) Aqueous iron(III) ions are present in the stream as complex ions.

(i) What is meant by the term ‘complex ion’?

(ii) Draw a diagram to show the shape of the aqueous iron(III) complex ions.
[2]

(c) The iron(III) ions are hydrolysed in the stream to form a precipitate which is basic in nature.
The precipitate is referred to as ‘yellow boy’ by the miners.

(i) Suggest the formula of the precipitate which gives rise to the yellow colour of ‘yellow
boy’.

(ii) Write an equation to show the hydrolysis of aqueous iron(III) ions in water to form the
‘yellow boy’, given that the final solution is acidic in nature.
[2]

1
Transition Elements

(d) As a result of the increased acidity of the stream water, other d-block metal ions are leached
into the water. Zinc(II) ions are often found in the water but are not noticed because they are
colourless.

(i) Suggest a reason why zinc(II) ions are colourless.

(ii) Explain why iron compounds are usually coloured.


[4]

3 [VJC 2011/III/3]

This question is about the transition elements and their compounds.

(a) The graph below shows the second to fourth ionisation energies for the first row dblock
elements scandium to zinc.

23
LL

(i) Explain what is meant by the term transition element.

(ii) Give an equation each to represent the second ionisation energy of chromium and the
fourth ionisation energy of iron respectively.

(iii) Explain briefly why the second ionisation energy of chromium is higher than that of
manganese.

(iv) Explain briefly why the fourth ionisation energy of cobalt is lower than that of iron.
[6]

2
Transition Elements

(b) In one experiment, when aqueous iron(III) sulfate is boiled with an excess of potassium
cyanide, a red solution A is obtained. In another experiment, when aqueous iron(III) sulfate
is added to aqueous potassium iodide, a brown solution B is obtained.

Solution B is found to catalyse the reaction between iodide and peroxodisulfate, S2O82,
whereas solution A does not affect the rate of this reaction.

When aqueous sodium thiosulfate is added to solution B followed by aqueous sodium


hydroxide, the brown colour is discharged and a green precipitate is observed. On standing,
the green precipitate turns red-brown.

Explain these observations as fully as you can, with the aid of equations and reference to
relevant Eo values.
[9]

(c) Cobalt is one of the transition elements which is capable of complex ion formation.

(i) Ethylenediamine, NH2CH2CH2NH2, is a ligand that can form a complex with Co3+ ions
in the mole ratio of 3:1. Deduce the formula of the cobalt-containing complex ion and
state its shape.

(ii) [Co(NH3)6]3+ appears yellow-brown while [CoF6]3 appears blue. Explain the difference
in colour.
23
[5]
[Total: 20]
LL

4 [CJC 2011/III/1(a)-(d)]
Nanochemistry is an important field of science that explores the use of extremely small
particles that have diameters about thousand times thinner than the width of a human hair.
Transition metals and its compounds are often the subject of interest in nanochemistry, as
many useful properties are improved at this ultra small scale.

(a) Nano-sized iron(III) hydroxide, Fe(OH)3 exists as a precipitate and can be used to remove
aqueous arsenate(V) ions, AsO43– from contaminated water. The Fe(OH)3 with the adsorbed
arsenate(V) ions are filtered off, and undergoes an ion-exchange with the arsenate(V) ions
removed and Fe(OH)3 precipitate recovered unchanged.
(i) State the electronic configuration of iron in Fe(OH)3.
(ii) Briefly explain why transition metal compounds are suitable for the above process.
(iii) Suggest why the use of nano-sized particles will improve this process.
[4]

(b) Nano-sized iron(II) hydroxide, Fe(OH)2 is often unstable, due to oxidation by water even
though oxygen is not present. The pH of the solution remains largely unchanged, a reddish-
brown precipitate and effervescence are observed.
(i) Identify the reddish-brown precipitate and suggest an equation for this reaction.

3
Transition Elements

(ii) The reaction is not expected to occur under standard conditions and can be
considered reversible. Suggest a possible reason why it does in fact occur for an
opened bottle of Fe(OH)2 suspended in water.
[3]
(c) Nano-sized iron(II) hydroxide, Fe(OH)2 is used for the purpose of removal of toxic
selenite(IV) ions, SeO32 from water, by reducing selenite(IV) ions to selenium, Se.
(i) Construct a half-equation for the reduction of SeO32 to Se in an alkaline solution.
(ii) In water that is heavily contaminated with CN ions, iron(II) hydroxide appears to
dissolve. State the type of reaction that is occurring and construct a balanced
chemical equation for it.
(iii) By selecting Eo values from the Data Booklet, and given that the redox potential of the
reaction in (i) is –0.37 V, explain and justify if SeO32 can be removed from water
contaminated with CN– ions.
[5]
[Total: 12]
5 [IJC 2012/II/6]

Many copper minerals are found in hydrothermal deposits where they were formed by
crystallization from very hot solutions which were trapped underground at high pressures.
One such copper mineral is chalcopyrite, CuFeS2.
23
Copper is extracted from the ore chalcopyrite (CuFeS2) in a three-stage process. In the first
stage of this extraction, the chalcopyrite is heated with silicon dioxide and oxygen.
LL

(a) Balance the following equation for this first stage in which copper sulfide is formed.
…...CuFeS2 + ……..SiO2 + ……..O2 Cu2S + ….…FeSiO3 + ……SO2
[1]

(b) Write the electronic configuration for Cu+ in Cu2S.


[1]

(c) When water is added to white anhydrous CuSO4, the solid dissolves to give a blue solution.
On addition of concentrated NH4Cl (aq), the solution changes to a yellow-green due to
formation of copper containing species D. Concentrating the solution produces green
crystals E of an ammonium salt with empirical formula CuN2H8Cl4.

(i) Suggest the formulae of cation present in E.

(ii) Suggest the formulae of anion D.

(iii) Suggest a balanced equation for the formation of D from aqueous CuSO4.

When excess of NH3 (aq) is added to species D, the yellow green solution turns to a deep
blue solution.

4
Transition Elements

(iv) Use this information and the information above to suggest the strength of NH3, H2O
and Cl– ligands in decreasing order.
[4]

(d) When a dilute aqueous solution containing a bidendate ligand, ethanedioate ion (C2O42–) is
added to a solution containing aqueous copper(II) ions, a ligand exchange reaction occurs.
In this reaction, four water molecules in the hydrated copper ion are replaced and a new
complex F is formed.

(i) Explain what is meant by a bidendate ligand.

(ii) Suggest the formulae of complex F formed.

(iii) In the complex F formed, the two water molecules are opposite each other. Draw a
diagram to show how the ethanedioate ions are bonded to a copper ion and give a
value for one of the O-Cu-O bond angles. You are not required to show the water
molecules.
[4]
[Total: 10]

6 [NJC 2011/II/5(a)-(c)]
23
LL

5
Transition Elements

(a)

[6]

(b)

[2]

(c)

[2]
[Total: 10]

7 [JJC 2012/II/4(a)]

Copper is a transition element that forms a rich variety of compounds with varying oxidation
states.

Cu(NO3)2(aq) is a blue solution which can undergo the following reactions.


23
LL

B, C and E contain complex ions of copper.


Both compound D and complex ion E contain copper and chlorine only.
The oxidation number of copper in C, D and E are the same.

(a) (i) What are the formulae of the compound A and of the cation present in B?

(ii) Suggest a balanced equation for the formation of B from A.

(iii) State the role of SO2 in the conversion of Cu(NO3)2 to Cu2O.

6
Transition Elements

(iv) The composition of complex ion C by mass is Cu, 65.1%, N, 28.7%, H, 6.2%.
Use the data to determine the empirical formula and identity of complex ion C.

(v) Suggest the formulae of compound D and complex ion E, given that the coordination
number of Cu in complex ion E is the same as that in C.
[8]
[Total: 8]

8 [MJC 2012/II/6(b)-(g)]

Iron is the most common element (by mass) on Earth. It exists in a wide range of oxidation
states, although +2 and +3 are the most common states. On the other hand, ruthenium is a
rare transition metal with oxidation states ranging from 0 to +8. The most common oxidation
states are also +2 and +3.

(a) The standard electrode potentials for Ru(III)/Ru(II) and Fe(III)/Fe(II) are given below.

[Ru(H2O)6]3+ + e- [Ru(H2O)6]2+ Eo = +0.23 V

[Fe(H2O)6]3+ + e- [Fe(H2O)6]2+ Eo = +0.77 V

Comment on the relative stability of the +2 and +3 oxidation state of ruthenium compared to
23
that of iron using the data above.
[1]

(b) When ruthenium(III) ions are treated with aqueous ammonia, an aqueous complex A is
LL

formed. Zinc is added to complex A and the resulting solution is treated with NaCl. Orange
crystals of compound B are formed which contains 37% of ruthenium by mass.

(i) Suggest a formula for complex A and compound B.

(ii) Aqueous ammonia is added to iron(III) chloride solution. The resulting mixture is
treated with Zn metal to form compound C. Using the Data Booklet, suggest with
reasoning the identity of compound C.
[4]

The following table lists some iron complexes together with their colours and their
equilibrium constants, Kc.

Complex Colour Kc
[Fe(SCN)(H2O)5]2+(aq) Deep red 1 × 102
[FeF6]3 (aq) Colourless 2 × 1015
[Fe(CN)6]4 (aq) Pale yellow 1 × 1024
[Fe(CN)6]3 (aq) Orange 1 × 1031
[Fe(edta)]2 (aq) Colourless 2 × 1014
[Fe(edta)] (aq) Yellow 1 × 1025

[edta = (O2CCH2)2NCH2CH2N(CH2COO2)2]

7
Transition Elements

(c) Write down the equation for the formation of the complex ion formed when Fe 3+ is mixed
with F ions.
[1]

(d) Write an expression for the equilibrium constant, Kc, of [FeF6]3.


[1]

(e) Use the data in the table to predict and explain:

(i) the effect of adding edta on the colour of a solution containing [Fe(SCN)(H2O)5]2+,
(ii) the effect of adding edta on the Eo value for the Fe3+ + e Fe2+ system.
[4]
(f) Explain why [Fe(CN)6]3 is orange in colour.
[3]
[Total: 14]

9 [NYJC 2012/II/4]
Cobalt and vanadium are transition metals.

(a) Give one characteristic chemical property of cobalt and vanadium which shows that they are
23
transition metals.
[1]
LL

(b) Aqueous cobalt(II) chloride, CoCl2 is a pink solution. When a mixture of the pink solution
and tartaric acid, HO2CCH(OH)CH(OH)CO2H, is added to aqueous hydrogen peroxide, the
following changes take place.

The initially pink solution turns green and then oxygen is vigorously evolved. Finally, the
solution turns pink again.

Suggest a role for the CoCl2(aq) and for the tartaric acid. Write an equation for the overall
reaction.
[3]
(c) Aqueous CoCl2 also undergoes the following reaction.

8
Transition Elements

(i) Crystals of salt, K has the following composition by mass:

Co, 22.0%; N, 31.4%; H, 6.7%; Cl, 39.8%

On adding an excess of AgNO3(aq) to an aqueous solution containing 0.01 mol of K,


4.29 g of AgCl(s) is precipitated.

Calculate the empirical formula of K, and draw the structure of the cation present in K
to show the geometry around the central ion.
(ii) State the types of reactions occurring in step I.
(iii) Eo data for some cobalt complex ions are given below.
[Co(C2O4)3]3(aq) + e– [Co(C2O4)3]4(aq) Eo = +0.57 V
3+ – 2+
[Co(NH3)6] (aq) + e [Co(NH3)6] (aq) Eo = +0.11 V

Use the Eo data given above to predict the reaction, if any, of adding C2O42 to K.
Explain your prediction.
[7]

(d) Some of the ions of vanadium and their corresponding colours are shown in the table below.

formula of vanadium ion VO3 VO2+ V3+ V2+


colour of aqueous solution yellow blue green violet
23
The colour of the reaction mixture changes from yellow to green when a transition metal is
added to an aqueous solution containing the salt of a vanadium ion.
LL

By reference to the Data Booklet, suggest a possible identity of the transition metal and
explain the colour change observed.
[3]
[Total: 14]

10 [ACJC 2011/II/5]

(a)

[1]

9
Transition Elements

(b)

23
LL

(i)

(ii)

10
Transition Elements

(iii)

[7]

(c)

(i)

23
(ii)
LL

(iii)

[7]
[Total: 15]

11
Transition Elements

Multiple Choice Questions (Section A)


1 [HCI 2011/I/16]

2 [HCI 2011/I/17]

23
LL

3 [ACJC 2011/I/30]

12
Transition Elements

4 [AJC 2011/I/19]

5 [CJC 2011/I/16]

23
LL

6 [DHS 2011/I/18]

13
Transition Elements

7 [IJC 2011/I/18]

23
LL

8 [JJC 2011/I/9]

14
Transition Elements

9 [VJC 2010/I/17]

10 [NJC 2011/I/21]

23
LL

11 [ACJC 2012/I/19]

15
Transition Elements

12 [SAJC 2011/I/20]

[Cr(OH)3](s)

23
13 [NJC 2011/I/23]
LL

16
Transition Elements

14 [YJC 2011/I/15]

15 [AJC 2012/I/18]

23
LL

17
Transition Elements

16 [JJC 2012/I/6]

23
17 [NYJC 2012/I/19]
LL

18
Transition Elements

18 [PJC 2012/I/18]

23
LL

19
Transition Elements

19 [PJC 2009/I/19]

20 [RI 2010/I/18]

23
Multiple Choice Questions (Section B)
LL

1 [HCI 2011/I/36]

20
Transition Elements

2 [ACJC 2011/I/40]

23
LL

3 [NYJC 2011/I/36]

21
Transition Elements

4 [JJC 2011/I/36]

23
LL

5 [NYJC 2010/I/36]

22
Transition Elements

6 [SRJC 2011/I/37] Modified

2 NH3 is a stronger ligand as compared to OH–.


23
7 [VJC 2011/I/36]
LL

23
Transition Elements

8 [NYJC 2012/I/36]

23
LL

9 [PJC 2012/I/35]

24
Transition Elements

10 [RI 2010/I/36]

23
LL

25
Transition Elements

TRANSITION ELEMENTS SUGGESTED SOLUTIONS

Multiple Choice Questions (Section A)


1. 2. 3. 4. 5. 6. 7. 8. 9. 10. 11. 12. 13. 14. 15. 16. 17. 18. 19. 20.
A C C D A B C D A B B D B B B C C D D D

Multiple Choice Questions (Section B)


1. 2. 3. 4. 5. 6. 7. 8. 9. 10.
C A A C B D C D D B

Structured/Free Response Questions

1
(a) Rhodium has valence electrons from both 5s and 4d orbitals / subshells available for
delocalisation, while strontium only has 2 valence electrons from 5s orbital i.e.
rhodium has more valence electrons than strontium.

Rhodium also has a smaller cationic radius than strontium. Thus, rhodium has
stronger metallic bonding between its cations and delocalised electrons, which
requires more energy to overcome.

(b) Although rhodium has more electrons than strontium, the electrons are added to the
23
inner 4d orbitals and thus provide poor shielding of the 5s electrons. Thus, effective
nuclear charge of rhodium increases relative to strontium and the atomic radius is
smaller.
LL

2
(a) 4Fe2+ (aq) + O2 (g) + 4H+ (aq)  4 Fe3+(aq) + 2H2O (l)

(b) (i) A complex ion is one which contains a central metal atom or ion closely surrounded
by ions or molecules called ligands through dative covalent bonds.
(ii)

(c) (i) Fe(OH)3

26
Transition Elements

(ii) [Fe(H2O)6] 3+(aq) Fe(OH)3(s) + 3H2O(l) + 3H+(aq)


(d) (i) Zn(II) ions has fully filled 3d subshell/orbitals.
Hence, d electrons cannot be promoted from a lower energy level to a higher energy
level (i.e. d-d transition does not occur).
(ii)  In a compound, the iron ions have partially filled 3d orbitals.
 In the presence of ligands, the d orbitals in Fe ions are split into two sets of
different energy levels. This effect is known as d orbital splitting.
 Since the 3d orbitals are partially filled, the electrons from the lower energy d
orbitals can absorb energy corresponding to certain wavelengths from the visible
spectrum and get promoted to the higher energy d orbitals.
 Such d-d transitions are responsible for the colour observed. The colour
observed is the complement of the colour absorbed.

3
(a) (i) A transition element (or transition metal) is a d-block element that forms at least one
ion with partially filled d orbitals.

(ii) Cr+(g)  Cr2+(g) + e


Fe3+(g)  Fe4+(g) + e
23
(iii) Cr+: [Ar]3d5, Mn+: [Ar]3d54s1

The second electron removed from manganese is a 4s−electron while the second
LL

electron removed from chromium is a 3d−electron.


Since a 3d−electron is closer to the nucleus and has lower energy, more energy is
required to remove it, causing second ionisation energy of Cr to be higher than that
of Mn.

(iv) Fe3+: [Ar]3d5, Co3+ : [Ar]3d6

The 4th electron from Co is removed from one d orbital containing a pair of electrons
while the 4th electron from Fe is removed from a singly occupied d orbital. Despite
the higher effective nuclear charge of Co3+ as compared to Fe3+, the removal of the
4th electron from Co is aided by inter-electronic repulsion. Hence, the 4th ionisation
energy of Co is lower than that of Fe.

(b) Solution A containing complex ion, [Fe(CN)6]3, is formed between Fe3+ and excess
CN. Ligand exchange reaction occurs to form a more stable complex.
[Fe(H2O)6]3+ + 6CN [Fe(CN)6]3 + 6H2O

When Fe3+ is added to KI, I is oxidised to brown I2 by Fe3+ while Fe3+ is reduced to
Fe2+. Hence, solution B contains I2 and Fe2+.
2Fe3+ + 2I  2Fe2+ + I2
This redox reaction is feasible as Eocell = 0.77 – 0.54 = +0.23 V > 0

Solution B containing Fe2 can act as a homogenous catalyst for the reaction

27
Transition Elements

between iodide and S2O82 as its reduction potential of 0.77 V lies between that of
Eo (I2/ I) = 0.54 V and Eo (S2O82/SO42) = 2.01 V.

The activation energy for the reaction between iodide and peroxodisulphate, S2O82–
is lowered as reactions are now between oppositely charged ions and are not
between two negatively charged ions.

The catalysed pathway involves 2 steps with each step having a lower activation
energy:
Step 1: 2Fe2 + S2O82  2Fe3 + 2SO42 Eo cell = 2.01 – 0.77 = +1.24 V
Step 2: 2Fe3 + 2I  2Fe2 + I2 Eo cell = 0.77 – 0.54 = +0.23 V

Solution A contains complex ion, [Fe(CN)6]3.


The redox potential of Eo([Fe(CN)6]3 / [Fe(CN)6]4) is +0.36 V, which is not within the
required range. Furthermore, both [Fe(CN)6]3 and I are negatively charged and the
activation energy of such a reaction is still high.

When Na2S2O3(aq) is added to solution B containing I2, a redox reaction occurs in


which I2 is reduced to I, hence brown colour of I2 is discharged.
I2 + 2S2O32  2I + S4O62

When NaOH(aq) is added to solution B containing Fe2+ ions, a green precipitate of


Fe(OH)2 is formed.
23
Fe2+(aq) + 2OH(aq) Fe(OH)2(s)
4Fe(OH)2(s) + O2 + 2H2O  4Fe(OH)3(s)
This redox reaction is feasible as Eocell = 0.40 – (–0.56) = +0.96 V > 0
LL

Hence Fe(OH)2 is oxidised by air to red-brown Fe(OH)3.

(c) (i) Since there is 1 lone pair of electrons on each N atom in NH2CH2CH2NH2, one
NH2CH2CH2NH2 molecule can form 2 dative covalent bonds with the central Co3+ in
the complex. With 3 NH2CH2CH2NH2 bonded to 1 Co3+ in the complex, there will be a
total of 6 dative bonds formed between the ligands and Co3+. Hence, the formula of
the complex is [Co(NH2CH2CH2NH2)3]3+ and the shape of the complex ion is
octahedral.

(ii) Different ligands have different ligand strength and thus the d orbitals are split to
different extents i.e. different E.
NH3 ligand results in a larger E and shorter  absorbed. Thus, [Co(NH3)6]3+ is
yellow-brown. F ligand results in a smaller E and longer  absorbed. Thus, [CoF6]3
appears blue.

4
(a) (i) 1s22s22p63s23p63d5

(ii) As transition metal compounds e.g. Fe(OH)3 contain transition metal ions which have
empty d orbitals which are able to accept the lone pair of electrons from reactant
species, and hence allow their adsorption onto the transition metal compounds.

(iii) Nano-scaled particles have a larger surface area resulting in more / faster adsorption
of the arsenate ions.

28
Transition Elements

(b) (i) Fe(OH)3


2Fe(OH)2 + 2H2O  2Fe(OH)3 + H2

(ii) As H2 is formed, it is constantly lost from the system, hence the position of
equilibrium will shift to the right, favoring the occurrence of the reaction.

(c) (i) SeO32 + 3H2O + 4e  Se + 6OH

(ii) Ligand exchange has occurred, forming soluble [Fe(CN)6]4 ions.

Fe(OH)2 + 6 CN  [Fe(CN)6]4 + 2OH


Or Fe(H2O)4(OH)2 + 6 CN  [Fe(CN)6]4 + 2OH + 4H2O

(iii) [Fe(CN)6]3 + e [Fe(CN)6]4 Eo = +0.36V


E reaction = –0.37 – (+0.36V) = –0.73 V < 0,hence the reaction is not spontaneous.
o

Thus removal of SeO32 ions cannot occur.

5
(a) 2CuFeS2 + 2SiO2 + 4O2 Cu2S + 2FeSiO3 + 3SO2

(b) 1s2 2s2 2p6 3s2 3p6 3d10


23
(c) (i) NH4+

(ii) CuCl42–
LL

(iii) Cu(H2O)62+ (aq) + 4Cl– (aq)  CuCl42– + 6H2O

(iv) Ligand strength: NH3 > Cl– > H2O

(d) (i) Ligands which contain two groups that each have at least a lone pair of electrons
which can be donated to the central metal ion in a complex, forming two dative
covalent bonds.

(ii) [Cu(C2O4)2(H2O)2]2–

(iii)

90o

29
Transition Elements

6
(a) L: Mg(OH)2
M: BaSO4
N: Cr(OH)63
P: CrO42
Q: Cr2O72
R: CH3COOH

(b) Cr2O3 exhibits amphoteric property as Cr2O3 is an ionic compound with significant
covalent character in its oxide.

(c) To ensure all Cr2O3 in the residue forms the soluble complex and can be removed
from Mg(OH)2 residue via filtration as filtrate.

Washed with deionised water to rinse down all the soluble ions on the surface of the
residue e.g. NaOH and the soluble chromium complex.

7
(a) (i) A: Cu(OH)2
B: [Cu(NH3)4(H2O)2]2+

(ii) Cu(OH)2(s) + 4NH3(aq) + 2H2O(l)  [Cu(NH3)4(H2O)2]2+ + 2OH–(aq)


23
(iii) Reducing agent.

(iv)
LL

Cu N H
65.1 28.7 6.2
63.5 14.0 1.0
1.025 2.05 6.2
1 2 6
Cu(NH3)2+

(v) D: CuCl
E: CuCl2–

8
(a) EFe3+/Fe2+ is more positive than ERu3+/Ru2+, the relative stability of +2 oxidation state
compared to +3 oxidation state is larger for iron than ruthenium.

(b) (i) Complex A [Ru(NH3)6]3+


Compound B [Ru(NH3)6]Cl2

Working:
Zn reduces Ru3+ to Ru2+ as Zn is a metal and is a reducing agent.

30
Transition Elements

Let the formula of the complex B be Ru(NH3)6xCl2. This is the general formula

(examples of Ru2+ complexes: [Ru(NH3)6]Cl2, [Ru(NH3)5Cl]Cl, [Ru(NH3)4Cl2])

Since it contains 37 % Ru by mass


101
101+17(6−𝑥)+2(35.5)
= 0.37
101 = 37.37 + 37.74 – 6.29 x + 26.27
0.38 = 6.29x
X=0

B is Ru(NH3)6]Cl2

(ii) Ecell = –0.56 – (–0.76) = +0.20V


Since Fe(OH)3 can be reduced by Zn, the compound C is Fe(OH)2

(c) Fe3+ + 6F– [FeF6]3–


or [Fe(H2O)6]3+ + 6F– [FeF6]3– + 6H2O

(d)
23
or

(e) (i) Kc of [Fe(edta)]– > Kc of [Fe(SCN)(H2O)5]2+.


Edta is a stronger ligand than H2O and SCN– and the deep red [Fe(SCN)(H2O)5]2+
LL

solution changes to yellow [Fe(edta)] –.

(ii) Addition of edta results in forming [Fe(edta)]2– and [Fe(edta)] –.


Since Kc of [Fe(edta)]– > Kc of [Fe(edta)]2–, edta ligand stabilises Fe(III) relative to
Fe(II) to a larger extent than water ligand.
Hence Eo value is less than +0.77 V / decreases.

(f) Fe3+ in [Fe(CN)6]3– has partially-filled d orbitals and d-orbital splitting occurs in the
presence of the CN– ligands. The d electrons from the lower energy d orbitals absorb
energy corresponding to the blue wavelength of light from the visible region of the
electromagnetic spectrum and get promoted to the higher energy d orbitals. The
remaining unabsorbed wavelengths are emitted and appear as the orange colour of
[Fe(CN)6]3– observed.

9
(a) variable oxidation state / complex ion formation / formation of coloured ions / catalytic
activity

(b) CoCl2(aq): catalyst


Tartaric acid: ligand
2H2O2  2H2O + O2

31
Transition Elements

Explanation
Eo/V
Co + e  Co
3+ - 2+
+ 1.82
O2 + 2 H+ + 2 e  H2O2 + 1.77

For aqueous Co2+ complex,


Eocell = 1.77 – 1.82 =  0.05 V (< 0, not feasible)

In the presence of tartaric acid, which is a ligand that stabilizes the +3 state of Co, E
value for Co3+/Co2+ will decrease until it is less than +1.77 V, for example, +1.60 V

Eo/V
Co(III)tartaric acid complex + e  Co(II) tartaric acid complex
-
+ 1.60
O2 + 2 H+ + 2 e  H2O2 + 1.77

For aqueous Co2+ complex,


Eocell = 1.77 – 1.60 = +0.17 V
Hence, Eocell > 0

The green intermediate is the Co3+ complex of tartaric acid.

After which, the green intermediate will be reduced back to the pink Co2+ complex
23
Eo/V
Co(III)tartaric acid complex + e-  Co(II) tartaric acid complex + 1.60
H2O2 + 2 H+ + 2 e-  2 H2O + 0.68
LL

For aqueous Co2+ complex,


Eocell = 1.60  0.68 = +0.92 V

Both steps are now feasible

32
Transition Elements

(c) (i)

(ii) Ligand exchange and redox.


23
(iii) Co3+ is more stable when NH3 is the ligand as seen from the smaller Eo value for the
reduction of [Co(NH3)6]3+ to [Co(NH3)6]2+.
LL

The ligand C2O42 is unable to displace the ammonia ligands in K.


No ligand exchange reaction takes place when C2O42 is added to K.

(d) Co, Fe or Ni

33
Transition Elements

10
(a) No. of co-ordinate bonds: 2

(b) (i) Solution A: green


Solution C: yellow / orange

(ii) d-orbital splitting: A < C < B

Different ligands have different capacities of splitting the d orbitals of a particular


transition metal cation. The longer the wavelength absorbed, the smaller the splitting
of d-orbitals.

(iii) NH3 < dmg < EDTA4–

Based on curve D, although the concentrations of the ligand are the same, the peak
23
of absorbance follows the order of [Ni(EDTA)]2– > [Ni(dmg)2] > [Ni(NH3)6]2+.

The higher the concentration of a particular complex ion present in an aqueous


LL

solution, the more stable the complex formed, hence the ligand will form a stronger
bond with Ni2+.

(c) (i) [Ni (CN)24 ][NH3 ] 6 (0.4)(0.6)6


Kc    933 mol2 dm-6 (3 s.f.)
[Ni (NH3 ) 26 ][CN ] 4 (0.2)(0.1)4

(ii) The intensity of blue colour of the solution will increase, due to the formation of blue
[Ni(NH3)6]2+ ions.

By Le Chatelier’s Principle, addition of excess aqueous ammonia will cause the


position of equilibrium of the above reaction to shift to the left and favour the
formation of the blue [Ni(NH3)6]2+ ions.

(iii) Based on the lg Kstab values, CN– ligands are able to form a more stable complex ion
as compared to NH3 ligands, since CN– ligands have stronger binding power than
NH3 ligands.

The position of equilibrium of the above reaction will thus shift to the right and favour
the formation of [Ni(CN)4]2–, hence Kc for the above reaction is much larger.

34

You might also like